Sei sulla pagina 1di 392

10

CBSE
TERM-2

MATHEMATICS
(As per the latest CBSE Syllabus)

R Chandra
Former Principal
DELHI PUBLIC SCHOOL
Taoru (Haryana)

FULL MARKS PVT LTD


Educational Publishers
New Delhi-110002
Published by:
FULL MARKS PVT LTD
4238A/1, Ansari Road, Daryaganj
New Delhi-110002
Phone: 011-40556600 (100 Lines)
Fax: 011-40556688
E-mail: delhi@fullmarks.org
Website: www.fullmarks.org

Author

All rights reserved. No part of this publication may be reproduced or transmitted, in any form or by any means,
without permission. Any person who does any unauthorised act in relation to this publication may be liable to criminal
prosecution and civil claims for damages.

Branches:
Ahmedabad Bengaluru Chennai Dehradun Guwahati Hyderabad Jaipur
Kolkata Lucknow Mumbai Ranchi

NEW EDITION

This book is meant for educational and learning purposes. The author(s) of the book has/have taken all reasonable
care to ensure that the contents of the book do not violate any existing copyright or other intellectual property rights of
any person in any manner whatsoever. In the event the author(s) has/have been unable to track any source and if any
copyright has been inadvertently infringed, please notify the publisher in writing for corrective action.

Printed at:
Note from the Publisher

FULL MARKS MATHEMATICS-10 has been thoroughly revised in view of the latest
Syllabus and Examination Specifications issued by CBSE.

As per the new modifications, Value-Based Questions carrying 3-5 marks have been
added in the curriculum. An Open-Text Based Assessment shall also be conducted by
the CBSE Board and its marks shall be added in the final examination.

Hence, the revised edition of the book has been divided into two parts. Part-A contains the
solved NCERT Exercises along with additional solved questions whereas Part-B contains
Value-Based Questions selected from the NCERT Textbook. We have also added Exemplar
Problems selected from the book Exemplar Problems Mathematics-10 published by
NCERT which is very helpful for the students in understanding the subject perfectly.

As the book has been divided in two volumes for different TermsSA-I and
SA-II, it lightens the burden of books for the students. It provides the students an
opportunity to have a clear and concentrated view of the Task they have to learn during
two different Terms. Complete CBSE Syllabus for each Term has been given in the
beginning of the course content of the Term-I and II respectively. It helps the students
not to get confused.

The book, therefore, is an up-to-date, dependable and learner-friendly resource. The


support website www.fullmarks.org is an added benefit for the users where one can get
much more and also an opportunity to share ones academic complexities.
SYLLABUS

SECOND TERM Marks: 90


Units Marks
II. Algebra (Contd.) 23
III. Geometry (Contd.) 17
IV. Trigonometry (Contd.) 08
V. Probability 08
VI. Coordinate Geometry 11
VII. Mensuration 23
Total 90

UNIT II: ALGEBRA (Contd.)


3. Quadratic Equations (15 Periods)
Standard form of a quadratic equation ax + bx + c = 0, (a 0). Solution of the quadratic
2

equations (only real roots) by factorization, by completing the square and by using quadratic
formula. Relationship between discriminant and nature of roots.
Situational problems based on quadratic equations related to day to day activities to be
incorporated.
4. Arithmetic Progressions (08 Periods)
Motivation for studying Arithmetic Progression Derivation of the n term and sum of the first
th

n terms of A.P. and their application in solving daily life problems.


UNIT III: GEOMETRY (Contd.)
2. Circles (08 Periods)
Tangent to a circle at a point.
1. (Prove) The tangent at any point of a circle is perpendicular to the radius through the point
of contact.
2. (Prove) The lengths of tangents drawn from an external point to circle are equal.
3. Constructions (08 Periods)
1. Division of a line segment in a given ratio (internally)
2. Tangent to a circle from a point outside it.
3. Construction of a triangle similar to a given triangle.
UNIT IV: TRIGONOMETRY (Contd.)
3. Heights and Distances (08 Periods)
Simple and believable problems on heights and distances. Problems should not involve more
than two right triangles. Angles of elevation/depression should be only 30, 45, 60.
UNIT V: STATISTICS AND PROBABILITY
2. Probability (10 Periods)
Classical definition of probability. Simple problems on single events (not using set notation).
UNIT VI: COORDINATE GEOMETRY
1. Lines (In two-dimensions) (14 Periods)
Review: Concepts of coordinate geometry, graphs of linear equations. Distance between two
points. Section formula (internal division). Area of a triangle.
UNIT VII: MENSURATION
1. Areas Related to Circles (12 Periods)
Motivate the area of a circle; area of sectors and segments of a circle. Problems based on areas
and perimeter/circumference of the above said plane figures. (In calculating area of segment of
a circle, problems should be restricted to central angle of 60, 90 and 120 only. Plane figures
involving triangles, simple quadrilaterals and circle should be taken.)
2. Surface Areas and Volumes (12 Periods)
(i) Surface areas and volumes of combinations of any two of the following: cubes, cuboids,
spheres, hemispheres and right circular cylinders/cones. Frustum of a cone.
(ii) Problems involving converting one type of metallic solid into another and other
mixed problems. (Problems with combination of not more than two different solids be taken.)

OTBA will appear only in SA-II for 10 marks, from statistic UnitVII. The unit will be tested on OTBA only.
QUESTION PAPER DESIGN

Very Short Short Long


Short Answer-I Answer-II Answer
Answer (SA) (SA) (LA) Total %
S. No. Typology of Questions
(VSA) Marks Weightage

1 Mark 2 Marks 3 Marks 4 Marks


1. REMEMBERING
(Knowledge based- Simple
recall questions, to know
specificfacts, terms, 1 2 2 3 23 26%
concepts, principles, or
theories, Identify, define, or
recite, information)
2. UNDERSTANDING-
(Comprehension- to be
familiar with meaning and
to understand conceptually, 2 1 1 4 23 26%
interpret, compare, contrast,
explain, paraphrase, or
interpret information)

3. APPLICATION
(Use abstract information
in concrete situation, to
apply knowledge to new
1 2 3 2 22 24%
situations. Use given
content to interpret a
situation, provide an
example, or solve a problem)
4. HIGH ORDER THINKING
SKILLS
(Analysis & Synthesis-
Classify, compare,
contrast or differentiate
1 4 14 16%
between different pieces of
information. Organize and/
or integrate unique pieces of
information from a variety of
sources)
5. CREATING, EVALUATION
AND MULTI-DISCIPLINARY
(Generating new ideas,
product or ways of viewing
things, Appraise, judge, 2* 8 8%
and/or justify the value
or worth of a decision or
outcome, or to predict
outcomes based on values)
TOTAL 41=4 62=12 103=30 114=44 90 100%

* One of the LA (4 marks) will be to assess the values inherent in the texts.
CONTENTS

II. ALGEBRA (Contd.)


4. Quadratic Equations.............................................................................................................. 9
5. Arithmetic Progressions........................................................................................................ 73

III. GEOMETRY (Contd.)


10. Circles....................................................................................................................................... 148
11. Constructions.......................................................................................................................... 176

IV. TRIGONOMETRY (Contd.)


9. Some Applications of Trigonometry................................................................................... 197

V. STATISTICS AND PROBABILITY


15. Probability............................................................................................................................... 227

VI. COORDINATE GEOMETRY


7. Coordinate Geometry............................................................................................................ 257

VII. MENSURATION
12. Areas Related to Circles........................................................................................................ 303
13. Surface Areas and Volumes................................................................................................. 334

Value Based Questions (Solved).......................................................................................... 373


SYLLABUS
CLASS-X

Second Term Marks: 90

Units Marks
II. Algebra (Contd.) 23
III. Geometry (Contd.) 17
IV. Trigonometry (Contd.) 08
V. Probability 08
VI. Coordinate Geometry 11
VII. Mensuration 23

TOTAL 90

DIVISION OF CHAPTERS
Sl. Nos. and Names of Chapters in NCERT Book
4. Quadratic Equations
5. Arithmetic Progressions
10. Circles
11. Constructions
9. Some Applications of Trigonometry
15. Probability
7. Coordinate Geometry
12. Areas Related to Circles
13. Surface Areas and Volumes
Term2

4
[Unit II: Algebra (Contd.)]

Quadratic Equations

Facts that Matter


z Quadratic Equations
An equation of the form ax2 + bx + c = 0, where a, b, c are real numbers, and a 0, is called
a quadratic equation in the variable x.
Examples:
x2 3x + 4 = 0
3
2y 2 + y2 = 0
2
2x2 149 = 0
z Roots of a Quadratic Equation:
A value x = is said to be a root of ax2 + bx + c = 0 if it satisfies the equation.
Example: 2 is a root of 3x2 x 10 = 0 because 3(2)2 2 10 = 0.
A quadratic equation has at most two roots (zeroes). Either it has no zero, or two zeroes.
We can solve a quadratic equation by factorisation.
Example: Solve 6x2 x 2 = 0
6x2 + 3x 4x 2 = 0
3x(2x + 1) 2(2x + 1) = 0
(3x 2) (2x + 1) = 0
3x 2 = 0 or 2x + 1 = 0
2 1
x = or x=
3 2
These are the two roots of the equation.

NCERT TEXTBOOK QUESTIONS SOLVED


EXERCISE 4.1
Q. 1. Check whether the following are quadratic equations:
(i) (x + 1)2 = 2(x 3) (ii) x2 2x = (2) (3 x)
(iii) (x 2) (x + 1) = (x 1) (x + 3) (iv) (x 3) (2x + 1) = x(x + 5)

9
(v) (2x 1) (x 3) = (x + 5) (x 1) (vi) x2 + 3x + 1 = (x 2)2
(vii) (x + 2)3 = 2x(x2 1) (viii) x3 4x2 x + 1 = (x 2)3
Sol. (i) (x + 1)2 = 2(x 3)
We have:
(x + 1)2 = 2 (x 3)
x2 + 2x + 1 = 2x 6
x2 + 2x + 1 2x + 6 = 0
x2 + 7 = 0
Since x2 + 7 is a quadratic polynomial
(x + 1)2 = 2(x 3) is a quadratic equation.
(ii) x2 2x = (
2) (3 x)
We have:
x2 2x = ( 2) (3 x)
x2 2x = 6 + 2x
x2 2x 2x + 6 = 0
x2 4x + 6 = 0
Since x2 4x + 6 is a quadratic polynomial
x2 2x = (2) (3 x) is a quadratic equation.
(iii) (x 2) (x + 1) = (x 1) (x + 3)
We have:
(x 2) (x + 1) = (x 1) (x + 3)
x2 x 2 = x2 + 2x 3
x2 x 2 x2 2x + 3 = 0
3x + 1 = 0
Since 3x + 1 is a linear polynomial
(x 2) (x + 1) = (x 1) (x + 3) is not quadratic equation.
(iv) (x 3) (2x + 1) = x(x + 5)
We have:
(x 3) (2x + 1) = x (x + 5)
2x2 + x 6x 3 = x2 + 5x
2x2 5x 3 x2 5x = 0
x2 + 10x 3 = 0
Since x2 + 10x 3 is a quadratic polynomial
(x 3) (2x + 1) = x(x + 5) is a quadratic equation.
(v) (2x 1) (x 3) = (x + 5) (x 1)
We have:
(2x 1) (x 3) = (x + 5) (x 1)

10 MathematicsX
2x2 6x x + 3 = x2 x + 5x 5
2 2
2x x 6x x + x 5x + 3 + 5 = 0
x2 11x + 8 = 0
2
Since x 11x + 8 is a quadratic polynomial
(2x 1) (x 3) = (x + 5) (x 1) is a quadratic equation.
(vi) x2 + 3x + 1 = (x 2)2
We have:
x2 + 3x + 1 = (x 2)2
x2 + 3x + 1 = x2 4x + 4
x + 3x + 1 x2 + 4x 4 = 0
2

7x 3 = 0
Since 7x 3 is a linear polynomial.
x2 + 3x + 1 = (x 2)2 is not a quadratic equation.
(vii) (x + 2)3 = 2x(x2 1)
We have:
(x + 2)3 = 2x(x2 1)
x + 3x (2) + 3x(2)2 + (2)3 = 2x3 2x
3 2

x3 + 6x2 + 12x + 8 = 2x3 2x


x + 6x + 12x + 8 2x3 + 2x = 0
3 2

x3 + 6x2 + 14x + 8 = 0
Since x + 6x2 + 14x + 8 is a polynomial of degree 3
3

(x + 2)3 = 2x(x2 1) is not a quadratic equation.


(viii) x3 4x2 x + 1 = (x 2)3
We have:
x3 4x2 x + 1 = (x 2)3
x3 4x2 x + 1 = x3 + 3x2( 2) + 3x( 2)2 + ( 2)3
x3 4x2 x + 1 = x3 6x2 + 12x 8
x3 4x2 x 1 x3 + 6x2 12x + 8 = 0
2x2 13x + 9 = 0
Since 2x2 13x + 9 is a quadratic polynomial
x3 4x2 x + 1 = (x 2)3 is a quadratic equation.
Q. 2. Represent the following situations in the form of quadratic equations:
(i) The area of a rectangular plot is 528 m2. The length of the plot (in metres) is one more than
twice its breadth. We need to find the length and breadth of the plot.
(ii) The product of two consecutive positive integers is 306. We need to find the integers.
(iii) Rohans mother is 26 years older than him. The product of their ages (in years) 3 years from
now will be 360. We would like to find Rohans present age.
(iv) A train travels a distance of 480 km at a uniform speed. If the speed had been 8 km/h less,
then it would have taken 3 hours more to cover the same distance. We need to find the speed
of the train.
Sol. (i) Let the breadth = x metres
Length = 2 (Breadth) + 1

Quadratic Equations 11
Length = (2x + 1) metres
Since Length Breadth = Area
(2x + 1) x = 528
2x2 + x = 528
2x2 + x 528 = 0
Thus, the required quadratic equation is
2x2 + x 528 = 0
(ii) Let the two consecutive numbers be x and (x + 1).
QProduct of the numbers = 306
x (x + 1) = 306
x2 + x = 306
x2 + x 306 = 0
Thus, the required equdratic equation is
x2 + x 306 = 0
(iii) Let the present age = x
Mothers age = (x + 26) years
After 3 years
His age = (x + 3) years
Mothers age = [(x + 26) + 3] years
= (x + 29) years
According to the condition,
LMProduct of their agesOP = 360
Nafter 3 years Q
(x + 3) (x + 29) = 360
x2 + 29x + 3x + 87 = 360
x2 + 29x + 3x + 87 360 = 0
x2 + 32x 273 = 0
Thus, the required quadratic equation is
x2 + 32x 273 = 0
(iv) Let the speed of the train = u km/hr
Distance covered = 480 km
Time taken = Distance Speed
= (480 u) hours
480
= hours
u
In second case,
Speed = (u 8) km/hour
Distance 480
= hours
Time taken =
speed u8b g
12 MathematicsX
According to the condition,
480 480
= 3
u8 u
480u 480(u 8) = 3u(u 8)
480u 480u + 3840 = 3u2 24u
3840 3u2 + 24u = 0
1280 u2 + 8u = 0
1280 + u2 8u = 0
u2 8u 1280 = 0
Thus, the required quadratic equation is
u2 8u 1280 = 0

NCERT TEXTBOOK QUESTIONS SOLVED


EXERCISE 4.2
Q. 1. Find the roots of the following quadratic equations by factorisation:
(i) x2 3x 10 = 0 (ii) 2x2 + x 6 = 0
1
(iii) 2x 2 + 7x + 5 2 = 0 (iv) 2x 2 x + = 0
8
(v) 100x2 20x + 1 = 0
Sol. (i) x2 3x 10 = 0
We have:
x2 3x 10 = 0
x2 5x + 2x 10 = 0
x (x 5) + 2 (x 5) = 0
(x 5) (x + 2) = 0
Either x 5 = 0 x = 5
or x+2 = 0 x=2
Thus, the required roots are x = 5 and x = 2.
(ii) 2x2 + x 6 = 0
We have:
2x2 + x 6 = 0
2
2x + 4x 3x 6 = 0
2x(x + 2) 3 (x + 2) = 0
(x + 2) (2x 3) = 0
Either x + 2 = 0 x = 2
3
or 2x 3 = 0 x=
2
3
Thus, the required roots are x = 2 and x = .
2
(iii) 2x 2 + 7 x + 5 2 = 0
We have:
2x2 + 7x + 5 2 = 0
2 x 2 + 2x + 5 x + 5 2 = 0

Quadratic Equations 13
2 x2 + e j
2 2 x + 5x + 5 2 = 0

2x x + 2 + 5 x + 2 = 0

ex + 2 je 2x + 5 j = 0

Either x+ 2 = 0 x = 2
5
or 2x + 5 = 0 x =
2
5
Thus, the required roots are x = 2 and x = .
2
1
(iv) 2x2 x + = 0
8
We have:
1
2x2 x + = 0
8
16x2 8x + 1 = 0
16x2 4x 4x + 1 = 0
4x(4x 1) 1(4x 1) = 0
(4x 1) (4x 1) = 0
1 1
x = and x =
4 4
1 1
Thus, the required roots are x = and x = .
4 4
(v) 100x2 20x + 1 = 0
We have:
100x2 20x + 1 = 0
2
100x 10x 10x + 1 = 0
10x (10x 1) 1 (10x 1) = 0
(10x 1) (10x 1) = 0
(10x 1) = 0 and (10x 1) = 0
1 1
x = and x =
10 10
1 1
Thus, the required roots are x = and x = .
10 10
Q. 2. Solve the problems given in Example 1.
Sol. (i) We have:
x2 45x + 324 = 0 Q ( 9) ( 36) = 324 and
2
x 9x 36x + 324 = 0 ( 9) + ( 36) = 45
x(x 9) 36(x 9) = 0
(x 9) (x 36) = 0
Either x 9 = 0 x = 9
or x 36 = 0 x = 36
Thus, x = 9 and x = 36

14 MathematicsX
(ii) We have:
x2 55x + 750 = 0
x2 30x 25x + 750 = 0 Q ( 30) ( 25) = 750 and
x (x 30) 25 (x 30) = 0 ( 30) + ( 25) = 55
(x 30) (x 25) = 0
Either x 30 = 0 x = 30
or x 25 = 0 x = 25
Thus, x = 30 and x = 25.
Q. 3. Find two numbers whose sum is 27 and product is 182.
Sol. Here, sum of the numbers is 27.
Let one of the numbers be x.
Other number = 27 x
According to the condition,
Product of the numbers = 182
x (27 x) = 182
27x x2 = 182
x2 + 27x 182 = 0
x2 27x + 182 = 0
x2 13x 14x + 182 = 0 Q 27 = ( 13) + ( 14) and
x (x 13) 14 (x 13) = 0 ( 13) ( 14) = 182
(x 13) (x 14) = 0
Either x 13 = 0 x = 13
or x 14 = 0 x = 14
Thus, the required numbers are 13 and 14.
Q. 4. Find two consecutive positive integers, sum of whose squares is 365.
Sol. Let the two consecutive positive integers be x and (x + 1).
Since the sum of the squares of the numbers = 365
x2 + (x + 1)2 = 365
x + [x2 + 2x + 1] = 365
2

x2 + x2 + 2x + 1 = 365
2
2x + 2x + 1 365 = 0
2x2 + 2x 364 = 0
x2 + x 182 = 0
x2 + 14x 13x 182 = 0 Q + 14 13 = 1 and
x (x + 14) 13 (x + 14) = 0 14 ( 13) = 182
(x + 14) (x 13) = 0
Either x + 14 = 0 x = 14
or x 13 = 0 x = 13
Since x has to be a positive integer
x = 13
x + 1 = 13 + 1 = 14
Thus, the required consecutive positive integers are 13 and 14.

Quadratic Equations 15
Q. 5. The altitude of a right triangle is 7 cm less than its base. If the hypotenuse is 13 cm, find the other
two sides.
Sol. Let the base of the given right triangle be x cm.
Its height = (x 7) cm
Q Hypotenuse= aBasef + bHeightg
2 2

13 = x + bx 7g
2 2

Squaring both sides, we get


169 = x2 + (x 7)2
169 = x2 + x2 14x + 49
2
2x 14x + 49 169 = 0
2x2 14x 120 = 0
x2 7x 60 = 0 Q ( 12) 5 = 60
2
x 12x + 5x 60 = 0 and ( 12) + 5 = 7
x (x 12) + 5 (x 12) = 0
(x 12) (x + 5) = 0
Either x 12 = 0 x = 12
or x+5 = 0 x=5
But the side of triangle can never be negative,
x = 12
Length of the base = 12 cm
Length of the height = (12 7) cm = 5 cm
Thus, the required base = 12 cm and height = 5 cm.
Q. 6. A cottage industry produces a certain number of pottery articles in a day. It was observed on a
particular day that the cost of production of each article (in rupees) was 3 more than twice the
number of articles produced on that day. If the total cost of production on that day was ` 90, find
the number of articles produced and the cost of each article.
Sol. Let the number of articles produced in a day = x
Cost of production of each article = ` (2x + 3)
According to the condition,
Total cost = 90
x (2x + 3) = 90
2x2 + 3x = 90
2x2 + 3x 90 = 0
2
2x 12x + 15x 90 = 0
2x (x 6) + 15 (x 6) = 0
(x 6) (2x + 15) = 0
Either x 6 = 0 x = 6
15
or 2x + 15 = 0 x =
2
But the number of articles cannot be negative.
15
x = is not required
2
16 MathematicsX
x = 6
Cost of each article = ` (2 6 + 3) = ` 15
Thus, the required number of articles produced is 6 and the cost of each article is ` 15.
z Solving a quadratic equation by completing the square:
Example: Solve x2 + 4x 5 = 0.
x2 + 4x 5 = 0
2
x + (2) (2) (1) x 5 = 0
x2 + (2) (2) (1) x + (2)2 (2)2 5 = 0
(x + 2)2 9 = 0
(x + 2)2 = 9
x +2 = 3
x = 32 or x = 3 2
x = 1 or x = 5
z Steps involved in solving ax2 + bx + c = 0
1. Divide the equation by a:
b c
x2 + x+ = 0
a a
b b FG IJ
b FG IJ FG b IJ
2 2
2. Write
a
as 2
2a H K
(1), and c as
2a H K H 2a K + c.

3. Therefore, the equation becomes:

x2 + 2
FG b IJ a1f + FG b IJ FG b IJ 2 2

H 2 a K H 2a K H 2a K +c = 0

Here, the first three terms are a perfect square.

FG b IJ 2 LMFG b IJ 2 OP
x+
H 2a K
MNH 2a K
c = 0
PQ
b
4. Now this can be solved easily to first find the value of x + and then x.
2a
z Quadratic Formula:
The roots of ax2 + bx + c = 0 are also given simply by the quadratic formula:
- b b 2 - 4 ac
x = 2a

-b b 2 - 4 ac
i.e., x = +
2a 2a

-b b 2 - 4 ac
or x = -
2a 2a
The term b2 4ac is said to be the discriminant D.
(i) If D > 0, roots exist and they are distinct.
(ii) If D = 0, the two roots are equal and real.

(iii) If D < 0, b 2 4 ac does not exist, so there are no real roots.

Quadratic Equations 17
NCERT TEXTBOOK QUESTIONS SOLVED
EXERCISE 4.3
Q. 1. Find the roots of the following quadratic equations, if they exist, by the method of completing the
square:
(i) 2x2 7x + 3 = 0 (ii) 2x2 + x 4 = 0
(iii) 4x 2 + 4 3x + 3 = 0 (iv) 2x2 + x + 4 = 0
Sol. (i) 2x2 7x + 3 = 0
Dividing throughout by the co-efficient of x2, we get
7 3
x2 x+ = 0
2 2
RSx 1 FG 7 IJ UV FG 7 IJ + 3
2 2

T 2 H 2KW H 4K 2 = 0


RSx 7 UV 49 + 3
2
= 0
T 4 W 16 2

RSx 7 UV 49 + 24
2
= 0
T 4 W 16 16

RSx 7 UV 25
2
= 0
T 4 W 16
RSx 7 UV 2
25 FG 5 IJ 2

T 4W =
16
=
H 4K
7 5
x =
4 4
Case I:
5
When is +ve, then
4
7 5 5 7
x = x= +
4 4 4 4

12
x= = 3
4
Case II:
5
When is ve, then
4
7 5 5 7
x = + x=
4 4 4 4
2 1
x= =
4 2
Thus, required roots are
1
x = 3 and x =
2
18 MathematicsX
(ii) 2x2 + x 4 = 0
We have:
2x2 + x 4 = 0
Dividing throughout by 2,
x
x2 + 2 = 0
2
RSx + 1 1 UV FG 1 IJ 2
2 2

T 2 2W H 4K = 0

FG x + 1 IJ 1 2
2

H 4 K 16 = 0

FG x + 1 IJ 33
2

H 4 K 16 = 0

FG x + 1 IJ 2
33 F 33 I 2


H 4K =
16
= GH 4 JK
FG x + 1 IJ 33

H 4K =
4
Case I:
33
When is positive, then
4
1 33 33 1
x+ = x=
4 4 4 4
33 1
x=
4
Case II:
33
When is negative, then
4
1 33 33 1
x+ = x=
4 4 4 4
33 1
x=
4
Thus, the required roots are
33 1 33 1
x = and x=
4 4
(iii) 4x2 + 4 3 x + 3 = 0
Dividing throughout by 4, we have:
3
x2 + 3 x + = 0
4

Quadratic Equations 19
LMx + FG 1 3 IJ OP FG 1 3 IJ
2 2
3

N H 2 KQ H 2 K +
4
= 0


LMx + 3 OP 3 + 3 = 0
2

MN 2 PQ 4 4

LMx + 3 OP = 0 2

MN 2 PQ
Fx + 3 I Fx + 3 I = 0
GH 2 JK GH 2 JK
3 3
x = and x =
2 2
(iv) 2x2 + x + 4 = 0
Dividing throughout by 2, we have:
x
x2 + +2 = 0
2
LMx + 1 OP FG 1 IJ + 2
2 2

N 4Q H 4K = 0


LMx + 1 OP 1 + 2 2
= 0
N 4 Q 16

LMx + 1 OP + 31 2
= 0
N 4 Q 16

LMx + 1 OP
31
2
=
N 4Q
16
But the square of a number cannot be negative.

x+LM 1 OP 2
cannot be a real value.
N 4 Q
So, no real roots exist.
There is no real value of x satisfying the given equation.
Q. 2. Find the roots of the following quadratic equations, using the quadratic formula:
(i) 2x2 7x + 3 = 0
(ii) 2x2 + x 4 = 0
(iii) 4x2 + 4 3 x + 3 = 0
(iv) 2x2 + x + 4 = 0
Sol. (i) 2x2 7x + 3 = 0
Comparing the given equation with ax2 + bx + c = 0, we have:
a = 2
b = 7
c = 3

20 MathematicsX
b2 4ac = ( 7)2 4 (2) (3)
= 49 24 = 25 0
2
Since b 4ac > 0
The given equation has real roots. The roots are given by

b b 2 4 ac
x =
2a
b g
7 25
2 a2f
x =

75
=
4
Taking +ve sign,
7+5 12
x = = = 3
4 4
Taking ve sign,
75 2 1
x = = =
4 4 2
Thus, the roots of the given equation are
1
x = 3 and x =
2
(ii) 2x2 + x 4 = 0
Comparing the given equation with ax2 + bx + c = 0 we have:
a = 2
b = 1
c = 4
b 4ac = (1)2 4 (2) ( 4)
2

= 1 + 32
= 33 > 0
Since b2 4ac > 0
The given equation has equal roots. The roots are given by

b b 2 4 ac
x =
2a
1 33 1 33
=
x =
2 2af 4
Taking +ve sign,
1 + 33
x =
4
Taking ve sign,
1 33
x =
4
Quadratic Equations 21
Thus, the required roots are:
1 + 33 1 33
x = and x=
4 4
(iii) 4x2 + 4 3 x + 3 = 0
Comparing the given equation with ax2 + bx + c = 0, we have:
a = 4
b = 4 3
c = 3

b2 4ac = e4 3 j 2
a fa f
4 4 3
= [16 3] 48
= 48 48 = 0
2
Since b 4ac = 0
The given equation has real roots, which are given by:

b b 2 4 ac
x =
2a
4 3 0
x =
2 4af
4 3 0 3 0
= =
8 2
3 3
x = and x =
2 2
(iv) 2x2 + x + 4 = 0
Comparing the given equation with ax2 + bx + c = 0, we have:
a = 2
b = 1
c = 4
b 4ac = (1)2 4 (2) (4)
2

= 1 32 = 31 < 0
2
Since b 4ac is less than 0, therefore the given equation does not have real roots.
Q. 3. Find the roots of the following equations:
1 1 1 11
(i) x = 3, x 0 (ii) = , x 4, 7 [CBSE 2012]
x x + 4 x 7 30
1
Sol. (i) x = 3, x 0
x
Here, we have:
1
x = 3
x
x2 1 = 3x

22 MathematicsX
x2 3x 1 = 0 ...(1)
Comparing (1) with ax2 + bx + c = 0, we have:
a = 1
b = 3
c = 1
b2 4ac = ( 3)2 4 (1) ( 1)
= 9 + 4 = 13 > 0

b b 2 4 ac
x =
2a
b g
3 13
2 a1f
x =

3 13
=
2
Now, taking +ve sign,
3 + 13
x =
2
Taking ve sign,
3 13
x =
2
Thus, the required roots of the given equation are:
3 + 13 3 13
x = and x =
2 2
1 1 11
(ii) = ; x 4, 7
x+4 x7 30
We have:
1 1 11
=
x+4 x7 30

(x 7) (x + 4) =
11
30
b
x+4 x7gb g
11 2
x7x4 = (x 3x 28)
30
11 30 = 11 (x2 3x 28)
30 = x2 3x 28
2
x 3x 28 + 30 = 0
x2 3x + 2 = 0 ...(1)
Comparing (1) with ax2 + bx + c = 0, we have:
a = 1
b = 3
c = 2
Quadratic Equations 23
b2 4ac = ( 3)2 4 (1) (2)
= 98=1>0
The quadratic equation (1) has real roots, which are given by:

b b 2 4 ac
x =
2a
b g
3 1
2 a1f
x =

31
=
2
Taking +ve sign, we have:
3+1 4
x = = =2
2 2
Taking ve sign, we have:
31
x = =1
2
Thus, the roots of the given equation are:
x = 2 and x = 1.
1
Q. 4. The sum of the reciprocals of Rehmans ages, (in years) 3 years ago and 5 years from now is .
3
Find his present age.
Sol. Let the present age of Rehman = x
3 years ago Rehmans age = (x 3) years
5 years later Rehmans age = (x + 5) years
Now, according to the condition,
1 1 1
+ =
x3 x+5 3


b x + 5g + b x 3g 1
bx 3gbx + 5g =
3
3 [x + 5 + x 3] = (x 3) (x + 5)
3 [2x + 2] = x2 + 2x 15
6x + 6 = x2 + 2x 15
2
x + 2x 6x 15 6 = 0
x2 4x 21 = 0 ...(1)
Now, comparing (1) with ax2 + bx + c = 0, we have:
a = 1
b = 4
c = 21
b 4ac = ( 4)2 4 (1) ( 21)
2

= 16 + 84
= 100
24 MathematicsX
-b b 2 - 4 ac
Since, x =
2a

- ( - 4 ) 100
x =
2 (1)

4 10
=
2
Taking positive sign, we have:

4 + 10 14
x = = = 7
2 2
Taking negative sign, we have:
4 10 6
x = = = 3
2 2
Since age cannot be negative,
x 3 x=7
So, the present age of Rehman = 7 years.
Q. 5. In a class test, the sum of Shefalis marks in Mathematics and English is 30. Had she got 2 marks
more in Mathematics and 3 marks less in English, the product of their marks would have been
210? Find her marks in the two subjects. [CBSE 2012]
Sol. Let, Shefalis marks in Mathematics = x
Marks in English = (30 x) [Q Sum of their marks in Eng. and Maths = 30]
Now, according to the condition,
(x + 2) [(30 x) 3] = 210
(x + 2) (30 x 3) = 210
(x + 2) (x + 27) = 210
x2 + 25x + 54 = 210
x2 + 25x + 54 210 = 0
x2 + 25x 156 = 0
x2 25x + 156 = 0 ...(1)
Now, comparing (1) with ax2 + bx + c = 0
a = 1
b = 25
c = 156
b2 4ac = (25)2 4(1) (156)
= 625 624 = 1

b b 2 4 ac
Since, x =
2a

Quadratic Equations 25
x =
b g
25 1
2 a1f
25 1
x =
2
Taking +ve sign, we have
25 + 1 26
x = = = 13
2 2
Taking ve sign, we get
25 1 24
x = = = 12
2 2
When x = 13, then 30 13 = 17
When x = 12, then 30 12 = 18
Thus, marks in Maths = 13, marks in English = 17
marks in Maths = 12, marks in English = 18
Q. 6. The diagonal of a rectangular field is 60 metres more than the shorter side. If the longer side is
30 metres more than the shorter side, find the sides of the field.
Sol. Let the shorter side (i.e., breadth) = x metres.
The longer side (length) = (x + 30) metres.
In a rectangle,

diagonal = abreadthf + blengthg


2 2

x + b x + 30g
2 2
x + 60 =

x + 60 = x 2 + x 2 + 60 x + 900
(x + 60)2 = 2x2 + 60x + 900
x + 120x + 3600 = 2x2 + 60x + 900
2

2x2 x2 + 60x 120x + 900 3600 = 0


x2 60x 2700 = 0 ...(1)
Comparing (1) with ax2 + bx + c = 0
a = 1
b = 60
c = 2700
b 4ac = ( 60)2 4 (1) ( 2700)
2

b2 4ac = 3600 + 10800


b2 4ac = 14400

b b 2 4 ac
Since, x =
2a

x =
b g
60 14400
af
2 1

26 MathematicsX
60 120
x =
2
Taking +ve sign, we get
60 + 120 180
x = = = 90
2 2
Taking ve sign,
60 120 60
x = = = 30
2 2
Since breadth cannot be negative,
x 30 x = 90
x + 30 = 90 + 30 = 120
Thus, the shorter side = 90 m
The longer side = 120 m.
Q. 7. The difference of squares of two numbers is 180. The square of the smaller number is 8 times the
larger number. Find the two numbers.
Sol. Let the larger number be x.
Since,
(smaller number)2 = 8 (larger number)
(smaller number)2 = 8x
smaller number = 8x
Now, according to the condition,

x2 e 8x j 2
= 180
x2
8x = 180
2
x 8x 180 = 0 ...(1)
Comparing (1) with ax2 + bx + c = 0, we have:
a = 1
b = 8
c = 180
b 4ac = ( 8)2 4 (1) ( 180)
2

= 64 + 720 = 784

b b 2 4 ac
Since, x =
2a
b g
8 784
x =
af
2 1
8 28
x = Q 784 = 28
2
Taking +ve sign, we get
8 + 28 36
x = = = 18
2 2
Quadratic Equations 27
Taking ve sign, we get
8 28 20
x = = = 10
2 2
But x = 10 is not admissible,
The smaller number = 18
8 18 = 144 = 12
Thus, the larger number = 12 or 12
Thus, the two numbers are:
18 and 12 or 18 and 12
Q. 8. A train travels 360 km at a uniform speed. If the speed had been 5 km/h more, it would have taken
1 hour less for the same journey. Find the speed of the train. [NCERT Exemplar]
Sol. Let the uniform speed of the train be x km/hr.
Distance
Since, time taken by the train =
Speed
360
hr time =
x
When speed is 5 km/hr more then time is 1 hour less.
360 360
= 1
x+5 x

360 360
= 1
x+5 x

360
LM 1 1 OP = 1
Nx + 5 xQ
1 1 1
=
x+5 x 360


x x+5 b g 1
b
x x+5 g =
360

x x5 =
b
1 x+5 x g
360
5 360 = 1 (x2 + 5x)
5 360 = x2 5x Q 360 5 = 1800
2
x + 5x 1800 = 0 ...(1)
Comparing (1) with
ax2 + bx + c = 0
a = 1
b = 5
c = 1800

28 MathematicsX
b2 4ac = (5)2 4 (1) ( 1800)
= 25 + 7200
= 7225
-b b 2 - 4 ac
Since x =
2a

5 7225
x =
af
2 1
5 85
x =
2 1af Q 7225 = 85

Taking +ve sign, we get:


5 + 85 80
x = = = 40
2 2
Taking ve sign, we get
5 85 90
x = = = 45
2 2
Since, the speed of a vehicle cannot be negative,
x 45
Thus, x = 40 speed of the train = 40 km/hr.
3
Q. 9. Two water taps together can fill a tank in 9 hours. The tap of larger diameter takes 10 hours
8
less than the smaller one to fill the tank separately. Find the time in which each tap can separately
fill the tank.
Sol. Let the smaller tap fills the tank in x hours.
The larger tap fills the tank (x 10) hours.
1
Time to fill the tank by smaller tap = hours.
x
1
Time to fill the tank by larger tap = hours.
x 10
1 1 x 10 + x
Since, the tank filled by the two taps together in 1 hour = + =
x x 10 b
x x 10 g
2 x 10
=
x 2 10 x
Now, according to the condition,
F
75 2 x 10 I 3 75
GH
8 x 2 10 x JK = 1 Q 9
8
=
8

75 b2 x 10g
= 1
8 e x 10 xj
2

Quadratic Equations 29
150 x 750
= 1
8 x 2 80 x
8x2 80x = 150x 750
8x2 80x 150x = 750
8x2 230x + 750 = 0 ...(1)
Comparing (1) with ax2 + bx + c = 0, we get
a = 8
b = 230
c = 750
2
b 4ac = (230)2 4 (8) (750)
= 52900 24000
= 28900

b b 2 4 ac
Since x =
2a

x =
b g
230 28900
2 8 af
230 170
x = Q 28900 = 170
16
Taking, the +ve sign, we get
230 + 170 400
x = = = 25
16 16
Taking the ve sign, we get
230 170 60 15
x = = =
16 16 4

15 15 25
For x = , (x 10) = 10 =
4 4 4
which is not possible, [ Time cannot be negative]
x = 25
x 10 = 25 10 = 15
Thus, time to fill the tank by the smaller tap alone = 25 hours and larger tap
alone = 15 hours.
Q. 10. An express train takes 1 hour less than a passenger train to travel 132 km between Mysore and
Bangalore (without taking into consideration the time they stop at intermediate stations). If the
average speed of the express train is 11 km/h more than that of the passenger train, find the
average speed of the two trains.
Sol. Let the average speed of the passenger train be x km/hr
Average speed of the express train = (x + 11) km/hr
Total distance covered = 132 km

30 MathematicsX
Distance
Also, Time =
Speed

132
Time taken by the passenger train = hour
x

132
Time taken by the express train = hour
x + 11
According to the condition, we get
132 FG 132 IJ 1
x + 11
=
HxK
132 132
= 1
x + 11 x

132
LM 1 1 OP = 1
N x + 11 x Q
132 M
L x x 11 OP

MN x bx + 11g PQ = 1

132 M
L 11 OP = 1
MN x + 11x PQ
2

11 (132) = 1 (x2 + 11x)


1452 = 1 (x2 + 11x)
x2 + 11x 1452 = 0 ...(1)
Comparing (1) with ax2 + bx + c = 0,
a = 1
b = 11
c = 1452
b2 4ac = (11)2 4 (1) (1452)
b2 4ac = 121 + 5808 = 5929

b b 2 4 ac
Since, x =
2a

11 5929 11 77
=
=
af
2 1 2
Q 5929 = 77

Taking +ve sign, we get


11 + 77 66
x = = = 33
2 2

Quadratic Equations 31
Taking ve sign, we get
11 77 88
x = = = 44
2 2
But average speed cannot be negative
x 44
x = 33
Average speed of the passenger train = 33 km/hr
Average speed of the express train = (x + 11) km/hr
= (33 + 11) km/hr
= 44 km/hr
Q. 11. Sum of the areas of two squares is 468 m2. If the difference of their perimeters is 24 m, find the
sides of the two squares.
Sol. Let the side of the smaller square be x m.
Perimeter of the smaller square = 4x m.
Perimeter of the larger square = (4x + 24) m
Perimeter
Side of the larger square =
4

=
4 x + 24
m =
4 x+6b g
m = x+6 m b g
4 4
Area of the smaller square = (side)2 = (x)2 m2
Area of the larger square = (x + 6)2 m2
According to the condition,
x2 + (x + 6)2 = 468
x2 + x2 + 12x + 36 = 468
2x2 + 12x 432 = 0
x2 + 6x 216 = 0 ...(1)
Comparing (1) with ax2 + bx + c = 0
a = 1
b = 6
c = 216
b 4ac = (6)2 4 (1) ( 216)
2

= 36 + 864 = 900

b b 2 4 ac
Since, x =
2a

6 900
x =
af
2 1

6 30
x =
2

32 MathematicsX
Taking +ve sign, we have:

6 + 30 24
x = = = 12
2 2
Taking ve sign, we get

6 30 36
x = = = 18
2 2
But the length of a square cannot be negative
x = 12
Length of the smaller square = 12 m
Thus, the length of the larger square = x + 6
= 12 + 6
= 18 m

NCERT TEXTBOOK QUESTIONS SOLVED


EXERCISE 4.4
Q. 1. Find the nature of the roots of the following quadratic equations. If the real roots exist, find them:

(i) 2x2 3x + 5 = 0 (ii) 3x 2 4 3 x + 4 = 0


(iii) 2x2 6x + 3 = 0
Sol. (i) 2x2 3x + 5 = 0
Comparing the given quadratic equation with ax2 + bx + c = 0, we have:
a = 2
b = 3
c = 5
The discriminant = b2 4ac
= ( 3)2 4 (2) (5)
= 9 40
= 31 < 0
Since b2 4ac is negative.
The given quadratic equation has no real roots.
(ii) 3x2 4 3 x + 4 = 0
Comparing the given quadratic equation with ax2 + bx + c = 0, we get
a = 3
b = 4 3
c = 4
b2 4ac = 4 3
2
a fa f
4 3 4
= (16 3) 48
= 48 48 = 0
Quadratic Equations 33
Thus, the given quadratic equation has two real roots which are equal. Here, the
roots are:
b b
and
2a 2a

i.e.,
e
4 3 j and
e
4 3 j
23 23
4 3 4 3
and Q 3 = 3 3
2 3 3 2 3 3
2 2
and
3 3
2 2
Thus, x = and x =
3 3
2
(iii) 2x 6x + 3 = 0
Comparing it with the general quadratic equation, we have:
a = 2
b = 6
c = 3
b 4ac = ( 6)2 4 (2) (3)
2

= 36 24
= 12 > 0
The given quadratic equation has two real and distinct roots, which are given by

b b 2 4 ac
x =
2a

x =
b g
6 12
22

62 3
=
4
3 3
=
2
Thus, the roots are:
3+ 3 3 3
x = and x =
2 2
Q. 2. Find the values of k for each of the following quadratic equations, so that they have two equal roots:
(i) 2x2 + kx + 3 = 0 (ii) kx (x 2) + 6 = 0
Sol. (i) 2x2 + kx + 3 = 0
Comparing the given quadratic equation with ax2 + bx + c = 0, we get
a = 2
b = k
c = 3

34 MathematicsX
b2 4ac = ( k)2 4 (2) (3)
= k2 24
Q For a quadratic equation to have equal roots,
b2 4ac = 0
k2 24 = 0 k = 24
k = 2 6
Thus, the required values of k are
2 6 and 2 6
(ii) kx (x 2) + 6 = 0
Comparing kx (x 2) + 6 = 0 i.e., kx2 2kx + 6 = 0 with ax2 + bx + c = 0, we get
a = k
b = 2k
c = 6
b 4ac = ( 2k)2 4 (k) (6)
2

= 4k2 24k
Since, the roots are real and equal,
b2 4ac = 0
4k2 24k = 0
4k (k 6) = 0
4k = 0 or k 6 = 0
k = 0 or k = 6
But k cannot be 0, otherwise, the given equation is no more quadratic. Thus, the
required value of k = 6.
Q. 3. Is it possible to design a rectangular mango grove whose length is twice its breadth, and the area
is 800 m? If so, find its length and breadth.
Sol. Let the breadth be x metres.
Length = 2x metres
Now, Area = Length Breadth
= 2x x metre2
= 2x2 sq. metre.
According to the given condition,
2x 2 = 800
800
x2 = = 400
2
x = 400 = 20
Therefore, x = 20 and x = 20
But x = 20 is possible (Q breadth cannot be negative).
x = 20
2x = 2 20 = 40
Thus, length = 40 m and breadth = 20 m
Quadratic Equations 35
Q. 4. Is the following situation possible? If so, determine their present ages.
The sum of the ages of two friends is 20 years. Four years ago, the product of their ages in years
was 48.
Sol. Let the age of one friend = x years
The age of the other friend = (20 x) years [Q Sum of their ages is 20 years]
Four years ago
Age of one friend = (x 4) years
Age of the other friend = (20 x 4) years
= (16 x) years
According to the condition,
(x 4) (16 x) = 48
16x 64 x2 4x = 48
x2 20x 64 48 = 0
x2 20x 112 = 0
x2 + 20x + 112 = 0 ...(1)
Here, a = 1, b = 20 and c = 112
b2 4ac = (20)2 4 (1) (112)
= 400 448
= 48 < 0
Since b2 4ac is less than 0.
The quadratic equation (1) has no real roots.
Thus, the given equation is not possible.
Q. 5. Is it possible to design a rectangular park of perimeter 80 m and area 400 m2? If so, find its length
and breadth.
Sol. Let the breadth of the rectangle be x metres.
Since, the parimetre of the rectangle = 80 m.
2 [Length + Breadth] = 80
2 [Length + x] = 80
80
Length + x = = 40
2
Length = (40 x) metres
Area of the rectangle = Length breadth
= (40 x) x sq. m
= 40x x2
Now, according to the given condition,
Area of the rectangle = 400 m2
40x x2 = 400
2
x + 40x 400 = 0
x2 40x + 400 = 0 ...(1)

36 MathematicsX
Comparing (1) with ax2 + bx + c = 0, we get
a = 1
b = 40
c = 400
b2 4ac = ( 40)2 4 (1) (400)
= 1600 1600 = 0
Thus, the equation (1) has two equal and real roots.
b b
Q x = and x =
2a 2a

breadth =
b g
40
=
40
= 20
2 1 af 2
Breadth, x = 20 m
Length = (40 x) = (40 20) m = 20 m.

MORE QUESTIONS SOLVED

I. VERY SHORT ANSWER TYPE QUESTIONS


Q. 1. If the roots of the quadratic equation ax2 + bx + c = 0 are equal then show that b2 = 4ac.
Sol. For equal roots, we have
b2 4ac = 0
b2 = 4ac
Q. 2. Find the value of k for which the quadratic equation kx2 5x + k = 0 have real roots.
Sol. Comparing kx2 5x + k = 0 with ax2 + bx + c = 0, we have:
a = k
b = 5
c = k
b2 4ac = ( 5)2 4 (k) (k)
= 25 4k2
For equal roots, b2 4ac = 0
25 4k2 = 0
4k 2 = 25
25
k2 =
4

25 5
k = =
4 2
Q. 3. If 4 is a root of the quadratic equation x2 + px 4 = 0 and x2 + px + k = 0 has equal roots,
find the value of k.
Sol. Q (4) is a root of x2 + px 4 = 0

Quadratic Equations 37
( 4)2 + p ( 4) = 0
16 4p 4 = 0
4p = 12 or p = 3
Now, x2 + px + k = 0
x2 + 3x + k = 0 [ Q p = 3]
Now, a = 1, b = 3 and c = + k
b2 4ac = (3)2 4 (1) (k)
= 9 4k
For equal roots, b2 4ac = 0
9 4k = 0 4k = 9
9
k =
4
Q. 4. If one root of the quadratic equation 2x2 3x + p = 0 is 3, find the other root of the quadratic
equation. Also find the value of p.
Sol. We have:
2x2 3x + p = 0 ...(1)
a = 2, b = 3 and c = p
b
Since, the sum of the roots =
a

=
3b g =
3
2 2
Q One of the roots = 3

3 3
The other root = 3 =
2 2
Now, substituting x = 3 in (1), we get
2 (3)2 3 (3) + p = 0
18 9 + p = 0
9+p = 0 p=9
Q. 5. If one of the roots of x2 + px 4 = 0 is 4 then find the product of its roots and the value of p.
Sol. If 4 is a root of the quadratic equation,
x2 + px 4 = 0
2
( 4) + ( 4) (p) 4 = 0
16 4p 4 = 0
12 4p = 0 p = 3
c
Now, in ax2 + bx + c = 0, the product of the roots =
a
Product of the roots in x2 px 4 = 0
4
= = 4
1
38 MathematicsX
Q. 6. For what value of k, does the given equation have real and equal roots?
(k + 1) x2 2 (k 1) x + 1 = 0.
Sol. Comparing the given equation with ax2 + bx + c = 0, we have:
a = k+1
b = 2 (k 1)
c = 1
For equal roots, b2 4ac = 0
[ 2 (k 1)]2 4 (k + 1) (1) = 0
4 (k 1)2 4 (k + 1) = 0
4 (k2 + 1 2k) 4k 4 = 0
4k2 + 4 8k 4k 4 = 0
4k2 12k = 0
4k (k 3) = 0
k = 0 or k = 3
Q. 7. Using quadratic formula, solve the following quadratic equation for x:
x2 2ax + (a2 b2) = 0
Sol. Comparing x2 2ax + (a2 b2) = 0, with ax2 + bx + c = 0, we have:
a = 1, b = 2a, c = a2 b2

b b 2 4 ac
x =
2a

x =
b g b 2ag 4 a1f ea
2a
2 2
b2 j
2 a1f

2a 4 a 2 4 a 2 + 4b 2
=
2

2a 4b 2 2a 2b
= = =ab
2 2
x = (a + b) or x = (a b)
Q. 8. If one of the roots of the quadratic equation 2x2 + kx 6 = 0 is 2, find the value of k. Also find
the other root.
Sol. Given equation:
2x2 + kx 6 = 0
one root = 2
Substituting x = 2 in 2x2 + kx 6 = 0
We have:
2 (2)2 + k (2) 6 = 0
8 + 2k 6 = 0
2k + 2 = 0 k=1
2x2 + kx 6 = 0 2x2 x 6 = 0

Quadratic Equations 39
b 1
Sum of the roots = =
a 2
1
other root = 2
2
3
=
2
Q. 9. Determine the value of k for which the quadratic equation 4x2 4kx + 1 = 0 has equal roots.
Sol. We have:
4x2 4kx + 1 = 0
Comparing with ax2 + bx + c = 0, we have
a = 4, b = 4k and c = 1
b2 4ac = ( 4k)2 4 (4k) (1)
= 16k2 16
For equal roots
b2 4ac = 0
16k2 16 = 0
16k 2 = 16 k2 = 1
k = 1
Q. 10. For what value of k, does the quadratic equation x2 kx + 4 = 0 have equal roots?
Sol. Comparing x2 kx + 4 = 0 with ax2 + bx + c = 0, we get
a = 1
b = k
c = 4
b 4ac = ( k)2 4 (1) (4) = k2 16
2

For equal roots,


b2 4ac = 0
k2 16 = 0
k 2 = 16
k = 4
Q. 11. What is the nature of roots of the quadratic equation 4x2 12x + 9 = 0?
Sol. Comparing 4x2 12x + 9 = 0 with ax2 + bx + c = 0 we get
a = 4
b = 12
c = 9
b 4ac = ( 12)2 4 (4) (9)
2

= 144 144 = 0
Since 2
b 4ac = 0
The roots are real and equal.
Q. 12. Write the value of k for which the quadratic equation x2 kx + 9 = 0 has equal roots.
(AI CBSE 2009 C)

40 MathematicsX
Sol. Comparing x2 kx + 9 = 0 with ax2 + bx + c = 0, we get
a = 1
b = k
c = 9
b 4ac = ( k)2 4 (1) (9)
2

= k2 36
For equal roots,
b2 4ac = 0
k2 36 = 0 k2 = 36
k = 6
Q. 13. For what value of k are the roots of the quadratic equation 3x2 + 2kx + 27 = 0 real and equal?
(CBSE 2008 C)
2 2
Sol. Comparing 3x + 2 kx + 27 = 0 with ax + bx + c = 0, we have:
a = 3
b = 2k
c = 27
b2 4ac = (2k)2 4 (3) (27)
= 4k2 (12 27)
For the roots to be real and equal
b2 4ac = 0
4k2 (12 17) = 0
4k 2 = 12 27
12 27
k2 = = 81
4
k = 9
Q. 14. For what value of k are the roots of the quadratic equation kx2 + 4x + 1 = 0 equal and real?
(CBSE 2008 C)
2 2
Sol. Comparing kx + 4x + 1 = 0, with ax + bx + c = 0, we get
a = k
b = 4
c = 1
2
b 4ac = (4)2 4 (k) (1)
= 16 4k
For equal and real roots, we have
b2 4ac = 0
16 4k = 0
4k = 16
16
k = = 4
4
Q. 15. For what value of k does (k 12) x2 + 2 (k 12) x + 2 = 0 have equal roots?
(AI CBSE 2008 C)

Quadratic Equations 41
Sol. Comparing (k 12) x2 + 2 (k 12) x + 2 = 0 with ax2 + bx + c = 0, we have:
a = (k 12)
b = 2 (k 12)
c = 2
b2 4ac = [2 (k 12)]2 4 (k 12) (2)
= 4 (k 12)2 8 (k 12)
= 4 (k 12) [k 12 2]
= 4 (k 12) (k 14)
For equal roots,
b2 4ac = 0
4 (k 12) [k 14] = 0
Either 4 (k 12) = 0 k = 12
or k 14 = 0 k = 14
But k = 12 makes k 12 = 0 which is not required
k 12
k = 14
Q. 16. For what value of k does the equation 9x2 + 3kx + 4 = 0 has equal roots?
(AI CBSE 2008 C)
Sol. Comparing 9x2 + 3kx + 4 = 0 with ax2 + bx + c = 0, we get
a = 9
b = 3k
c = 4
b2 4ac = (3k)2 4 (9) (4)
= 9k2 144
For equal roots,
b2 4ac = 0
9k2 144 = 0
9k 2 = 144
144
k2 = = 16
9
k = 4

II. SHORT ANSWER TYPE QUESTIONS


Q. 1. Solve 2x2 5x + 3 = 0.
Sol. We have:
2x2 5x + 3 = 0 ...(1)
Comparing (1) with ax2 + bx + c = 0,
a = 2
b = 5
c = 3
b 4ac = ( 5)2 4 (2) (3)
2

= 25 24 = 1

42 MathematicsX
b b 2 4 ac
Since, x =
2a

x =
b g
5 1
2 a2f
51
=
4
Taking, + ve sign,
5+1 6 3
x = = =
4 4 2
Taking, ve sign,
51 4
x = = = 1
4 4
Thus, the required roots are
3
x = and x = 1
2
Q. 2. Solve the following quadratic equation:
2x2 + 4x 8 = 0
Sol. We have:
2x2 + 4x 8 = 0
Dividing by 2, we get
x2 + 2x 4 = 0 ...(1)
Comparing (1) with ax2 + bx + c = 0,
a = 1,
b = 2
c = 4
b 4ac = (2)2 4 (1) ( 4)
2

= 4 + 16 = 20

b b 2 4 ac
Since, x =
2a
2 20
x =
af
2 1

22 5
x =
2

2 1 5
x = = 1 5
2
Taking +ve sign, we get

x = e 1 + 5 j
Quadratic Equations 43
Taking ve sign we get,

x = e 1 5 j
e j
Thus, the required roots are x = 1 + 5 and x = 1 5 . e j
x+1 x2
Q. 3. Solve: + = 3. [CBSE 2012]
x1 x+2
Sol. We have,
x+1 x2
+ = 3
x1 x+2


bx + 2gbx + 1g + bx 2gbx 1g = 3
bx 1gbx + 2g

e x + 2 x + x + 2j + e x x 2 x + 2j
2 2

= 3
x2 x + 2x 2


ex 2
j e
+ 3x + 2 + x2 3x + 2 j = 3
2
x +x2
x2 + 3x + 2 + x2 3x + 2 = 3 (x2 + x 2)
2x2 + 4 = 3x2 + 3x 6
3x + 3x 6 2x2 4
2 = 0
x2 + 3x 10 = 0
x2 + 5x 2x 10 = 0
x (x + 5) 2 (x + 5) = 0
(x + 5) (x 2) = 0
Either x+5 = 0 x=5
or x2 = 0 x=2
Thus, the required roots are
x = 5 and x = 2
Q. 4. Solve (using quadratic formula):
x2 + 5x + 5 = 0
Sol. We have:
x2 + 5x + 5 = 0
Comparing (1) with ax2 + bx + c = 0, we have:
a = 1
b = 5
c = 5
b 4ac = (5)2 4 (1) (5)
2

= 25 20 = 5

44 MathematicsX
b b 2 4 ac
Since, x =
2a
5 5
x =
af
2 1
5 5
x =
2
Taking +ve sign, we have:
5+ 5
x =
2
Taking ve sign, we have:
5 5
x =
2
Thus, the required roots are:
5+ 5 5 5
x = and x =
2 2
Q. 5. Solve for x: 36x2 12ax + (a2 b2) = 0.
Sol. We have:
36x2 12ax + (a2 b2) = 0 ...(1)
Comparing (1) with Ax2 + Bx + C = 0, we have:
A = 36
B = 12a
C = (a2 b2)
B2 4AC = [ 12a]2 4 (36) [a2 b2]
= 144 a2 144 (a2 b2)
= 144 a2 144 a2 + 144 b2
= 144 b2
B B 2 4 AC
Since, x =
2A

x =
b g
12a 144 b 2
2 a 36f
12a 12b
x =
72
Taking +ve sign, we have:
12a + 12b
x =
72

x =
12
72
a+b =
1
6
a+b b g
Taking ve sign, we get
12a 12b
x =
72
Quadratic Equations 45
x =
12
72
b g
ab =
1
6
abb g
Thus, the required roots are:
1 1
x = (a + b) and x = (a b)
6 6
Q. 6. Find the roots of the quadratic equation 2 x 2 - 5x - 2 = 0, using the quadratic formula.
[NCERT Exemplar]
Sol. Comparing the given equation with the general equation ax2 + bx + c = 0, we have
a=2

b = - 5 b 2 - 4 ac = ( - 5)2 - 4 2 ( -2)
c = -2
= 5 + 16
= 21
Now, using the quadratic formula, we have:

- b b 2 - 4 ac
x =
2a
-( - 5) 21
x =
2( -2)
5 21
=
4
Taking the, positive sign, we get
5 + 21
x =
4
Taking the negative sign, we get
5 - 21
x =
4
5 + 21 5 - 21
Thus, x = and
4 4
Q. 7. Solve: 16x2 8a2 x + (a4 b4) = 0 for x.
Sol. We have:
16x2 8a2 x + a4 b4 = 0 ...(1)
Comparing (1) with ax2 + bx + c = 0, we get
a = 16
b = 8a2
c = (a4 b4)
b2 4ac = [ 8a2]2 4 (16) (a4 b4)
= 64 a4 64 (a4 b4)
= 64 a4 64 a4 + 64 b4
= 64 b4
46 MathematicsX
b b 2 4 ac
Since, x =
2a


e j
8a 2 64 b 4
x =
a f
2 16

8a 2 8b 2
x =
32

8 a2 b2 a2 b2
x = =
32 4
Now, taking +ve sign, we get
a2 + b2
x =
4
Taking ve sign, we get
a2 b2
x =
4
Thus, the required roots are:
a2 + b 2 a2 b 2
x = and x =
4 4
2 2 2
Q. 8. Solve for x: 9x 6ax + a b = 0.
Sol. We have:
9x2 6ax + (a2 b2) = 0 ...(1)
Comparing (1) with ax2 + bx + c = 0, we get
a = 9, b = 6a and c = (a2 b2)
b2 4ac = ( 6a)2 4 (9) (a2 b2)
= 36a2 36 (a2 b2)
= 36a2 36a2 + 36b2
= 36b2 = (6b)2

b b 2 4 ac
Since, x =
2a

b g a6bf
6a
2

2 a9f
x =

6a 6b
x =
18
6 ab ab
x = =
18 3
Taking the +ve sign, we get

Quadratic Equations 47
a+b
x =
3
Taking the ve sign, we get
ab
x =
3
The required roots are:
a+b ab
x = and x=
3 3
Q. 9. Evaluate 20 + 20 + 20 + ...

Sol. Let 20 + 20 + 20 ... = x


The given expression can be written as

x = 20 + ( )
20 + 20 ...

or x = 20 + x
squaring both side, we have

( )
2
x2 = 20 + x
x 2 = 20 + x
2
x2 x 20 = 0, where x = -b b - 4 ac
2a
Here : a = 1, b = 1 and c = 20
-( -1) ( -1)2 - 4 1 ( -20)
x =
2(1)

= 1 1 + 80 = 1 81 = 1 9
2 2 2
Since the given expression is positive,
Rejecting the negative sign, we have:
x = 1 + 9 = 10 = 5
2 2
Thus, 20 + 20 + 20 + ... = 5
Q. 10. Using quadratic formula, solve the following quadratic equation for x:
x2 4ax + 4a2 b2 = 0
Sol. Comparing the given equation with ax2 + bx + c = 0, we have:
a = 1
b = 4a
c = 4a2 b2
b 4ac = [ (4a)]2 4 (1) [4a2 b2]
2

= 16a2 4 (4a2 b2)


= 16a2 16a2 + 4b2
= 4b2 = (2b)2

48 MathematicsX
b b 2 4 ac
Since, x =
2a
b g a2bf
4a
2

2 a1f
x =

4 a 2b
x =
2
2
x = 2a b = 2a b
2
Taking the +ve sign, x = 2a + b
Taking the ve sign, x = 2a b
Thus, the required roots are:
x = 2a + b and x = 2a b
Q. 11. Using quadratic formula, solve the following quadratic equation for x:
x2 2ax + (a2 b2) = 0
Sol. Comparing the given equation with ax2 + bx + c = 0, we have:
a = 1, b = 2a and c = (a2 b2)
b 4ac = ( 2a)2 4 (1) (a2 b2)
2

= 4a2 4 (a2 b2)


= 4a2 4a2 + 4b2 = 4b2
b b 2 4 ac
Since, x =
2a


b g
2a 4b 2
2 a1f
x =

2a 2b
x =
2
2
x = ab = ab
2
Taking the +ve sign, we get
x = a+b
Taking the ve sign, we get
x = ab
Thus, the required roots are:
x = a + b and x = a b
Q. 12. Find the roots of the equation:
1 1 11 1 9
+ = ; x 3, , (CBSE 2009 C)
x + 3 2x 1 7x + 9 2 7
Sol. We have:
1 1 11
+ =
x + 3 2x 1 7x + 9
2x 1 + x + 3 11
=
b gb
x + 3 2x 1 g 7x + 9

Quadratic Equations 49
3x + 2 11
2 =
2x x + 6x 3 7x + 9


b3x + 2gb7 x + 9g = 11
2x2 + 5x 3
(3x + 2) (7x + 9) = 11 (2x2 + 5x 3)
2
21x + 27x + 14x + 18 = 22x2 + 55x 33
21x2 + 41x + 18 = 22x2 + 55x 33
2
(21 22) x + (41 55)x + (18 + 33) = 0
x2 + ( 14x) + (51) = 0
x2 + 14x 51 = 0
2
x + 17x 3x 51 = 0
x (x + 17) 3 (x + 17) = 0
(x + 17) (x 3) = 0
Either x + 17 = 0 x = 17
or x3 = 0 x=3
Thus, the required roots of the given equation are:
3 and 17
Q. 13. Find the roots of the equation:
1 1 4
= ; x 0, 3 (CBSE 2009 C)
x x3 3
Sol. We have:
1 1 4
=
x x3 3


b x 3g x =
4
x b x 3g 3
33 = 4 (x2 3x)
9 = 4x2 12x
4x2 12x + 9 = 0 ...(1)
Comparing (1), with ax2 + bx + c = 0, we get
a = 4
b = 12
c = 9
b2 4ac = ( 12)2 4 (4) (9)
= 144 144 = 0
Now, the roots are:
b b 2 4 ac
x =
2a


b g
12 0
2 a4 f
x =

12 3
x = =
8 2
3 3
Thus, the roots are: and
2 2
50 MathematicsX
Q. 14. Find the roots of the equation:
1 1 8 5
+ = ; x 0, 2, (CBSE 2008 C)
x2 x 2x + 5 2
Sol. We have:
1 1 8
+ =
x2 x 2x + 5
x+x2 8
=
b
x x2 g
2x + 5
2x 2 8
2 =
x 2x 2x + 5
(2x 2) (2x + 5) = 8 (x2 2x)
4x2 + 10x 4x 10 = 8x2 16x
4x2 + 22x 10 = 0
2x2 11x + 5 = 0 ...(1)
Comparing (1) with ax2 + bx + c = 0, we have:
a = 2
b = 11
c = 5
b 4ac = ( 11)2 4 (2) (5)
2

= 121 40 = 81
Now, the roots are given by
b b 2 4 ac
x =
2a

x =
b g
11 81
2 a2f
11 9
=
4
Taking the +ve sign,
11 + 9 20
x = = = 5
4 4
Taking the ve sign,
11 9 2 1
x = = =
4 4 2
1
Thus, the required roots are: 5 and .
2
Q. 15. Find the roots of the following equation:
1 1 11
= ; x = 4, 7 (CBSE 2012)
x+4 x 7 30
Sol. We have:
1 1 11
=
x+4 x7 30

Quadratic Equations 51

bx 7g bx + 4g 11
bx + 4gbx 7g =
30
x7x4 11
2 =
x 3 x 28 30
11 30 = 11 (x2 3x 28)
30 = x2 3x 28
2
x 3x 28 + 30 = 0
x2 3x + 2 = 0
2
x 2x x + 2 = 0
x (x 2) 1 (x 2) = 0
(x 1) (x 2) = 0
Either x1 = 0 x=1
or x2 = 0 x=2
Thus, the required roots are: 1 and 2.
2a 2b
Q. 16. If and are roots of the equation x2 1 = 0, form an equation whose roots areand .
b a
Sol. Q and are roots of x2 1 = 0 and x2 1 = 0 can be written as x2 + 0x 1= 0 where
a = 1, b = 0 and c = 1.
-b 0
Sum of roots = =0
=
a 1
+ = 0
c -1
Also, product of roots == = 1
a 1
= 1

Now, the roots of the new equation are 2a and 2b


b a
2 a 2b
Sum of the roots of the new equation = +
b a
2 2 2 2
= 2a + 2b = 2(a + b )
ab ab
2
= 2[(a + b ) - 2ab ]
ab
(0)2 - 2( -1) Q a+b = 0
= 2
( -1) and ab = ( -1)
0 + 2
= 2
( -1)
= 2 (2) = 4
2 a 2b ab
Product of the roots of the new equation = =4 =4
b a ab
52 MathematicsX
Since, a quadratic equation is given by
Sum of the Product
x2 - x+ =0
roots of the roots
The required quadratic equation is
x2 (4) x + 4 = 0
or x2 + 4x + 4 = 0
Remember
2 + 2 + 2 = ( + )2
2 + 2 = ( + )2 2

III. LONG ANSWER TYPE QUESTIONS


Q. 1. A two-digit number is 5 times the sum of its digits and is also equal to 5 more than twice the
product of its digits. Find the number.
Sol. Let the tens digit = x
And the ones digit = y
The number = 10x + y
According to the conditions,
10x + y = 5 (x + y) ...(1)
10x + y = 2xy + 5 ...(2)
From (1), we have
10x + y = 5x + 5y
10x + y 5x 5y = 0
5x 4y = 0
5
5x = 4y or y = x
4
5
Substituting y = x in (2), we get
4
5x 5x 2
10 x + = +5
4 2
40x + 5x = 10x2 + 20 [Multiplying both sides by 4]
45x = 10x2 + 20
10x2 45x 20 = 0
2x2 9x 4 = 0
2x2 8x x + 4 = 0
2x (x 4) 1 (x 4) = 0
(x 4) (2x 1) = 0
Either x4 = 0 x=4
1
or 2x 1 = 0 x=
2
But a digit cannot be a fraction,
x = 4 The tens digit = 4

Quadratic Equations 53
54
Now, the ones digit y = = 5
4
x = 4 and y = 5
The required number = 10 4 + 5
= 40 + 5 = 45
Q. 2. The denominator is one more than twice the numerator. If the sum of the fraction and its reciprocal
16
is 2 , find the fraction.
21
Sol. Let the numerator be x
Denominator = (2x + 1)
x
Fraction =
2x + 1

2x + 1
Reciprocal of the fraction =
x
According to the condition,
x 2x + 1 16
+ = 2
2x + 1 x 21


b gb g
x x + 2x + 1 2x + 1 58
x b2 x + 1g
=
21

x + b2 x + 1g
2 2
58
=
2x2 + x 21
x2 + 4 x2 + 4 x + 1 58
=
2x 2 + x 21
5x 2 + 4 x + 1 58
2 =
2x + x 21
21 (5x2 + 4x + 1) = 58 (2x2 + x)
105x2 + 84x + 21 = 116x2 + 58x
105x2 116x2 + 84x 58x + 21 = 0
11x2 + 26x + 21 = 0
11x2 26x 21 = 0 ...(1)
Comparing (1) with ax2 + bx + c = 0, we have:
a = 11
b = 26
c = 21
b2 4ac = ( 26)2 4 11 ( 21)
= 676 + 924 = 1600

b b 2 4 ac
Since, x =
2a
54 MathematicsX
b g
26 1600
2 a11f
x =

26 40
x =
22
Taking the +ve sign,
26 + 40 66
x = = = 3
22 22
Taking the ve sign,
26 40 14 7
x = = =
22 22 11
7
But the numerator cannot be
11
x = 3 The numerator = 3
Denominator = 2 (3) + 1 = 7
3
Thus, the required fraction =
7
10
Q. 3. The sum of a number and its reciprocal is . Find the number.
3
Sol. Let the required number = x
1
Its reciprocal =
x
According to the condition, we have:

The number +
LMReciprocal of OP =
10
Nthe number Q 3
1 10
x+ =
x 3
x2 + 1 10
=
x 3
3 (x2 + 1) = 10x
3x2 + 3 10x = 0
3x2 10x + 3 = 0
3x2 9x x + 3 = 0
3x (x 3) 1 (x 3) = 0
(x 3) (3x 1) = 0
Either x3 = 0 x=3
1
or 3x 1 = 0 x=
3
1
Thus, the required number is 3 or .
3

Quadratic Equations 55
Q. 4. The hypotenuse of a right triangle is 3 10 cm. If the smaller side is tripled and the longer side
doubled, new hypotenuse will be 9 5 cm. How long are the sides of the triangle?
Sol. Let the smaller side = x

Longer side = bHypotenuseg bsmaller sideg


2 2

= e3 10 j x2 2

= 9 10 x 2

= 90 x 2
According to the condition, we have
[3 (Smaller side)]2 + [2 (Longer side)]2 = [New Hypotenuse]2

a f + LMN2 e90 x j OPQ


2
2 2 2
3 x = 9 5
9x2 + 4 (90 x2) = 81 5
9x2 + 360 4x2 = 405
5x 2 = 405 360
5x 2 = 45
45
x2 = = 9
5
x = 9 = 3
But x = 3 is not required, because the side of a triangle cannot be negative.
x = 3 Smaller side = 3 cm
Longer side = 90 3 2
= 90 9
= 81 = 9 cm
Thus, the required sides of the triangle are 3 cm and 9 cm.
Q. 5. A motor-boat goes 10 km upstream and returns back to the starting point in 55 minutes. If the
speed of the motor boat in still water is 22 km/hr, find the speed of the current.
Sol. Let the speed of the current = x km/hr
The speed downstream = (22 + x) km/hr
The speed upstream = (22 x) km/hr
Distance
Since, Time =
Speed
F 10 I hrs
Time for going 10 km downstream = GH 22 + x JK
Time for returning back 10 km upstream = G
F 10 I hours
H 22 x JK
According to the condition,

56 MathematicsX
10 10 55
+ =
22 + x 22 x 60

10
LM 1 + 1 OP =
11
[Q 55 minutes =
55
hours]
N 22 + x 22 x Q 12 60
L 22 + x + 22 x OP
10 12 M

NM b22 + xgb22 xg QP = 11

120 M
L 44 OP = 11
MN 484 x PQ
2

11 (484 x2) = 120 44


5324 11x2 = 5280
11x 2 = 5324 5280 = 44
44
x2 = = 4
11
x = 2
But speed of the current cannot be negative,
x = 2
Speed of current = 2 km/hr
Q. 6. A motor boat whose speed in still water is 5 km/hr, takes 1 hour more to go 12 km upstream than
to return downstream to the same spot. Find the speed of the stream. (AI CBSE 2009 C)
Sol. Let the speed of the stream be x km/hr
Downstream speed of the motor boat = (x + 5) km/hr
12
Time taken to go 12 km upstream = hours
5x
12
Time taken to return 12 km downstream = hours
5+x
According to the condition
12 12
= 1
5x 5+x
12 (5 + x) 12 (5 x) = 1 (5 x) (5 + x)
60 + 12x 60 + 12x = 25 x2
24x = 25 x2
2
x + 24x 25 = 0
2
x + 25x x 25 = 0
x (x + 25) 1 (x + 25) = 0
(x 1) (x 25) = 0
Either x 1 = 0 x = 1
or x + 25 = 0 x = 25
But x = 25 is not admissible, because the speed of the stream cannot be negative.
x = 1
speed of the stream = 1 km/hr.

Quadratic Equations 57
Q. 7. Sum of the areas of two squares is 260 m2. If the difference of their perimeters is 24 m, then find
the sides of the two squares. (AI CBSE 2009 C)
Sol. Let the side of one of the squares be x metres
Perimeter of square-I = 4 x metres = 4x metres
Perimeter of square-II = (24 + 4x) metres
1
Side of the square-II = (24 + 4x) metres = (6 + x) metres
4
Now, according to the condition, we have:
x2 + (6 + x)2 = 260
x2 + 36 + x2 + 12x 260 = 0
2x2 + 12x 224 = 0
x2 + 6x 112 = 0 ...(1)
Comparing (1) with ax2 + bx + c = 0, we get,
a = 1
b = 6
c = 112
b2 4ac = (6)2 4 (1) ( 112)
= 36 + 448 = 484

b b 2 4 ac
x =
2a
6 484
x =
af
2 1
6 22
=
2
Taking +ve sign, we have
6 + 22 16
x = = = 8
2 2
Taking ve sign, we have
6 22 28
x = = = 14
2 2
But x = 14 is not required, as the length of a side cannot be negative.
x = 8
Side of square-I = 8 m
Side of square-II = 6 + 8 m = 14 m.
Q. 8. The age of a father is twice the square of the age his son. Eight years hence, the age of the father
will be 4 years more than three times the age of his son. Find their present ages.
(AI CBSE 2009 C)
Sol. Let the present of son be x years.
Fathers present age = 2x2 years
8 years hence:
Age of son = (x + 8) years
Age of father = (2x2 + 8) years

58 MathematicsX
According to the condition:
(2x2 + 8) = 3 (x + 8) + 4
2x2 + 8 3x 24 4 = 0
2x2 3x + 8 28 = 0
2x2 3x 20 = 0 ...(1)
Comparing (1) with ax2 + bx + c = 0, we get
a = 2
b = 3
c = 20
b2 4ac = ( 3)2 4 (2) ( 20)
= 9 + 160 = 169
b b 2 4 ac
Now, x =
2a
b g
3 169 3 13
x =
af
2 2
=
4
Taking +ve sign,
3 + 13 16
x = = = 4
4 4
Taking ve sign,
3 13 10 5
x = = =
4 4 2
5
But x = is not required, as the age cannot be negative.
2
x = 4
Present age of son = 4 years
Present age of father = 2 42 = 32 years.
Q. 9. The diagonal of a rectangular field is 60 m more than the shorter side. If the longer side is 30
metres more than the shorter side, find the sides of the field. (CBSE 2009 C)
Sol. See Q-6 of Textbook Exercise 4.3.
Q. 10. A motor boat whose speed in still water is 16 km/h, takes 2 hours more to go 60 km upstream
than to return to the same spot. Find the speed of the stream. (CBSE 2009 C)
Sol. Let the speed of the stream = x km/hr
For the motor boat, we have:
Downstream speed = (16 + x) km/hr
Upstream speed = (16 x) km/hr
For going 60 km:
60
Downstream = hours
16 + x
60
Upstream = hours
16 x
According to the condition:
60 60
= 2
16 x 16 + x
Quadratic Equations 59
60 (16 + x) 60 (16 x) = 2 (16 x) (16 + x)
960 + 60x 960 + 60x = 2 (256 x2)
2x2 + 120x = 2 256 2x2
x2 + 60x = 256
x2 + 60x 250 = 0 ...(1)
Comparing (1) with ax2 + bx + c = 0,
a = 1
b = 60
c = 256
b2 4ac = (60)2 4 (1) ( 256)
= 3600 + 1024 = 4624
b b 2 4 ac
x =
2a
60 4624
x =
2 1 af
60 68
x =
2
Taking +ve sign,
60 + 68 8
x = = = 4
2 2
Taking ve sign,
60 68 128
x = = = 64
2 2
Since, the speed of a stream cannot be negative,
x = 64 is not admissible
x = 4
speed of the stream = 4 km/hr.
Q. 11. A train travels 288 km at a uniform speed. If the speed had been 4 km/hr more, it would have
taken 1 hour less for the same journey. Find the speed of the train. (CBSE 2009 C)
Sol. Let the speed of the train be x km/hr
Total distance travelled = 288 km
288
Time taken = hours
x
In the other case,
Speed of the train = (x + 4) km/hr
288
Time taken = hours
x+4
According to the condition,
288 288
= 1
x x+4


b g
288 x + 4 288 x
b
x x+4 g = 1

60 MathematicsX
288x + 1152 288x = 1 (x) (x + 4)
288x + 1152 288x = x2 + 4x
1152 = x2 + 4x
2
x + 4x 1152 = 0 ...(1)
Comparing (1) with ax2 + bx + c = 0,
a = 1
b = 4
c = 1152
b2 4ac = (4)2 4 (1) ( 1152)
= 16 + 4608

b b 2 4 ac
x =
2a


af
4 4608
=
4 68
x =
2 1af 2
Taking +ve sign,
4 + 68 64
x = = = 32
2 2
Taking ve sign,
4 68 72
x = = = 36
2 2
Q speed cannot be negative,
x 36
x = 32
speed of the train = 32 km/hr.
Q. 12. A train travels at a certain average speed for a distance of 63 km and then travels a distance of
72 km at an average speed of 6 km/hr more than its original speed of. If takes 3 hours to complete
the total journey, what is its original average speed? [NCERT Exemplar]
Sol. Let the original average speed = x km/hr
72
Time taken to cover 72 km = hours
x+6
63
Time taken to cover 63 km = hours
x
Since, total time = 3 hours
72 63
+ = 3
x+6 x
1 72 63 Q HCF of 72
+ = 1 3
9 x + 6 x 9 and 63 is 9
8 7 1
+ =
x+6 x 3
8x + 7( x + 6) 1
=
x( x + 6) 3
Quadratic Equations 61
x 2 + 6x
8x + 7x + 42 =
3
x 2 + 6x
15x + 42 =
3
3[15x + 42] = x2 + 6x
45x + 126 x2 6x = 0
x2 39x 126 = 0
2
x 42x + 3x 126 = 0
x(x 42) + 3(x 42) = 0
(x + 3) (x 42) = 0
Either x+3 = 0 x =3
or x 42 = 0 x = 42
Since, speed cannot be negative,
x = 3 is not desired.
Thus, the original speed of the train is 42 km/hr.
Q. 13. If 5 is a root of the quadratic equation 2x2 + px 15 = 0 and the quadratic equation
p (x2 + x) + k = 0 has equal roots, then find the values of p and k. (AI CBSE 2009)
Sol. Since 5 is a root of 2x2 + px 15 = 0,
Substituting x = 5 in the given equation, we get
2 ( 5)2 + p ( 5) 15 = 0
2 (25) + ( 5p) 15 = 0
50 5p 15 = 0
5p + 35 = 0
5p = 35
35
p = = 7
5
Now, comparing the another quadratic equation p (x2 + x) + k = 0, i.e., px2 + px + k = 0
with ax2 + bx + c = 0, we have:
a = p
b = p
c = k
b2 4ac = p2 4 (p) (k)
= p2 4pk
2
Since p (x x) + k = 0 has equal roots,
p2 4pk = 0
2
(7) 4 (7) k = 0 |Q p = 7
49 28k = 0
49 7
k = =
28 4
Thus, the required values of
7
p = 7 and k = .
4

62 MathematicsX
Q. 14. In a class test, the sum of Gagans marks in Mathematics and English is 45. If he had 1 more
mark in Mathematics and 1 less in English, the product of marks would have been 500. Find the
original marks obtained by Gagan in Mathematics and English separately. (AI CBSE 2008 C)
Sol. Let Gagans marks in maths = x
and Marks in English = (45 x)
According to the condition,
(x + 1) (45 x + 1) = 500
(x + 1) (44 x) = 500
44x x2 + 44 x = 500
x2 + 44x 456 x = 0
x2 43x + 456 = 0
x2 19x 24x + 456 = 0 Q 24 19 = 45
x (x 19) 24 (x 19) = 0 43 = ( 24) + ( 19)
(x 19) (x 24) = 0
Either x 19 = 0 x = 19
or x 24 = 0 x = 24
When x = 19, then 45 19 = 26
When x = 24, then 45 24 = 21
Gagans marks in Maths = 19 and in English = 26
Or
Gagans marks in Maths = 24 and in English = 21.
Q. 15. The sum of areas of two squares is 640 m2. If the difference of their perimeters is 64 m, find the
sides of two squares. (CBSE 2008 C)
Sol. Let the side of square I be x metres.
Perimeter of square I = 4x metres
Perimeter of square II = (64 + 4x) m
1
Side of square II = (64 + 4x) m
4
= (16 + x) m
Now Area of square I = x x = x2
Area of square II = (16 + x) (16 + x) = (16 + x)2
= 256 + x2 + 32x
According to the condition,
LMArea of OP + LMArea of theOP = 640
Nsquare IQ Nsquare II Q
x2 + [256 + x2 + 32x] = 640
x2 + x2 + 32x + 256 640 = 0
2x2 + 32x 384 = 0
x2 + 16x 192 = 0
x2 + 24x 8x 192 = 0 Q 24 8 = 16
x (x + 24) 8 (x + 24) = 0 and 24 8 = 192

Quadratic Equations 63
(x + 24) (x 8) = 0
Either x + 24 = 0 x = 24
or x8 = 0 x=+8
Q side of a square cannot be negative,
Rejecting x = 24, we have x = 8
Side of smaller square = 8 m
Side of larger square = 8 + 16 m = 24 m.
Q. 16. In a class test, the sum of Kamals marks in Mathematics and English is 40. Had he got 3 marks
more in Mathematics and 4 marks less in English, the product of his marks would have been 360.
Find his marks in two subjects separately. (CBSE 2012)
Sol. Let Kamals marks in Maths = x
His marks in English = (40 x)
According to the condition,
(x + 3) [40 x 4] = 360
(x + 3) (36 x) = 360
36x x2 + 108 3x 360 = 0
x2 + 33x 252 = 0
x2 33x + 252 = 0
2 Q ( 21) ( 12) = 252
x 21x 12x + 252 = 0
( 21) + ( 12) = 33
x (x 21) 12 (x 21) = 0
(x 21) (x 12) = 0
Either (x 21) = 0 x = 21
or (x 12) = 0 x = 12
For x = 21, Marks of Kamal
in Maths = 21
in English = 40 21 = 19
For x = 12, Marks of Kamal
in Maths = 12
in English = 40 12 = 28.
Q. 17. Find the value of p for which the quadratic equation 4x2 + px + 3 = 0 has equal roots.
(Al CBSE 2014)

Hint:
For equal roots, b2 4ac = 0
p2 4(4) (3) = 0 or p2 48 = 0 p = 48 = 4 3

Q. 18. Solve the quadratic equation 2x2 + ax a2 = 0 [Al CBSE (Delhi) 2014]
Sol. 2x2 + ax a2 = 0
2x + 2ax ax a2 = 0
2
splitting ax into
2x[x + a] a[x + a] = 0
2ax and ax
(x + a) (2x a) = 0
a
x = a or x =
2

64 MathematicsX
IV. HOTS QUESTIONS
Q. 1. Had Ravita scored 10 more marks in her Mathematics test out of 30 marks, 9 times these marks
would have been the square of her actual marks. How many marks did she get in the test?
[NCERT Exemplar]
Sol. Let actual marks be x
9 [ Actual marks + 10] = [Square of actual marks]
or 9 (x + 10) = x 2
9x + 90 = x 2
x2 9x 90 = 0
2
x 15x + 6x 90 = 0
x(x 15) + 6(x 15) = 0
(x + 6) (x 15) = 0
Either x+6 = 0 x =6
or x 15 = 0 x = 15
But marks cannot be less than 0.
x = 6 is not desired.
Thus, Ravita got 15 marks in her Mathematics test.
Q. 2. A motor boat whose speed is 18 km/h in still water takes 1 hour more to go 24 km upstream than
to return downstream to the same spot. Find the speed of the stream. (AI CBSE 2008)
Sol. Let the speed of the stream = x km/hr
speed of the motor boat:
upstream = (18 x) km/hr
downstream = (18 + x) km/hr
Time taken by the motor boat in going:
24 Distance
24 km downstream = hours Q Time = Speed
18 + x
20
24 km upstream = hours
18 x
According to the condition:
24 24
= 1
18 x 18 + x
24 (18 + x) 2 (18 x) = 1 (18 x) (18 + x)
24 [18 + x 18 + x] = 182 x2
24 [2x] = 324 x2
48x = 324 x2
2
x + 48x 324 = 0
x2 + 54x 6x 324 = 0
x (x + 54) 6 (x + 54) = 0
(x 6) (x + 54) = 0
Either x6 = 0 x=6
or x + 54 = 0 x = 54
But speed cannot be negative
Rejecting x = 54, we have
x = 6 Speed of the boat = 6 km/hr.

Quadratic Equations 65
Q. 3. In a class test, the sum of marks obtained by P in Mathematics and Science is 28. Had he got 3 more
marks in Mathematics and 4 marks less in Science, the product of marks obtained in the two
subjects would have been 180? Find the marks obtained in two subjects separately. (CBSE 2008)
Sol. Let marks obtained by P in Maths be x.
His marks in Science = (28 x)
According to the condition,
(x + 3) (28 x 4) = 180
(x + 3) ( x + 24) = 180
24x x2 + 72 3x = 180
x2 + 21x + 72 180 = 0
x2 + 21x 108 = 0
x2 21x + 108 = 0
2
x 12x 9x + 108 = 0
x (x 12x) 9(x 12) = 0
(x 9) (x 12) = 0
(x 9) (x 12) = 0
Either x9 = 0 x=9
or x 12 = 0 x = 12
When x = 9 then 28 x = 28 9 = 19
When x = 12 then 28 x = 28 12 = 16
Thus Ps marks in Maths = 9 and Science = 19
Or
Ps marks in Maths = 12 and Science = 16.
Q. 4. Solve for x:
3x 2 - 2 2x - 2 3 = 0 [CBSE (Foreign) 2014]

Hint:
3 x 2 2 2 x 2 3 = 0 , we have

a = 3 , b = ( -2 2 ) and c = ( -2 3 )
2
b2 4ac = ( -2 2 ) - 4 ( 3 )( -2 3 ) = 32

b b 2 - 4ac
Using Quadratic formula, x =
2a
we get, x = 6
Q. 5. Solve for x :
x2 + 5 5 x 70 = 0

Hint:
a = 1, b = 5 5 and c = 70

(5 5)
2
b2 4ac = 4 (1) (70) = 9 5
Now use quad. formula to get x = 2 5 , 7 x

66 MathematicsX
Q. 6. At t minutes past 2 pm, the time needed by the minute hand of a clock to show 3 pm was found to
t2
be 3 minutes less than minutes. Find t.
4
Sol. For a minute-hand time needed to show 2 pm to 3 pm is 60 minutes.
It has already covered t minutes.
Time required by the minute-hand to reach to 12 (at 3 pm) = (60 t) minutes.
t2
3 =(60 t)
4
t2
+ t 63 = 0
4
t2+ 4t 252 = 0
Solving, we get, t = 14 or 18
But t = 18 is not desirable (being negative)
Thus, t = 14 minutes.
Q. 7. A train, travelling at a uniform speed for 360 km, would have taken 48 minutes less to travel the
same distance if its speed were 5 km/hr more. Find the original speed of the train.
[NCERT Exemplar]
Sol. Let the original speed be x km/hr
360
Original time taken = hours
x
New speed = (x + 5) km/hr
360
New time = km / hr
x+5
According to the condition,
360 360 48
= +
x x + 5 60
x +5 x 4
360 =
x( x + 5) 5
5 4
360 2 = x2 + 5x 2250 = 0
x + 5x 5
Solving for x, we get x = 50 or 45
Speed cannot be negative
Rejecting x = 50, we have x = 45
Thus, the original speed of the train = 45 km/hr.
Q. 8. If the roots of the equation (b - c)x2 + (c a)x + (a b) = 0 are equal, then prove that 2b = a + c
Hint: For equal roots D =0
Here, D = (c a)2 4(b c) ( a b)
(c a)2 4(b c)(a b) =0
c2 + a2 + 4b2 2ac 4ab + 4ac 4bc = 0
(c + a 2b)2 =0
(c + a 2b) =0 c + a = 2b (Hence Proved)

Quadratic Equations 67
Q. 9. If the roots of the equations

ax2 + 2bx + c = 0 and bx2 2 ac x + b = 0


are simultanously real then prove that b2 = 4ac
Hint: Let D1 and D2 be the discriminants
D1 0 and D2 b2
4b2 4ac 0 and 4ac 4b2 0
b2 ac and ac b2
b2 = ac
Q. 10. If the roots of the equation
(c2 ab)x2 2(a2 bc)x + b2 ac = 0 are equal, then prove that
either a = 0 or a3 + b3 + c3 = 3abc
Hint: D = b2 4ac
= [2(a2 bc)]2 4 (a2 ab) (b2 ac)
= 4a (a3 + b3 + c3 3abc)
For equal roots, D= 0
4a (a3 + b3 + c3 3abc) = 0
a = 0 or a3 + b3 + c3 = 3abc

TEST YOUR SKILLS


1. A natural number, when increased by 12, equals 160 times its reciprocal. Find the number.
[NCERT Exemplar]
Hint: Let the natural number be x
160
x + 12 =
x
x(x + 12) = 160
x2 + 12x 160 = 0

2. By increasing the list price of a book by ` 10, a person can buy 10 books less for
` 1200. Find the original list price of the book. (CBSE 2007)
3. The hypotenuse of a right-angled triangle is 1 cm more than twice the shortest side. If the
third side is 2 cm less than the hypotenuse, find the sides of the triangle. (CBSE 2007)
4. A passenger train takes 2 hours less for a journey of 300 km, if its speed is increased by
5 km/hr from its usual speed. Find its usual speed. (CBSE 2006, 2007)
5. The numerator of a fraction is one less than its denominator. If three is added to each of
3
the numerator and denominator, the fraction is increased by . Find the fraction.
28
(AI CBSE 2007)
6. The difference of squares of two natural numbers is 45. The square of the smaller number
is four times the larger number. Find the numbers. (AI CBSE 2007)
7. Solve: x 2 + 5 5x - 70 [NCERT Exemplar]

68 MathematicsX
8. A train travels a distance of 300 km at a uniform speed. If the speed of the train is
increased by 5 km an hour, the journey would have taken two hours less. Find the original
speed of the train. (CBSE 2006)
9. The speed of a boat in still water is 11 km/hr. It can go 12 km upstream and returns
downstream to the original point in 2 hours 45 minutes. Find the speed of the stream.
(AI CBSE 2006)
10. Determine the value of k for which the quadratic equation 4x2 3kx + 1 = 0 has equal
roots. (CBSE 2006 C)
11. Using quadratic formula, solve the following equation for x:
ab x2 + (b2 ac) x bc = 0 (AI CBSE 2006 C)
12. The sum of the numerator and the denominator of a fraction is 12. If the denominator is
1
increased by 3, the fraction becomes . Find the fraction. (AI CBSE 2006 C)
2
13. Rewrite the following as a quadratic equation in x and then solve for x:
4 5 3
3 = ; x 0, (AI CBSE 2006 C)
x 2x + 3 2
14. A two digit number is such that the product of its digits is 18. When 63 is subtracted from
the number, the digits interchange their places. Find the number. (AI CBSE 2006 C)
15. A train covers a distance of 90 km at a uniform speed. Had the speed been 15 km/hr more,
it would have taken 30 minutes less for the journey. Find the original speed of the train.
(AI CBSE 2006 C)
16. Solve for x:
x+1 x2
+ = 3; (x 1, 2) (CBSE 2012)
x1 x+2
17. Using quadratic formula, solve the following for x:
9x2 3 (a2 + b2) x + a2 b2 = 0 (CBSE 2012)
18. Find the equation whose roots are reciprocal of the roots of
3x2 5x + 7 = 0
19. A number consists of two digits whose product is 18. When 27 is subtracted from the
number, the digits change their places. Find the number. (AI CBSE 2005 C)
20. A number consisting of two digits is seven times the sum of its digits. When 27 is
subtracted from the number, the digits are reversed. Find the number. (AI CBSE 2005 C)
21. The sum of the squares of two consecutive odd numbers is 394. Find the nimbers.
[CBSE (Foreign) 2014]
Hint:
Let the two consecutive odd numbers be x and x + 2.
x2 + (x + 2)2 = 394
x + x2 + 4x + 4 = 394
2

2x2 + 4x 390 = 0
x2 + 2x 195 = 0
x2 + 15x 13x 195 = 0 or x(x + 15) 13(x + 15) = 0
x = 13 or x = 15
For x = 13, x + 2 = 13 + 2 = 15
Thus, the required numbers are 13 and 15.
Quadratic Equations 69
22. An aeroplane left 30 minutes later than its scheduled time and in order to reach its
destination 1500 km away in time, it has to increase its speed by 250 km/hr from its usual
speed. Determine its usual speed. (AI CBSE 2005 C)
23. Using quadratic formula, solve for x:
9x2 3 (a + b) x + ab = 0 (CBSE 2012)
24. Find the number which exceeds its positive square root by 20.
1
25. The sum of two numbers is 15 and the sum of their reciprocals is . Find the numbers.
3
(CBSE 2005)
26. A two digit number is such that the product of its digits is 14. If 45 is added to the number,
the digits interchange their places. Find the number. (AI CBSE 2005)
27. A two digit number is such that the product of its digits is 20. If 9 is added to the number,
the digits interchange their places. Find the number. (AI CBSE 2005)
28. A two digit number is such that the product of its digits is 15. If 8 is added to the number,
the digits interchange their places. Find the number. (AI CBSE 2005)
29. Solve for x:
x1 x3 1
+ = 3 ; (x 2, 4) (AI CBSE 2005)
x2 x4 3
30. Solve for x:
ab x2 + (b2 ac) x bc = 0 (AI CBSE 2005)
1
31. The sum of two numbers is 18. The sum of their reciprocals is . Find the numbers.
4
(AI CBSE 2005)
1
32. The sum of two numbers is 16, and the sum of their reciprocals is . Find the numbers.
3
(AI CBSE 2005)
1 1 3
33. The sum of two numbers a and b is 15, and sum of their reciprocals and is .
a b 10
Find the numbers a and b. (CBSE 2005)
34. Solve for x:
1 1 1 1
= + + ; a 0, b 0, x 0 (CBSE 2012)
a+b+x a b x
35. Find the roots of the following quadratic equation:
2 2 3
x x = 0 (CBSE Sample Paper 2011)
5 5
Hint:

2 2 3
x x = 0 2x2 5x 3 = 0
5 5
(2x + 1) (x 3) = 0
1
x = and x = 3
2

70 MathematicsX
36. Find the roots of the equation:

1 1 3
+ = 1; x ,5 (CBSE Sample Paper 2011)
2x 3 x 5 2
37. A natural number when subtracted from 28, becomes equal to 160 times its reciprocal.
Find the number. (CBSE Sample Paper 2011)
38. Find two consecutive odd positive integers, sum of whose squares is 290.
(CBSE Sample Paper 2011)
39. Find the values of k for which the quadratic equation (CBSE Delhi 2014)
(k + 4) x2 + (k + 1) x + 1 = 0 has equal roots. Also find these roots.

Hint:
From (k + 4)x2 + (k + 1)x + 1 = 0, we get
a = (k + 4), b = (k + 1) and c = 1
b2 4ac = (k + 1)2 4 (k + 4) (1)
= k2 2k 15
For equal roots, b2 4ac = 0
k2 2k 15 = 0
k = 5 or k = 3
For k = 5, we have (k + 4)x2 + (k + 1) x + 1 = 0
9x2 + 6x + 1 = 0

1 1
Solving for x, we get x = ,
3 3
For k = 3, we have
x2 2x + 1 = 0
solving it, we get x = 1, x = 1

40. Solve for x :

16 15
1 = ; x 0, 1. (AI CBSE 2014)
x x+1

Hint:

16 15 16 15
1 = =1
x x+1 x x+1
or 16(x + 1) 15(x) = x(x + 1) x2 = 16
x = 4

41. Solve for x :

x 2 x 4 10
+ = ; x 3, 5 (AI CBSE 2014)
x3 x5 3

Quadratic Equations 71
Hint:
x2 x4 ( x 5 )( x 2) + ( x 4 )( x - 3)
L.H.S. = + (By cross multiplication)
x3 x 5 ( x 3)( x 5 )
2x 2 14x + 22

x 2 8x + 15
2x 2 14x + 22 10
= or 3[2x2 14x + 22] = 10[x2 8x + 15]
x 2 8x + 15 3
7
On simplification, we get 2x2 19x + 42 = 0 x =
2

ANSWERS
Test Your Skills
1. 8 2. ` 30 3. 8 cm, 15 cm, 17 cm 4. 25 km/hr
3
5. 6. 9 and 6 7. 7 5; 2 5 8. 25 km/hr
4
4 4 c b 5
9. 5 km/hr 10. k or k 11. x , x 12.
3 3 b a 7
13. 2, 1 14. 92 15. 45 km/hr 16. x = 5, 2
2 2
a b
17. , 18. 7x2 5x + 3 = 0 19. 63 20. 63
3 3
a b
21. 14 22. 750 km/h 23. x , 24. x = 16
3 3
25. 5, 10 26. 27 27. 45 28. 35
29. 5 , 5 30. x = b 31. 6, 12 32. 12, 14
2
1
33. a = 5 or 10, b = 10 or 5 34. x = a 35. x = ,3
2
3 2
36. 4 37. 8 38. 11 and 13
2

72 MathematicsX
5 Arithmetic Progressions

Facts that Matter


z Sequence
When some numbers are arranged in a definite order, according to a definite rule, they are said
to form a sequence.
The number occurring at the 1st place is called the 1st term, denoted by T1.
The number occurring at the nth place is called the nth term, denoted by Tn.
Example: Let us consider a rule
Tn = 3n + 1
Putting n = 1, 2, 3, 4, 5, ..... we get
T1 = 3 (1) + 1 = 4
T2 = 3 (2) + 1 = 7
T3 = 3 (3) + 1 = 10
T4 = 3 (4) + 1 = 13
T5 = 3 (5) + 1 = 16
...... ...... ......
...... ...... ......
...... ...... ......
The numbers 4, 7, 10, 13, 16, ..... ..... form a sequence.
The pattern followed by its terms is:
To start with 4 and add 3 to each term to get the next term.
z Finite Sequence
A sequence containing definite number of terms is called a finite sequence.
z Infinite Sequence
A sequence containing infinite number of terms is called an infinite sequence.
z Progressions
The sequences in which each term (other than the first and the last) is related to its succeeding
term by a fixed rule, are called progressions.
NOTE:
There are three types of progressions:
I. Arithmetic Progressions (A.P.)
II. Geometric Progressions (G.P.)
III. Harmonic Progressions (H.P.)
But, here we shall learn about arithmetic progression.
73
z Arithmetic Progression (A.P.)
An arithmetic progression is a list of numbers in which each term is obtained by adding a fixed
number to the preceding term except the first term.
The fixed number is called the common difference, which can be positive, negative or zero.
The common difference of an A.P. can be obtained by subtracting any term from its following
term.
Generally the first term is denoted by a and the common difference is denoted by d.
z If three numbers a, b, c are in order, then (b a) = common difference = (c d)
z When certain number of terms of an A.P. are required, then we select the terms in the following
manner:
Number of Terms Terms Common difference
3 a d, a, a + d d
4 a 3d, a d, a + d, a + 3d 2d
5 a 2d, a d, a, a + d, a + 2d d
6 a 5d, a 3d, a 3, a + d, a + 3 d, a + 5d 2d

NCERT TEXTBOOK QUESTIONS SOLVED


EXERCISE 5.1
Q. 1. In which of the following situations, does the list of numbers involved make an arithmetic
progression, and why?
(i) The taxi fare after each km when the fare is ` 15 for the first km and ` 8 for each additional
km.
1
(ii) The amount of air present in a cylinder when a vacuum pump removes of the air
4
remaining in the cylinder at a time.
(iii) The cost of digging a well after every metre of digging, when it costs ` 150 for the first
metre and rises by ` 50 for each subsequent metre.
(iv) The amount of money in the account every year, when ` 10,000 is deposited at compound
interest at 8% per annum.
Sol. (i) Let us consider,
The first term (T1) = Fare for the first km = ` 15 since, the taxi fare beyond the first
km is ` 8 for each additional km. T1 = 15
Fare for 2 km = ` 15 + 1 ` 8 T2 = a + 8 [where a = 15]
Fare for 3 km = ` 15 + 2 ` 8 T3 = a + 16
Fare for 4 km = ` 15 + 3 ` 8 T4 = a + 24
Fare for 5 km = ` 15 + 4 ` 8 T5 = a + 32
M
Fare for n km = ` 15 + (n 1) 8 Tn = a + (n 1) 8
We see that above terms form an A.P.
(ii) Let the amount of air in the cylinder = x
1
Air removed in 1st stroke = x
4
74 MathematicsX
1 3x
Air left after 1st stroke = x x =
4 4
3 x 1 3x FG IJ
3x 3 x 9
Air left after 2nd stroke =
4

4 4 H K
=
4

16
=
16
x

9 1 9 FG IJ
9x 9x 27 x
Air left after 3rd stroke =
16
x
4 16 H
x =
16 64K =
64
27 1 27 x FG IJ
27 x 27 91x
Air left after 4th stroke =
64
x
4 64
=
H 64 K
256
=
256
Thus, the terms are:
3x 9 27 91x
x, , x, x,
4 16 64 256
3x x
Here, x =
4 4
9 3x 3x
x =
16 4 16
FG IJ
x FG
3x IJ
Since
H K
4

H 16 K
The above terms are not in A.P.
(iii) Here, The cost of digging for 1st metre = ` 150
The cost of digging for first 2 metres = ` 150 + ` 50 = ` 200
The cost of digging for first 3 metres = ` 150 + (` 50) 2 = ` 250
The cost of digging for first 4 metres = ` 150 + (` 50) 3 = ` 300
The terms are: 150, 200, 250, 300, ...
Since, 200 150 = 50
And 250 200 = 50
(200 150) = (250 200)
The above terms form an A.P.
FG 8 IJ 1
(iv) The amount at the end of 1st year = 10000 1 +
H 100 K
FG 8 IJ 2

H 100 K
The amount at the end of 2nd year = 10000 1 +

F 8 IJ
The amount at the end of 3rd year = 10000 G 1 +
3

H 100 K
F 8 IJ
The amount at the end of 4th year = 10000 G 1 +
4

H 100 K
M
The terms are
LM10000 FG 1 + 8 IJ OP , LM10000 FG 1 + 8 IJ OP , LM10000 FG 1 + 8 IJ
2 3 OP , .....
[10000],
N H 100 K Q MN H 100 K PQ MN H 100 K PQ
Obviously,
LM10000 FG 1 + 8 IJ OP 10000 LM10000 FG 1 + 8 IJ OP L10000 FG 1 + 8 IJ O
2

N H 100 K Q MN H 100 K PQ MN H 100 K PQ


The above terms are not in A.P.
Arithmetic Progressions 75
Q. 2. Write first four terms of the AP, when the first term a and the common difference d are given as
follows:
(i) a = 10, d = 10 (ii) a = 2, d = 0
1
(iii) a = 4, d = 3 (iv) a = 1, d =
2
(v) a = 1.25, d = 0.25
Sol. (i) Tn = a + (n 1) d
For a = 10 and d = 10, we have:
T1 = 10 + (1 1) 10 = 10 + 0 = 10
T2 = 10 + (2 1) 10 = 10 + 10 = 20
T3 = 10 + (3 1) 10 = 10 + 20 = 30
T4 = 10 + (4 1) 10 = 10 + 30 = 40
Thus, the first four terms of A.P. are:
10, 20, 30, 40.
(ii) Tn = a + (n 1) d
For a = 2 and d = 0, we have:
T1 = 2 + (1 1) 0 = 2 + 0 = 2
T2 = 2 + (2 1) 0 = 2 + 0 = 2
T3 = 2 + (3 1) 0 = 2 + 0 = 2
T4 = 2 + (4 1) 0 = 2 + 0 = 2
The first four terms are:
2, 2, 2, 2.
(iii) Tn = a + (n 1) d
For a = 4 and d = 3, we have:
T1 = 4 + (1 1) ( 3) = 4 + 0 = 4
T2 = 4 + (2 1) ( 3) = 4 + ( 3) = 1
T3 = 4 + (3 1) ( 3) = 4 + ( 6) = 2
T4 = 4 + (4 1) ( 3) = 4 + ( 9) = 5
Thus, the first four terms are:
4, 1, 2, 5.
(iv) Tn = a + (n 1) d
1
For a = 1 and d = , we get
2
1
T1 = 1 + (1 1) =1+0=1
2
1 1 1
T2 = 1 + (2 1) =1+ =
2 2 2
1
T3 = 1 + (3 1) =1+1=0
2
1 3 1
T4 = 1 + (4 1) = 1+ =
2 2 2
The first four terms are:
1 1
1, , 0, .
2 2

76 MathematicsX
(v) Tn = a + (n 1) d
For a = 1.25 and d = 0.25, we get
T1 = 1.25 + (1 1) ( 0.25) = 1.25 + 0 = 1.25
T2 = 1.25 + (2 1) ( 0.25) = 1.25 + ( 0.25) = 1.50
T3 = 1.25 + (3 1) ( 0.25) = 1.25 + ( 0.50) = 1.75
T4 = 1.25 + (4 1) ( 0.25) = 1.25 + ( 0.75) = 2.0
Thus, the four terms are:
1.25, 1.50, 1.75, 2.0
Q. 3. For the following APs, write the first term and the common difference:
(i) 3, 1, 1, 3, ... (ii) 5, 1, 3, 7, ...
1 5 9 13
(iii) , , , , ... (iv) 0.6, 1.7, 2.8, 3.9, ...
3 3 3 3
Sol. (i) We have: 3, 1, 1, 3, .....
T1 = 3 a = 3
T2 = 1
T3 = 1
T4 = 3
T2 T1 = 1 3 = 2
T4 T3 = 3 ( 1) = 3 + 2 = 2
UV d=2
Thus, a = 3 and d = 2 W
(ii) We have: 5, 1, 3, 7, .....
U|
T1 = 5 a = 5
|V
T2 = 1 d = T2 T1 = 1 ( 5) = 1 + 5 = 4
T3 = 3 ||
d= 1+5=4
UV
T4 = 7 W T4 T3 = 7 3 = 4 d = 4
Thus, a = 5 and d = 4 W
1 5 9 13
(iii) We have: , , , , .....
3 3 3 3
1 1
T1 = a=
3 3
5 5 1 4
T2 = d = T2 T1 = =
3 3 3 3
T3 =
9 U| 13 9 4
13
13 V|
d = T4 T3 =
3

3
=
3
T4 =
1
3 W 4
Thus, a = and d =
3 3
(iv) We have: 0.6, 1.7, 2.8, 3.9, .....
T1 = 0.6 a = 0.6
T2 = 1.7 d = T2 T1 = 1.7 0.6 = 1.1
T3 = 2.8
T4 = 3.9 d = T4 T3 = 3.9 2.8 = 1.1
Thus, a = 0.6 and d = 1.1

Arithmetic Progressions 77
Q. 4. Which of the following are APs? If they form an AP, find the common difference d and write three
more terms.
5 7
(i) 2, 4, 8, 16, ... (ii) 2, , 3, , ...
2 2
(iii) 1.2, 3.2, 5.2, 7.2, ... (iv) 10, 6, 2, 2, ...
(v) 3, 3 + 2 , 3 + 2 2 , 3 + 3 2 , ... (vi) 0.2, 0.22, 0.222, 0.2222, ...
1 1 1 1
(vii) 0, 4, 8, 12, ... (viii) , , , , ...
2 2 2 2
(ix) 1, 3, 9, 27, ... (x) a, 2a, 3a, 4a, ...
(xi) a, a2, a3, a4, ... (xii) 2 , 8 , 18 , 32 , ...
(xiii) 3 , 6 , 9 , 12 , ... (xiv) 12, 32, 52, 72, ...
2 2 2
(xv) 1 , 5 , 7 , 73, ...
Sol. (i) We have: 2, 4, 8, 16, .....
T1 = 2 UV T2 T1 = 4 2 = 2
T2 = 4 W
T3 = 8
T4 = 16
UV T4 T3 = 16 8 = 8
Since 2 8
T2 T1 T4 T3
W
The given numbers do not form an A.P.
5 7
(ii) We have: 2, , 3, , .....
2 2
5 7
T1 = 2, T2 = , T3 = 3, T4 =
2 2
5 1
T2 T1 = 2 =
2 2
5 1
T3 T2 = 3 =
2 2
7 1
T4 T3 = 3 =
2 2
1 1
T2 T1 = T3 T2 = T4 T3 = d=
2 2
The given numbers form an A.P.
1 7 1
T5 = T4 + = + = 4
2 2 2
1 1 9
T6 = T5 + = 4+ =
2 2 2
1 9 1
T7 = T6 + = + = 5
2 2 2
1 9
Thus, d = and T5 = 4, T6 = and T7 = 5
2 2
(iii) We have: 1.2, 3.2, 5.2, 7.2, .....
T1 = 1.2, T2 = 3.2, T3 = 5.2, T4 = 7.2

78 MathematicsX
T2 T1 = 3.2 + 1.2 = 2
T3 T2 = 5.2 + 3.2 = 2
T4 T3 = 7.2 + 5.2 = 2
T2 T1 = T3 T2 = T4 T3 = 2 d = 2
The given numbers form an A.P.
Such that d = 2.
Now, T5 = T4 + ( 2) = 7.2 + ( 2) = 9.2
T6 = T5 + ( 2) = 9.2 + ( 2) = 11.2
T7 = T6 + ( 2) = 11.2 + ( 2) = 13.2
Thus, d = 2 and T5 = 9.2, T6 = 11.2 and T7 = 13.2
(iv) We have: 10, 6, 2, 2, .....
T1 = 10, T2 = 6, T3 = 2, T4 = 2
T2 T1 = 6 + 10 = 4
T3 T2 = 2 + 6 = 4
T4 T3 = 2 + 2 = 4
T2 T1 = T3 T2 = T4 T3 = 4 d = 4
The given numbers form an A.P.
Now, T 5 = T4 + 4 = 2 + 4 = 6
T6 = T5 + 4 = 6 + 4 = 10
T7 = T6 + 4 = 10 + 4 = 14
Thus, d = 4 and T5 = 6, T6 = 10, T7 = 14
(v) We have:
3, 3+ 2 , 3+2 2 , 3 + 3 2 , .....
T1 = 3, T2 = 3 + 2 , T3 = 3 + 2 2 , T4 = 3 + 3 2

T2 T1 = 3 + 2 3 = 2

T3 T2 = 3 + 2 2 3 2 = 2

T4 T3 = 3 + 3 2 3 2 2 = 2

T2 T1 = T3 T2 = T4 T3 = 2 d= 2
The given numbers form an A.P.
Now, T5 = T4 + 2

= 3+3 2 + 2 = 3+4 2

T6 = T5 + 2

= 3+4 2 + 2 = 3+5 2

T7 = T6 + 2

= 3+5 2 + 2 = 3+6 2

Thus, d = 2 and T5 = 3 + 4 2 , T6 = 3 + 5 2 , T7 = 3 + 6 2 .
Arithmetic Progressions 79
(vi) We have: 0.2, 0.22, 0.222, 0.2222, .....
T1 = 0.2 UV T2 T1 = 0.22 0.2 = 0.02
T2 = 0.22 W
T3 = 0.222 UV
T4 T3 = 0.2222 0.222 = 0.0002.
T4 = 0.2222 W
Since,
T2 T1 T4 T3
The given numbers do not form an A.P.
(vii) We have: 0, 4, 8, 12, .....
T1 = 0, T2 = 4, T3 = 8, T4 = 12
T2 T1 = 4 0 = 4
T3 T2 = 8 + 4 = 4
T4 T3 = 12 + 8 = 4
T2 T1 = T3 T2 = T4 T3 = 4 d = 4
The given numbers form an A.P.
Now, T5 = T4 + ( 4) = 12 + ( 4) = 16
T6 = T5 + ( 4) = 16 + ( 4) = 20
T7 = T6 + ( 4) = 20 + ( 4) = 24
Thus, d = 4 and T5 = 16, T6 = 20, T7 = 24
(viii) We have:
1 1 1 1
, , , , .....
2 2 2 2
1
T 1 = T2 = T3 = T4 =
2
T2 T1 = 0
T3 T2 = 0
T4 T3 = 0
T2 T1 = T3 T2 = T4 T3 = 0 d = 0
The given number form an A.P.
1 1
Now, T5 = + 0 =
2 2
1 1
T6 = + 0 =
2 2
1 1
T7 = + 0 =
2 2
1 1 1
Thus, d = 0 and T5 = , T6 = , T7 =
2 2 2
(ix) We have: 1, 3, 9, 27, .....
Here, T1 = 1 UV T2 T1 = 3 1 = 2
T2 = 3 W
T3 = 9 UV T4 T3 = 27 9 = 18
T4 = 27 W
T2 T1 T4 T3
The given numbers do not form an A.P.
80 MathematicsX
(x) We have: a, 2a, 3a, 4a, .....
T1 = a, T2 = 2a, T3 = 3a, T4 = 4a
T2 T1 = 2a a = a
T3 T2 = 3a 2a = a
T4 T3 = 4a 3a = a
T2 T1 = T3 T2 = T4 T3 = a d=a
The numbers form an A.P.
Now, T5 = T4 + a = 4a + a = 5a
T6 = T5 + a = 5a + a = 6a
T7 = T6 + a = 6a + a = 7a
Thus, d = a and T5 = 5a, T6 = 6a, T7 = 7a
(xi) We have: a, a2, a3, a4, .....
T1 = a UV T2 T1 = a2 a = a [a 1]
T2 = a2 W
T3 = a3 UV T4 T3 = a4 a3 = a3 [a 1]

Since,
T4 = a 4 W
T2 T1 T4 T3
The given terms are not in A.P.
(xii) We have: 2, 8, 18 , 32 , .....
T1 = 2 , T2 = 8 , T3 = 18 , T4 = 32
T2 T1 = 8 2 = 2 2 2 = 2
T3 T2 = 18 8 = 3 2 2 2 = 2
T4 T3 = 32 18 = 4 2 3 2 = 2
T2 T1 = T3 T2 = T4 T3 = 2 d= 2
The given numbers form an A.P.
Now, T5 = 4 2 + 2 = 5 2 = 50
T6 = 5 2 + 2 = 6 2 = 72
T7 = 6 2 + 2 = 7 2 = 98
Thus, d = 2 and T5 = 50 , T6 = 72 , T7 = 98
(xiii) We have: 3, 6 , 9, 12 , .....
T1 = 3 UV T2 T1 = 6 3 = 3 e j
2 1
T2 = 6 W
and T3 = UV
9 T4 T3 = 12 9 = 2 3 3 = e
3 2 3 j
T4 = 12 W
T2 T1 T T
4 3
The given terms do not form an A.P.

Arithmetic Progressions 81
(xiv) We have: 12, 32, 52, 72, .....
T 1 = 12 = 1 UV T2 T1 = 9 1 = 8
T 2 = 32 = 9 W
T 3 = 52 = 25 U
T 4 = 72 = 49 W
V T4 T3 = 49 25 = 24

T2 T1 T4 T3
The given terms do not form an A.P.
(xv) We have: 12, 52, 72, 73, .....
T1 = 12 = 1, T2 = 52 = 25, T3 = 72 = 49, T4 = 73
T2 T1 = 25 1 = 24
T3 - T2 = 49 25 = 24
T4 - T3 = 73 49 = 24
T2 T1 = T3 T2 = T4 T3 = 24 d = 24
The numbers form an A.P.
Now, T5 = T4 + 24 = 73 + 24 = 97
T6 = T5 + 24 = 97 + 24 = 121
T7 = T6 + 24 = 121 + 24 = 145
Thus, d = 24 and T5 = 97, T6 = 121, T7 = 145
z nth Term of an A.P.
The nth term Tn of the A.P. with first term a and common difference d is given by
Tn = a + (n 1) d
Tn is also called the general term of the A.P. If there are m terms in the A.P., then Tm
represents the last term which is generally denoted by l.

NCERT TEXTBOOK QUESTIONS SOLVED


EXERCISE 5.2
Q. 1. Fill in the blanks in the following table, given that a is the first term, d the common difference
and an the nth term of the A.P.:

a d n an

(i) 7 3 8
(ii) 18 10 0
(iii) 3 18 5
(iv) 18.9 2.5 3.6
(v) 3.5 0 105

Sol. (i) an = a + (n 1) d
a 8 = 7 + (8 1) 3
= 7+73
= 7 + 21
a 8 = 28
82 MathematicsX
(ii) an = a + (n 1) d
a 10 = 18 + (10 1) d
0 = 18 + 9d
18
9d = 18 d= = 2
9
d = 2
(iii) an = a + (n 1) d
5 = a + (18 1) ( 3)
5 = a + 17 ( 3)
5 = a 51
a = 5 + 51 = 46
Thus, a = 46
(iv) an = a + (n 1) d
3.6 = 18.9 + (n 1) 2.5
(n 1) 2.5 = 3.6 + 18.9
(n 1) 2.5 = 22.5
22.5
n1 = = 9
2.5
n = 9 + 1 = 10
Thus, n = 10
(v) an = a + (n 1) d
an = 3.5 + (105 1) 0
an = 3.5 + 104 0
an = 3.5 + 0 = 3.5
Thus, an = 3.5
Q. 2. Choose the correct choice in the following and justify:
(i) 30th term of the A.P.: 10, 7, 4, ...., is
(A) 97 (B) 77 (C) 77 (D) 87
1
(ii) 11th term of the A.P.: 3, , 2 , ...., is
2
1
(A) 28 (B) 22 (C) 38 (D) 48
2
Sol. (i) Here, a = 10, n = 30
Tn = a + (n 1) d and d = 7 10 = 3
T30 = 10 + (30 1) ( 3)
T30 = 10 + 29 ( 3)
T30 = 10 87 = 77
Thus, the correct choice is (C) 77.

Arithmetic Progressions 83
(ii) Here, a = 3, n = 11 and d =
1
2
b g 1
3 = +3 =
2
5
2
Tn = a + (n 1) d
5
T11 = 3 + (11 1)
2
T11 = 3 + 25 = 22
Thus, the correct choice is (B) 22.
Q. 3. In the following A.Ps., find the missing terms in the boxes:

(i) 2, , 26

(ii) , 13, ,3

1
(iii) 5, , , 9
2

(iv) 4, , , , ,6

(v) , 38, , , , 22
Sol. (i) Here, a = 2, T3 = 26
Let common difference = d
Tn = a + (n 1) d
T3 = 2 + (3 1) d
26 = 2 + 2d
2d = 26 2 = 24
24
d = = 12
2
The missing term = a + d

= 2 + 12 = 14
(ii) Let the first term = a and common difference = d
Here, T2 = 13 and T4 = 3
T2 = a + d = 13
T4 = a + 3d = 3
T4 T2 = (a + 3d) (a + d) = 3 13
2d = 10
10
= 5
d =
2
Now, a + d = 13 a + ( 5) = 13
a = 13 + 5 = 18
Thus, missing terms are a and a + 2d or 18 and 18 + ( 10) = 8
i.e., T1 = 18 and T3 = 8

84 MathematicsX
1
(iii) Here, a = 5 and T4 = 9
2
since, T4 = a + 3d
1
9 = 5 + 3d
2
1 1
3d = 9 5 = 4
2 2
1 9 1 3
d = 4 3 = =
2 2 3 2
The missing terms are:
3 1
T2 = a + d = 5 + = 6
2 2
FG 3 IJ
T3 = a + 2d = 5 + 2
H 2K = 8

(iv) Here, a = 4 and T6 = 6


Tn = a + (n 1) d
T6 = 4 + (6 1) d
6 = 4 + 5d
5d = 6 + 4 = 10
d = 10 5 = 2
T2 = a+d=4+2=2
T3 = a + 2d = 4 + 2 (2) = 0
T4 = a + 3d = 4 + 3 (2) = 2
T5 = a + 4d = 4 + 4 (2) = 4

The missing terms are 2 , 0 , 2 , 4

(v) Here, T2 = 38 and T6 = 22


T2 = a + d = 38
T6 = a + 5d = 22
T6 T2 = a + 5d (a + d) = 22 38
4d = 60
60
d = = 15
4
a+d = 38 a + ( 15) = 38
a = 38 + 15 = 53
Now, T3 = a + 2d = 53 + 2 ( 15) = 53 30 = 23
T4 = a + 3d = 53 + 3 ( 15) = 53 45 = 8
T5 = a + 4d = 53 + 4 ( 15) = 53 60 = 7

Thus, the missing terms are 53 , 23 , 8 , 7

Arithmetic Progressions 85
Q. 4. Which term of the A.P.: 3, 8, 13, 18, ..., is 78?
Sol. Let the nth term = 78
Here, a = 3, T1 = 3 and T2 = 8
d = T2 T1 = 8 3 = 5
Now, Tn = a + (n 1) d
78 = 3 + (n 1) 5
78 3 = (n 1) 5
75 = (n 1) 5
(n 1) = 75 5 = 15
n = 15 + 1 = 16
Thus, 78 is the 16th term of the given A.P.
Q. 5. Find the number of terms in each of the following A.Ps.:
1
(i) 7, 13, 19, ..., 205 (ii) 18, 15 , 13, ..., 47
2
Sol. (i) Here, a = 7
d = 13 7 = 6
Let the number of terms be n
Tn = 205
Now, Tn = a + (n 1) d
7 + (n 1) 6 = 205
(n 1) 6 = 205 7 = 198
198
n1 = = 33
6
n = 33 + 1 = 34
Thus, the required number of terms is 34.
(ii) Here, a = 18
1 1
d = 15 18 = 2
2 2
Let the nth term = 47
Tn = a + (n 1) d
FG 2 1 IJ
47 = 18 + (n 1)
H 2K
FG 5 IJ
47 18 = (n 1)
H2K
= (n 1) G
F 5 IJ
65
H2K
F 2 IJ
= 65 G
n1
H5K
n 1 = ( 13) ( 2) = 26
n = 26 + 1 = 27
Thus, the required number of terms is 27.

86 MathematicsX
Q. 6. Check whether 150 is a term of the A.P.: 11, 8, 5, 2 ...
Sol. For the given A.P., we have
a = 11
d = 8 11 = 3
Let 150 is the nth term of the given A.P.
Tn = a + (n 1) d
150 = 11 + (n 1) ( 3)
150 11 = (n 1) ( 3)
161 = (n 1) ( 3)
161 161
n1 = =
3 3
161 164 2
n = +1 = = 54
3 3 3
But n should be a positive integer.
Thus, 150 is not a term of the given A.P.
Q. 7. Find the 31st term of an A.P. whose 11th term is 38 and the 16th term is 73.
Sol. Here, T31 = ?
T11 = 38
T16 = 73
If the first term = a and the common difference = d.
Then,
a + (11 1) d = 38
a + 10 d = 38 ...(1)
and a + (16 1) d = 73
a + 15d = 73 ...(2)
Subtracting (1) from (2), we get
(a + 15d) (a + 10 d) = 73 38
5d = 35
35
d = = 7
5
From (1),
a + 10 (7) = 38
a + 70 = 38
a = 38 70 = 32
T31 = 32 + (31 1) 7
T31 = 32 + 30 7
T31 = 32 + 210
T31 = 178
Thus, the 31st term is 178.

Arithmetic Progressions 87
Q. 8. An A.P. consists of 50 terms of which 3rd term is 12 and the last term is 106. Find the 29th term.
Sol. Here, n = 50
T3 = 12
Tn = 106 T50 = 106
If first term = a and the common difference = d
T3 = a + 2d = 12 ...(1)
T50 = a + 49d = 106 ...(2)
T50 T3 a + 49d (a + 2d) = 106 12
47d = 94
94
d = = 2
47
From (1), we have
a + 2d = 12 a + 2 (2) = 12
a = 12 4 = 8
Now, T29 = a + (29 1) d
= 8 + (28) 2
= 8 + 56 = 64
Thus, the 29th term is 64.
Q. 9. If the 3rd and the 9th terms of an A.P. are 4 and 8 respectively, which term of this A.P. is zero ?
Sol. Here, T3 = 4 and T9 = 8
Using Tn = a + (n 1) d
T3 = a + 2d = 4 ...(1)
T9 = a + 8d = 8 ...(2)
Subtracting (1) from (2) we get
(a + 8d) (a + 2d) = 8 4
6d = 12
12
d = = 2
6
Now, from (1), we have:
a + 2d = 4
a + 2 ( 2) = 4
a4 = 4
a = 4+4=8
Let the nth term of the A.P. be 0.
Tn = a + (n 1) d = 0
8 + (n 1) ( 2) = 0
(n 1) 2 = 8
8
n1 = = 4
2
n = 4+1=5
Thus, the 5th term of the A.P. is 0.

88 MathematicsX
Q. 10. The 17th term of an A.P. exceeds its 10th term by 7. Find the common difference.
Sol. Let a be the first term and d be the common difference of the given A.P.
Now, using Tn = a + (n 1) d
T17 = a + 16d
T10 = a + 9d
According to the condition,
Tn + 7 = T17
(a + 9d) + 7 = a + 16d
a + 9d a 16d = 7
7d = 7 d = 1
Thus, the common difference is 1.
Q. 11. Which term of the A.P.: 3, 15, 27, 39, ... will be 132 more than its 54th term?
Sol. Here, a = 3
d = 15 3 = 12
Using Tn = a + (n 1) d, we get
T54 = a + 53d
= 3 + 53 12
= 3 + 636 = 639
Let an be 132 more than its 54th term.
an = T54 + 132
an = 639 + 132 = 771
Now an = a + (n 1) d = 771
3 + (n 1) 12 = 771
(n 1) 12 = 771 3 = 768
768
(n 1) = = 64
12
n = 64 + 1 = 65
Thus, 132 more than 54th term is the 65th term.
Q. 12. Two A.Ps. have the same common difference. The difference between their 100th terms is 100, what
is the difference between their 1000th terms?
Sol. Let for the 1st A.P., the first term = a
T100 = a + 99d
And for the 2nd A.P., the first term = a
= a + 99d
T100
According to the condition, we have:
T100 T100
= 100
a + 99d (a + 99d) = 100
a a = 100
Let, T1000 T1000
= x
a + 999d (a + 999d) = x
a a = x x = 100
The difference between the 1000th terms is 100.
Arithmetic Progressions 89
Q. 13. How many three-digit numbers are divisible by 7?
Ans. The first three digit number divisible by 7 is 105.
The last such three digit number is 994.
The A.P. is 105, 112, 119, ....., 994
Here, a = 105 and d = 7
Let n be the required number of terms.
Tn = a + (n 1) d
994 = 105 + (n 1) 7
(n 1) 7 = 994 105 = 889
889
(n 1) = = 127
7
n = 127 + 1 = 128
Thus, 128 numbers of 3-digit are divisible by 7.
Q. 14. How many multiples of 4 lie between 10 and 250?
Sol. The first multiple of 4 beyond 10 is 12.
The multiple of 4 just below 250 is 248.
The A.P. is given by:
12, 16, 20, ....., 248
Here, a = 12 and d = 4
Let the number of terms = n
Using Tn = a + (n 1) d, we get
Tn = 12 + (n 1) 4
248 = 12 + (n 1) 4
(n 1) 4 = 248 12 = 236
236
n1 = = 59
4
n = 59 + 1 = 60
Thus, the required number of terms = 60.
Q. 15. For what value of n, are the nth terms of two A.Ps.: 63, 65, 67, ... and 3, 10, 17, ... equal?
Sol. For the 1st A.P.
a = 63 and d = 65 63 = 2
Tn = a + (n 1) d
Tn = 63 + (n 1) 2
For the 2nd A.P.
a = 3 and d = 10 3 = 7
Tn = a + (n 1) d
Tn = 3 + (n 1) 7
Now, according to the condition,
3 + (n 1) 7 = 63 + (n 1) 2
(n 1) 7 (n 1) 2 = 63 3
90 MathematicsX
7n 7 2n + 2 = 60
5n 5 = 60
5n = 60 + 5 = 65
65
n = = 13
5
Thus, the 13th terms of the two given A.Ps. are equal.
Q. 16. Determine the A.P. whose third term is 16 and the 7th term exceeds the 5th term by 12.
Sol. Let the first term = a and the common difference = d.
Using Tn = a + (n 1) d, we have:
T3 = a + 2d
a + 2d = 16 ...(1)
And T7 = a + 6d, T5 = a + 4d
According to the condition,
T7 T5 = 12
(a + 6d) (a + 4d) = 12
a + 6d a 4d = 12
2d = 12
12
d = = 6 ...(2)
2
Now, from (1) and (2), we have:
a + 2 (6) = 16
a + 12 = 16
a = 16 12 = 4
The required A.P. is
4, [4 + 6], [4 + 2 (6)], [4 + 3 (6)], .....
or 4, 10, 16, 22, .....
Q. 17. Find the 20th term from the last term of the A.P.: 3, 8, 13, ..., 253.
Sol. We have, the last term l = 253
Here, d = 8 3 = 5
Since, the nth term before the last term is given by l (n 1) d,
We have
20th term from the end = l (20 1) 5
= 253 19 5
= 253 95 = 158
Q. 18. The sum of the 4th and 8th terms of an A.P. is 24 and the sum of the 6th and 10th terms is 44.
Find the first three terms of the A.P.
Sol. Let the first term = a
And the common difference = d
Using Tn = a + (n 1) d,
T4 + T8 = 24
(a + 3d) + (a + 7d) = 24
Arithmetic Progressions 91
2a + 10d = 24
a + 5d = 12 ...(1)
And T6 + T10 = 44
(a + 5d) + (a + 9d) = 44
2a + 14d = 44
a + 7d = 22 ...(2)
Now, subtracting (1) from (2), we get
(a + 7d) (a + 5d) = 22 12
2d = 10
10
d = = 5
2
From (1), a + 5 5 = 12
a + 25 = 12
a = 12 25 = 13
Now, the first three terms of the A.P. are given by:
a, (a + d), (a + 2d)
or 13, ( 13 + 5), [ 13 + 2 (5)]
or 13, 8, 3
Q. 19. Subba Rao started work in 1995 at an annual salary of ` 5000 and received an increment of
` 200 each year. In which year did his income reach ` 7000?
Sol. Here, a = ` 5000 and d = ` 200
Say, in the nth year he gets ` 7000.
Using Tn = a + (n 1) d, we get
7000 = 5000 + (n 1) 200
(n 1) 200 = 7000 5000 = 2000
2000
n1 = = 10
200
n = 10 + 1 = 11
Thus, his income becomes ` 7000 in 11 years.
Q. 20. Ramkali saved ` 5 in the first week of a year and then increased weekly savings by ` 1.75. If in
the nth week, her weekly savings become ` 20.75, find n.
Sol. Here, a = ` 5 and d = ` 1.75
In the nth week her savings become ` 20.75.
Tn = ` 20.75
Using Tn = a + (n 1) d, we have
20.75 = 5 + (n 1) (1.75)
(n 1) 1.75 = 20.75 5
(n 1) 1.75 = 15.75
15.75
n1 = = 9
1.75
n = 9 + 1 = 10
Thus, the required number of years = 10.

92 MathematicsX
z Sum of First n Terms of an A.P.
(i) If the first term of an A.P. is a and the common difference is d then the sum of
its first n terms is given by:
n
Sn = [2a + (n 1) d]
2
(ii) If the last term of the A.P. is l then
n
Sn = (a + l)
2
Remember,
The sum of first n positive integers is given by:

Sn =
b
n n+1 g
2

NCERT TEXTBOOK QUESTIONS SOLVED


EXERCISE 5.3
Q. 1. Find the sum of the following A.Ps.:
(i) 2, 7, 12, ..., to 10 terms. (ii) 37, 33, 29, ..., to 12 terms.
1 1 1
(iii) 0.6, 1.7, 2.8, ..., to 100 terms. (iv) , , , ..., to 11 terms.
15 12 10
Sol. (i) Here, a = 2
d = 72=5
n = 10
n
Since, Sn = [2a + (n 1) d]
2
10
S10 = [2 2 + (10 1) 5]
2
S10 = 5 [4 + 9 5]
S10 = 5 [49] = 245
Thus, the sum of first 10 terms is 245.
(ii) We have:
a = 37
d = 33 ( 37) = 4
n = 12
n
Sn = [2a + (n 1) d]
2
12
S12 = [2 ( 37) + (12 1) 4]
2
= 6 [ 74 + 11 4]
= 6 [ 74 + 44]
= 6 [ 30] = 180
Thus, sum of first 12 terms = 180.
Arithmetic Progressions 93
(iii) Here, a = 0.6
d = 1.7 0.6 = 1.1
n = 100
n
Sn = [2a + (n 1) d]
2
100
S100 = [2 (0.6) + (100 1) 1.1]
2
= 50 [1.2 + 99 1.1]
= 50 [1.2 + 108.9]
= 50 [110.1]
= 5505
Thus, the required sum of first 100 terms is 5505.
1
(iv) Here, a =
15
1 1 1
d = =
12 15 60
n = 11
n
Sn = [2a + (n 1) d]
2
11 LMFG 2 1 IJ + b11 1g 1 OP
S11 =
2 NH 15 K 60 Q
11 L 2 F 1 IO
M + G 10 J P
=
2 N 15 H 60 K Q
11 L 2 1O
= M
2 N 15 6 Q
+ P

11 L 4 + 5 O
2 MN 30 PQ
=

11 9 99 33
= = =
2 30 60 20
33
Thus, the required sum of first 11 terms = .
20
Q. 2. Find the sums given below:
1
(i) 7 + 10 + 14 + ... + 84
2
(ii) 34 + 32 + 30 + ... + 10
(iii) 5 + ( 8) + ( 11) + ... + ( 230)
Sol. (i) Here, a = 7
1 1 7
d = 10 7 = 3 =
2 2 2
l = 84
94 MathematicsX
Let n be the number of terms
Tn = a + (n 1) d
7
84 = 7 + (n 1)
2
7
(n 1) = 84 7 = 77
2
2
n 1 = 77 = 22
7
n = 22 + 1 = 23

Now, Sn =
n
2
b g
a+l

S23 =
23
2
b g
7 + 84

23 2093 1
= 91 = = 1046
2 2 2
1
Thus, the required sum = 1046 .
2
(ii) Here, a = 34
d = 32 34 = 2
l = 10
Let the number of terms be n
Tn = 10
Now Tn = a + (n 1) d
10 = 34 + (n 1) ( 2)
(n 1) ( 2) = 10 34 = 24
24
n1 = = 12
2
n = 13
n
Now, Sn = [2a + (n 1) d]
2
13
S13 = [2 34 + (13 1) ( 2)]
2
13
= [68 + 12 ( 2)]
2
13
= [68 24]
2
13
= 44
2
= 13 22 = 286
OR
n
S13 = (a + l)
2
13
= (34 + 10)
2
13
= 44 = 13 22 = 286
2
Thus, the required sum is 286.

Arithmetic Progressions 95
(iii) Here, a = 5
d = 8 ( 5) = 3
l = 230
Let n be the number of terms.
Tn = 230
230 = 5 + (n 1) ( 3)
(n 1) ( 3) = 230 + 5 = 225
225
n1 = = 75
3
n = 75 + 1 = 76
76
Now, S76 = [( 5) + ( 230)]
2
= 38 ( 235)
= 8930
The required sum = 8930.
Q. 3. In an A.P.:
(i) given a = 5, d = 3, an = 50, find n and Sn.
(ii) given a = 7, a13 = 35, find d and S13.
(iii) given a12 = 37, d = 3, find a and S12.
(iv) given a3 = 15, S10 = 125, find d and a10.
(v) given d = 5, S9 = 75, find a and a9.
(vi) given a = 2, d = 8, Sn = 90, find n and an.
(vii) given a = 8, an = 62, Sn = 210, find n and d.
(viii) given an = 4, d = 2, Sn = 14, find n and a.
(ix) given a = 3, n = 8, S = 192, find d.
(x) given l = 28, S = 144, and there are total 9 terms. Find a.
Sol. (i) Here, a = 5, d = 3 and an = 50 = l
an = a + (n 1) d
50 = 5 + (n 1) 3
50 5 = (n 1) 3
(n 1) 3 = 45
45
(n 1) = = 15
3
n = 15 + 1 = 16
n
Now Sn = (a + l)
2
16
= (5 + 50)
2
= 8 (55) = 440
Thus, n = 16 and Sn = 440
96 MathematicsX
(ii) Here, a = 7 and a13 = 35 = l
an = a + (n 1) d
35 = 7 + (13 1) d
35 7 = 12d
28 = 12d
28 7
d = =
12 3
Now, using
n
Sn = (a + l)
2
13
S13 = (7 + 35)
2
13
= 42
2
= 13 21 = 273
7
Sn = 273 and d =
3
(iii) Here, a 12 = 37 = l and d = 3
Let the first term of the A.P. be a.
Now a 12 = a + (12 1) d
37 = a + 11d
37 = a + 11 3
37 = a + 33
a = 37 33 = 4
n
Now, Sn = (a + l)
2

12
S12 = (4 + 37)
2
S12 = 6 (41) = 246
Thus, a = 4 and S12 = 246
(iv) Here, a3 = 15 = l
S10 = 125
Let first term of the A.P. be a and the common difference = d
a3 = a + 2d
a + 2d = 15 ...(1)
n
Again Sn = [2a + (n 1) d]
2
10
S10 = [2a + (10 1) d]
2
125 = 5 [2a + 9d]

Arithmetic Progressions 97
125
2a + 9d = = 25
5
2a + 9d = 25 ...(2)
Multiplying (1) by 2 and subtracting (2) from it, we get
2 [a + 2d = 15] [2a + 9d = 25]
2a + 4d 2a 9d = 30 25
5d = 5
5
d = = 1
5
From (1), a + 2 ( 1) = 15 a = 15 + 2 a = 17
Now, a10 = a + (10 1) d
= 17 + 9 ( 1)
= 17 9 = 8
Thus, d = 1 and a10 = 8
(v) Here, d = 5, S9 = 75
Let the first term of the A.P. is a.
9
S9 = [2a + (9 1) 5]
2
9
75 = [2a + 40]
2
2
75 = 2a + 40
9
50
= 2a + 40
3
50 70
2a = 40 =
3 3
70 1 35
a = =
3 2 3
Now, a 9 = a + (9 1) d
35
=
3
b
+ 85 g
35
= + 40
3
35 + 120 85
= =
3 3
35 85
Thus, a = and a9 =
3 3
(vi) Here, a = 2, d = 8 and Sn = 90
n
Sn = [2a + (n 1) d]
2
98 MathematicsX
n
90 = [2 2 + (n 1) 8]
2
90 2 = 4n + n (n 1) 8
180 = 4n + 8n2 8n
180 = 8n2 4n
45 = 2n2 n
2n2 n 45 = 0
2n2 10n + 9n 45 = 0
2n (n 5) + 9 (n 5) = 0
(2n + 9) (n 5) = 0
9
Either 2n + 9 = 0 n =
2
or n5 = 0 n=5
9
But n = is not required.
2
n = 5
Now, an = a + (n 1) d
a 5 = 2 + (5 1) 8
= 2 + 32 = 34
Thus, n = 5 and a5 = 34.
(vii) Here, a = 8, an = 62 = l and Sn = 210
Let the common difference = d
Now, Sn = 210
n
210 = (a + l)
2
n n
210 = (8 + 62) = 70 = 35n
2 2
210
n = = 6
35
Again an = a + (n 1) d
62 = 8 + (6 1) d
62 8 = 5d
54
54 = 5d d =
5
54
Thus, n = 6 and d = .
5
(viii) Here, an = 4, d = 2 and Sn = 14
Let the first term be a.
an = 4
a + (n 1) 2 = 4

Arithmetic Progressions 99
a + 2n 2 = 4
a = 4 2n + 2
a = 6 2n ...(1)
Also Sn = 14
n
(a + l) = 14
2
n
(a + 4) = 14
2
n (a + 4) = 28 ...(2)
Substituting the value of a from (1) into (2),
n [6 2n + 4] = 28
n [10 2n] = 28
2n [5 n] = 28
n (5 n) = 14 [Dividing throughout by 2]
5n n2 + 14 = 0
n2 5n 14 = 0
n2 7n + 2n 14 = 0
n (n 7) + 2 (n 7) = 0
(n 7) (n + 2) = 0
Either n7 = 0 n=7
or n+2 = 0 n=2
But n cannot be negative,
n = 7
Now, from (1), we have
a = 627 a=8
Thus, a = 8 and n = 7
(ix) Here, a = 3, n = 8 and Sn = 192
Let the common difference = d.
n
Sn = [2a + (n 1) d]
2
8
192 = [2 (3) + (8 1) d]
2
192 = 4 [6 + 7d]
192 = 24 + 28d
28d = 192 24 = 168
168
d = = 6
28
Thus, d = 6.
(x) Here, l = 28 and S9 = 144
Let the first term be a.

100 MathematicsX
n
Then Sn = (a + l)
2
9
S9 = (a + 28)
2
9
144 = (a + 28)
2
2
a + 28 = 144 = 16 2 = 32
9
a = 32 28 = 4
Thus, a = 4.
Q. 4. How many terms of the A.P.: 9, 17, 25, ... must be taken to give a sum of 636?
Sol. Here, a = 9
d = 17 9 = 8
Sn = 636
n
Sn = [2a + (n 1) d] = 636
2
n
[(2 9) + (n 1) 8] = 636
2
n [18 + (n 1) 8] = 1272
n (8n + 10) = 1272
8n2 + 10n 1272 = 0
4n2 + 5n 636 = 0
2
4n + 53n 48n 636 = 0
n (4n + 53) 12 (4n + 53) = 0
53
(n 12) (4n + 53) = 0 n = 12 and n=
4
53
Rejecting n = , we have n = 12.
4
Q. 5. The first term of an A.P. is 5, the last term is 45 and the sum is 400. Find the number of terms
and the common difference.
Sol. Here, a = 5
l = 45 = Tn
Sn = 400
Tn = a + (n 1) d
45 = 5 + (n 1) d
(n 1) d = 45 5
(n 1) d = 40 ...(1)
n
Also Sn = (a + l)
2
n
400 = (5 + 45)
2
Arithmetic Progressions 101
400 2 = n 50
400 2
n = = 16
50
From (1), we get
(16 1) d = 40
15d = 40
40 8
d = =
15 3
Q. 6. The first and the last terms of an A.P. are 17 and 350 respectively. If the common difference is
9, how many terms are there and what is their sum?
Sol. We have,
First term a = 17
Last term l = 350 = Tn
Common difference d = 9
Let the number of terms be n
Tn = a + (n 1) d
350 = 17 + (n 1) 9
(n 1) 9 = 350 17 = 333
333
n1 = = 37
9
n = 37 + 1 = 38
n
Since, Sn = (a + l)
2
38
S38 = (17 + 350)
2
= 19 (367) = 6973
Thus, n = 38 and Sn = 6973
Q. 7. Find the sum of first 22 terms of an A.P. in which d = 7 and 22nd term is 149.
Sol. Here, n = 22, T22 = 149 = l
d = 7
Let the first term of the A.P. be a.
Tn = a + (n 1) d
Tn = a + (22 1) 7
a + 21 7 = 149
a + 147 = 149
a = 149 147 = 2
n
Now, S22 = [a + l]
2
22
S22 = [2 + 149]
2
= 11 [151] = 1661
Thus S22 = 1661
102 MathematicsX
Q. 8. Find the sum of first 51 terms of an A.P. whose second and third terms are 14 and 18 respectively.
Sol. Here, n = 51, T2 = 14 and T3 = 18
Let the first term of the A.P. be a and the common difference is d.
We have:
T2 = a + d a + d = 14 ...(1)
T3 = a + 2d a + 2d = 18 ...(2)
Subtracting (1) from 2, we get
a + 2d a d = 18 14
d = 14
From (1), we get
a + d = 14 a + 4 = 14
a = 14 4 = 10
n
Now, Sn = [2a + (n 1) d]
2
51
S51 = [(2 10) + (51 1) 4]
2
51
= [20 + 200]
2
51
= 220
2
= 51 110 = 5610
Thus, the sum of 51 terms is 5610.
Q. 9. If the sum of first 7 terms of an A.P. is 49 and that of 17 terms is 289, find the sum of first n
terms.
Sol. Here, we have:
S7 = 49 and S17 = 289
Let the first term of the A.P. be a and d be the common difference, then
n
Sn = [2a + (n 1) d]
2

7
S7 = [2a + (7 1) d] = 49
2
7 (2a + 6d) = 2 49 = 98
98
2a + 6d = = 14
7
2 [a + 3d] = 14
14
a + 3d = = 7
2
a + 3d = 7 ...(1)
17
Also, S17 = [2a + (17 1) d] = 289
2
Arithmetic Progressions 103
17
(2a + 16d) = 289
2
289
a + 8d = = 17
17
a + 8d = 17 ...(2)
Subtracting (1) from (2), we have:
a + 8d a 3d = 17 7
5d = 10
10
d = =2
5
Now, from (1), we have
a + 3 (2) = 7
a = 76=1
n
Now, Sn = [2a + (n 1) d]
2
n
= [2 1 + (n 1) 2]
2
n
= [2 + 2n 2]
2
n
= 2n
2
= n n = n2
Thus, the required sum of n terms = n2.
Q. 10. Show that a1, a2, ..., an, ... form an A.P. where an is defined as below:
(i) an = 3 + 4n (ii) an = 9 5n
Also find the sum of the first 15 terms in each case.
Sol. (i) Here, an = 3 + 4n
Putting n = 1, 2, 3, 4, ..... n, we get:
a 1 = 3 + 4 (1) = 7
a 2 = 3 + 4 (2) = 11
a 3 = 3 + 4 (3) = 15
a 4 = 3 + 4 (4) = 19
..... ..... .....
an = 3 + 4n
The A.P. in which a = 7 and d = 11 7 = 4 is:
7, 11, 15, 19, ....., (3 + 4n).
15
Now S15 = [(2 7) + (15 1) 4]
2
15
= [14 + (14 4)]
2
104 MathematicsX
15
= [14 + 56]
2
15
= 70
2
= 15 35 = 525
(ii) Here, an = 9 5n
Putting n = 1, 2, 3, 4, ....., n, we get
a 1 = 9 5 (1) = 4
a 2 = 9 5 (2) = 1
a 3 = 9 5 (3) = 6
a 4 = 9 5 (4) = 11
..... .....
The A.P. is:
4, 1, 6, 11, ..... 9 5 (n) [having first term as 4 and d = 1 4 = 5]
15
S15 = [(2 4) + (15 1) ( 5)]
2
15
= [8 + 14 ( 5)]
2
15
= [8 70]
2

=
15
2
b g
62

= 15 ( 31) = 465.
Q. 11. If the sum of the first n terms of an A.P. is 4n n2, what is the first term (that is S1)? What is
the sum of first two terms? What is the second term? Similarly, find the 3rd, the 10th and the
nth terms.
Sol. We have:
Sn = 4n n2
S1 = 4 (1) (1)2
= 4 1 = 3 First term = 3
S2 = 4 (2) (2)2
= 8 4 = 4 Sum of first two terms = 4
Second term (S2 S1) = 4 3 = 1
S3 = 4 (3) (3)2
= 12 9 = 3 Sum of first 3 terms = 3
Third term (S3 S2) = 3 4 = 1
S9 = 4 (9) (9)2
= 36 81 = 45
S10 = 4 (10) (10)2
= 40 100 = 60
Tenth term = S10 S9 = [ 60] [ 45] = 15

Arithmetic Progressions 105


Now, Sn = 4 (n) (n)2 = 4n n2
Also Sn 1 = 4 (n 1) (n 1)2
= 4n 4 [n2 2n + 1]
= 4n 4 n2 + 2n 1
= 6n n2 5
nth term = Sn Sn 1
= [4n n2] [6n n2 5]
= 4n n2 6n + n2 + 5 = 5 2n
Thus, S1 = 3 and a1 = 3
S2 = 4 and a2 = 1
S3 = 3 and a3 = 1
a 10 = 15 and an = 5 2n
Q. 12. Find the sum of the first 40 positive integers divisible by 6.
Sol. The first 40 positive integers divisible by 6 are:
6, 12, 18, ....., (6 40).
And, these numbers are in A.P. such that
a = 6
d = 12 6 = 6 and an = 6 40 = 240 = l
40
S40 = [(2 6) + (40 1) 6]
2
= 20 [12 + 39 6]
= 20 [12 + 234]
= 20 246 = 4920
OR
n
Sn = [a + l]
2
40
S40 = [6 + 240]
2
= 20 246 = 4920
Thus, the sum of first 40 multiples of 6 is 4920.
Q. 13. Find the sum of the first 15 multiples of 8.
Sol. The first 15 multiples of 8 are:
8, (8 2), (8 3), (8 4), ....., (8 15)
or 8, 16, 24, 32, ....., 120.
These numbers are in A.P., where
a = 8 and l = 120
15
S15 = [a + l]
2
15
= [8 + 120]
2
15
= 128
2
= 15 64 = 960
Thus, the sum of first positive 15 multiples of 8 is 960.

106 MathematicsX
Q. 14. Find the sum of the odd numbers between 0 and 50.
Sol. Odd numbers between 0 and 50 are:
1, 3, 5, 7, ....., 49
These numbers are in A.P. such that
a = 1 and l = 49
Here, d = 3 1 = 2
Tn = a + (n 1) d
49 = 1 + (n 1) 2
49 1 = (n 1) 2
48
(n 1) = = 24
2
n = 24 + 1 = 25
25
Now, S25 = 1 + 49
2
25
= 50
2
= 25 25 = 625
Thus, the sum of odd numbers between 0 and 50 is 625.
Q. 15. A contract on construction job specifies a penalty for delay of completion beyond a certain date
as follows: ` 200 for the first day, ` 250 for the second day, ` 300 for the third day, etc., the
penalty for each succeeding day being ` 50 more than for the preceding day. How much money
the contractor has to pay as penalty, if he has delayed the work by 30 days? (CBSE 2012)
Sol. Here, penalty for delay on
1st day = ` 200
2nd day = ` 250
3rd day = ` 300
...............
...............
Now, 200, 250, 300, ..... are in A.P. such that
a = 200, d = 250 200 = 50
S30 is given by

30 LM
using Sn =
n
a f OPQ
2a + n 1 d
S30 =
2
[2 (200) + (30 1) 50]
N 2
= 15 [400 + 29 50]
= 15 [400 + 1450]
= 15 1850 = 27,750
Thus, penalty for the delay for 30 days is ` 27,750.
Q. 16. A sum of ` 700 is to be used to give seven cash prizes to students of a school for their overall
academic performace. If each prize is ` 20 less than its preceding prize, find the value of each
of the prizes. (CBSE 2012)

Arithmetic Progressions 107


Sol. Sum of all the prizes = ` 700
Let the first prize = a
2nd prize = (a 20)
3rd prize = (a 40)
4th prize = (a 60)
........................................
Thus, we have, first term = a
Common difference = 20
Number of prizes, n = 7
Sum of 7 terms Sn = 700
n
Since, Sn = [2a + (n 1) d]
2
7
700 = [2 (a) + (7 1) ( 20)]
2
7
700 = [2a + (6 20)]
2
2
700 = 2a 120
7
200 = 2a 120
2a = 200 + 120 = 320
320
a = = 160
2
Thus, the values of the seven prizes are:
` 160, ` (160 20), ` (160 40), ` (160 60), ` (160 80), ` (160 100) and
` (160 120)
` 160, ` 140, ` 120, ` 100, ` 80, ` 60 and ` 40.
Q. 17. In a school, students thought of planting trees in and around the school to reduce air pollution.
It was decided that the number of trees that each section of each class will plant, will be the same
as the class, in which they are studying, e.g., a section of Class I will plant 1 tree, a section of
Class II will plant 2 trees and so on till Class XII. There are three sections of each class. How many
trees will be planted by the students? (CBSE 2012)
Sol. Number of classes = 12
Each class has 3 sections.
Number of plants planted by class I = 1 3 = 3
Number of plants planted by class II = 2 3 = 6
Number of plants planted by class III = 3 3 = 9
Number of plants planted by class IV = 4 3 = 12
.......................................................................................................
Number of plants planted by class XII = 12 3 = 36
The numbers 3, 6, 9, 12, ..........., 36 are in A.P.
Here, a = 3 and d = 6 3 = 3

108 MathematicsX
Number of classes = 12
i.e., n = 12
Sum of the n terms of the above A.P., is given by
12 LMusing S =
n
a f OPQ
2a + n 1 d
S12 =
2
[2 (3) + (12 1) 3]
N n
2
= 6 [6 + 11 3]
= 6 [6 + 33]
= 6 39 = 234
Thus, the total number of trees = 234.
Q. 18. A spiral is made up of successive semi-circles, with centres alternately at A and B, starting with
centre at A, of radii 0.5 cm, 1.0 cm, 1.5 cm, 2.0 cm, ..... as shown in figure. What is the total
22
length of such a spiral made up of thirteen consecutive semi-circles? (Take = )
7
l3

l1

A B

l2

l4
[Hint: Length of successive semi-circles is l1, l2, l3, l4, ... with centres at A, B, A, B, ..., respectively.]
Sol. Length of a semi-circle = semi-circumference
1
= (2 r)
2
= r
l 1 = r1 = 0.5 cm = 1 0.5 cm
l 2 = r2 = 1.0 cm = 2 0.5 cm
l 3 = r3 = 1.5 cm = 3 0.5 cm
l 4 = r4 = 2.0 cm = 4 0.5 cm
...... ............... ......................
l 13 = r13 cm = 6.5 cm = 13 0.5 cm
Now, length of the spiral
= l1 + l2 + l3 + l4 + ..... + l13
= 0.5 [1 + 2 + 3 + 4 + ..... + 13] cm ...(1)
1, 2, 3, 4, ....., 13 are in A.P. such that
a = 1 and l = 13
13 n
S13 = 1 + 13 [using Sn = (a + l)]
2 2

Arithmetic Progressions 109


13
= 14
2
= 13 7 = 91
From (1), we have:
Total length of the spiral
= 1.5 [91] cm
5 22
91 cm
LMQ = 22 OP
=
10 7 N 7 Q
1 22
= 91 cm
2 7
= 11 13 cm = 143 cm.
Q. 19. 200 logs are stacked in the following manner: 20 logs in the bottom row, 19 in the next row, 18
in the row next to it and so on (see figure). In how many rows are the 200 logs placed and how
many logs are in the top row?

Sol. We have:
The number of logs:
1st row = 20
2nd row = 19
3rd row = 18
obviously, the numbers
20, 19, 18, ....., are in A.P. such that
a = 20
d = 19 20 = 1
Let the numbers of rows be n.
Sn = 200
n
Now, using, Sn = [2a + (n 1) d], we get
2
n
Sn = [2 (20) + (n 1) ( 1)]
2
n
200 = [40 (n 1)]
2
2 200 = n 40 n (n 1)
400 = 40n n2 + n
n2 41n + 400 = 0
2
n 16n 25n + 400 = 0
n (n 16) 25 (n 16) = 0
(n 16) (n 25) = 0
110 MathematicsX
Either
n 16 = 0 n = 16
or n 25 = 0 n = 25
Tn = 0 a + (n 1) d = 0 20 + (n 1) ( 1) = 0
n 1 = 20 n = 21
i.e., 21st term becomes 0
n = 25 is not required.
Thus, n = 16
Number of rows = 16
Now, T16 = a + (16 1) d
= 20 + 15 ( 1)
= 20 15 = 5
Number of logs in the 16th (top) row is 15.
Q. 20. In a potato race, a bucket is placed at the starting point, which is 5 m from the first potato, and
the other potatoes are placed 3 m apart in a straight line. There are ten potatoes in the line
(see figure).

A competitor starts from the bucket, picks up the nearest potato, runs back with it, drops it in the
bucket, runs back to pick up the next potato, runs to the bucket to drop it in, and she continues
in the same way until all the potatoes are in the bucket. What is the total distance the competitor
has to run?
[Hint: To pick up the first potato and the second potato, the total distance (in metres) run by a
competitor is 2 5 + 2 (5 + 3)]
Sol. Here, number of potatoes = 10
The up-down distance of the bucket:
From the 1st potato = [5 m] 2 = 10 m
From the 2nd potato = [(5 + 3) m] 2 = 16 m
From the 3rd potato = [(5 + 3 + 3) m] 2 = 22 m
From the 4th potato = [(5 + 3 + 3 + 3) m] 2 = 28 m
.................................. ...........................
10, 16, 22, 28, ..... are in A.P. such that
a = 10 and d = 16 10 = 6
n
Using Sn = [2a + (n 1) d], we have:
2
10
S10 = [2 (10) + (10 1) 6]
2
= 5 [20 + 9 6]

Arithmetic Progressions 111


= 5 [20 + 54]
= 5 [74]
= 5 74 = 370
Thus, the sum of above distances = 370 m.
The competitor has to run a total distance of 370 m.

NCERT TEXTBOOK QUESTIONS SOLVED


EXERCISE 5.4
Q. 1. Which term of the A.P.: 121, 117, 113, ..., is its first negative term?
[Hint: Find n for an < 0] (CBSE 2012)
Sol. We have the A.P. having a = 121 and d = 117 121 = 4
an = a + (n 1) d
= 121 + (n 1) ( 4)
= 121 4n + 4
= 125 4n
For the first negative term, we have
an < 0
(125 4n) < 0
125 < 4n
125
< n
4
1
31 < n
4
1
or n > 31
4
Thus, the first negative term is 32nd term.
Q. 2. The sum of the third and the seventh terms of an A.P. is 6 and their product is 8. Find the sum
of first sixteen terms of the A.P..
Sol. Here, T3 + T7 = 6 and T3 T7 = 8
Let the first term = a and the common difference = d
T3 = a + 2d and T7 = a + 6d
T3 + T7 = 6
(a + 2d) + (a + 6d) = 6
2a + 8d = 6
a + 4d = 3 ...(1)
Again T3 T7 = 8
(a + 2d) (a + 6d) = 8
(a + 4d 2d) (a + 4d + 2d) = 8
[(a + 4d) 2d] [(a + 4d) + 2d] = 8
[(3) 2d] [(3) + 2d] = 8 [From (1)]
32 (2d)2 = 8
9 4d2 = 8

112 MathematicsX
4d2 = 8 9 = 1
1 1
d2 = =
4 4
1
d =
2
1
When d = .
2
From (1), we have:
FG 1 IJ
a+4
H 2K = 3

a + 2 = 3 or a = 3 2 = 1
n
Now, Using Sn = [2a + (n 1) d], we get
2
16 LM a f b
2 1 + 16 1 g OPQ
1
S16 =
2 N 2

=
L 15 O
8 M2 + P
N 2Q
= 16 + 60 = 76
i.e., the sum of first 16 terms = 76
1
When d = .
2
From (1), we have:
FG 1 IJ
a+4
H 2K = 3

a2 = 3 a = 5
Again, the sum of first sixteen terms
16 LM a f b g FG IJ OP
1
S16 =
2 N H KQ
2 5 + 16 1
2
L F 15 IJ OP
8 M10 + G
=
N H 2 KQ
25 cm

= 80 60 = 20
i.e., the sum of first 16 terms = 20
Q. 3. A ladder has rungs 25 cm apart (see figure). The rungs
decrease uniformly in length from 45 cm at the bottom to 25
1m
1 2
2
cm at the top. If the top and the bottom rungs are 2 m
2
apart, what is the length of the wood required for the rungs?
250
[Hint: Number of rungs = + 1]
25 25 cm
Sol. Total distance between bottom to top rungs
45 cm
1
= 2 m
2
Arithmetic Progressions 113
5
= 100 cm
2
= 250 m
Distance between two consecutive rungs = 25 cm
250
Number of rungs = + 1 = 10 + 1 = 11
25
Length of the 1st rung (bottom rung) = 45 cm
Length of the 11th rung (top rung) = 25 cm
Let the length of each successive rung decrease by x cm
Total length of the rungs
= 45 cm + (45 x) cm + (45 2x) cm + ..... + 25 cm
Here, the numbers 45, (45 x), (45 2x), ....., 25 are in an A.P. such that
First term a = 45
Last term l = 25
Number of terms n = 11
n
Using, Sn = [a + l], we have
2
11
S11 = [45 + 25]
2
11
S11 = 70
2
S11 = 11 35 = 385
Total length of 11 rungs = 385 cm
i.e., Length of wood required for the rungs is 385 cm.
Q. 4. The houses of a row are numbered consecutively from 1 to 49. Show that there is a value of x such
that the sum of the numbers of the houses preceding the house numbered x is equal to the sum
of the numbers of the houses following it. Find this value of x.
[Hint: Sx 1 = S49 Sx]
Sol. We have, the following consecutive numbers on the houses of a row ;
1, 2, 3, 4, 5, ....., 49.
These numbers are in an A.P., such that
a = 1
d = 21=1
n = 49
Let one of the houses be numbered as x
Number of houses preceding it = x 1
Number of houses following it = 49 x
Now, the sum of the house-numbers preceding x is given by:
n
Using Sn = [2a + (n 1) d]
2
x1
Sx 1 = [2 (1) + (x 1 1) 1]
2

114 MathematicsX
x1
= [2 + x 2]
2
x1
= x
2

=
x x1 b g
2
x2 x
=
2 2
The houses beyond x are numbered as
(x + 1), (x + 2), (x + 3), ....., 49
For these house numbers (which are in an A.P.),
First term (a) = x + 1
Last term (l) = 49
n
Using Sn = [a + l], we have
2
49 x
S49 x = [(x + 1) + 49]
2
49 x
= x + 50
2
49 x x 2
=
2

2
b g
+ 49 25 25 x

FG 49x 25xIJ x + b49 25g


2
=
H2 K 2
2
x x
= + b 49 25g
2 2
According to the question,
[Sum of house numbers preceding x] = [Sum of house numbers following x]
i.e., Sx 1 = S49 x
x2 x x x2

2

2
=
2

2
+ 49 25b g
Fx 2
x2 I
x x
GH 2 +
2 JK
+
2 2
= (49 25)

2x2
= (49 25)
2
x 2 = (49 25)
x = 49 25
x = (7 5) = 35
But x cannot be taken as ve
x = 35

Arithmetic Progressions 115


Q. 5. A small terrace at a football ground comprises of 15 steps each of which is 50 m long and built
of solid concrete.
1 1
Each step has a rise of m and a tread of m . (see Fig.). Calculate the total volume of concrete
4 2
required to build the terrace.
1 1
[Hint: Volume of concrete required to build the first step = 50 m 3 ]
4 2

1m
2

m
50
1m
4
Sol. For 1st step:
1 1
Length = 50 m, Breadth = m , Height = m
2 4
Volume of concrete required to build the 1st step
= Volume of the cuboidal step
= Length Breadth height
1 1
= 50 m 3
2 4
25
= 1 m3
4
For 2nd step:
1 FG IJ 1 1 1
Length = 50 m, Breadth =
2 H K
m , Height = +
4 4
m = 2 m
4
Volume of concrete required to build the 2nd step
1 1
= 50 2 m 3
2 4
25
= 2 m3
4
For 3rd step:
1 FG IJ
1 1 1 1
Length = 50 m, Breadth =
2 H
m , Height =
K+ +
4 4 4
m = 3 m
4
Volume of concrete required to build the 3rd step
1 1
= 50 3 m 3
2 4
25
= 3 m3
4
........... ........... ........... ........... ...........
Thus, the volumes (in m3) of concrete required to build the various steps are:
116 MathematicsX
FG 25 1IJ , FG 25 2IJ , FG 25 3IJ , ..... .....
H4 K H4 K H4 K
obviously, these numbers form an A.P. such that
25
a =
4

25 25 25
d = =
2 4 4
Here, total number of steps n = 15
Total volume of concrete required to build 15 steps is given by the sum of their individual
volumes.
n
Using, Sn = [2 (a) + (n 1) d], we have:
2

15 LM FG IJ b g
25 25
m3
OP
S15 =
2
2
NH K
4
+ 15 1
4 Q
15 L 25 25 O
2 MN 2
= + 14 P m 3
4Q
15 L 25 175 O
2 MN 2 2 PQ
3
= + m

15 200 3
= m
2 2
= 15 50 m3 = 750 m3
Thus, the required volume of concrete is 750 m3.

MORE QUESTIONS SOLVED


I. VERY SHORT ANSWER TYPE QUESTIONS
Q. 1. If the numbers x 2, 4x 1 and 5x + 2 are in A.P. Find the value of x.
[NCERT Exemplar Problem, (CBSE Sample Paper 2011)]
Sol. x 2, 4x 1 and 5x + 2 are in A.P.
(4x 1) (x 2) = (5x + 2) (4x 1)
3x + 1 = x + 3
2x = 2 x = 1
Q. 2. Which term of the A.P. 4, 9, 14, ..... is 109?
Sol. Let 109 is the nth term,
Using Tn = a + (n 1) d, we have:
109 = 4 + (n 1) 5 [ a = 4 and d = 9 4 = 5]

Arithmetic Progressions 117


109 4 105
n1 = = = 21
5 5
n = 21 + 1 = 22
Thus, the 22nd term is 109.
Q. 3. If a, (a 2) and 3a are in A.P. then what is the value of a?
Sol. a, (a 2) and 3a are in A.P.
(a 2) a = 3a (a 2)
a 2 a = 3a a + 2
2 = 2a + 2
2a = 2 2 = 4
4
a = = 2
2
Thus, the required value of a is 2.
Q. 4. How many terms are there in the A.P.?
7, 10, 13, ....., 151
Sol. Here, a = 7, d = 10 7 = 3
Let there are n-terms.
Tn = a + (n 1) d
T51 = 7 + (n 1) 3
151 7
= n1
3
144
= n 1 n = 48 + 1 = 49
3
i.e., n = 49
Q. 5. Which term of the A.P. 72, 63, 54, ..... is 0?
Sol. Here, a = 72
d = 63 72 = 9
Let nth term of this A.P. be 0
Tn = a + (n 1) d
72 + (n 1) ( 9) = 0
72
(n 1) = = 8
9
n = 8+1=9
Thus the 9th term of the A.P. is 0.
Q. 6. The first term of an A.P. is 6 and its common difference is 2. Find its 18th term.
Sol. Using Tn = a + (n 1) d, we have:
T18 = 6 + (18 1) ( 2)
= 6 + 17 ( 2)
= 6 34 = 28
Thus, the 18th term is 28.
Q. 7. The 4th term of an A.P. is 14 and its 12th term 70. What is its first term?
Sol. Let the first term = a
If d is the common difference,
Then T4 = a + 3d = 14 ...(1)
And T12 = a + 11d = 70 ...(2)
118 MathematicsX
Subtracting (1) from (2),
a + 11d a 3d = 70 14
56
8d = 56 d= = 7
8
From (1), a + 3 (7) = 14
a + 21 = 14
a = 14 21 = ( 7)
Thus, the first term is 7.
Q. 8. Which term of A.P. 5, 2, 1, 4 ..... is 40?
Sol. Here, a = 5
d = 25=3
Let nth term be 40
Tn = a + (n 1) d
40 = 5 + (n 1) ( 3)
40 5 45
n1 = = = 15
3 3
n = 15 + 1 = 16
i.e., The 16th term of the A.P. is 40.
Q. 9. What is the sum of all the natural numbers from 1 to 100?
Sol. We have:
1, 2, 3, 4, ....., 100 are in an A.P. such that
a = 1 and l = 100
n
Sn = [a + l]
2
100
S100 = [1 + 100] = 50 101 = 5050.
2
Q. 10. For an A.P., the 8th term is 17 and the 14th term is 29. Find its common difference.
Sol. Let the common difference = d and first term = a
T8 = a + 7d = 17 ...(1)
T14 = a + 13d = 29 ...(2)
Subtracting (1) from (2), we have:
a + 13d a 7d = 29 17
6d = 12
12
d = = 2
6
The required common difference = 2.
Q. 11. If the first and last terms of an A.P. are 10 and 10. How many terms are there? Given that
d = 1.
Sol. Let the required number of terms is n and
1st term a = 10
nth term Tn = 10
Let common difference be d then using,
Tn = a + (n 1) d, we have:

Arithmetic Progressions 119


10 = 10 + (n 1) ( 1)
10 = 10 n + 1
n + 1 = 10 10 = 20
n = 20 1 = 21
n = 21
Q. 12. The nth term of an A.P. is (3n 2) find its first term.
Sol. Tn = 3n 2
T1 = 3 (1) 2 = 3 2 = 1
First term = 1
Q. 13. The nth term of an A.P. is (2n 3) find the common difference.
Sol. Here, Tn = 2n 3
T1 = 2 (1) 3 = 1
T2 = 2 (2) 3 = 1
d = T2 T1 = 1 ( 1) = 2
Thus the common difference is 2.
Q. 14. If the nth term of an A.P. is (7n 5). Find its 100th term.
Sol. Here, Tn = 7n 5
T1 = 7 (1) 5 = 2
T2 = 7 (2) 5 = 9
a = 2
and d = T2 T1
= 92=7
Now T100 = 2 + (100 1) 7 [using Tn = a + (n 1) d]
= 2 + 99 7
= 2 + 693 = 695.
Q. 15. Find the sum of first 12 terms of the A.P. 5, 8, 11, 14, ...... .
Sol. Here, a = 5
d = 85=3
n = 12
n
Using Sn = [2 (a) + (n 1) d]
2
12
we have: S12 = [2 (5) + (12 1) 3]
2
= 6 [10 + 33]
= 6 43 = 258
Q. 16. Write the common difference of an A.P. whose nth term is 3n + 5. (AI CBSE 2009 C)
Sol. Tn = 3n + 5
T1 = 3 (1) + 5 = 8
T2 = 3 (2) + 5 = 11
d = T2 T1
= 11 8 = 3
Thus, the common difference = 3.

120 MathematicsX
Q. 17. Write the value of x for which x + 2, 2x, 2x + 3 are three consecutive terms of an A.P.
(CBSE 2009 C)
Sol. Here, T1 = x + 2
T2 = 2x
T3 = 2x + 3
For an A.P., we have:
2x (x + 2) = 2x + 3 2x
2x x 2 = 2x + 3 2x
x2 = 3
x = 3+2=5
Thus, x = 5
Q. 18. What is the common difference of an A.P. whose nth term is 3 + 5n? (CBSE 2009 C)
Sol. Tn = 3 + 5n
T1 = 3 + 5 (1) = 8
And T2 = 3 + 5 (2) = 13
d = T2 T1
d = 13 8 = 5
Thus, common difference = 5.
Q. 19. For what value of k, are the numbers x, (2x + k) and (3x + 6) three consecutive terms of an A.P.?
(AI CBSE 2009)
Sol. Here, T1 = x, T2 = (2x + k) and T3 = (3x + 6)
For an A.P., we have
T2 T1 = T3 T2
i.e., 2x + k x = 3x + 6 (2x + k)
x + k = 3x + 6 2x k
x+k = x+6k
k+k = x+6x
2k = 6
6
k = = 3
2
4
Q. 20. If , a, 2 are three consecutive terms of an A.P., then find the value of a? (AI CBSE 2009)
5
4
Sol. Here, T1 =
5
T2 = a
T3 = 2
For an A.P.,
T2 T1 = T3 T2
4
a = 2a
5
4
a + a = 2+
5
14
2a =
5
Arithmetic Progressions 121
14 1 7
a = =
5 2 5
7
Thus, a =
5
Q. 21. For what value of p are 2p 1, 7 and 3p three consecutive terms of an A.P.? (CBSE 2009)
Sol. Here, T1 = 2p 1
T2 = 7
T3 = 3p
For an A.P., we have:
T2 T1 = T3 T2
7 (2p 1) = 3p 7
7 2p + 1 = 3p 7
2p 3p = 7 1 7
5p = 15
15
p = = 3
5
Thus, p = 3
Q. 22. For what value of p are 2p + 1, 13 and 5p 3 three consecutive terms of an A.P.?
(CBSE 2009)
Sol. Here, T1 = 2p + 1
T2 = 13
T3 = 5p 3
For an A.P., we have:
T2 T1 = T3 T2
13 (2p + 1) = 5p 3 13
13 2p 1 = 5p 16
2p + 12 = 5p 16
2p 5p = 16 12 = 28
7p = 28
28 28
p = = = 4
7 7
p = 4
Q. 23. The nth term of an A.P. is 7 4n. Find its common difference. (CBSE 2008)
Sol. Tn = 7 4n
T1 = 7 4 (1) = 3
T2 = 7 4 (2) = 1
d = T2 T1
= ( 1) 3 = 4
Thus, common difference = 4
Q. 24. The nth term of an A.P. is 6n + 2. Find the common difference. (CBSE 2008)
Sol. Here, Tn = 6n + 2
T1 = 6 (1) + 2 = 8
T2 = 6 (2) + 2 = 14
d = T2 T1 = 14 8 = 6
Common difference = 6.

122 MathematicsX
Q. 25. Write the next term of the A.P. 8, 18 , 32 , ..... (AI CBSE 2008)
Sol. Here, T1 = 8 = 42 = 2 2
T2 = 18 = 92 = 3 2
T3 = 32 = 16 2 = 4 2

a = 2 2
Now, d = T2 T1
= 3 2 2 2 = b
2 32 = g 2
T4 = a + 3d

= 2 2+3 e 2j
= 2 2 +3 2

= b g
2 2 + 3 = 5 2 or 50

Thus, the next term of the A.P. is 5 2 or 50 .


Q. 26. The first term of an A.P. is p and its common difference is q. Find the 10th term.
(AI CBSE 2008)
Sol. Here, a = p and d = q
Tn = a + (n 1) d
T10 = p + (10 1) q
= p + 9q
Thus, the 10th term is p + 9q.
Q. 27. Find the next term of the A.P. 2, 8, 18 ..... (CBSE 2008 C)

Sol. Here, T1 = 2 a= 2
T2 = 8 = 2 2

T3 = 18 = 3 2
Now, d = T2 T1
= 2 2 2

= 2
Now, using Tn = a + (n 1) d, we have
T4 = a + 3d
= 2+3 e 2j
= 2 1+3 = 4 2

= 16 2 = 32

Thus, the next term = 32 .


Arithmetic Progressions 123
Q. 28. Which term of the A.P.:
21, 18, 15, ..... is zero? (CBSE 2008 C)
Sol. Here, a = 21
d = 18 21 = 3
Since Tn = a + (n 1) d
0 = 21 + (n 1) ( 3)
3 (n 1) = 21
21
(n 1) = = 7
3
n = 7+1=8
Thus, the 8th term of this A.P. will be 0.
Q. 29. Which term of the A.P.:
14, 11, 8, ..... is 1? (AI CBSE 2008 C)
Sol. Here, a = 14
d = 11 14 = 3
Let the nth term be ( 1)
Using Tn = a + (n 1) d, we get
1 = 11 + (n 1) ( 3)
1 14 = 3 (n 1)
15 = 3 (n 1)
15
n1 = = 5
3
n = 5+1=6
Thus, 1 is the 6th term of the A.P.
Q. 30. The value of the middlemost term (s) of the AP : 11, 7, 3, ...49.
[NCERT Exemplar]
Sol. Q a = 11, an = 49 and d = (7) (11) = 4
an = a + (n 1)d
49 = 11 + (n 1) 4 n = 16
Since, n is an even number
There will be two middle terms, which are:
16 16
th and + 1 th
2 2
or 8th and 9th
Now, a 8 = a + (8 1)d
= 11 + 7 4 = 17
a 9 = a + (9 1)d
= 11 + 8 4 = 21
Thus, the values of the two middlemost terms are : 17 and 21.

124 MathematicsX
II. SHORT ANSWER TYPE QUESTIONS
Note : For an A.P. with the 1st term and common difference a and d respectively, we have :
(a) nth term from the end = (m n +1)th term from the beginning, where m is the number
of terms in the A.P.
nth term from the end = (a) + (m n)d
(b) If l is the last term of the A.P., then
nth term from the end is the nth term of an A.P. whose first term is l and common
difference is d
nth term from the end = l + (n 1) (d)

Q. 1. If 9th term of an A.P. is zero, prove that its 29th term is double of its 19th term.
[NCERT Exemplar]
Sol. Let a be the first term and d be the common difference.
Now, Using Tn = a + (n 1) d, we have
T9 = a + 8d a + 8d = 0 ...(1) [ T9 = 0 Given]
T19 = a + 18d = (a + 8d) + 10d = (0) + 10d = 10d ...(2) [ a + 8d = 0]
T29 = a + 28d
= (a + 8d) + 20d
= 0 + 20d = 20d [ a + 8d = 0]
= 2 (10d) = 2 (T19) [ T19 = 10d]
T29 = 2 (T19)
Thus, the 29th term of the A.P. is double of its 19th term.
Q. 2. If Tn = 3 + 4n then find the A.P. and hence find the sum of its first 15 terms.
Sol. Let the first term be a and the common difference be d.
Tn = a + (n 1) d
T1 = a + (1 1) d = a + 0 d = a
T2 = a + (2 1) d = a + d
But it is given that
Tn = 3 + 4n
T1 = 3 + 4 (1) = 7
First term, a = 7
Also, T2 = a + d = 3 + 4 (2) = 11
d = T2 T1 = 11 7 = 4
n
Now, using Sn = [2a + (n 1) d], we get
2
15
S15 = [2 (7) + (15 1) 4]
2
15
= [14 + 14 4]
2
15
= 70
2
= 15 35 = 525
Thus, the sum of first 15 terms = 525.

Arithmetic Progressions 125


Q. 3. Which term of the A.P.:
3, 15, 27, 39, ..... will be 120 more than its 53rd term?
Sol. The given A.P. is:
3, 15, 27, 39, .....
a = 3
d = 15 3 = 12
Using, Tn = a + (n 1) d, we have:
T53 = 3 + (53 1) 12
= 3 + (52 12)
= 3 + 624 = 627
Now, T53 + 120 = 627 + 120 = 747.
Let the required term be Tn
Tn = 747
or a + (n 1) d = 747
3 + (n 1) 12 = 747
(n 1) 12 = 747 3 = 744
744
n1 = = 62
12
n = 62 + 1 = 63
Thus, the 63rd term of the given A.P. is 120 more than its 53rd term.
Q. 4. Find the 31st term of an A.P. whose 10th term is 31 and the 15th term is 66.
Sol. Let the first term is a and the common difference is d.
Using Tn = a + (n 1) d, we have:
T10 = a + 9d
31 = a + 9d ...(1)
Also T15 = a + 14d
66 = a + 14d ...(2)
Subtracting (1) from (2), we have:
a + 14d a 9d = 66 31
5d = 35
35
d = = 7
5
From (1), a + 9d = 31
a + 9 (7) = 31
a + 63 = 31
a = 31 63
a = 32
Now, T31 = a + 30d
= 32 + 30 (7)
= 32 + 210 = 178
Thus, the 31st term of the given A.P. is 178.
Q. 5. If the 8th term of an A.P. is 37 and the 15th term is 15 more than the 12th term, find the A.P.
Hence find the sum of the first 15 terms of the A.P.
Sol. Let the 1st term = a
126 MathematicsX
And the common difference = d
Using Tn = a + (n 1) d
T8 = a + 7d
37 = a + 7d ...(1)
Also T15 = a + 14d
And T12 = a + 11d
According to the question,
T15 = T12 + 15
a + 14d = a + 11d + 15
a a + 14d 11d = 15
15
3d = 15 d= = 5
3
From (1), we have:
a + 7 (5) = 37
a + 35 = 37
a = 37 35 = 2
Since an, A.P. is given by :
a, a + d, a + 2d, a + 3d, ....
The required A.P. is given by 2, 2 + 5, 2 + 2(5),...
2, 7, 12, ...
n
Now, using Sn = [2a + (n 1) d]
2
15
S15 = [2 (2) + 14 5]
2
15
= [4 + 70]
2
15
= 74 = 15 37 = 555.
2
Q. 6. The 5th and 15th terms of an A.P. are 13 and 17 respectively. Find the sum of first 21 terms
of the A.P.
Sol. Let a be the first term and d be the common difference.
Using Tn = a + (n 1) d, we have:
T15 = a + 14d = 17 ...(1)
T5 = a + 4d = 13 ...(2)
Subtracting (2) from (1), we have:
(T15 T5) = 17 13 = 30
a + 14d a 4d = 30
10d = 30 d = 3
Substituting d = 3 in (2), we get
a + 4d = 13
a + 4 ( 3) = 13
a + ( 12) = 13
a = 13 + 12 = 25
Arithmetic Progressions 127
n
Now using Sn = [2a + (n 1) d] we have:
2
21
S21 = [2 (25) + (21 1) ( 3)]
2
21
= [50 + ( 60)]
2
21
= 10
2
= 21 ( 5) = 105
Thus, the sum of first fifteen terms = 105.
Q. 7. The 1st and the last term of an A.P. are 17 and 350 respectively. If the common difference is 9
how many terms are there in the A.P.? What is their sum?
Sol. Here, first term, a = 17
Last term Tn = 350 = l
Common difference (d) = 9.
Using Tn = a + (n 1) d, we have:
350 = 17 + (n 1) 9
350 17
n1 =
9
333
= = 37
9
n = 37 + 1 = 38
Thus, there are 38 terms.
n
Now, using, Sn = [a + l], we have
2
38
S38 = [17 + 350]
2
= 19 [367] = 6973
Thus, the required sum of 38 terms = 6973.
Q. 8. If the sum of first 7 terms of an A.P. is 49 and that of first 17 terms is 289, find the sum of n
terms. (CBSE 2008 C)
Sol. Let the first term = a and the common difference = d.
n
Using Sn = [2a + (n 1) d]
2
7
S7 = [2a + 6d] = 49
2
7
2 a + 3d = 49
2
7 [a + 3d] = 49
49
a + 3d = = 7
7
i.e., a + 3d = 7 ...(1)
17
Also S17 = [2a + 16d] = 289
2
128 MathematicsX
17
2 [a + 8d] = 289
2
17 [a + 8d] = 289
289
a + 8d = = 17
17
a + 8d = 17 ...(2)
Subtracting (2) from (1), we have:
a + 8d a 3d = 17 7
5d = 10 d = 2
From (1), we have
a + 3 (2) = 7
a + 6 =7 a =76=1
n
Now, Sn = [2a + (n 1) d]
2
n
= [2 1 + (n 1) 2]
2
n
= [2 + 2n 2]
2
n
= 2n
2
Thus, the sum of n terms is n2.
Q. 9. The first and last term of an A.P. are 4 and 81 respectively. If the common difference is 7, how
many terms are there in the A.P. and what is their sum?
Sol. Here, first term = 4 a = 4 and d = 7.
Last term, l = 81 Tn = 81
Tn = a + (n 1) d
81 = 4 + (n 1) 7
81 4 = (n 1) 7
77
77 = (n 1) 7 n = + 1 = 11 + 1 = 12
7
There are 12 terms.
Now, using
n
Sn = (a + l)
2
12
S12 = (4 + 81)
2
S12 = 6 85 = 510
The sum of 12 terms of the A.P. is 510.
Q. 10. The angles of a quadrilateral are in A.P. whose common difference is 15. Find the angles.
Sol. Let one of the angles = a
The angles are in an A.P.
The angles are:
a, (a + d), (a + 2d) and (a + 3d)
d = 15 [Given]
Arithmetic Progressions 129
The angles are:
a, (a + 15), [a + 2 (15)] and [a + 3 (15)]
i.e., a, (a + 15), (a + 30) and (a + 45).
The sum of the angles of a quadrilateral is 360.
a + (a + 15) + (a + 30) + (a + 45) = 360
4a + 90 = 360
4a = 360 90 = 270
270 1
a = = 67
4 2
The four angles are:

67
1 FG 1 IJ FG
o
1 IJ o
, 67 + 15 , 67 + 30 , and 67 1 + 45 FG
IJ
2 H 2 K H 2 K H 2 K
1 1 1 1
or 67 , 82 , 97 , and 112 .
2 2 2 2
Q. 11. The angles of a triangle are in AP. The greatest angle is twice the least. Find all the angles of the
triangle. [NCERT Exemplar]
Sol. Let a, b, c are the angles of the triangle, such that
c = 2a ...(1)
Since a, b, c are in A.P.
a+c
Then b = ...(2)
2
From (1) and (2), we get
a + 2a
a, , 2 a are the three angles of the triangle.
2
a + 2a
a+ + 2 a = 180
2
2a + a + 2a + 4a = 360
9a = 360
a = 360 = 40
9
The smallest angle = 40
The greatest angle = 2a = 2 40 = 80
a + c 40 + 80
The third angle = = = 60
2 2
Thus the angles of the triangle are : 40, 60, 80.
Q. 12. Find the middle term of the A.P. 10, 7, 4, ....., 62. (AI CBSE 2009 C)
Sol. Here, a = 10
d = 7 10 = 3
Tn = ( 62)
Using Tn = a + (n 1) d, we have
62 = 10 + (n 1) ( 3)
62 10 72
n1 = = = 24
3 3

130 MathematicsX
n = 24 + 1 = 25
Number of terms = 25
FG n + 1IJ th term
Middle term =
H 2 K
25 + 1
= th term
2
= 13th term
Now T13 = 10 + 12d
= 10 + 12 ( 3)
= 10 36 = 26
Thus, the middle term = 26.
Q. 13. Find the sum of all three digit numbers which are divisible by 7. (CBSE 2012)
Sol. The three digit numbers which are divisible by 7 are:
105, 112, 119, ....., 994.
It is an A.P. such that
a = 105
d = 112 105 = 7
Tn = 994 = l
Tn = a + (n 1) d
994 = 105 + (n 1) 7
994 105 889
n1 = = = 127
7 7
n = 127 + 1 = 128
n
Now, using Sn = [a + l]
2
128
We have S128 = [105 + 994]
2
= 64 [1099]
= 70336
Thus, the required sum = 70336.
Q. 14. Find the sum of all the three digit numbers which are divisible by 9. (AI CBSE 2009 C)
Sol. All the three digit numbers divisible by 9 are:
117, 126, ....., 999 and they form an A.P.
Here, a = 108
d = 117 108 = 9
Tn = 999 = l
Now, using Tn = a + (n 1) d, we have
999 = 108 + (n 1) (9)
999 108 = (n 1) 9
891 = (n 1) 9
891
n1 = = 99
9
n = 99 + 1 = 100
Now, the sum of n term of an A.P. is given
Arithmetic Progressions 131
n
Sn = [a + l]
2
100
S100 = [108 + 999]
2
= 50 [1107]
= 55350
Thus, the required sum is 55350.
Q. 15. Find the sum of all the three digit numbers which are divisible by 11. (CBSE 2009 C)
Sol. All the three digit numbers divisible by 11 are 110, 121, 132, ....., 990.
Here, a = 110
d = 121 110 = 11
Tn = 990
Using Tn = a + (n 1) d, we have
990 = 110 + (n 1) 11
990 110
n1 = = 80
11
n = 80 + 1 = 81
n
Now, using Sn = [a + l], we have
2
81
S81 = [110 + 990]
2
81
= 1100
2
= 81 550 = 44550
Thus, the required sum = 44550.
Q. 16. The sum of first six terms of an AP is 42. The ratio of 10th term to its 30th term is 1 : 3. Calculate
the first term and 13th term of A.P.
6
Sol. Q S6 =
2
{2 a + (6 - 1)d} = 42
6a + 15d = 42 ...(1)
Also, (a10) : (a30) = 1 : 3
a + 9d 1
or =
a + 29d 3
3(a + 9d) = a + 29d
3a + 27d = a + 27d
2a = 2d
a = d ...(2)
From (1) 6d + 15d = 42 d =2
From (2) a = d d =2
Now, a 13 = a + 12d
= 2 + 12 2 = 26
Q. 17. If Sn the sum of n terms of an A.P. is given by Sn = 3n2 4n, find the nth term.
(CBSE 2012)
Sol. We have:
Sn 1 = 3 (n 1)2 4 (n 1)
132 MathematicsX
= 3 (n2 2n + 1) 4n + 4
= 3n2 6n + 3 4n + 4
= 3n2 10n + 7
nth term = Sn Sn 1
= 3n2 4n [3n2 10n + 7]
= 3n2 4n 3n2 + 10n 7
= 6n 7.
Q. 18. The sum of 4th and 8th terms of an A.P. is 24, and the sum of 6th and 10th terms is 44. Find
the A.P. (CBSE 2009)
Sol. Let, the first term = a
Common difference be = d
Using Tn = a + (n 1) d, we have
T4 = a + 3d
T6 = a + 5d
T8 = a + 7d
T10 = a + 9d
T4 + T8 = 24
(a + 3d) + (a + 7d) = 24
2a + 10d = 24
a + 5d = 12 [Dividing by 2] ...(1)
Also T6 + T10 = 44
(a + 5d) + (a + 9d) = 44
2a + 14d = 44
a + 7d = 22 [Dividing by 2] ...(2)
Subtracting (1) from (2), we have:
(a + 7d) (a + 5d) = 22 12
2d = 10 d=5
From (1), a + 5 (5) = 12
a = 12 25 = 13
Since, the A.P. is given by:
a, a + d, a + 2d, .....
We have the required A.P. as:
13, ( 13 + 5), [ 13 + 2 (5)], .....
or 13, 8, 3, .....
Q. 19. If Sn, the sum of first n terms of an A.P. is given by
Sn = 5n2 + 3n
Then find the nth term. (CBSE 2009)
Sol. Sn = 5n2 + 3n
Sn 1 = 5 (n 1)2 + 3 (n 1)
= 5 (n2 2n + 1) + 3 (n 1)
Arithmetic Progressions 133
= 5n2 10n + 5 + 3n 3
= 5n2 7n + 2
Now, nth term = Sn Sn 1
The required nth term
= [5n2 + 3n] [5n2 7n + 2]
= 10n 2.
Q. 20. The sum of 5th and 9th terms of an A.P. is 72 and the sum of 7th and 12th term of 97. Find
the A.P. (CBSE 2009)
Sol. Let a be the 1st term and d be the common difference of the A.P.
Now, using Tn = a + (n 1) d, we have
T5 = a + 4d
T7 = a + 6d
T9 = a + 8d
T12 = a + 11d
T5 + T9 = 72
a + 4d + a + 8d = 72
2a + 12d = 72
a + 6d = 36 [Dividing by 2] ...(1)
Also T7 + T12 = a + 6d + a + 11d = 97
36 + a + 11d = 97 [From (1)]
a + 11d = 97 36
a + 11d = 61 ...(2)
Subtracting (1) from (2), we get
a + 11d a 6d = 61 36
5d = 25
25
d =
5
From (1), we have
a + 11 (5) = 61
a + 55 = 61
a = 61 55 = 6
Now, an A.P. is given by
a, a + d, a + 2d, a + 3d, .....
The required A.P. is:
6, (6 + 5), [6 + 2 (5)], [6 + 3 (5)], .....
or 6, 11, 16, 24, .....
Q. 21. In an A.P. the sum of its first ten terms is 150 and the sum of its next term is 550. Find the
A.P.
Sol. Let the first term = a
And the common difference = d
10
S10 =
2
[2 a + (10 - 1)d ] = 150

134 MathematicsX
10a + 45d = 150
2a + 9d = 30 ...(1)
Q The sum of next 10 terms (i.e. S20 S10) = 550
20

2
[2 a + (20 - 1)d ] - (-150) = 550

20a + 190d + 150 = 550


2a + 19d + 15 = 55
2a + 19d = 55 15
2a + 19d = 70 ...(2)
Subtracting (1) from (2), we get
2a + 19d = 70
2a + 9d = 30
+
-40
10d = 40 d= = -4
10
6
From (1), 2(a) + 9(4) = 30 or a = =3
2
Thus, AP is a, a + d, a + 2d ...
or 3, [3 + (4)], [3 + 2(4)], ...
or 3, 1, 5, ...
Q. 22. Which term of the A.P. 3, 15, 27, 39, ..... will be 120 more than its 21st term?
(AI CBSE 2009)
Sol. Let the 1st term is a and common difference = d
a = 3 and d = 15 3 = 12
Now, using Tn = a + (n 1) d
T21 = 3 + (21 1) 12
= 3 + 20 12
= 3 + 240 = 243
Let the required term be the nth term.
nth term = 120 + 21st term
= 120 + 243 = 363
Now Tn = a + (n 1) d
363 = 3 + (n 1) 12
363 3 = (n 1) 12
360
n1 = = 30
12
n = 30 + 1 = 31
Thus the required term is the 31st term of the A.P.
Q. 23. Which term of the A.P. 4, 12, 20, 28, ..... will be 120 more than its 21st term?
(AI CBSE 2009)

Arithmetic Progressions 135


Sol. Here, a = 4
d = 12 4 = 8
Using Tn = a + (n 1) d
T21 = 4 + (21 1) 8
= 4 + 20 8 = 164
The required nth term = T21 + 120
nth term = 164 + 120 = 284
284 = a + (n 1) d
284 = 4 + (n 1) 8
284 4 = (n 1) 8
280
n1 = = 35
8
n = 35 + 1 = 36
Thus, the required term is the 36th term of the A.P.
Q. 24. The sum of n terms of an A.P. is 5n2 3n. Find the A.P. Hence find its 10th term.
(CBSE 2008)
Sol. We have:
Sn = 5n2 3n
S1 = 5 (1)2 3 (1) = 2
First term T1 = (a) = 2
S2 = 5 (2)2 3 (2) = 20 6 = 14
Second term T2 = 14 2 = 12
Now the common difference = T2 T1
d = 12 2 = 10
An A.P. is given by
a, (a + d), (a + 2d) .....
The required A.P. is:
2, (2 + 10), [2 + 2 (10)], .....
2, 12, 22, .....
Now, using Tn = a + (n 1) d, we have
T10 = 2 + (10 1) 10
= 2 + 9 10
= 2 + 90 = 92.
Q. 25. Find the 10th term from the end of the A.P.:
8, 10, 12, ....., 126 (CBSE 2008)
Sol. Here, a = 8
d = 10 8 = 2
Tn = 126
Using Tn = a + (n 1) d
126 = 8 + (n 1) 2
126 8
n1 = = 59
2
n = 59 + 1 = 60

136 MathematicsX
l = 60
Now 10th term from the end is given by
l (10 1) = 60 9 = 51
Now, T51 = a + 50d
= 8 + 50 2
= 8 + 100 = 108
Thus, the 10th term from the end is 108.
Q. 26. The sum of n terms of an A.P. is 3n2 + 5n. Find the A.P. Hence, find its 16th term.
(CBSE 2012)
Sol. We have,
Sn = 3n2 + 5n
S1 = 3 (1)2 + 5 (1)
= 3+5=8
T1 = 8 a = 8
S2 = 3 (2)2 + 5 (2)
= 12 + 10 = 22
T2 = 22 8 = 14
Now d = T2 T1 = 14 8 = 6
An A.P. is given by,
a, (a + d), (a + 2d), .....
The required A.P. is:
8, (8 + 6), [8 + 2 (6)], .....
8, 14, 20, .....
Now, using Tn = a + (n 1) d, we hve
T16 = a + 15d
= 8 + 15 6 = 98
Thus, the 16th term of the A.P. is 98.
Q. 27. The sum of 4th and 8th terms of an A.P. is 24 and the sum of 6th and 10th terms is 44. Find
the first three terms of the A.P. (AI CBSE 2008)
Sol. Let the first term be a and the common difference be d.
Using Tn = a + (n 1) d, we have
T4 = a + 3d, T6 = a + 5d
T8 = a + 7d and T10 = a + 9d
Since T4 + T8 = 24
a + 3d + a + 7d = 24
2a + 10d = 24 a + 5d = 12 ...(1)
Also, T6 + T10 = 44
a + 5d + a + 9d = 44
2a + 14d = 44 a + 7d = 22 ...(2)
Subtracting (2) from (1), we get,
a + 7d a 5d = 22 12
2d = 10 d = 5

Arithmetic Progressions 137


Now from (1),
a + 5 (5) = 12
a + 25 = 12 a = 13
First term (T1) = a + 0 = 13
Second term (T2) = a+d
= 13 + 5 = 8
Third term T3 = a + 2d
= 13 + 10 = 3
Q. 28. In an A.P., the first term is 8, nth term is 33 and sum of first n terms is 123. Find n and d, the
common difference. (AI CBSE 2009)
Sol. Here,
First term T1 = 8 a = 8
nth term Tn = 33 = l
Sn = 123 [Given]
n
Using, Sn = [a + l], we have
2
n
Sn = [8 + 33]
2
n
123 = 41
2
123 2
n = = 6
41
Now, T6 = 33
a + 5d = 33
8 + 5d = 33
5d = 33 8 = 25
25
d = = 5
5
Thus, n = 6 and d = 5.
Q. 29. For what value of n are the nth terms of two A.P.s 63, 65, 67, ..... and 3, 10, 17, ..... equal?
[NCERT Exemplar (AI CBSE 2009)]
Sol. For the 1st A.P.
a = 63
d = 65 63 = 2
Tn = a + (n 1) d Tn = 63 + (n 1) 2
For the 2nd A.P.
a = 3
d = 10 3 = 7
Tn = a + (n 1) d Tn = 3 + (n 1) 7
[Tn of 1st A.P.] = [Tn of 2nd A.P.]
63 + (n 1) 2 = 3 + (n 1) 7

138 MathematicsX
63 3 + (n 1) 2 (n 1) 7
=
60 + (n 1) 2 (n 1) 7 0=
60 + (n 1) [2 7] 0=
60 + (n 1) ( 5) 0=
60
(n 1) = = 12
5
n = 12 + 1 = 13
Thus, the required value of n is 13.
Q. 30. If m times the mth term of an A.P. is equal to n times the nth term, find the (m + n)th term of
the A.P. [(AI CBSE 2008), (CBSE 2012)]
Sol. Let the first term (T1) = a and the common difference be d.
nth term = a + (n 1) d
And mth term = a + (m 1) d
Also, (m + n)th term = a + (m + n 1) d ...(1)
m (mth term) = n (nth term)
m [a + (m 1) d] = n [a + (n 1) d]
ma + m (m 1) d = na + n (n 1) d
ma + (m2 m) d na (n2 n) d = 0
ma - na + (m2 m) d (n2 n) d = 0
a [m n] + [m2 m n2 + n] d = 0
a [m n] + [(m2 n2) (m n)] d = 0
a [m n] + [(m + n) (m n) (m n)] d = 0
a [m n] + (m n) [m + n 1] d = 0
Dividing throughout by (m n), we have:
a + [m + n 1] d = 0
a + [(m + n) 1] d = 0 ...(2)
(m + n) th term = 0 [From (1) and (2)]
Q. 31. In an A.P., the first term is 25, nth term is 17 and sum of first n terms is 60. Find n and d,
the common difference. (AI CBSE 2008)
Sol. Here, the first term a = 25
And the nth term = 17 = l
Using Tn = a + (n 1) d, we have:
17 = 25 + (n 1) d
(n 1) d = 17 25 = 42

(n 1) d = 42 = d =
LM 42 OP ...(1)
Nn 1 Q
n
Also, Sn = [a + l]
2
n
60 = [25 + ( 17)]
2
n
60 = 8
2
60
60 = 4n n = = 15
4
Arithmetic Progressions 139
From (1), we have
42 42
d = = = 3
15 1 14
Thus, n = 15 and d = 3
Q. 32. In an A.P., the first term is 22, nth term is 11 and sum of first n terms is 66. Find n and d,
the common difference. (AI CBSE 2008)
Sol. We have
1st term (T1) = 22 a = 22
Last term (Tn) = 11 l = 11
n
Using, Sn = [a + l], we have:
2
n
66 = [22 + ( 11)]
2
66 2 = n [11]
66 2
n = = 12
11
Again using
Tn = a + (n 1) d
We have:
T12 = 22 + (12 1) d
11 = 22 + 11d [ nth term = 11]
11d = 22 11 = 33
33
d = = 3
11
Thus, n = 12 and d = 3

III. HOTS QUESTIONS

Q. 1. Find the 6th term of the A.P. :


2m + 1 2m 1 2m 3
, , , ....
m m m
2m + 1 2m 1
Sol. Here, a1 = , a2 =
m m
d = a2 a1
2m 1 2 m + 1 2 m 1 2m 1
= =
m m m
( 2 )
=
m
Now, an = a + (n 1)d
2m + 1 2
an = + (n 1)
m m
140 MathematicsX
2m + 1 2n 2
= + 1
m m m
2m + 1 2n 2
= +
m m m
2m + 1 2 n + 2
=
m
2 m 2n + 3
=
m
2m 2 n + 3
Thus, the nth term is
m
Again, we have
2 m 2n + 3
an =
m
2m 2(6) + 3 2 m 12 + 3
a6 = =
m m
2m 9
=
m
2 m 9
i.e., the 6th term is
m
Q. 2. If the numbers a, b, c, d and e form an A.P., then find the value of a 4b + 6c 4d + e
Sol. We have the first term of A.P. as a.
Let D be the common difference of the given A.P.,
Then :
b = a + D, c = a + 2D, d= a + 3D and e = a + 4D
Q 2nd term = a + common difference
rd
3 term = a + 2 common difference etc.
a 4b + 6c 4d + e
= a 4(a + D ) + 6 (a + 2D) 4 (a + 3D) + (a + 4D)
= a 4a + 6a 4a + a 4D + 12D 12D + 4D
= 8a 8a + 16D 16D = 0
Thus, a 4b + 6c 4d + e = 0

an + 1 + b n + 1
Q. 3. If is the arithmetic mean between a and b, then, find the value of n.
an + b n
1
Sol. Note : A.M., between a and b = (a + b)
2
We know that :
a+b
A.M. between a and b =
2
It is given that,
an+ 1 + bn+ 1
is the A.M. between a and b
an + b n

Arithmetic Progressions 141


an + 1 + b n + 1 a + b
=
an + b n 2
By cross multiplication, we get :

2 a
n+1
+ b n + 1 = an + b n [ a + b ]
2an + 1 + 2bn + 1= an + 1 + abn + anb + bn + 1
2an + 1 an + 1 + 2bn + 1 bn + 1= abn + anb
an + 1 + bn + 1 = abn +anb
an+1 anb = abn bn+1
an[ab] = bn[a b]
an (a b)
=
bn (a b) = 1
0
an an
= n Q x0 = 1
bn b
n =0

1 1 1
Q. 4. If pth term of an A.P. is and qth term , prove that the sum of the first pq terms is pq + 1 .
q p 2

1 1
Hint: pth term = a + (p 1)d = ...(1)
q q
1 1
qth term = a + (q 1)d = p ...(2)
p
1 1
Solving (1) and (2), d = pq and q = pq

n
Using Sn =
2
[ 2 a + (n 1)d ] , we get :
pq 2 1 1
Spq = 2 pq + ( pq 1) pq Spq = 2 ( pq + 1)

1 1 1
Q. 5. If , , are in A.P., prove that a2, b2, c2 are also in A.P.
b+c c + a a+b

1 1 1 1
Hint: =
c+a b+c a+b c+a
[ using the fact, that in an A.P.
(2nd term 1st term) = (3rd term 2nd term)]

Q. 6. Solve the equation :


1 + 4 + 7 + 10 + ... + x = 287 [NCERT Exemplar]

142 MathematicsX
Sol. Since, 4 - 1 = 3

7 - 4 = 3 1, 4, 7, 10, ..., x form an A.P.
10 - 7 = 3
a = 1, d=3 and an = x
an = a + (n 1)d
x = 1 + (n 1) 3 or x = 3n 2
n
Also, (a + l)
Sn =
2
n
287 = (1 + x )
2
2(287) = n[1 + (3n 2)]
574 = n[3n 1]
3n2 n 574 = 0
Solving the above quadratic equation, we get
-( -1) 1 + 4 3 574 1 6888
n = =
6 6
1 83 -41
or n = n = 14 or
6 3
But, negative n is not desirable.
n = 14
x = 3n 2
Now, x = 3(14) 2 = 42 2 = 40
Thus, x = 40
Q. 7. Find three numbers in A.P. whose sum is 21 and their product is 231.
Sol. Let the three numbers in A.P. are:
a d, a, a+d
(a d) + a + (a + d) = 21
a d + a + a + d = 21
or 3a = 21 a=7
Also, (a d) a (a + d) = 231
(7 d) 7 (7 + d) = 231
(7 d) (7 + d) 7 = 231
231
72 d2 = = 33
7
49 d2 = 33
or d 2 = 49 33 = 16
d = 4
Now, when d = 4, then three numbers in AP are : (7 4), 7, (7 + 4) i.e. 3, 7, 11.
When d = 4, then three numbers in AP are : [7 (4)], 7, [7 + (4)]
or 11, 7, 3

Arithmetic Progressions 143


TEST YOUR SKILLS
1. Find the value of p if the numbers x, 2x + p, 3x + p are three successive terms of the AP.
2. Find p and q such that: 2p, 2p + q, p + 4q, 35 are in AP
3. Find a, b and c such that the following numbers are in A.P. :
a, 7, b, 23, c [NCERT Exemplar]

Hint:
7 a = b7 a + b = 14
23 b = b 7 2b = 30 b = 15
23 b = c 23 c + b = 46 c = 46 b
= 46 15
= 31
And a = 14 b = 14 15 = 1

4. Determine k so that k2 + 4k + 8, 2k2 + 3k + 6, 3k2 + 4k + 4 are three consecutive terms of


an AP. [NCERT Exemplar]
4 12
5. If , a, are three consecutive terms of an AP, find the value of a.
5 5

11
6. For what value of p, are (2p 1), 7 and p three consecutive terms of an AP?
2
7. If (x + 2), 2x, (2x + 4) are three consecutive terms of an AP, find the value of x.
(CBSE 2012)
8. For what value of p are (2p 1), 13 and (5p 10) are three consecutive terms of an A.P.?
9. Find the 10th term from the end of the A.P. 4, 9, 14, ... 254.
10. Find the 6th term of the AP 54, 51, 48...
11. Find the 8th term from the end of the AP : 7, 10, 13, ..., 184.
12. Find the 16th term of the AP 3, 5, 7, 9, 11, ...
13. Find the 12th term of the AP:
14, 9, 4, 1, 6, ...
14. Find the middle term of the AP :
20, 16, ..., 180
15. Find the 6th term from the end of the A.P.
17, 14, 11, ..., (40)
16. Find the middle term of the AP :
10, 7, 4, ..., (62)

144 MathematicsX
17. Which term of the AP : 24, 21, 18, 13, ... is the first negative term?

Hint: The first negative term will be the term immediately less than 0. i.e. Tn < 0.
Here , a = 24
[a + (n 1)d] < 0 d = (21 - 24) = -3
3n > 27 n>9 n = 10

18. The 6th term of an AP is 10 and its 10th term is 26. Determine the 15th term of the A.P.
19. For what value of n are the nth terms of the following two APs the same:
13, 19, 25, ... and 69, 68, 67, ....
20. The 8th term of an AP is zero. Prove that its 38th term is triple its 18th term.
Hint:
T8 = 0 a + 7d = 0 a = 7d
T38 = a + 37d = 7d + 37d = 30d
Also, T18 = a + 17d = 7d + 17d = 10d
30d = 3 (10d) T38 = 3 T18
21. For what value of n, the nth terms of the following two APs are equal?
23, 25, 27, 29, ... and 17, 10, 3, 4, ... [NCERT Exemplar]
22. Which term of the AP : 5, 15, 25, ... will be 130 more than 31st term?
Hint: Let an be the required term
i.e. an be 130 more than a31
an a31 = 130

23. Which term of the AP : 3, 15, 27, 39, ... will be 130 120 more than its 64th term?
24. The 9th term of an AP is 499 and its 499th term is 9. Which of its term is equal to zero.
25. Determine A.P. whose fourth term is 18 and the difference of the ninth term from fifteenth
term is 30.
26. How many natural numbers are there between 200 and 500 which are divisible by 7?

Hint: 200 ... 203 ... 497 ... 500

Divisible by 7

a = 203, d = 7 and an = 497


a + (n 1) d = an 203 + (n 1) 7 = 497

27. How many multiples of 7 are there between 100 and 300?
28. Find the value of the middle term of the following A.P. : 11, 7, 3, ..., 49.
29. Find the value of the middle term of the following A.P. : 6, 2, 2, ..., 58.
30. How many two digit numbers are divisible by 3?
Hint: Here, a = 12, d = 3 and an = 99

Arithmetic Progressions 145


31. If the 9th term of an AP is zero, show that 29th term is double the 19th term.

Also, a + (9 - 1)d = 0
a29 a + ( 29 - 1)d a + 8d = 0
Hint: a19 = a + (19 - 1)d = 2
a = -8d
a + 28d
=2
a + 18d
-8d + 28d
=2
-8d + 18d
20d
=2 20d = 20d
10d
a29 = a19
32. If in an AP, the sum of its first ten terms is 80 and the sum of its next ten terms is 280.
Find the AP.
33. If in an A.P.
an = 20 and Sn = 399
then find n
Hint: an = a + (n 1)d (n 1)d = 19
n
Sn = [2a + (n - 1)d ] = 399
2
n
= [2(1) + 19] = 399 n = 38
2
34. Find the sum of all natural numbers from 1 to 100.
35. The first and last terms of an AP are 4 and 81 respectively. If the common difference is
7, how many terms are there in the A.P. and what is their sum?
36. How many terms of A.P.
a, 17, 25, ...
must be taken to get a sum of 450?
37. Find the sum of first hundred even natural numbers which are multiples of 5.
38. Find the sum of the first 30 positive integers divisible by 6.
39. Find the sum of those integers from 1 to 500 which are multiples of 2 or 5.
[NCERT Exemplar]

Hint: Multiples of 2 are : 2, 4, 6, 8, 10, 12, 14, 16, ..., 500.


Multiples of 5 are : 5, 10, 15, 20, 25, 30, ..., 500.
Multiples of 2 as well as 5 : 10, 20, 30, 40, ..., 500.
The required sum

= Sum of multiplies of 2 + Sum of multiples of 5 - Multiples of 2 as well as 5

40. If the nth term of an A.P. is 2n + 1, find Sn of the A.P.

146 MathematicsX
41. An A.P. consists of 37 terms. The sum of the three middle most terms is 225 and the sum
of the last three terms is 429. Find the A.P. [NCERT Exemplar]
42. If Sn denotes the sum of n-terms of A.P. whose common differences is d and first term is
a find:
Sn 2Sn1 + Sn2 (CBSE 2012)
Hint: an = Sn Sn1

43. If the ratio of 11th term to 18th term of an A.P. is 2 : 3. Find the ratio of the 5th term
to the 21st term and also the ratio of the sum of the first five terms to the sum of first 21
terms. (CBSE 2012)
44. If in an A.P. the first term is 2, the last term is 29 and sum of the terms is 155. Find the
common difference of the A.P.
5n 2 3n
45. The sum of n terms of an A.P. is + . Find the 20th term.
2 2
46. If Sn denotes the sum of first n terms of an A.P., prove that
S30 = 3(S20 S10) [Al CBSE Foreign 2014]

ANSWERS
Test Your Skills
1. p=0 2. p = 10, q = 5 3. a = 1, b = 15, c = 31 4. k = 0
5. a = 8/5 6. p=2 7. x =6 8. p = 5
9. 209 10. 69 11. 163 12. 33
13. 41 14. 80 15. 25 16. 26
17. n = 10 18. 46 19. n=9 21. n = 9
22. 44th 23. 74th 24. 508 25. 3, 8, 13, 18, ...
26. 43 27. 28 28. 17; 21 29. 26
30. 30 32. 1, 1, 3, 5, 7... 33. 38
34. 5050 35. 12, 510 36. 10 37. 50500
38. 2790 39. 27250 40. n(n + 2) 41. 3, 7, 11, 15, ...
42. d 43. 1 : 3; 5 : 49 44. d =3 45. 99

Arithmetic Progressions 147


[Unit III: Geometry (Contd.)]

10 Circles

Facts that Matter


z Tangent to a Circle
A tangent to a circle is a line that touches the circle at only one point.

NOTE:
I. There is only one tangent at a point of the circle.
II. The tangent to a circle is a special case of the secant, when the two end points of its corresponding
chord coincide.

Theorem 1
The tangent at any point of a circle is perpendicular to the radius, through the point of contact.
[AI CBSE 2014] [CBSE 2011]
Proof: We have the centre O of the given circle and XY is the
tangent to the circle at a point P.
Let us take a point Q on XY other than P. Join OQ.
O
Obviously, Q lies outside the circle.
i.e., OQ > OP
Since, all the points on XY, except P lies outside the circle.
X P Q Y
i.e., OP is smaller than all the distance of the point O from XY.
i.e., OP is the smallest distance of O from XY.
i.e., OP XY

NCERT TEXTBOOK QUESTIONS SOLVED


EXERCISE 10.1
Q. 1. How many tangents can a circle have?
Sol. A circle can have an infinite number of tangents.
Q. 2. Fill in the blanks:
(i) A tangent to a circle intersects it in .......... point(s).
(ii) A line intersecting a circle in two points is called a .......... .
(iii) A circle can have .......... parallel tangents at the most.
(iv) The common point of a tangent to a circle and the circle is called .......... .
Sol. (i) exactly one (ii) secant (iii) two (iv) point of contact.
148
Q. 3. A tangent PQ at a point P of a circle of radius 5 cm meets a line through the centre O at a point
Q so that OQ = 12 cm. Length of PQ is:
(A) 12 cm (B) 13 cm

(C) 8.5 cm (D) 119 cm

Sol. Since PQ = OQ 2 OP 2

= 12 2 5 2

= 144 25

= 119
The option (D) is correct.
Q. 4. Draw a circle and two lines parallel to a given line such that one is a tangent and the other a secant
to the circle.
Sol. We have the required figure.
Here, l is the given line and a circle with centre O is drawn.

The line PT is drawn which is parallel to l and tangent to the circle.


Also, AB is drawn parallel to line l and is a secant to the circle.

z Number of Tangents from a Point on a Circle


I. There is no tangent to a circle passing through a point lying inside the circle.
II. There is one and only one tangent to a circle passing through a point lying on the circle.
III. There are exactly two tangents to a circle through a point lying outside the circle.
Theorem 2
The lengths of tangents drawn from an external point to a circle are equal.
[NCERT Exemplar, (CBSE 2010, 2011, 2014, CBSE Foreign 2014)]
Given: We have a circle with centre O and a point P lying outside the circle. Two tangents PQ and
PR on the circle from P.
To Prove: PR = PQ
Construction: Join OP, OQ and OR
Proof: OQ is a radius and PQ is a tangent.
PQO = 90
Similarly, PRO = 90

Circles 149
Now, in right OQP and right ORP, we have:
OP = OP [Common]
R

P O

Q
OQ = OR [Radii of the same circle]
PQO = PRO [As Proved above]
OQP ORP [R.H.S.]
Their corresponding parts are equal.
PQ = PR

NCERT TEXTBOOK QUESTIONS SOLVED


EXERCISE 10.2
Q. 1. Choose the correct option:
From a point Q, the length of the tangent to a circle is 24 cm and the distance of Q from the centre
is 25 cm. The radius of the circle is
T
(A) 7 cm (B) 12 cm
m
(C) 15 cm (D) 24.5 cm 24 c
Sol. QT is a tangent to the circle at T and OT is
radius Q 25 cm O
Also, OQ = 25 cm and QT = 24 cm
Using Pythagoras theorem, we get
OQ2 = QT2 + OT2
OT2 = OQ2 QT2
= 252 242 = (25 24) (25 + 24)
= 1 49 = 49 = 72
OT = 7
Thus, the required radius is 7 cm.
The correct option is (A).
Q. 2. Choose the correct option:
T
In figure, if TP and TQ are the two tangents to a circle with centre P
O so that POQ = 110, then PTQ is equal to
(A) 60 (B) 70 110
(C) 80 (D) 90 Q
O
Sol. TQ and TP are tangents to a circle with centre O.
such that POQ = 110
OP PT and OQ QT
150 MathematicsX
OPT = 90 and OQT = 90
Now, in the quadrilateral TPOQ, we get
PTQ + 90 + 110 + 90 = 360
PTQ + 290 = 360
PTQ = 360 290 = 70
Thus, the correct option is (B).
Q. 3. Choose the correct option:
If tangents PA and PB from a point P to a circle with centre O are inclined to each other at angle
of 80, then POA is equal to
(A) 50 (B) 60 (C) 70 (D) 80
Sol. Since, O is the centre of the circle and two tangents from P to the circle are PA and PB.
OA AP and OB BP
OAP = OBP = 90 A
Now, in quadrilateral PAOB, we have:
APB + PAO + AOB + PBO = 360
80 + 90 + AOB + 90 = 360 P
260 + AOB = 360 O
AOB = 360 260
AOB = 100.
In rt OAP and rt OBP, we have B
OP = OP [Common]
OAP = OBP [Each = 90]
OA = OB [Radii of the same circle]
OAP OBP
Their corresponding parts are equal
POA = POB
1 1
POA = AOB = 100 = 50 .
2 2
Thus, the option (A) is correct.

Q. 4. Prove that the tangents drawn at the ends of a diameter of a circle are parallel.
[CBSE 2012, CBSE Foreign 2014]
Sol. In the figure, we have:
PQ is diameter of the given circle and O is its centre.
Let tangents AB and CD be drawn at the end points of the diameter PQ.
Since the tangent at a point to a circle is perpendicular to the radius through the point.
PQ AB APQ = 90
C Q D

A P B
Circles 151
And PQ CD PQD = 90
APQ = PQD
But they form a pair of alternate angles.
AB y CD.

Q. 5. Prove that the perpendicular at the point of contact to the tangent to a circle passes through the centre.
Sol. In the figure, the centre of the circle is O and tangent AB touches the circle at P.
If possible, let PQ be perpendicular to AB such that it is not passing through O.
Join OP.
Since tangent at a point to a circle is perpendicular to the radius through that point,
AB OP i.e. OPB = 90 ...(1)
But by construction,
AB PQ QPB = 90 ...(2)
From (1) and (2),
QPB = OPB
which is possible only when O and Q coincide.
Thus, the perpendicular at the point of contact
to the tangent passes through the centre.

Q. 6. The length of a tangent from a point A at distance 5 cm from the centre of the circle is 4 cm. Find
the radius of the circle.
Sol. The tangent to a circle is perpendicular to the radius through the point of contact.
OTA = 90
T
Now, in the right OTA, we have:
OP2 = OT2 + PT2 4 cm

5 2 = OT2 + 42 A
OT2 = 52 42 5 cm
O
2
OT = (5 4) (5 + 4)
OT2 = 1 9 = 9 = 32
OT = 3
Thus, the radius of the circle is 3 cm.
Q. 7. Two concentric circles are of radii 5 cm and 3 cm. Find the length of the chord of the larger circle
which touches the smaller circle.
Sol. In the figure, O is the common centre, of the given
concentric circles.
AB is a chord of the bigger circle such that it is a
tangent to the smaller circle at P.
O
Since OP is the radius of the smaller circle through 5 cm 3 cm
P, the point of contact, A B
P
OP AB
APB = 90
Also, a radius perpendicular to a chord bisects the chord.

152 MathematicsX
1
OP bisects AB AP = AB
2
Now, in right APO,
OA2 = AP2 OP2
52 = AP2 32
AP2 = 52 32
AP2 = (5 3) (5 + 3) = 2 8
AP2 = 16 = (4)2
AP = 4 cm
1
AB = 4 AB = 2 4 = 8 cm
2
Hence, the required length of the chord AB is 8 cm.
Q. 8. A quadrilateral ABCD is drawn to circumscribe a circle (see figure). Prove that:
AB + CD = AD + BC [CBSE (Foreign) 2014, CBSE 2012] (AI CBSE 2008 C)
Sol. Since the sides of quadrilateral ABCD, i.e., AB, BC, CD and DA
touch the circle at P, Q, R and S respectively, and the lengths of
two tangents to a circle from an external point are equal.
AP = AS
BP = BQ
DR = DS
CR = CQ
Adding them, we get
(AP + BP) + (CR + RD) = (BQ + QC) + (DS + SA)
AB + CD = BC + DA
which was to be proved.

Q. 9. In the figure, XY and XY are two parallel tangents to a circle with centre O and another tangent
AB with point of contact C intersecting XY at A and XY at B. Prove that AOB = 90.
X P Y
A

O
C

X Y
Q B
[CBSE 2012]
Sol. The tangents drawn to a circle from an external point are equal.
AP = AC
In PAO and AOC, we have:
AO = AO [Common]
OP = OC [Radii of the same circle]
AP = AC

Circles 153
PAO AOC [SSS Congruency]
PAO = CAO
PAC = 2 CAO ...(1)
Similarly CBQ = 2 CBO ...(2)
Again, we know that sum of internal angles on the same side of a transversal is 180.
PAC + CBQ = 180
2 CAO + 2 CBO = 180 [From (1) and (2)]
180
CAO + CBO = = 90 ...(3)
2
Also CAO + CBO + AOB = 180 [Sum of angles of a triangle]
90 + AOB = 180
AOB = 180 90
AOB = 90.
Q. 10. Prove that the angle between the two tangents drawn from an external point to a circle is
supplementary to the angle subtended by the line-segment joining the points of contact at the
centre.
Sol. Here, let PA and PB be two tangents drawn from an external point P to a circle
with centre O.
A

P B

Now, in right OAP and right OBP, we have


PA = PB [Tangents to circle from an external point P]
OA = OB [Radii of the same circle]
OP = OP [Common]
By SSS congruency,
OAP OBP
Their corresponding parts are equal.
OAA = OPB
And AOP = BOP
APB = 2 OPA and AOB = 2 AOP
But AOP = 90 OPA
2 AOP = 180 2 OPA
AOB = 180 APB
AOB + APB = 180.

154 MathematicsX
Q. 11. Prove that the parallelogram circumscribing a circle is a rhombus. (CBSE 2012, CBSE Delhi 2014)
Sol. We have ABCD, a parallelogram which circumscribes a
circle (i.e., its sides touch the circle) with centre O.
Since tangents to a circle from an external point are equal in length,
AP = AS
BP = BQ D

CR = CQ
DR = DS
S
Adding, we get
R
(AP + BP) + (CR + DR) = (AS + DS) + (BQ + CQ) A O
AB + CD = AD + BC C
P
But AB = CD [opposite sides of ABCD]
Q
and BC = AD
AB + CD = AD + BC 2 AB = 2 BC
AB = BC
Similarly AB = DA and DA = CD B
Thus, AB = BC = CD = AD
Hence ABCD is a rhombus.
A
Q. 12. A triangle ABC is drawn to circumscribe a circle of radius 4 cm
such that the segments BD and DC into which BC is divided
by the point of contact D are of lengths 8 cm and 6 cm
respectively (see figure). Find the sides AB and AC. E
F
[CBSE 2012] O
Sol. Here ABC subscribe the circle with centre O.
Also, radius = 4 cm C B
The sides BC, CA and AB touch the circle at D, E and 6 cm D 8 cm
F respectively.
BF = BD = 8 cm
CE = CD = 6 cm A
AF = AE = x cm (say) x cm
x cm
The sides of the triangle are:
14 cm, (x + 6) cm and (x + 8) cm
Perimeter of ABC
O 8 cm
= [14 + (x + 6) + (x + 8)] cm 6 cm

= [14 + 6 + 8 + 2x] cm
B
= 28 + 2x cm C
6 cm D 8 cm
Semi perimeter of ABC
1
S= [28 + 2x] cm = (14 + x) cm
2
S AB = (14 + x) (8 + x) = 6
S BC = (14 + x) (14) = x
S AC = (14 + x) (16 + x) = 8
Area of ABC = b gb gb
S S AB S BC S AC g = b14 + xga6faxfa8f cm 2

Circles 155
= b14 + xg 48x cm2 ...(1)
1
Now, ar ( OBC) = BC OD
2
1
= 14 4 [ OD = Radius]
2
= 28 cm2
1 1
ar ( OCA) = CA OE = (x + 6) 4
2 2
1
= 4 (x + 6) = (2x + 12) cm2
2
1 1
ar ( OAB) = AB OF = (x + 8) 4
2 2
= (2x + 16) cm2
ar ( ABC) = ar ( OBC) + ar ( OCA) + ar ( OAB)
= 28 cm2 + (2x + 12) cm2 + (2x + 16) cm2
= (28 + 12 + 16) + 4x cm2
= (56 + 4x) cm2 ...(2)
From (1) and (2), we have:
56 + 4x = b14 + xg 48x
4 [14 + x] = 4 b14 + x g 3 x

14 + x = b14 + xg 3x
Squaring both sides
(14 + x)2 = (14 + x) 3x
196 + x2 + 28x = 42x + 3x2
2x2 + 14x 196 = 0 x2 + 7x 98 = 0
(x 7) (x + 14) = 0
Either x 7 = 0 x = 7
or x + 14 = 0 x = (14)
But x = ( 14) is not required
x = 7 cm
Thus, AB = 8 + 7 = 15 cm
BC = 8 + 6 = 14 cm
CA = 6 + 7 = 13 cm.

Q. 13. Prove that opposite sides of a quadrilateral circumscribing a circle subtend supplementary angles
at the centre of the circle. [CBSE 2012]
Sol. We have a circle with centre O.
A quadrilateral ABCD is such that the sides AB, BC, CD and DA touch the circle at P, Q,
R and S respectively.
Let us join OP, OQ, OR and OS. We know that two tangents drawn from an external point
to a circle subtend equal angles at the centre.

156 MathematicsX
1 = 2
3 = 4
5 = 6 and 7 = 8 D

Also, the sum of all the angles around a point is 360.


1 + 2 + 3 + 4 + 5 + 6 + 7 + 8 = 360
2 [1 + 8 + 5 + 4] = 360
(1 + 8 + 5 + 4) = 180 (1)
And 2 [2 + 3 + 6 + 7] = 360
S
(2 + 3) + (6 + 7) = 180 (2)
Since, 2 + 3 = AOB 8
O
6 + 7 = COD 1 7
R
A
1 + 8 = AOD 2 6
5
4 + 5 = BOC 3 4

From (1) and (2), we have: P C


AOD + BOC = 180 and B
Q

AOB + COD = 180

MORE QUESTIONS SOLVED


I. VERY SHORT ANSWER TYPE QUESTIONS
Q. 1. In the adjoining figure, PA and PB are tangents from P to a circle with centre C. If APB = 40
then find ACB.
Sol. Since a tangent to a circle is perpendicular
to the radius through the point of contact,
1 = 90 and 2 = 90
Now, in quadrilateral APBC, we have: A
1 + ACB + 2 + P = 360 1
90 + ACB + 90 + 40 = 360
C
ACB + 220 = 360 40 P

ACB = 360 220 2


= 140. B

Q. 2. In the given figure, PA and PB are tangents from P to a circle with centre O. If AOB = 130,
then find APB.
Sol. Since a tangent to a circle is
perpendicular to the radius through A
the point of contact, 1
1 = 2 = 90
O 130
Now, in quadrilateral AOBP, we have: P

1 + AOB + 2 + APB = 360


2
90 + 130 + 90 + APB = 360 B

Circles 157
310 + APB = 360
APB = 360 310 = 50
Thus, APB = 50.
Q. 3. In the given figure, PT is a tangent to a circle whose centre is O. If PT = 12 cm and PO = 13
cm then find the radius of the circle.
T
Sol. Since a tangent to a circle is perpendicular
to the radius through the point of contact, r
12 cm
OTP = 90 O
In rt OTP, using Pythagoras theorem, we get 13 c
m
OP2 = OT2 + PT2 P
2
13 = OT + 122 2

OT2 = 132 122 = (13 12) (13 + 12) = 1 25 = 25


OT2 = 5 2
OT = 5
Thus, radius (r) = 5 cm.
Q. 4. In the given figure, PT is a tangent to the circle and O is its centre. Find OP.
T

8 cm 15
cm

O
P

Sol. Since, a tangent to a circle is perpendicular to the radius through the point of contact.
OTP = 90
In right OTP, using Pythagoras theorem, we get
OP2 = OT2 + PT2
= 82 + 152 = 64 + 225 = 289 = 172
OP = 17 2 = 17 cm.
Q. 5. If O is the centre of the circle, then find the length of the tangent AB in the given figure.
A

6 cm

O B
10 cm

Sol. A tangent to a circle is perpendicular to the radius through the point of contact.
OAB = 90
Now, in right OAB, we have
OB2 = OA2 + AB2
10 2 = 62 + AB2
AB2 = 102 62 = (10 6) (10 + 6) = 4 16 = 64 = 82
AB = 82 = 8 .
158 MathematicsX
Q. 6. In the figure, PA is a tangent from an external point P to a circle with centre O. If POB = 115
then find APO. (AI CBSE 2009 C)
A

P O

B
Sol. Here, PA is a tangent and OA is radius. Also, a radius through the point of contact is
perpendicular to the tangent.
OA = PA
PAO = 90
In OAP, POB is an external angle,
APO + PAO = POB
APO + 90 = 115
APO = 115 90 = 25

Q. 7. In the following figure, PA and PB are tangents drawn from a point P to the circle with centre
O. If APB = 60, then what is AOB? (CBSE 2009 C)
A
P

Sol. The radius of the circle through the point of contact is perpendicular to the tangent.
OA AP and OB BP
PAO = PBO = 90
Now, in quadrilateral OAPB,
OAP + APB + PBO + AOB = 360
90 + 60 + 90 + AOB = 360
AOB + 240 = 360
AOB = 360 240 = 120

Q. 8. In the figure, CP and CQ are tangents to a circle with centre O. ARB is another tangent touching
the circle at R. If QC = 11 cm, BC = 7 cm then find, the length of BR. (CBSE 2009)
P
A

O R C

B
Q

Circles 159
Sol. Tangents drawn from an external point are equal,
BQ = BR ...(1)
And CQ = CP
Since, BC + BQ = QC
7 + BR = 11 [ BQ = BR]
BR = 11 7 = 4 cm.
Q. 9. In the figure, ABC is circumscribing a circle. Find the length of BC. (AI CBSE 2009)
A

4 cm
11
Q cm
R
3 cm

B P C
Sol. Since tangents drawn from an external point to the circle are equal,
AR = AQ = 4 cm ...(1)
BR = BP = 3 cm ...(2)
PC = QC ...(3)
QC = AC AQ
= 11 4 = 7 cm [From (1)]
BC = BP + PC [From (3)]
= 3 + QC
= (3 + 7) cm = 10 cm
Q. 10. In the figure, if ATO = 40, find AOB. [AI CBSE 2008]
A

O
T

B
Sol. Since the tangent is perpendicular to the radius through the point of contact,
1 = 4 = 90
Also, OA = OB [Radii of the same circle]
OT = OT [Common]
OAT OBT [RHS congruency]
3 = 2

A
4

O 3
40
2 5
T

1
B

160 MathematicsX
Now, in OAT,
3 + 4 + 5 = 180
3 + 90 + 40 = 180
3 = 180 90 40 = 50
AOB = 50 + 50 = 100.

Q. 11. From a point P, the length of the tangent to a circle is 15 cm and distance of P from the centre
of the circle is 17 cm, then what is the radius of the circle? [CBSE 2008 C]

A
15 c
m
r
P
O 17 cm

Sol. Since radius is perpendicular to the tangent through the point of contact,
OA AP
OAP = 90
In rt OAP, we have:
OA2 + AP2 = OP2
r2 + (15)2 = (17)2
r2 = 172 152 = (17 15) (17 + 15) = 2 32 = 64
r = 64 = 8
Thus, radius = 8 cm.
Q. 12. The two tangents from an external point P to a circle with centre O are PA and PB. If
APB = 70, then what is the value of AOB? (AI CBSE 2008 C)
Sol. Since tangent is perpendicular to the radius through the point of contact.
1 = 2 = 90
In quadrilateral OABP,
AOB + 1 + 2 + APB = 360
AOB + 90 + 90 + 70 = 360

A
1

O 70 P

2
B

AOB + 250 = 360


AOB = 360 250 = 110
Circles 161
II. SHORT ANSWER TYPE QUESTIONS
Q. 1. Prove that the tangents drawn at the ends of a chord of a circle make equal angles with the chord.
[NCERT Exemplar]
M
Sol.

1
L C
O
2

N
Let NM be a chord of a circle with centre C.
Let the tangents at M and N meet at O.
Q OM is a tangent at M
\ OMC = 90 ...(1)
Similarly ONC = 90 ...(2)
Since, CM = CN [Radii of the same circle]
\ In D CMN, 1 = 2
From (1) and (2), we have
OMC 1 = ONC 2
OML = ONL
Thus, tangents make equal angles with the chord.
Q. 2. Two concentric circles have a common centre O. The chord AB to
the bigger circle touches the smaller circle at P. If OP = 3 cm and
AB = 8 cm then find the radius of the bigger circle.
Sol. AB touches the smaller circle at P.
O
OP AB OPA = 90
Now, AB is a chord of the bigger circle.
Since, the perpendicular from the centre to a chord, bisects A B
P
the chord,
P is the mid-point of AB
8
AP = = 4 cm
2
In right APO, we have
AO2 = OP2 + AP2
AO2 = 32 + 42
AO2 = 9 + 16 = 25 = 52

AO = 5 2 = 5 cm Q
Thus, the radius of the bigger circle is 5 cm.
Q. 3. In the given figure, O is the centre of the circle and
PQ is a tangent to it. If its circumference is 12
O
cm, then find the length of the tangent.
Sol. Circumference of the circle = 12 cm
10
2 r = 12 cm
[ r is the radius of the circle] P

162 MathematicsX
12
r = = 6 cm
2
Radius of the circle = 6 cm = OQ
Since a tangent to circle is perpendicular to the radius through the point of contact,
OQP = 90
Now, in rt OQP, we have:
OQ2 + QP2 = OP2
62 + QP2 = 10 2
QP2 = 102 62 = (10 6) (10 + 6) = 4 16 = 64 = 82
QP = 82 = 8
Thus, the length of the tangent is 8 cm.
Q. 4. Given two concentric circles of radii 10 cm and 6 cm. Find the
length of the chord of the larger circle which touches the other A
circle.
Sol. The chord AB touches the inner circle at P.
AB is tangent to the inner circle. 6m
O P
OP AB
[ O is the centre and OP is radius
through the point of contact P] 10 m

OPB = 90.
Now, in right OPB, we have: B

OP2 + PB2 = OB2


62 + PB2 = 10 2
PB2 = 102 62 = (10 6) (10 + 6)
PB2 = 4 16
PB2 = 64 = 82
PB = 8 2 = 8 cm
The radius perpendicular to a chord bisects the chord.
P is the mid-point of AB
AB = 2 PB = 2 8 = 16 cm.
Q. 5. Two tangents TP and TQ are drawn to a circle with centre O from an external point T. Prove that
PTQ = 2 OPQ. (CBSE Sample Paper 2011)
P

O T

Q
Sol. Tangent to a circle is perpendicular to the radius through the point of contact.
In quadrilateral. OPTQ,
OPT + OQT + POQ + PTQ = 360
or 90 + 90 + POQ + PTQ = 360
POQ + PTQ = 360 90 90 = 180 ...(1)
In OPQ, 1 + 2 + POQ = 180 ...(2)
Circles 163
Since OP = OQ [Radii of the same circle]
1 = 2 [Angles opposite to equal sides]
OPT = 90 = OQT
From (2), we have
1 + 1 + POQ = 180
2 1 + POQ = 180 ...(3)
From (1) and (3), we have
2 1 + POQ = POQ + PTQ
2 1 = PTQ
2 OPQ = PTQ.
Q. 6. In the figure, the incircle of ABC touches the sides BC, CA and AB at D, E and F respectively.
If AB = AC, prove that BD = CD.
Sol. Since the lengths of tangents drawn from an external point to a circle are equal,
We have AF = AE
A
BF = BD
CD = CE
Adding them, we get F E

(AF + BF) + CD = (AE + CE) + BD


AB + CD = AC + BD
But AB = AC B D C (Given)
CD = BD.
Q. 7. A circle is touching the side BC of a D ABC at P and touching AB and AC produced at Q and
R. Prove that:
1
AQ = (Perimeter of D ABC) [NCERT Exemplar CBSE 2011, 2012]
2
Sol. Since, the two tangents drawn to a circle from an
external point are equal. A

\ AQ = AR ...(1)
Similarly,
BQ = BP ...(2) P
B C
and CR = CP ...(3) Q R
Now, Perimeter of D ABC O

= AB + BC + AC
= AB + (BP + PC) + AC
= AB + (BQ + CR) + AC [From (2) and (3)]
= (AB + BQ) + (CR + AC)
= AQ + AR
= AQ + AQ [From (1)]
= 2AQ
1
AQ = (Perimeter D ABC)
2
164 MathematicsX
Q. 8. In two concentric circles, a chord of the larger circle touches the smaller circle. If the length of this
chord is 8 cm and the diameter of the smaller circle is 6 cm, then find the diameter of the larger
circle. (CBSE 2009 C)
Sol. Let the common centre be O. Let AB be the chord of
the larger circle.
C
AB = 8 cm
And CD is the diameter of the smaller circle i.e.,
CD = 6 cm O
1
OD = (6) = 3 cm
2 A
D
B
Join OA. D is the point of contact. 8 cm
OD AB
D is the mid point of AB
AD = 4 cm
Now, in right ADO, we have:
AO2 = AD2 + OD2
= 42 + 32 = 16 + 9 = 25 = 52
AO = 5 cm
2AO = 2(5 cm) = 10 cm
The diameter of the bigger circle is 10 cm.
Q. 9. In the following figure, PA and PB are two tangents drawn to a circle with centre O, from an
external point P such that PA = 5 cm and APB = 60. Find the length of chord AB.
(CBSE 2009 C)
A
5 cm

P 60 O

B
Sol. Since the tangents to a circle from an external point are equal,
PA = PB = 5 cm
In PAB, we have
PAB = PBA [ PA = PB]
PAB + PBA + APB = 180
PAB + PAB + 60 = 180
2 PAB + 60 = 180
2 PAB = 180 60 = 120
PAB = 60
Each angle of PAB is 60.
PAB is an equilateral triangle.
PA = PB = AB = 5 cm
Thus, AB = 5 cm
Circles 165
Q. 10. In the following figure, AB is a chord of length 9.6 cm of a circle with centre O and radius 6 cm.

The tangents at A and B intersect at P. Find the length PA. (AI CBSE 2009 C)
Sol.
A
x
6 cm

cm cm
4.8 4.8
y R O
P
6 cm

Join OB.
Let PA = x cm and PR = y cm
Since, OP is perpendicular bisector of AB
9.6
AR = BR = = 4.8 cm
2
Now, in rt OAR, we have:
OA2 = OR2 + AR2 [By Pythagoras theorem]
OR2 = OA2 AR2
= 62 (4.8)2 = (6 4.8) (6 + 4.8) = 1.2 10.8
= 12.96
OR = 3.6 cm.
Again, in right OAP,
OP2 = AP2 + OA2
OP2 = (AR2 + PR2) + OA2 [ AP2 = AR2 + PR2]
(y + 3.6)2 = (4.8)2 + y2 + 62
y2 + 12.96 + 7.2 y = 23.04 + y2 + 36
7.2 y = 46.08
46.08
y = = 6.4
7.2
PR = 6.4 cm
Now, AP2 = AP2 + PR2
= (4.8)2 + (6.4)2 = 23.04 + 40.96 = 64
AP = 64 = 8 cm

166 MathematicsX
Q. 11. Two tangents PA and PB are drawn to a circle with centre O from an external point P. Prove that
APB = 2OAB (CBSE 2009)
A
P

Sol. We have PA and PB, the tangents to the circle and O is the centre of the circle.
PA = PB
2 = 4 ...(1)

A
2
1
O
P 3
4
B

Since the tangent is perpendicular to the radius through the point of contact,
OAP = 90
1 + 2 = 90 ...(2)
2 = 90 1
Now, in ABP, we have:
2 + 3 + 4 = 180
2 + 3 + 2 = 180 [From (1)]
2 + 3 = 180
2 (90 1) + 3 = 180 [From (2)]
180 2 1 + 3 = 180
2 1 = 3 3 = 21
APB = 2OAB

Q. 12. ABC is an isosceles triangle, in which AB = AC, circumscribed about a circle. Show that BC is
bisected at the point of contact. [CBSE 2012]
Sol. We know that the tangents to a circle from an external point are equal.
A
AD = AF
Similarly,
BD = BE
and CE = CF D F

Since AB = AC [Given]
AB AD = AC AD
AB AD = AC AF [ AD = AF]
B E C

Circles 167
BD = CF ...(1)
But BF = BD and CF = CE
From (1), we have:
BE = CE
Q. 13. If a, b, c are the sides of a right triangle where c is hypotenuse, prove that the radius r of the circle
a+b c
which touches the sides of the triangle is given by r = [(NCERT Exemplar)
2
(CBSE 2012)]
Sol. Here, a, b and c are the sides of rt D ABC A
Such that BC = a, CA = b and AB = c
Let the circle touches the sides BC, CA, AB at
D, E and F respectively.
= AE = AF and BD = BF c
F b
Also, CE = CD = r
r
\ AF = b r O r
E
BF = a r r

Now, AB = c (AF + BF) = (b r) + (a r) B


D
a C
c = b + a 2r
2r = a + b c
a+b c
r=
2
Q. 14. In a right ABC, right angled at B, BC = 5 cm and AB = 12 cm. The circle is touching the sides
of ABC. Find the radius of the circle. [CBSE 2014]
Sol. Let the circle with centre O and radius r touches AB, BC and AC at P, Q, R, respectively.
Now,
AR = AP
AP = AB BP = (12 r) cm
AR = (12 r)cm
Similarly, CR = (5 r)cm
Now, using Pythagoras theorem in rt ABC, we have
AC2 = AB2 + BC2
AC2 = 122 + 52
AC = 13 cm
But AC = AR + CR
= (12 r) + (5 r)
(12 r) + (5 r) = 13 cm
17 2r = 13 cm
2 r = 17 13 = 4 cm
4
r= = 2 cm
2
Thus, the radius of the circle is 2 cm.
168 MathematicsX
Q. 15. Prove that the parallelogram circumscribing a circle is a rhombus.
[CBSE 2012] (CBSE Sample Paper 2011)
Sol. Since ABCD is a ygm
AB = CD
and AD = BC
Tangents from an external point to a circle are equal, R
D C
AP = AS
BP = BQ
RC = QC S Q
DR = DS
(AP + PB) + (RC + DR) = (AS + DS) + (BQ + QC)
A B
AB + CD = AD + BC P

2 AB = 2 AD AB = AD
AB = AD = CD = BC
i.e., ABCD is a rhombus.
Q. 16. In the following figure, OP is equal to diameter of the circle. Prove that ABP is an equilateral
triangle. (AI CBSE 2008)
A

O P

B A
Sol. Since the tangent is perpendicular to
5
the radius through the point of
contact, C 2
OAP = 90 O 1 P
3
Let us join AB and AC. 4
In right OAP, OP is the hypotenuse
and C is the mid point of OP. B
[ OP is a diameter of the circle (given)]
CA = CP = CO = Radius of the circle.
OAC is an equilateral triangle.
Since all angles in an equilateral triangle are 60,
1 = 60
Now, in OAP, we have
1 + OAP + 2 = 180
60 + 90 + 2 = 180

Circles 169
2 = 180 90 60 = 30
Since PA and PB make equal angles with OP,
2 = 3 3 = 30
APB = 2 + 3 = 30 + 30 = 60
Again, PA = PB.
In ABP, 4 = 5 [Angles opposite to equal sides are equal]
Now, in ABP,
4 + 5 + APB = 180
4 + 4 + APB = 180
24 + 60 = 180
24 = 180 60 = 120
120
4 = = 60
2
Since, 4 = 60
5 = 60 ABP is an equilateral .
APB = 60
Q. 17. Prove that the angle between the two tangents to a circle drawn from an external point is
supplementary to the angle subtended by the line segment joining the points of contact at the centre.
(CBSE 2008 C)
Or
Two tangents PA and PB are drawn from an external point P to a circle with centre O. Prove that
AOBP is a cyclic quadrilateral. (CBSE Sample Paper 2011)
Sol.

We have tangents PA and PB to the circle from the external point P. Since a tangent to
a circle is perpendicular to the radius through the point of contact,
2 = 90 and 4 = 90
Now, in quadrilateral OAPB,
1 + 2 + 3 + 4 = 360
1 + 90 + 3 + 90 = 360
1 + 3 = 360 90 90 = 180
i.e., 1 and 3 are supplementary angles.
AOB and APB are supplementary
AOBP is a cyclic quadrilateral.
Q. 18. Out of two concentric circles, the radius of the outer circle is 5 cm and the chord AC of length
8 cm is a tangent to the inner circle. Find the radius of the inner circle.
[NCERT Exemplar, CBSE (Foreign) 2014]

170 MathematicsX
O
m
5c
A C
L

Sol. Let the given chord AC of the larger circle touch the smaller circle at L.
Q AC is a tangent at L to the smaller circle with centre O
\ OL ^ AC
Also AC is a chord of the bigger circle
1
\ AL = AC
2
[Q A perpendicular from centre to a chord of the circle, divides the
chord into two equal parts.]
But AC = 8 cm
1
\ AL = (8 cm) = 4 cm.
2
Now, in rt. DOAL,
OL2 = OA2 AL2
or OL2 = 52 42
= (5 + 4) (5 4)
= 91=9
OL = 9 = 3 cm
Thus, the radius of the inner circle is 3 cm.

TEST YOUR SKILLS


1. In the following figure, two circles touch each other externally at C. Prove that the common
tangent at C bisects the other two common tangents.
P
E
Q

C
A B

F M
L
2. In the figure, if AB = AC, prove that BE = CE. [CBSE 2006]
A

D F

B E C

Circles 171
3. A point P is 13 cm from the centre of the circle. The length of the tangent drawn from P
to the circle is 12 cm. Find the radius of the circle.
4. A circle is touching the side BC of ABC at P and touching AB and AC produced at Q
and R respectively.
1
Prove that: AQ = (Perimeter of ABC)
2
5. The incircle of ABC touches the sides BC, CA and AB at D, E and F respectively.
Show that: [CBSE 2012]
AF + BD + CD = AE + BF + CE
1
= (Perimeter of ABC)
2
A

F
E

B C
D
6. Show that the tangents drawn at the end points of a diameter of a circle are parallel.
7. In the figure PQ is a chord of a circle and PT is the tangent at P such that
QPT = 60, Find PRQ. [NCERT Exemplar]

Hint: Q

OPQ = OQP = 30 POQ = 120 O


1
Also, PRQ = reflex POQ R
2
T
P

8. In the figure, PQL and PRM are tangents to the circle with centre O at the points Q and
R, respectively and S is a point on the circle such that SQL = 50 and SRM = 60. Find
the measure of QSR. [NCERT Exemplar]

L
Q
50

S P
O

60
R
M

Hint:
OQS = 90 50 = 40 = OQQ
ORS = 90 60 = 30 = OSR
QSR = OSQ + OSR = 40 + 30

172 MathematicsX
9. PQ is a chord of length 8 cm of a circle of radius 5 cm. The tangents at P and Q intersect
at a point T. Find the length of TP.
P
T
5 cm
8
cm O

Q
A D
10. In the following figure, l and m are two parallel l
tangents to a circle with centre F. DE is the
tangent segment between the two parallel
tangents touching the circle at C. Show that DFE F C
= 90.

m
R E
11. In the figure, two circles with centres A and B and radii 5 cm and 3 cm touching each
other internally. If the perpendicular bisector of segment AB, meets the bigger circle at P
and Q, find the length of PQ.

12. Two tangents making an angle of 120 with each other, are drawn to a circle of radius
6 cm. Show that the length of each tangent is 2 3 cm.
13. In the figure BOA is a diameter of a circle and the tangent at a point P meets BA extended
at T. If PBO = 30, then find PTA.
P

30
B T
O A

Hint:
Q BPA = 90
\ PAB = 60 = OPA
Since, OP ^ PT APT = 30 and PTA = 60 30.

14. In the given figure, ABC is a right D right angled at B such that BC = 6 cm and
AB = 8 cm. Find the radius of the circle.

Circles 173
A

B
R C
15. If from an external point B of a circle with centre O, two tangents BC and BD are drawn
such that DBC = 120 , prove that BC + BD = BO i.e., BO = 2 BC
[NCERT Exemplar, CBSE 2014]

1
O B
2
C

Hint:
DOC = 180 120 = 60 [Q 1 = 2 = 90]
rt D OBD @ DOBC
BOC = BOD = 30
BD 1
In rt D OBD, = sin 30 =
OB 2
1
\ BD = OB
2
1
Similarly, BC = OB
2
16. In the figure, XP and XQ are two tangents to the circle with centre O, drawn from an
external point X. ARB is another tangent touching the circle at R. Prove that XA + AR
= XB + BR. [AI. CBSE (Foreign) 2014
Hint: P
A
AP = AR and BQ = BR
XP = XA + AP XA + AR [Q AP = AR O R x
XQ = XB + BQ XB + BR [Q BQ = BR
B
XP = XQ gives XA + AR = XB + BR Q

17. Prove that the line segment joining the points of contact of two parallel tangents of a circle,
passes through its centre. [CBSE (Delhi) 2014]
Hint:
Let PAQ and RBS be two parallel tangents to circle with centre O.
Join OA, OB and draw OCyPQ

174 MathematicsX
PAyCO
PAO + COA = 180 [co-interior angles]
90 + COA = 180 COA = 90
Similarly, COB = 90
COA + COB = 90 + 90 = 180. Hence, AOB is a st. line passing through O (centre
of the circle).
P A Q

O
C

R B S

18. In the figure, AB is a chord of length 16cm, of a circle of radius 10cm. The tangents at
A and B intersect at a point P. Find the length of PA. [AI. CBSE. 2010, 2014]

Hint:
AB = 16 cm AL = BL = 8 cm
In OLB, OB2 = LB2 + OL2 [By Pythagoras Theorem]

102 = 82 + OL2 OL = 100 64 = 36 = 6


Let PL = x and PB = y, So that OP = (x + 6)cm
In PLB and OBP, we have:
PB2 = PL2 + BL2 and OP2 = OB2 + PB2 A
32
Substituting x and y and simplifying, we get x = cm O L
3 P

1600
y2 = x2 + 64 y2 = y = 40 3 B
9
40
Thus, PA = cm.
3

ANSWERS
Test Your Skills
3. 5 cm 7. 120 8. 70 13. 30 14. 2 cm

Circles 175
11 Constructions

Facts that Matter


We know that:
I. To divide a line segment in a given ratio m : n, we divide this segment into (m + n) equal parts.
Then we take m parts on one side and n on the other.
II. The idea of dividing a line segment in any ratio is used in construction of a triangle similar
to a given triangle, whose sides are in a given ratio with the corresponding sides of the
given triangle.
III. The scale-factor means the ratio of the sides of the triangle to be constructed with the
corresponding sides of the given triangle.

NCERT TEXTBOOK QUESTIONS SOLVED


EXERCISE 11.1
In each of the following, give the justification of the construction also:
Q. 1. Draw a line segment of length 7.6 cm and divide it in the ratio 5 : 8. Measure the two parts.
Sol. Steps of construction:
I. Draw a line segment AB = 7.6 cm.
II. Draw a ray AX making an acute angle with AB.
III. Mark 13 (8 + 5) equal points on AX, and mark them as X1, X2, X3, ........, X13.
IV. Join point X13 and B.
V. From point X5, draw X5C y X13B, which meets AB at C.
Thus, C divides AB in the ratio 5 : 8
On measuring the two parts, we get:
AC = 4.7 cm and BC = 2.9 cm
X
X13
X12
X11
X10
X9
X8
X7
X6
X5
X4
X3
X2
X1

A C B
7.6 cm

176
Justification:
In ABX13 and ACX5, we have
CX5 y BX 13
AC AX5 5
= =
CB X5 X13 8
AC : CB = 5 : 8.
Q. 2. Construct a triangle of sides 4 cm, 5 cm and 6 cm and then a triangle similar to it whose sides
2
are of the corresponding sides of the first triangle. [CBSE 2012]
3
Sol. Steps of construction:
I. Draw a ABC such that BC = 6
cm, AC = 5 cm and AB = 4 cm. A

II. Draw a ray BX making an acute


angle CBX. A

III. Mark three points X1, X2, X3 on 5 cm


BX such that BX1 = X1X2 = X2X3.
IV. Join X3 C. C
B C
V. Draw a line through X2 such that 6 cm

it is parallel to X3 C and meets


BC at C. X
1

VI. Draw a line through C parallel


X
to CA to intersect BA at A. 2

Thus, ABC is the required triangle. X


3

Justification: X
By construction, we have:
X3C y X2C
BX 2 BC
= [Using BPT]
X2X3 C C
BX 2 2
But =
X2X3 1
BC 2
=
C C 1
C C 1
=
BC 2
Adding, 1 to both sides, we get
C C 1
+1 = +1
BC 2
C C + BC 1+2
=
BC 2
Constructions 177
BC 3
=
BC 2
Now, in BCA and BCA
we have CA y C A
Using AA similarity, we have:
BCA ~ BCA


A B
=
A C
=
BC
.
LMeach equal to 2 OP
AB AC BC N 3Q
Q. 3. Construct a triangle with sides 5 cm, 6 cm and 7 cm and then another triangle whose sides are
7
of the corresponding sides of the first triangle.
5
Sol. Steps of construction: Z
A
I. Construct a ABC such that AB =
5 cm, BC = 7 cm and AC = 6 cm. A
II. Draw a ray BX such that CBX is
an acute angle.
III. Mark 7 points of X1, X2, X3, X4, X5, 5 cm 6 cm
X6 and X7 on BX such that
BX1 = X1X2 = X2X3 = X3X4 = X4X5
= X5X6 = X6X7 B Y
7 cm C C
IV. Join X5 to C. X
1
V. Draw a line through X7 intersecting X
2
X
BC (produced) at C such that 3
X
4
X5 C y X7 C X
5
X
VI. Draw a line through C parallel to 6
X
7
CA to intersect BA (produced) at
A. Thus, ABC is the required
triangle. X

Justification:
By construction, we have
CA y CA
Using AA similarity, ABC ~ A BC
A B A C BC
= =
AB AC BC
Also X7C y X5C [By construction]
BC BX 5
BX7C ~ BX5C =
BC BX 7
BX 5 5 BC 5 BC 7
= = or =
BX 7 7 BC 7 BC 5
A B A C BC 7
= = = .
AB AC BC 5

178 MathematicsX
Q. 4. Construct an isosceles triangle whose base is 8 cm and altitude 4 cm and then another triangle
1
whose sides are 1 times the corresponding sides of the isosceles triangle.
2
Sol. Steps of construction:
I. Draw BC = 8 cm
II. Draw the perpendicular A
bisector of BC which
intersects BC at D.
A
III. Mark a point A on the
above perpendicular such
that DA = 4 cm.
IV. Join AB and AC.
Thus, ABC is the required C
B
isosceles triangle. D C
X
V. Now, draw a ray BX such 1 X
2
that CBX is an acute X
3
angle. X
VI. On BX, mark three points
X1, X2 and X3 such that:
BX1 = X1X2 = X2X3
VII. Join X2 to C.
VIII. Draw a line through X3 parallel to X2 C and intersecting BC (extended) to C.
IX. Draw a line through C parallel to CA intersecting BA (extended) at A, thus,
ABC is the required triangle.
Justification:
We have CA y CA [By construction]
Using AA similarity, ABC ~ A BC
A B A C BC
= =
AB AC BC
Since, X3C y X2C [By construction]
BX3 C ~ BX2 C
BC BX 3
= [By BPT]
BC BX 2
BX 3 3
But =
BX 2 2
BC 3
=
BC 2
A B A C BC 3
Thus, = = = .
AB AC BC 2
Q. 5. Draw a triangle ABC with side BC = 6 cm, AB = 5 cm and ABC = 60. Then construct a triangle
3
whose sides are of the corresponding sides of the triangle ABC. [CBSE 2011, 2012]
4
Constructions 179
Sol. Steps of construction: A

I. Construct a ABC such that


BC = 6 cm, AB = 5 cm and A
ABC = 60.

m
5c
II. Draw a ray BX such that
CBX is an acute angle.
III. Mark four points X1, X2, X3 60
and X4 on BX such that B
6 cm C C
BX1 = X1X2 = X2X3 = X3X4 X
1
IV. Join X4C and draw X3C y X4C
X
such that C is on BC. 2

V. Also draw another line X


3
through C and parallel to CA
X
to intersect BA at A. 4

Thus, ABC is the required triangle. X

Justification:
By construction, we have:
X4C y X3C
BX 3 BC
= [By BPT]
BX 4 BC
BX 3 3
But = [By construction]
BX 4 4
BC 3
= ...(1)
BC 4
Now, we also have
CA y CA [By construction]
BC A ~ BCA [using AA similarity]
A B A C BC 3
= = = . [From (1)]
AB AC BC 4
Q. 6. Draw a triangle ABC with side BC = 7 cm, B = 45, A = 105. Then, construct a triangle
4
whose sides are times the corresponding sides of ABC.
3
Sol. Steps of construction:
I. Construct a ABC such that BC = 7 cm, B = 45 and A = 105.
II. Draw a ray BX making an acute angle CBX with BC.
III. On BX, mark four points X1, X2, X3 and X4 such that
BX1 = X1X2 = X2X3 = X3X4.
IV. Join X3 to C.
V. Draw X4C y X3C such that C lies on BC (extended).
VI. Draw a line through C parallel to CA intersecting the extended line segment BA at
A.
180 MathematicsX
Y

A
10
5

45 30 C
B C
7 cm
X
1
X
2
X
3
X
4

X
Thus, A BC is the required triangle.
Justification:
By construction, we have:
CA y CA
ABC ~ ABC [AA similarity]
A B A C BC
= = ...(1)
AB AC BC
Also, by construction,
X4 C y X3 C
BX4C ~ BX3C
BC BX 4
=
BC BX 3
BX 4 4
But =
BX 3 3
BC 4
= ...(2)
BC 3
From (1) and (2), we have:
A B A C BC 4
= = = .
AB AC BC 3
Q. 7. Draw a right triangle in which the sides (other than hypotenuse) are of lengths 4 cm and 3 cm.
5
Then construct another triangle whose sides are times the corresponding sides of the given
3
triangle.
Sol. Steps of construction:
I. Construct the right triangle ABC such that B = 90, BC = 4 cm and BA = 3 cm.
Constructions 181
II. Draw a ray BX such that an acute
angle CBX is formed.
III. Mark 5 points X1, X2, X3, X4 and X5 A
on BX such that
BX1 = X1X2 = X2X3 = X3X4 = X4X5.
IV. Join X3 to C. A
V. Draw a line through X5 parallel to

3 cm
X3 C, intersecting the extended line
segment BC at C.
VI. Draw another line through C
90
parallel to CA intersecting the B 4 cm C C
extended line segment BA at A. X1 X
2
X3 X
Thus, A BC is the required 4 X5
triangle.
Justification: X

By construction, we have:
CA y CA
ABC ~ A BC [By AA similartiy]
A B A C BC
= = ...(1)
AB AC BC
Also, X5C y X3C [By construction]
BX5 C ~ BX3 C
BC BX 5
=
BC BX 3
BX 5 5
But = ...(2)
BX 3 3
From (1) and (2) we get
A B A C BC 5
= = = .
AB AC BC 3

TANGENTS TO A CIRCLE

+ Remember:
I. If a point lies inside a circle, then there cannot be a tangent to the circle through this point.
II. If a point lies on the circle, then there is only one tangent to the circle at this point and it is
perpendicular to the radius through that point.
III. If the point lies outside the circle, there will be two tangents to the circle from this point.

NOTE:
(i) For drawing a tangent at a point of a circle, simply draw the radius through this point and draw
a line perpendicular to this radius through this point.
(ii) The two tangents to a circle from an external point are equal.

182 MathematicsX
Construction of tangents to a circle from a point outside it.
Steps of construction:
I. Let the centre of the circle be O
and P be a point outside the circle. A

II. Join O and P.


III. Bisect OP and let M be the mid
point of OP.
P O
IV. Taking M as centre and MP or M
MO as radius, draw a circle
intersecting the given circle at the
points A and B.
V. Join PA and PB. B

Thus, PA and PB are the required


two tangents.

NOTE:
In case, the centre of the circle is not known, then to locate its centre, we take any two non-parallel
chords and then find the point of intersection of their perpendicular bisectors.

NCERT TEXTBOOK QUESTIONS SOLVED


EXERCISE 11.2
Q. 1. In each of the following, give also the justification of the construction:
Draw a circle of radius 6 cm. From a point 10 cm away from its centre, construct the pair of
tangents to the circle and measure their lengths.
Sol. Steps of construction:
I. With O as centre and radius 6 cm, draw a circle.
II. Take a point P at 10 cm away from the centre.
III. Join O and P. A

IV. Bisect OP at M.
V. Taking M as centre and MP or MO
as radius, draw a circle. O

VI. Let the new circle intersects the


M
given circle at A and B.
VII. Join PA and PB.
B
Thus, PA and PB are the required
P
two tangents.
By measurement, we have:
PA = PB = 9.6 cm.

Constructions 183
Justification:
Join OA and OB
Since PO is a diameter.
OAP = 90 = OBP [Angles in a semicircle]
Also, OA and OB are radii of the same circle.
PA and PB are tangents to the circle.
Q. 2. Construct a tangent to a circle of radius 4 cm from a point on the concentric circle of radius 6 cm
and measure its length. Also verify the measurement by actual calculation.
Sol. Steps of construction:
I. Join PO and bisect it such that the mid
point of PO is represented by M.
II. Taking M as centre and OM or MP as A
radius, draw a circle such that this circle
intersects the circle (of radius 4 cm) at
A and B.
III. Join A and P. O
M
P
Thus, PA is the required tangent.
By measurement, we have:
PA = 4.5 cm
B
Justification:
Join OA such that
PAO = 90 [Angle in a semi-circle]
PA OA
OA is a radius of the inner circle.
PA has to be a tangent to the inner circle.
Q. 3. Draw a circle of radius 3 cm. Take two points P and Q on one of its extended diameters each at
a distance of 7 cm from its centre. Draw tangents to the circle from these two points P and Q.
Sol. Steps of construction:
I. Join P and O.
II. Bisect PO such that M be its mid-point.

A C

O
P Q
M N

B D

184 MathematicsX
III. Taking M as centre and MO as radius, draw a circle. Let it intersect the given circle
at A and B.
IV. Join PA and PB.
Thus, PA and PB are the two required tangents from P.
V. Now, join O and Q.
VI. Bisect OQ such that N is its mid point.
VII. Taking N as centre and NO as radius, draw a circle. Let it intersect the given circle
at C and D.
VIII. Join QC and QD.
Thus, QC and QD are the required tangents to the given circle.
Justification:
Join OA such that OAP = 90 [Angle in a semi-circle]
PA OA PA is a tangent.
Similarly, PB OA PB is a tangent
Now, join OC such that QCO = 90 [Angle in a semi-circle]
QC OC QC is a tangent.
Similarly, QD OC QD is a tangent.
Q. 4. Draw a pair of tangents to a circle of radius 5 cm which are inclined to each other at an angle
of 60.
Sol. Steps of construction:
I. With centre O and radius = 5 A
cm, draw a circle. 90 60
C

II. Draw an angle AOB = 120.


III. Draw a perpendicular on OA at
A.
IV. Draw another perpendicular on
120

OB at B. O

V. Let the two perpendiculars meet


at C. 90

CA and CB are the two required B


tangents to the given circle
which are inclined to each other
at 60.
Justification:
In a quadrilateral OACB, using angle sum property, we have:
120 + 90 + 90 + ACB = 360
300 + ACB = 360
ACB = 360 - 300 = 60.

Q. 5. Draw a line segment AB of length 8 cm. Taking A as centre, draw a circle of radius 4 cm and
taking B as centre, draw another circle of radius 3 cm. Construct tangents to each circle from the
centre of the other circle. [CBSE 2012]
Constructions 185
Sol. Steps of construction:
I. Bisect the line segment AB. Let its mid point be M.
II. With centre as M and MA (or MB) as radius, draw a circle such that it intersects the
circle with centre A at the points P and Q.
III. Join BP and BQ.
Thus, BP and BQ are the required two tangents from B to the circle with centre A.
IV. Let the circle with centre M, intersects the circle with centre B at R and S.
V. Join RA and SA.
Thus, RA and SA are the required two tangents from A to the circle with centre B.

P
R

A B
M

S
Q

Justification:
Let us join A and P.
APB = 90 [Angle in a semi circle]
BP AP
But AP is radius of the circle with centre A.
BP has to be a tangent to the circle with centre A.
Similarly, BQ has to be tangent to the circle with centre A.
Also, AR and AS have to be tangent to the circle with centre B.

Q. 6. Let ABC be a right triangle in which AB = 6 cm, BC = 8 cm and B = 90. BD is the


perpendicular from B on AC. The circle through B, C, D is drawn. Construct the tangents from
A to this circle. [CBSE 2012]
Sol. Steps of construction:
I. Join AO (O is the centre of the circle passing through B, C and D.)
II. Bisect AO. Let M be the mid point of AO.
III. Taking M as centre and MA as radius, draw a circle intersecting the given circle at
B and E.
IV. Join AB and AE. Thus, AB and AE are the required two tangents to the given circle
from A.

186 MathematicsX
Justification
Join OE, then AEO = 90 [Angle being in a semi circle]
AE OE.
But OE is a radius of the given circle.
AE has to be a tangent to the circle.
Similarly, AB is also a tangent to the given circle.

D
E

C
B O

Q. 7. Draw a circle with the help of a bangle. Take a point outside the circle. Construct the pair of
tangents from this point to the circle. [CBSE 2012]
Sol. Steps of construction:
I. Draw the given circle using
a bangle.
II. Take two non parallel chords
PQ and RS of this circle.
III. Draw the perpendicular
bisectors of PQ and RS such
that they intersect at O.
Therefore, O is the centre of
the given circle.
IV. Take a point P outside this
circle.
V. Join OP and bisect it. Let M
be the mid point of OP.
VI. Taking M as centre and OM
as radius, draw a circle. Let
it intersect the given circle at
A and B.
VII. Join PA and PB. Thus, PA and PB are the required two tangents.
Constructions 187
Justification:
Join OA and OB.
Since OAP = 90 [Angle in a semi-circle]
PA OA
Also OA is a radius.
PA has to be a tangent to the given circle.
Similarly, PB is also a tangent to the given circle.

MORE QUESTIONS SOLVED

I. SHORT ANSWER TYPE QUESTIONS


Q. 1. Draw a circle of diameter 6.4 cm. Then draw two tangents to the circle from a point P at a distance
6.4 cm from the centre of the circle. A

Sol. Steps of construction:


I. Draw a circle with centre O and radius
6.4
= cm or 3.2 cm.
2 P
M 3.2 cm
O
6.4 cm
II. Mark a point P outside the circle such
that OP = 6.4 cm.
III. Join OP.
IV. Bisect OP such that its mid point is at B
M.
V. With centre M and radius OM, draw a circle intersecting the given circle at A and B.
VI. Join PA and PB. Thus, PA and PB are the two tangents to the given circle.

Q. 2. Draw a circle of radius 3.4 cm. Draw two tangents to it inclined at an angle of 60 to each other:
[NCERT Exemplar]
Sol. Steps of construction:
I. Draw a circle with centre O and radius
as 3.4 cm.
A
P
II. Draw two radii OA and OB such that 90 60
AOB = 120.
3.4 cm

III. Draw perpendiculars at A and B such


that these perpendiculars meet at P.
12
0

Obviously, APB = 60. O


[Using Angle sum property of a 90
quadrilateral] B

IV. Thus, PA and PB are the required


tangents to the given circle.

188 MathematicsX
Q. 3. Draw ABC in which AB = 3.8 cm, B = 60 and median AD = 3.6 cm. Draw another triangle
FG 4 IJ AB.
ABC similar to the first such that AB =
H 3K C

Sol. Steps of construction: Y

I. Draw AB = 3.8 cm. C

II. Construct ABY = 60.


III. With centre A and radius as 3.6 cm mark a ray
to intersect BY at D.
IV. With centre D and radius BD, mark an arc to D
intersect BY at C.
cm
V. Join CA. Thus, ABC is a triangle. 3.
6

VI. Draw a ray AX, such that BAX is an acute


60
angle. A
B
B
3.8 cm
VII. Mark 4 points X1, X2, X3 and X4 such that
X
AX1 = X1X2 = X2 X3 = X3 X4. 1

VIII. Join X3B. X


2

IX. Through X4 draw X4B y X3B X


3
X. Through B draw B C y BC where C lies on AC
X X
(produced). 4

Thus, CAB is the required triangle.


Q. 4. Draw an equiliateral triangle of height 3.6 cm. Draw another triangle similar to it such that its
2
side is of the side of the first.
3
Sol. Steps at construction:
I. Draw a line segment RS.
Z
II. Mark a point Y on it.
III. Through Y, draw YZ RS A

IV. Mark a point A on YZ such that


YA = 3.6 cm A
3.6 cm

V. At A draw YAB = 30 such that the


point B is on RS.
VI. With centre A and radius = AB, mark
a point C on RS. C
VII. Join AC. R B Y C S

VIII. Draw a ray BX such that CBX is an X


1
acute angle. X
2
IX. Mark three points X1, X2, X3 such that
X
AX1 = X1X2 = X2 X3. 3

X. Join X3 and C. X
XI. Through X2 draw X2C y X3C.
XII. Through C draw CA y CA.
Thus, ABC is the required triangle.
Constructions 189
Q. 5. Draw an isosceles ABC, in which AB = AC = 5.6 cm and ABC = 60. Draw another ABC
FG 2 IJ
similar to ABC such that AB = H 3K AB.

Sol. Steps of Construction:


I. Draw a ray BD.
II. Through B, draw another ray BE such that DBE = 60.
III. Cut off BA = 5.6 cm. E
IV. With A as centre and A
radius 6 cm, mark an arc
intersecting BD at C.
A
V. Join A and C to get ABC.
VI. Draw a ray BX such that 5.6 cm
5.6 cm

CBX is an acute angle.


VII. Mark three point X1, X2
and X3 such that
B
BX1 = X1X2 = X2X3. C C
D

VIII. Join X3 and C. X1

IX. Through X2 draw X2C y X3C X2


X3
X. Through C, draw CA y CA
Thus, A BC is the required
triangle. X

Q. 6. Construct an isosceles triangle whose base is 9 cm and altitude is 5 cm. Then construct another
3
triangle whose sides are of the corresponding sides of the first isosceles triangle.
4
(CBSE 2009 C)
Sol. Steps of construction:
A
I. Construct a ABC such that
AB = AC, BC = 9 cm and
A
altitude AD = 5 cm.
II. Through B, draw a ray BX
such that CBX is an acute
angle.
III. Mark 4 equal points X1, X2,
X3 and X4 on BX. such that B C
C
BX1 = X1 X2 = X2 X3 = X3 X4
X
IV. Join X4 and C.
1

X
V. Through X3, draw X3C y X4C, 2

X
intersecting BC in C. 3

X
VI. Through C, draw CA y CA, 4

intersecting AB in A.
Thus, ABC is the required
triangle. X

190 MathematicsX
Q. 7. Draw a line segment AB of length 7 cm. Taking A as centre draw a circle of radius 3 cm and taking
B as centre, draw another circle of radius 2.5 cm. Construct tangents to each circle from the centre
of the other circle. [CBSE 2009 C.]
Sol. Steps of construction:
I. Draw a line segment AB = 7 cm
II. With centre A and radius 3 cm, draw a circle.

R
P

M
A B
7 cm

Q
S

III. With centre B and radius 2.5 cm, draw another circle.
IV. Bisect AB and let M be the mid point of AB.
V. With centre M and radius AM, draw a circle intersecting the two circles in P,Q and
R,S.
VI. Join AP, AQ, BR and BS.
Thus, AP, AQ, BR and BS are required tangents.
Q. 8. Construct a ABC in which BC = 6.5 cm, C
AB = 4.5 cm and ABC = 60. Construct a
triangle similar to this triangle whose sides are
3
of the corresponding sides of the ABC. C
4
(CBSE 2009) 6.5 cm

Sol. Steps of construction:


I. Construct the ABC such that
AB = 4.5 cm, B = 60 and
BC = 6.5 cm. 60
II. Construct an acute angle BAX. A
B
4.5 cm B
III. Mark 4 points X1, X2, X3 and X4 on
AX such that AX 1 = X1X2 = X2X3 X
1
=X3X4. X
2
IV. Join X4 and B.
X
V. Draw X3 B y BC, meeting AC at C. 3

X
Thus, CAB is the required . 4

Constructions 191
Q. 9. Draw a right triangle in which sides (other than hypotenuse) are of lenghts 8 cm and 6 cm. Then
3
construct another triangle whose sides are times the corresponding sides of the first triangle.
4
(AI CBSE 2009)
Sol. Steps of construction:
I. Draw a ABC such that AB = 8 cm, B = 90 and BC = 6 cm.
II. Construct an acute angle BAX.
III. Mark 4 points X1, X2, X3 and X4 on AX such that AX1 = X1X2, = X2X3 = X3X4.

A B
8 cm B
X1

X2

X3

X4

IV. Join X4 and B.


V. Draw X3B y X4B.
VI. Draw BC y BC.
Thus, ABC is the required rt .
Q. 10. Construct a ABC in which BC = 5 cm, CA = 6 cm and AB = 7 cm. Construct a ABC similar
7
to ABC, each of whose sides are times the corresponding sides of ABC.
5
Sol. Steps of construction:
I. Construct ABC such that:
BC = 5 cm, CA = 6 cm and AB = 7 cm.
II. Draw a ray BX such that CBX is an acute angle.
III. Mark 7 points X1, X2, .............. X7 such that:
BX1 = X1X2, = X2X3 = X3X4 = X4X5 = X5X6 = X6X7
IV. Join X7 and C.
V. Draw a line through X5 parallel to X7 C to meet BC extended at C.
VI. Through C, draw a line parallel to CA to meet BA extended at A.

192 MathematicsX
A

m
6 cm

7c
C
B C
5 cm
X
1
X
2
X
3
X
4
X
5
X
6
X X
7

Thus, A BC is the required triangle.


Q. 11. Construct a triangle with sides 4 cm, 5 cm and 7 cm. Then construct a triangle similar to it whose
2
sides are of the corresponding sides of the given triangle. (AI CBSE 2008 C)
3
Sol. Steps of construction:
I. Construct the ABC such that A

BC = 7 cm, CA = 5 cm and BA
= 4 cm. A

II. Draw a ray BX such that CBX


is an acute angle.
III. Mark three points X1, X2 and X3
B
on BX such that: C C

BX1 = X1 X2 = X2 X3
X
IV. Join X3 and C. 1

X
V. Draw X2C y X3C. 2

X
VI. Draw CA y CA 3

X
Thus, A BC is the required triangle.
Q. 12. Construct a ABC in which AB = 6.5 cm, B = 60 and BC = 5.5 cm. Also construct a triangle
3
ABC similar to ABC whose each side is times the corresponding side of the ABC.
2
(CBSE 2008)
Sol. Steps of construction:
I. Construct a ABC such that AB = 6.5 cm, B = 60 and BC = 5.5 cm.
II. Draw a ray AX making an acute angle BAX.
III. Mark three points X1, X2, X3 on the ray AX such that
AX1 = X1 X2 = X2 X3
IV. Join X2 and B.
V. Draw X3B y X2B such that B is a point on extended AB.

Constructions 193
VI. Join B C y BC such that C is a point on AC (extended).
Thus, C AB is the required triangle.
C

5.5 cm

A B
B
6.5 cm

X
1
X
2
X
3

Q. 13. Draw a ABC with side BC = 6 cm, AB = 5 cm and ABC = 60. Construct ABC similar
3
to ABC such that sides of ABC are of the corresponding sides of ABC.
4
(AI CBSE 2008)
Sol. Steps of construction: C

I. Construct the given ABC.


C
II. Draw a ray AX such that BAC is
6c

an acute angle.
m

III. Mark 4 points X1, X2, X3 and X4 on


AX such that
AX1 = X1 X2 = X2 X3 = X3 X4. A
5 cm 60
B
B
IV. Join X4 B.
X
V. Draw X3B y X4B 1
X
2
VI. Through B draw B C y BC. X
3
X
Thus, B AC is the required triangle. 4 X

Q. 14. Draw a circle of radius 3 cm. From a point P, 6 cm away from its centre, construct a pair of tangents
to the circle. Measure the lengths of the tangents. (AI CBSE F 2009)
Sol. Steps of construction:
I. Draw the given circle such that its A
centre is at O and radius = 3 cm.
II. Mark a point P such that
OP = 6 cm.
M
III. Bisect OP. Let M be the mid point O P

of OP.
IV. Taking M as centre and OM as
radius draw a circle intersecting
B
the given circle at A and B.
194 MathematicsX
V. Join PA and PB.
Thus, PA and PB are the required tangents to the given circle.
Q. 15. Construct a triangle whose perimeter is 13.5 cm and the ratio of the three sides is 2 : 3 : 4.
(CBSE 2012)
Sol. Steps of construction:
I. Draw a line PQ = 13.5 cm
II. At P, draw a ray PR making a convenient acute angle QPR with PQ.
III. On PR mark (2 + 3 + 4), 9 points at equal distances.
IV. Join Q and the mark 9.
V. Through the points 2 and 5 draw lines R

2-A and 5-B parallel to 9-Q. Let these lines 9


8
meet PQ at A and B respectively. 7
6
VI. With A as centre and radius = AP, draw 5
4
an arc. 3
2
VII. With B as centre and radius = BQ, draw 1
another arc which intersects the arc of
P
step VI at C. A B Q
VIII. Join CA and CB.
ABC is the required triangle. C

TEST YOUR SKILLS


1. Draw a line AB = 12 cm and divide it in the ratio 3 : 5. Measure the two parts.
(CBSE 2007)
2. Draw a rt. ABC, in which B = 90, BC = 5 cm, AB = 4 cm. Then construct another
5
A BC whose sides are times the corresponding sides of ABC. (AI CBSE 2008)
3
2
3. Construct a triangle similar to a given ABC such that each of its sides is rd of the
3
corresponding side of the ABC. It is given that AB = 4 cm, BC = 5 cm and AC = 6 cm.
Also write the steps of construction. (CBSE 2012)
4
4. Construct a triangle similar to a given triangle ABC with its sides th of the corresponding
5
sides of ABC. Also, write the steps of construction. (AI CBSE 2006)
5. Draw a circle of radius 3 cm. Take a point at a distance of 5 cm from the centre of the
circle. Measure the length of each tangent. (CBSE 2006 C)
6. Divide a line segment of 7 cm internally in the ratio 2 : 3. (AI CBSE 2006 C)
7. Draw any triangle ABC. Construct another triangle ABC similar to the triangle ABC with
4
each side equal to th of the corresponding side of triangle ABC. (CBSE 2004)
5
8. Divide a line segment of length 6 cm internally in the ratio 3 : 2. (AI CBSE 2004)

Constructions 195
9. Divide a line segment of length AB = 6 cm into 2 : 3 internally. (AI CBSE 2004)
10. Draw a circle of radius 3.5 cm, from a point P outside the circle at a distance of 6 cm from
the centre of circle, draw two tangents to the circle. (AI CBSE 2005)
5
11. Construct a triangle similar to a given DABC with sides equal to of the corresponding
3
sides of DABC. (AI CBSE 2005)
12. Construct a DPQS such that PQ = 4.5 cm, PS = 4 cm and SQ = 5.4 cm. Construct another
triangle PQS similar to DPQS with side SQ = 7.2 cm. (CBSE 2005)
13. Construct a DABC in which AB = 6 cm, B = 60 and AC = 7 cm. Construct a D similar
4
to the DABC whose sides are of the corresponding sides. (AI CBSE 2005, CBSE 2012)
7
14. Draw a right triangle in which the sides (other than hypotenuse) are of lengths 4 cm and
3
3 cm. Then construct another triangle whose sides are times the corresponding sides
5
of the given triangle. [CBSE 2012]
15. Draw a right triangle ABC in which BC = 12 cm, AB = 5 cm and B = 90. Construct a
2
right D similar to it and of scale factor . (NCERT Exemplar)
3
16. Draw a right triangle in which the sides (other than hypotenuse) are of lengths 4 cm and
5
3 cm. Then construct a triangle similar to it and of scale factor .
3
17. Draw a triangle ABC with side BC = 7 cm, B = 45, A = 105. Then, construct a D whose
4
sides are times the corresponding sides of DABC. (CBSE 2011)
3
18. Construct a rhombus ABCD in which AB = 4 cm and ABC = 60. Divide it into two
triangles ABC and ADC. Construct the triangle ABC similar to DABC with the scale factor
2
. Draw a line segment CD parallel to CD, where D lies on AD. Is AB C D a rhombus?
3
Give reasons. (CBSE 2011, 2012)
19. Draw a circle of radius 1.5 cm. Take a point P outside it. Without using the centre, draw
two tangents to the circle from the point P. (CBSE 2011, 2012)
20. Draw a right triangle ABC in which AB = 6 cm, BC = 8 cm and B = 90. Draw BD
perpendicular from B on AC and draw a circle passing through the points B, C and D.
Construct tangents from A to this circle. [CBSE (Delhi) 2014]
21. Construct a triangle with sides are 5 cm, 5.5 cm and 6.5 cm. Now construct another triangle,
3
whose sides are times the corresponding sides of the given triangle. (AI CBSE 2014)
5
22. Construct a triangle ABC, in which AB = 5 cm, BC = 6cm and AC = 7cm. Then construct
3
another triangle whose sides are times the corresponding sides of ABC.
5
[AI CBSE (Foreign) 2014)

196 MathematicsX
[Unit IV: Trigonometry (Contd.)]

9
Some Applications of
Trigonometry

Facts that Matter


z Heights and Distances
1. Line of sight
A line drawn from the eye of the observer to the point in the object viewed by the observer
is called the line of sight.

ht
f Sig The point
eo viewed by
Lin the observer

90
B Object
Observer

2. Angle of elevation
The angle of elevation of the point ht
f Sig
viewed is the angle formed by the line eo
Lin
of sight with the horizontal when the
point being viewed is above the
horizontal-level. In the figure, AOB O q
is the angle of elevation. A
Observer Horizontal Level
3. Angle of depression
The angle of depression of the point viewed is the angle formed by the line of sight with the
horizontal level when the point being viewed is below the horizontal level. In the figure,
AOC is the angle of depression.
O
A
q Horizontal Level

observer Lin
eo
fS
igh
t

object
C

197
NOTE:
In case of an angle of elevation the observer has to raise his head to look at the object whereas
in case of an angle of depression he lowers his head to look at the point being viewed.

NCERT TEXTBOOK QUESTIONS SOLVED


EXERCISE 9.1
Q. 1. A circus artist is climbing a 20 m long rope, which is tightly stretched and tied from the top of
a vertical pole to the ground. Find the height of the pole, if the angle made by the rope with the
ground level is 30 (see figure).
Sol. In the figure, let AC is the rope and AB is the pole. In right ABC, we have:
AB A
= sin 30
AC
1 20 m
But sin 30 =
2
AB 1
=
AC 2 30
AB 1 B C
= [ AC = 20 m]
20 2
1
AB = 20 = 10 m
2
Thus, the required height of the pole is 10 m.

Q. 2. A tree breaks due to storm and the broken part bends so that the top of the tree touches the ground
making an angle 30 with it. The distance between the foot of the tree to the point where the top
touches the ground is 8 m. Find the height of the tree. [CBSE 2012]
Sol. Let the original height of the tree = OP.
It is broken at A and its top is touching the ground at B.
Now, in right AOB, we have
P
AO
= tan 30
OB
1
But tan 30 =
3
AO 1 A
=
OB 3
AO 1 8
= AO =
8 3 3
AO
Also, = sec 30 30
OB
B 8m O
AB 2 28 16
= AB = =
8 3 3 3
Now, height of the tree
OP = OA + AP = OA + AB

198 MathematicsX
8 16
= + [ AB = AP]
3 3
24 24 3
= m = m = 8 3m
3 3 3
Q. 3. A contractor plans to install two slides for the children to play in a park. For the children below
the age of 5 years, she prefers to have a slide whose top is at a height of 1.5 m, and is inclined
at an angle of 30 to the ground, whereas for older children, she wants to have a steep slide at
a height of 3 m, and inclined at an angle of 60 to the ground. What should be the length of the
slide in each case?
Sol. In the figure, DE is the slide for younger children whereas AC is the slide for older
children.
In right ABC,
AB = 3 m
AC = length of the slide
AB
= sin 60
AC
3 3
=
AC 2
23
AC = = 2 3m
3
A

D
3m

1.5m

60
30
E
B C

Again in right BDE,


DE
= cosec 30 = 2
BD
DE
= 2
1.5
DE = 2 1.5 m
DE = 3 m
Thus, the lengths of slides are 3 m and 2 3 m .

Q. 4. The angle of elevation of the top of a tower from a point on the ground, which is 30 m away from
the foot of the tower, is 30. Find the height of the tower.
Sol. In right ABC, AB = the height of the tower. The point C is 30 m away from
the foot of the tower,

Some Applications of Trigonometry 199


AC = 30 m B
AB
Now, = tan 30
AC
h 1 1
= [ tan 30 = ] h
30 3 3

30 30 3
h = = = 10 3
3 3 3 30
A
C 30 m
Thus, the required height of the tower is 10 3 m .

Q. 5. A kite is flying at a height of 60 m above the ground. The string attached to the kite is temporarily
tied to a point on the ground. The inclination of the string with the ground is 60. Find the
length of the string, assuming that there is no slack in the string.
Sol. Let in the right AOB,
OB = Length of the string
AB = 60 m = Height of the kite. B

OB 2
= cosec 60 =
AB 3
60 m
OB 2 2 60
= OB =
60 3 3

120 3 60 A
OB = = 40 3 O
3 3

Thus, length of the string is 40 3 m .

Q. 6. A 1.5 m tall boy is standing at some distance from a 30 m tall building. The angle of elevation
from his eyes to the top of the building increases from 30 to 60 as he walks towards the
building. Find the distance he walked towards the building.

30 m
B 30 C 60
D
1.5 m

x O

Sol. Here, OA is the building.


In right ABD,
AD 1
= tan 30 =
BD 3

200 MathematicsX
BD = AD 3 = 28.5 3 [ AD = 30 m 1.5 m
= 28.5 m]
Also, in right ACD,
AD
= tan 60 = 3
CD
AD 28.5
CD = =
3 3
28.5
Now, BC = BD CD = 28.5 3
3

BC = 28.5
LM 3 1 OP
N 3Q

= 28.5
LM 3 1 OP
N 3Q
2 3
= 28.5
3 3
28.5 2 3
=
3
= 9.5 2 3
= 19 3 m
Thus the distance walked by the man towards the building = 19 3 m.

Q. 7. From a point on the ground, the angles of elevation of the bottom and the top of a transmission
tower fixed at the top of a 20 m high building are 45 and 60 respectively. Find the height of
the tower. (CBSE 2010)
Sol. Let the height of the building be BC
BC = 20 m
And height of the tower be CD.
Let the point A be at a distance y metres from the top B of the building.
Now, in right ABC, D

BC
= tan 45 = 1
AB xm

20
y = 1 y = 20 m i.e., AB = 20 m. C

Now, in right ABD,


BD 20 m
= tan 60 = 3
AB
BD 45
= 3 60
20 A
B

Some Applications of Trigonometry 201


20 + x
= 3 20 + x = 20 3
20
x = 20 3 20 = 20 [ 3 1]
x = 20 [1.732 1]
x = 20 0.732 = 14.64
Thus, the height of the tower is 14.64 m.

Q. 8. A statue, 1.6 m tall, stands on the top of a pedestal. From a point on the ground, the angle of
elevation of the top of the statue is 60 and from the same point the angle of elevation of the top
of the pedestal is 45. Find the height of the pedestal. (CBSE 2012)
Sol. In the figure,
DC represents the statue.
BC represents the pedestal. D

Now in right ABC, we have


AB
= cot 45 = 1 1.6 m
BC
AB C
= 1 AB = h metres.
h
Now in right ABD, we get
h metra
BD
= tan 60 = 3 45
AB
60
A
BD = 3 AB = 3 h B

h + 1.6 = 3h
h + 1.6
= 3 h( 3 1) = 1.6
h
1.6 1.6 3 +1
h = =
3 1 3 1 3 +1
1.6
h = ( 3 + 1)
31
1.6
= 3 +1
2
= 0.8 ( 3 + 1) m
Thus, the height of the pedestal = 0.8 ( 3 + 1) m.

Q. 9. The angle of elevation of the top of a building from the foot of the tower is 30 and the angle of
elevation of the top of the tower from the foot of the building is 60. If the tower is 50 m high, find
the height of the building. (CBSE Delhi 2014)
Sol. In the figure, let height of the building = AB = h m
Let CD be the tower.

202 MathematicsX
CD = 50 m
Now, in right BAC,
AC
= cot 30 = 3
AB
D
AC
= 3 AC = h 3 ...(1)
h
Again, in right DCA,
B
DC 50 m
= tan 60
AC
hm
50 50
= 3 AC = ...(2)
AC 3 60 30
A C
From (1) and (2),
50
3h =
3

50 1 50
h = =
3 3 3

2
Thus, the height of the building = 16 m
3
Q. 10. Two poles of equal heights are standing opposite each other on either side of the road, which is
80 m wide. From a point between them on the road, the angles of elevation of the top of the poles
are 60 and 30 respectively. Find the height of the poles and the distances of the point from the
poles. (CBSE 2012)
Sol. Let AB and CD are the two poles such that:
AB = h metres
CD = h metres
B D

h h

60 30
A C
P
x metre (80x)m

80 m
Let P be the point on the road such that
AP = x m
CP = (80 x) m
Some Applications of Trigonometry 203
Now, in right APB, we have
AB
= tan 60
AP
h
= 3 h=x 3 ...(1)
x
Again in right CPD,
CD
= tan 30
CP
h 1
=
(80 x) 3
80 x
h = ...(2)
3
From (1) and (2), we get
80 x
3x =
3
3 3x = 80 x
3x = 80 x
3x + x = 80
4x = 80
80
x = = 20
4
80 x = 80 20 = 60
Now, from (1), we have:
h = 3 20 = 1.732 20
= 34.64
Thus, (i) The required point is 20 m away from the first pole and 60 m away from the
second pole.
(ii) Height of each pole = 34.64 m.

Q. 11. A TV tower stands vertically on a bank of a canal. From a point on the other bank directly opposite
the tower, the angle of elevation of the top of the tower is 60. From another point 20 m away from
this point on the line joining this point to the foot of the tower, the angle of elevation of the top
of the tower is 30 (see figure). Find the height of the tower and the width of the canal.
Sol. Let the TV Tower be AB = h m. A
Let the point C be such that
BC = x and CD = 20 m.
Now, in right ABC, we have:
AB
= tan 60 30 60
BC D
C
h 20m B
= 3 h= 3x ...(1)
x
In right ABD, we have:

204 MathematicsX
AB
= tan 30
BD
h 1
=
x + 20 3
x + 20
h = ...(2)
3
From (1) and (2), we get
x + 20
3x = 3x = x + 20
3
3x x = 20
20
2x = 20 x= = 10 m
2
Now, from (1), we get
h = 3 10 = 1.732 10 = 17.32
Thus, the height of the tower = 17.32 m.
Also width of the river = 10 m.
Q. 12. From the top of a 7 m high building, the angle of elevation of the top of a cable tower is 60 and
the angle of depression of its foot is 45. Determine the height of the tower.
Sol. In the figure, let AB be the height of the tower.
D
AB = 7 metres.
Let CD be the cable tower.
hm
In right DAE, we have
DE
= tan 60 60
EA A E
h 45
= 3
x
h = 3.x ...(1)
7m
Again, in right ABC,
AB
= tan 45
BC
7
= 1 B xm C
x
x = 7 ...(2)
From (1) and (2),
h = 7 3 = DE
CD = CE + ED
= 7 + 7 3 = 7 (1 + 3 ) m
= 7 (1 + 1.732) m = 7 2.732 m = 19.124 m
Thus, the height of the cable tower is 19.124 m.
Q. 13. As observed from the top of a 75 m high lighthouse from the sea-level, the angles of depression
of two ships are 30 and 45. If one ship is exactly behind the other on the same side of the
lighthouse, find the distance between the two ships.

Some Applications of Trigonometry 205


B
45
30

75 m

30 45 A
D C
Sol. In the figure, let AB represent the light house.
AB = 75 m.
Let the two ships be C and D such that angles of depression from B are 45 and 30
respectively.
Now in right ABC, we have:
AB
= tan 45
AC
75
= 1 AC = 75 ...(1)
AC
Again, in right ABD, we have:
AB
= tan 30
AD
75 1
= AD = 75 3 ...(2)
AD 3
Since the distance between the two ships = CD
= AD AC
= 75 3 75 = 75 [ 3 1]
= 75 [1.732 1] = 75 0.732 = 54.9
Thus, the required distance between the ships = 54.9 m.

Q. 14. A 1.2 m tall girl spots a balloon moving with the wind in a horizontal line at a height of
88.2 m from the ground. The angle of elevation of the balloon from the eyes of the girl at any
instant is 60. After some time, the angle of elevation reduces to 30 (see figure). Find the distance
travelled by the balloon during the interval. (AI CBSE 2009)

88.2 m

60
30

206 MathematicsX
Sol. In the figure, let C be the position of the observer (the girl).
A and P are two positions of the balloon.
CD is the horizontal line from the eyes of the (observer) girl.
Here PD = AB = 88.2 m 1.2 m = 87 m
In right ABC, we have
AB
= tan 60
BC
87 87
= 3 BC = m
BC 3
A P

88.2 m

60
30 B
C D
1.2 m

In right PDC, we have


PD
= tan 30
CD
87 1
= CD = 87 3
CD 3
Now, BD = CD BC
87
= 87 3
3

= 87
LM 3
1OP = 87 FG 3 1 IJ 2 87
N 3Q H 3K =
3
m

2 87 3 2 87 3
= = = 2 29 3 m
3 3 3

= 58 3 m
Thus, the required distance between the two positions of the balloon = 58 3 m

Q. 15. A straight highway leads to the foot of a tower. A man standing at the top of the tower observes
a car at an angle of depression of 30, which is approaching the foot of the tower with a uniform
speed. Six seconds later, the angle of depression of the car is found to be 60. Find the time taken
by the car to reach the foot of the tower from this point. (CBSE 2009)
Sol. In the figure, let AB is the height of the tower and C and D be the two positions of the
car.

Some Applications of Trigonometry 207


B
30 60

30 60
C D A

In right ABD, we have:


AB
= tan 60
AD
AB
= 3 AB = 3 AD ...(1)
AD
In right ABC, we have:
AB
= tan 30
AC
AB 1 AC
= AB = ...(2)
AC 3 3
From (1) and (2)
AC
3 AD =
3

AC = 3 3 AD = 3 AD
Now CD = AC AD
= 3 AD AD = 2 AD
Since the distance 2 AD is covered in 6 seconds,
6
The distance AD will be covered in i.e., 3 seconds.
2
Thus, the time taken by the car to reach the tower from D is 3 seconds.

Q. 16. The angles of elevation of the top of a tower from two points at a distance of 4 m and 9 m from
the base of the tower and in the same straight line with it are complementary. Prove that the
height of the tower is 6 m.
Sol. Let the tower be represented by AB in the figure.
Let AB = h metres.
In right ABC, we have:
208 MathematicsX
AB B
= tan
AC
h
= tan ...(1)
9
h
In right ABD, we have:
AB
= tan (90 ) = cot

q

AD

90
q
h C D A
= cot ...(2) 4m
4 9m
Multiplying (1) and (2), we get
h h
= tan cot = 1 [ tan cot = 1]
9 4

h2
= 1 h2 = 36
36
h = 6m
h = 6m [ Height is positive only]
Thus, the height of the tower is 6 m.

NCERT TEXTBOOK QUESTIONS SOLVED


EXERCISE 9.2

Q. 1. The angles of depression of the top and the bottom of a building 50 m high as observed from the
top of a tower are 30 and 60 respectively. Find the height of the tower and also the horizontal
distance between the building and the tower.
Sol. In the figure,

E
30
60
x

30 D
A

50 m 50m

60

B C

Some Applications of Trigonometry 209


Let AB = 50 m be the building.
Let CE be the tower such that CE = (50 + x) m
In right ADE, we have:
DE 1
= tan 30 =
AD 3
x 1
= AD x 3 or BC = x 3 ...(1)
AD 3
In right ACE, we have:
CE
= tan 60 = 3
BC
50 + x 50 + x
= 3 BC = ...(2)
BC 3
From (1) and (2), we get
50 + x
3x =
3
3 x 3 = 50 + x
3x x = 50 x = 25
Height of the tower = 50 + x
= 50 + 25
= 75 m
Now from (1), BC = 3 x
= 3 25 m
= 1.732 25 m
= 43.25 m
i.e., The horizontal distance between the building and the tower = 43.25 m.
Q. 2. The angle of elevation of the top of a tower as observed from a point on the ground is and
on moving a metres towards the tower, the angle of elevation is . Prove that the height of the
a tan tan
tower is .
tan tan
A
Sol. In the figure, let the tower be
represented by AB.
In right ABC, we have:
AB h
tan = =
BC x h
x tan = h
h
x = ...(1)
tan
Now, in right ABD, we have: a b
D
a C x B

210 MathematicsX
AB
= tan
BD
h
= tan
x+a
h = (x + a) tan
h = x tan + a tan
h h
h = tan + a tan [ x = from (1)]
tan tan

h tan + a tan . tan


h =
tan
h tan = h tan + a tan tan
h tan h tan = a tan tan
h (tan tan ) = a tan tan
a tan tan
h =
tan tan
D
Q. 3. A vertical tower stands on a horizontal plane and is
surmounted by a vertical flag staff of height 5 m. From
a point on the plane the angles of elevation of the bottom
and top of the flag staff are respectively 30 and 60. 5m
Find the height of the tower.
Sol. Let in the figure, BC be the tower such that
BC = y metres. C
CD be the flag staff such that
CD = 5 m
60 ym
BD = (y + 5) m.
In right ABC, we have:
30
A B
BC 1 xm
= tan 30 =
AB 3

y 1
= x= 3.y ...(1)
x 3
In right ABD, we have:
BD
= tan 60 = 3
AB
( y + 5)
= 3 y+5= 3x
x

y+5 = 3 ( 3 y) [x = 3 y from (1)]

Some Applications of Trigonometry 211


y + 5 = 3y
5
3y y = 5 y = = 2.5 m
2
The height of the tower = 2.5 m.

Q. 4. The length of the shadow of a tower standing on level plane is found to be 20 m longer when the
suns altitude is 30 than when it was 60. Find the height of the tower.
Sol. In the figure, let CD be the tower such that
CD = h metres
D
Also BC = x metres
In right BCD, we have:
CD
= tan 60 = 3
BC h
h h
= 3 x= ...(1)
x 3
In right ACD, we have: 30 60
A C
CD 1 20 m B x
= tan 30 =
AC 3
h 1
= 3 h = 20 + x
20 + x 3
h h
3 h = 20 + [From (1), x = ]
3 3
3 3 h = 20 3 + h

3h h = 20 3
20
2h = 20 3 3 = 10 3
h=
2
h = 10 1.732 = 17.32 m
Thus, the height of the tower = 17.32 m.
Q. 5. From the top of a hill 200 m high, the angles of depression of the top and bottom of a pillar are 30
and 60 respectively. Find the height of the pillar and its distance from the hill. [CBSE 2014]
Sol. In the figure, let AD is the hill such that A
30
AD = 200 m and CE is the pillar.
60
In right ADE, we have:
AD
= tan 60 = 3 30
DE C B
200 m
200
= 3
DE
200 200 3
DE = = 60
3 3 3 D
E
212 MathematicsX
3 200 1.73 200
DE = =
3 3
346
= = 115.33 m
3
Distance between pillar and hill = 115.33 m
200
Now, BC = DE = m [ DE = BC]
3
In right ABC, we have:
AB 1
= tan 30 =
BC 3
BC 200 1 200 200
AB = = = [ BC = ]
3 3 3 3 3
= 66.67 m
Height of the pillar
CE = AD AB [ CE = BD]
= 200 66.67 m
= 133.33 m
Q. 6. The angles of elevation of the top of a tower from two points on the ground at distances a and
b units from the base of the tower and in the same straight line with it are complementary. Prove
that the height of the tower is ab units.
Sol. In the figure, AB is the tower, such that:
A
AB = h
BD = b
BC = a
In right ABD, we have h

AB
= tan (90 )
BD q
q 90
h C B
= tan (90 ) D b
b a
h = b cot ...(1)
In right ABC, we have
AB
= tan
BC
h
= tan h = a tan ...(2)
a
Multiplying (1) and (2), we get
h h = b cot a tan
h2 = a b (cot tan )
h2 = ab [ cot tan = 1]

h = ab

Some Applications of Trigonometry 213


Q. 7. A pole 5 m high is fixed on the top of a tower. The angle of elevation of the top of the pole observed
from a point A on the ground is 60 and the angle of depression of the point A from the top
of the tower is 45. Find the height of the tower. [A.I. CBSE 2004]
Sol. In the figure, let BC be the tower and CD be the pole.
Let BC = x metres and AB = y metres
In right ABC, we get
BC D
= tan 45 = 1
AB
BC = AB y = x ... (1) 5m
In right ABD, we have:
BD C
= tan 60 = 3
AB
xm
x+5 60
= 3 45
y
A y B
y 3 = x+5

x 3 = x+5 [ x = y from (1)]

3xx = 5
( 3 1) x = 5

5 5 3 +1
x = =
3 1 3 1 3 +1

5 ( 3 + 1) 5 (1.732 + 1)
= =
31 2
5
= 2.732 m
2
= 5 1.366 m
= 6.83 m
Thus, the height of the tower = 6.83 m

MORE QUESTIONS SOLVED


I. SHORT ANSWER TYPE QUESTIONS
Q. 1. A tower stands vertically on the ground. From a point on the ground which is 20 m away from
the foot of the tower, the angle of elevation of the top of the tower is found to be 60. Find the
height of the tower. (CBSE 2010)
Sol. In the figure, AB is the tower,
AB = h metres
In rt ABC, we have:

214 MathematicsX
BC
= tan 60
AC
h
= 3
20
| tan 60 = 3 and AB = 20 m
h = 20 3 metre
Thus, the height of the tower = 20 3 m.

Q. 2. The angle of depression of the top and the bottom of a 9 m high building from the top of a tower
are 30 and 60 respectively. Find the height of the tower and the distance between the building
and the tower.
C
30
60

30 E
A

9m 9m

60

B D

Sol. Let AB represents the building and CD be the tower.


AB = 9 m
In right BDC, we have:
CD
= tan 60 = 3
DB
CD = DB 3 ...(1)
In right AEC, we have:
CE 1
= tan 30 =
AE 3
CD 9 1
= AE = 3 CD 9 3
AE 3
BD = 3 (DB 3 ) 9 3
BD = 3 BD 9 3
2BD = 9 3
9 9 1.732
BD = 3 =
2 2
BD = 7.8 m

Some Applications of Trigonometry 215


From (1), we have,
9 27
CD = 3
3 = = 13.5
2 2
Thus, height of the tower = 13.5 m
Distance between the building and the tower = 7.8 m

II. LONG ANSWER TYPE QUESTIONS


Q. 1. A boy whose eye level is 1.3 m from the ground, spots a balloon moving with the wind in a
horizontal level at some height from the ground. The angle of elevation of the balloon from the
eyes of the boy at any instant is 60. After 2 seconds, the angle of elevation reduces to 30. If
the speed of the wind at that moment is 29 3 m/s, then find the height of the balloon from
ground. (CBSE 2009 C)
E D

60

H
A 30 G

1.3 m 1.3 m

B F C

Sol. Let E and D be the two positions of the balloon.


Let AB be the position of the boy.
AB = 1.3 m
HF = CG = 1.3 m

Also speed of the wind = 29 3 m/s


Distance covered by the balloon in 2 seconds

= ED = HG = 2 29 3 m

= 58 3 m
AG = AH + HG

= AH + 58 3 m ...(1)
Now, in right AEH, we have
EH
= tan 60 = 3
AH

216 MathematicsX
EH
EH = AH 3 AH = ...(2)
3
In right AGD, we have
DG 1
= tan 30 =
AG 3
DG 1
= [From (1)]
( AH + 58 3 ) 3

3 DG = AH + 58 3
EH
3 DG = + 58 3 [From (2)]
3
3 3 DG =EH + 58 3 3
3 DG =EH + 3 58
3 DG =EH + 174
3 DG EH =174
2 DG =174 [ DG = EH]
174
DG = = 87 m
2
CD = DG + GC = (87 + 1.3) m
= 88.3 m
Thus, the height of the balloon = 88.3 m.

Q. 2. A statue, 1.5 m tall stands on the top of a pedestal. From a point on the ground, the angle of
elevation of the top of the statue is 45 and from the same point the angle of elevation of the top
of the pedestal is 30. Find the height of the pedestal from the ground. (CBSE 2012, 2009-C)
Sol. Let AB be the pedestal and AB = h
Let C be the point on the ground such that D
BC = x metres.
In right ACB, we have: 1.5 m
AB
= tan 30
BC
h 1 A
=
x 3
45 hm
x = 3 h ...(1)
In right DCB, we have: 30
C
BD B
= tan 45 x
BC
BD
= 1
x
AB + AD
= 1
x

Some Applications of Trigonometry 217


h + 1.5
= 1
x
h + 1.5 = x
h + 1.5 = 3h [From (1)]
3 h h = 1.5
h ( 3 1) = 1.5
1.5 3 +1
h =
3 1 3 +1
1.5 ( 3 + 1) 1.5 ( 3 + 1)
h = = m
31 2
h = 0.75 ( 3 + 1) m
Thus, the height of the pedestal = 0.75 ( 3 + 1) m.
Q. 3. The angles of depression of the top and battom of an 8 m tall building from the top of a multi-
storeyed building are 30 and 45, respectively. Find the height of the multi-storeyed building
and the distance between the two buildings. (CBSE 2009)
Sol. Let the multistoreyed building be AB.
AB = q metres
AD = (q 8) m [ BD = 8 m]
Let EC be the small building.
Now, in right ABC, we have: A

AB 30
45
= tan 45 = 1
BC
AB = BC
q = p ...(1) (q8)m

In right ADE, we have: q metres

AD 1 E 30
= tan 30 = D
DE 3 p metres

8m
3 AD = DE
45
3 (q 8) = p C p metres B

3q8 3 = q [From (1)]

3qq = 8 3
q ( 3 1) = 8 3

8 3
q =
3 1

8 3 3 +1
q = m
3 1 3 +1

218 MathematicsX
8 3 ( 3 + 1) 8 3 ( 3 + 1)
= m= m
e 3j 2
12 2

= 4 (3 + 3 ) m = 4 (3 + 1.732) m
= 18.928 m
Since p = q
p = 18.928 m
Distance between the two buildings = 18.928 m
Height of the multi-storeyed building = 18.928 m.
Q. 4. From the top of a building 60 m high, the angles of depression of the top and bottom of a vertical
lamp post are observed to be 30 and 60 respectively. Find:
(i) The horizontal distance between the building and the lamp post.
(ii) The height of the lamp post. [Take 3 = 1.732] (CBSE 2012)
Sol. In the figure, let CE be the building and AB be the lamp post
CE = 60 m
In right BCE, we have:
E
CE 30
= tan 60 = 3 60
BC
60
= 3
BC
60 60 3
BC = = m A
30
3 3 3 D

60 3 60 m
BC = = 20 3 m
3
In right ADE, we have:
DE 1
= tan 30 =
AD 3
DE 1
= 60
20 3 3
B C
[ BC = AD = 20 3 m]
20 3
DE = = 20 m
3
Height of the lamp post = AB = CD
= CE DE
= 60 m 20 m
= 40 m.
Also, the distances between the lamp post and the building
= 20 3 m
= 20 1.732 m [ 3 = 1.732]
= 34.64 m

Some Applications of Trigonometry 219


Q. 5. The angle of elevation of a cloud from a point h meters above the surface of a lake is q and the
angle of depression of its reflection in the lake is f. Prove that the height of the clouds above the
tan f + tan q
lake is h . [NCERT Exemplar]
tan f tan q
Sol. Let P be the cloud and Q be its reflection in the lake. As shown in the figure, let A be the
point of observation such that AB = h
Let the height of the cloud above the lake = x
Let AL = d
PL PN LN
From rt DPLA, tan q = =
AL AL
xh
tan q = ...(1)
d
x+h
similarly, tan f = ...(2)
d
P


A L

h
d
B
N

tan f x + h
From (1) and (2), =
tan q x h

2x tan f + tan q tan f + tan q


or = tan f tan q x = h tan f tan q
2h
Q. 6. From a point 100 m above a lake, the angle of elevation of a stationary helicopter is 30 and the
angle of depression of reflection of the helicopter in the lake is 60. Find the height of the helicopter.
(AI CBSE 2008 C)
Sol. In the figure, A is the stationary helicopter and F is its reflection in the lake.
In right AED, we have:
AE
tan 30 =
DE
220 MathematicsX
1
But tan 30 =
3
AE 1
=
DE 3 A
x 100 1 (x100)
=
y 3 m
x metres
D 30 E
x 100 y
y = ...(1)
3 100 m 100 m
EF
In right DEF, tan 60 =
DE
EF
= 3 x metres
DE
x + 100
= 3
y
3 y = x + 100 F

But y = 3 (x 100)
3 3 (x 100) = x + 100
3 (x 100) = x + 100
3x 300 x = 100
2x = 100 + 300
2x = 400
400
x = = 200
2
Thus, the height of the stationary helicopter = 200 m.

Q. 7. The angle of elevation of an aeroplane from a point on the ground is 60. After a flight of 15
seconds, the angle of elevation changes to 30. If the aeroplane is flying at a constant height of
1500 3 m, find the speed of the aeroplane. (AI CBSE 2008 C)
Sol. In the figure, let E and C be the two locations of the aeroplane.
Height BC = ED
= 1500 3 m E C
In right ABC, we have:
BC 1
= tan 30 =
AB 3
1500 3 m

1500 3 1
=
AB 3
60
AB = 3 1500 3 m
= 3 1500 m = 4500 m 30
In right ADE, we have: A D B

Some Applications of Trigonometry 221


ED
= tan 60 = 3
AD
1500 3
= 3 [ ED = BC]
AD
1500 3
AD = = 1500 m
3
Since the distance travelled in 15 seconds = AB AD
= 4500 1500 = 3000 m
Distance
Since, Speed =
Time
3000
Speed of the aeroplane = m s = 200 m/s.
15
Q. 8. A spherical balloon of radius r subtends an angle q at the eye of the observer. If the angle of elevation
of its centre is f, find the heights of centre of the balloon. [NCERT Exemplar]
Sol. In the figure, let O be the centre of the balloon, and A be the eye of the observer. r be the
radius.
\ OP = r and PAQ = q
Also, OAB = f
Let the height of the centre of the balloon be h OB = h.
In DOAP, OPA = 90 Q
q r
sin = , where OA = s ...(1) O
2 s
h
From, DOAB, sin f = ...(2) r
s
Now, from (1) and (2), h
P
q q
h 2
q

sin f s h s h f
2

q = r = sr = r A
B
sin
2 s
sin f
h = r q
sin
2
1 q
q Q = cose c
h = r . sin f . cosec q 2
2 sin
2
Q. 9. As observered from the top of a light house, 100 m high above sea level, the angle of depression
of a ship sailing directly towards it, changes from 30 to 60. Determine the distances travelled
by the ship during the period of observation. [Use 3 = 1.732] (AI CBSE 2004)
Sol. Let A represents the position of the observer such that
AB = 100 m

222 MathematicsX
In right ABC, we have
AB
= tan 60
BC
100
= 3 3 BC = 100
BC
100 3 100 100 3
BC = = =
3 3 3 3 A
30
100 1.732 60
= = 57.73 m
3
In right ABD, we have:

100 m
AB 1
= tan 30 =
BD 3
100 1
= 30 60
BD 3 D C B
BD = 3 100 = 1.732 100
BD = 173.2 m
The distance travelled
CD = BD BC
= (173.2 57.73) m = 115.47 m
Q. 10. The angles of elevation and depression of the top and the bottom of a tower from the top of a
building, 60m high, are 30 and 60 respectively. Find the difference between the heights of the
building and the tower and the distance between them. (CBSE Delhi 2014)
Sol. Let AB is building = 60 m and DC is the tower
DE 1 D
In rt. AED, = tan 30 =
x 3
x = 3 DE ...(1)

Tower
AB
In rt. ABC, = tan 60 = 3
BC
30 x
60 60 A E
= 3 x= ...(2) 60
x 3
60m

Substituting the value of x from (2) in (1), we have :


60 60 B 60 C
3 DE = 3 DE = 3 3 = 20
Difference between the heights of building and tower = 20 m
Distance between the tower and building
= x = 3 20 = 1.732 20m = 34.64m

TEST YOUR SKILLS


1. A person standing on the bank of a river observes that the angle of elevation of the top
of a tower standing on the opposite bank is 60. When he moves 40 m away from the bank,
he finds the angle of elevation to be 30. Find the height of the tower and the width of
the river. [Use 3 = 1.732] [CBSE 2008]

Some Applications of Trigonometry 223


2. A straight highway leads to the foot of a tower. A man standing at top of the tower
observes a car at angle of depression of 30, which is approaching the foot of the tower
with a uniform speed. Six seconds later, the angle of depression of the car is found to be
60. Find the time taken by the car to reach the foot of the tower from this point.
[AI CBSE 2008]
3. An aeroplane, when 3000 m high, passes vertically above another aeroplane at an instant,
when the angle of elevation of the two aeroplanes from the same point on the ground are
60 and 45 respectively. Find the vertical distance between the aeroplanes.

[Use 3 = 1.732] [CBSE 2011, 2012 CBSE 2008 F]


4. The angle of elevation of an aeroplane from a point A on the ground is 60. After a flight
of 30 seconds, the angle of elevation changes to 30. If the plane is flying at a constant
height of 3600 3 m, find the speed, in km/hr, of the plane. [CBSE 2008 F]
5. The angle of elevation of a jet fighter from a point A on the ground is 60. After a flight
of 10 seconds, the angle of elevation changes to 30. If the jet is flying at a speed of
648 km/hr, find the constant height at which the jet is flying [Use 3 = 1.732]
[CBSE 2012] [AI CBSE 2008]
6. The angle of elevation of a jet fighter from a point A on the ground is 60. After a flight
of 10 seconds, the angle of elevation changes to 30. If the jet is flying at a speed of
432 km/hr, find the constant height at which the jet is flying [Use 3 = 1.732]
[CBSE 2012] [AI CBSE 2008]
7. The angle of elevation of a jet fighter from a point A on the ground is 60. After a flight
of 15 seconds, the angle of elevation changes to 30. If the jet is flying at a speed of
720 km/hr, find the constant height at which the jet is flying. [use 3 = 1.732]
[AI CBSE 2008, 2014] [CBSE 2012]
8. A statue 1.46 m tall standing on the top of a pedestal. From a point on the ground, the
angle of elevation of the top of the statue is 60 and from the same point, the angle of
elevation of the top of the pedestal is 45. Find the height of the pedestal. [Use 3 = 1.73]
[CBSE 2008, 2012]
9. From the top of a house, h metres high from the ground, the angles of elevation and
depression of the top and bottom of a tower on the other side of the street are and
respectively. Prove that the height of the tower is h (1 + tan cot ). [AI CBSE, 2006, 2007]
10. A window in a building is at a height of 10 m from the ground. The angle of depression
of a point P on the ground from the window is 30. The angle of elevation of the top of
the building from the point P is 60. Find the height of the building. [AI CBSE 2007]
11. A pole 5 m high is fixed on the top of a tower. The angle of elevation of the top of the
pole observed from a point A on the ground is 60 and the angle of depression of point
A from the top of the tower is 45. Find the height of the tower. [Take 3 = 1.732]
[AI CBSE 2004, 2007]
12. A boy standing on a horizontal plane finds a bird flying at a distance of 100 m from him
at an elevation of 30. A gril standing on the roof of 20 m high building finds the angle
of elevation of the same bird to be 45. Both the boy and the girl are on opposite sides
of the bird. Find the distance of bird from the girl. [CBSE 2007]
224 MathematicsX
13. The angle of elevation of the top of a hill at the foot of the tower is 60 and the angle of
elevation of the top of the tower from the foot of the hill is 30. If the tower is 50 m high,
find the height of the hill. [AI CBSE 2006 C]
14. The angle of elevation of the top of a tower from a point on the same level as the foot of
the tower is 30. On advancing 150 metres towards the foot of the tower, the angle of
elevation becomes 60. Show that the height of the tower is 129.9 metres.
[Use 3 = 1.732] [CBSE 2006 C]
15. From a window 15 m high above the ground in a street, the angles of elevation and
depression of the top and foot of another house on the opposite side of the street are 30
and 45 respectively. Show that the height of the opposite house is 23.66 metres.

[Take 3 = 1.732] [CBSE 2006 C]


16. A man standing on the deck of a ship, which 10 m above the water level, observes the
angle of elevation of the top of a hill as 60 and the angle of depression of the base of
the hill as 30. Calculate the distance of the hill from the ship and the height of the hill.
[CBSE 2012] [AI CBSE 2006]
17. From a point A on a straight road the angle of elevation of the top of a vertical tower
situated on the roof of a vertical building on the sam road is . The angle of elevation of
the bottom of the tower from a point B on the road is again . The height of the building
is 50 m. If AB : BY is 2 : 5, where Y is the base of building, then show that the height of
the tower is 20 m.
18. The angle of elevation of the top of a tower 30 m high from the foot of another tower in
the same plane is 60 and the angle of elevation of the top of the second tower from the
foot of the first tower is 30. Find the distance between the two towers and also the height
of the other tower. [NCERT Exemplar]
19. An observer 1.5 m tall is 20.5 m away from a tower 22 m high. Determine the angle of
elevation of the top of the tower from the eye of the observer.
[NCERT Exemplar]
20. From a point on the ground the angles of elevation of the bottom and top of a transmission
tower fixed at the top of 20 m high building are 45 and 60. Find the height of the tower.
21. The angle of elevation of a cloud from a point 200 m, above a lake is 30 and the angle
of depression of the reflection of the cloud in the lake is 60. Find the height of the cloud.
[CBSE 2011, 2012]
22. A tree 12 m high is broken by the wind in such a way that its top touches the ground
and makes an angle of 60 with the ground. At what height from the bottom the tree is
broken by the wind? [CBSE 2011]
23. Two ships are there in the sea on either side of a lighthouse in such a way that the ships
and the lighthouse are in the same straight line. The angles of depression of two ships
as observed from the top of the lighthouse are 60 and 45. If the height of the lighthouse
is 200m, find the distance between the two ships. [Use 3 = 1.732] [CBSE 2014]

Hint:
In rt AMP, we have:
PM 200
= tan 60 = 3
AM x

Some Applications of Trigonometry 225


200 P
x= ...(1)
3 45 60
In rt BMP, we have :

Lighthouse
PM 200
= tan 45 y = 1 200 m
BM
y = 200 ...(2) 60 A
B 45
Solving (1) and (2), Ship-II M Ship-I
y x
we get required distance (x + y ) = 315.4 m

24. Two ships are approaching a ligh house from opposite directions. The angles of depression
of the two ships from the top of the light house are 30 and 45. If the distance between
the two ships is 100 m, find the height of the light house. [Use 3 = 1.732]
[AI. CBSE (Foreign) 2014]
25. The angle of elevation of the top of a tower at a distance of 120m from a point A on the
ground is 45. If the angle of elevation of the top of a flagstaff fixed at the top of the tower,
at A is 60, then find the height of the flagstaff. [Use 3 = 1.73] [AI. CBSE 2014]

ANSWERS
TEST YOUR SKILLS
1. 34.64 m; 20 m 2. 3 seconds 3. 1268 m 4. 864 km/h 5. 1558.8 m
6. 1039.2 m 7. 2598 m 8. 2 m 10. 30 m 11. 6.82 m 12. 30 2 m
13. 150 m 15. 23.6 m 16. 40 m; 10 2 m 18. 10 3 m , 10 m
19. 45 20. 20 ( 3 1) m 21. 400 m 22. 5.569 m. 23. 315.4 m
24. 36.6 m 25. 87.6 m

226 MathematicsX
[Unit V: Statistics and Probability]

15 Probability

Facts that Matter


z We know that experimental (or empirical) probabilities of events are based on the results of
actual experiments.
z If we perform the same experiment for different number of times, we get different data giving
different probability.
Number of trials in which event happened
z P(E) =
Total number of trials
where, P(E) is the experimental (or empirical probability).
Remember:
(i) The experimental or empirical probability of an event is based on what has actually happened
whereas the theoretical probability of the event tries to predict what will happen on the basis of certain
assumptions.
(ii) An experiment in which all possible outcomes are known in advance and the exact outcome of
specific event cannot be predicted in advance, is called a random experiment. The word random
means all outcomes have equal chances of occurrence.
(iii) Performing a random experiment is trial.
(iv) Die is a solid cube having six faces marked as 1, 2, 3, 4, 5, 6 respectively. Plural of die is dice.
(v) Events are said to be equally likely, if under the given condition, we cannot prefer one event to
another event.
(vi) Favourable outcomes are those which result in occurrence of an event in question.
(vii) A pack of playing cards consists of four suits: Spades (z), Hearts (y), Diamond (x) and Clubs (w).
Each suit has 13 cards ( total cards 52).
Kings, Queens and Jacks are called Face-cards.
( There are 12-face cards in a pack)
(viii) If we toss a coin, two outcomes are possible : Head or Tail.
(ix) Probability of an impossible event = 0.
(x) Probability of a sure event = 1.
(xi) Probability of a possible event can be between 0 and 1.

NCERT TEXTBOOK QUESTIONS SOLVED


EXERCISE 15.1

Q. 1. Complete the following statements:


(i) Probability of an event E + Probability of the event not E = .................. .
(ii) The probability of an event that cannot happen is ............... . Such an event is called
................ .

227
(iii) The probability of an event that is certain to happen is .............. . Such an event is called
................ .
(iv) The sum of the probabilities of all the elementary events of an experiment is .............. .
(v) The probability of an event is greater than or equal to .................. and less than or equal to
................ .
Sol. (i) Probability of an event E + Probability of the event not E = 1.
(ii) The probability of an event that cannot happen is 0. Such an event is called
impossible event.
(iii) The probability of an event that is certain to happen is 1. Such an event is called
sure or certain event.
(iv) The sum of the probabilities of all the elementary events of an experiment is 1.
(v) The probability of an event is greater than or equal to 0 and less than or equal to 1.
Q. 2. Which of the following experiments have equally likely outcomes? Explain.
(i) A driver attempts to start a car. The car starts or does not start.
(ii) A player attempts to shoot a basketball. She/he shoots or misses the shot.
(iii) A trial is made to answer a true-false question. The answer is right or wrong.
(iv) A baby is born. It is a boy or a girl.
Sol. (i) Since, the car may or may not start, thus the outcomes are not equally likely.
(ii) The player may shoot or miss the shot.
The outcomes are not equally likely.
(iii) In advance it is known that the answer is to be either right or wrong.
The outcomes right or wrong are equally likely to occur.
(iv) In advance it is known the newly born baby has to be either a boy or a girl.
The outcomes either a boy or a girl are equally likely to occur.
Q. 3. Why is tossing a coin considered to be a fair way of deciding which team should get the ball at
the beginning of a football game?
Sol. Since on tossing a coin, the outcomes head and tail are equally likely, the result of
tossing a coin is completely unpredictable and so it is a fair way.
Q. 4. Which of the following cannot be the probability of an event?
2
(A) (B) 1.5 (C) 15% (D) 0.7
3
Sol. Since, the probability of an event cannot be negative,
(B) 1.5 cannot be the probability of an event.
Q. 5. If P(E) = 0.05, what is the probability of not E?
Sol. P(E) + P(not E) = 1
0.05 + P(not E) = 1 P(not E) = 1 0.05
= 0.95
Thus, probability of not E = 0.95.
Q. 6. A bag contains lemon flavoured candies only. Malini takes out one candy without looking into
the bag. What is the probability that she takes out
(i) an orange flavoured candy?
(ii) a lemon flavoured candy?
Sol. (i) Since, there are lemon flavoured candies only in the bag,
Taking out any orange flavoured candy is not possible.
Probability of taking out an orange flavoured candy = 0.
(ii) Also, probability of taking out a lemon flavoured candy = 1.
Q. 7. It is given that in a group of 3 students, the probability of 2 students not having the same birthday
is 0.992. What is the probability that the 2 students have the same birthday?
228 MathematicsX
Sol. Let the probability of 2 students having same birthday = P(SB)
And the probability of 2 students not having the same birthday = P(nSB)
P(nSB) + P(nSB) = 1
P(SB) + 0.992 = 1
P(SB) = 1 0.992 = 0.008
So, the required probability of 2 boys having the same birthday = 0.008.
Q. 8. A bag contains 3 red balls and 5 black balls. A ball is drawn at random frm the bag. What is the
probability that the ball drawn is (i) red? (ii) not red? (CBSE 2012)
Sol. Total number of balls = 3 + 5 = 8
Number of all possible outcomes = 8
(i) For red balls:
There are 3 red balls.
Number of favourable outcomes = 3
Number of favourable outcomes
PRed =
Number of all possible outcomes
3
=
8
(ii) For not red balls:
Probability of the ball drawn which is not red
3 83 5
= 1 = = .
8 8 8
Q. 9. A box contains 5 red marbles, 8 white marbles and 4 green marbles. One marble is taken out of
the box at random. What is the probability that the marble taken out will be (i) red? and
(ii) white? (iii) not green?
Sol. Total number of marbles = 5 + 8 + 4 = 17
(i) For red marbles:
Number of red marbles = 5
Number of favourable outcomes = 5
5
Probability of red marbles, P(red) =
17
(ii) For white balls:
Number of white balls = 8
Probability of white balls,
8
P(white) =
17
(iii) For not green balls:
Number of white balls = 4
Number of not green balls = 17 4 = 13
i.e., Favourable outcomes = 13
Probability of ball not green
13
P(not green) =
17
OR
Number of green marbles = 4
Number of not green balls = 17 4 = 13
Favourable outcomes = 13
13
P(not green) =
17

Probability 229
Q. 10. A piggy bank contains hundred 50p coins, fifty Re 1 coins, twenty ` 2 coins and ten ` 5 coins.
If it is equally likely that one of the coins will fall out when the bank is turned upside down, what
is the probability that the coin (i) will be a 50p coin? and (ii) will not be ` 5 coin?
Sol. Number of:
50 p coins = 100
Re 1 coins = 50
` 2 coins = 20
` 5 coins = 10
Total number of coins = 100 + 50 + 20 + 5 = 180
(i) For a 50 p coin:
Favourable events = 100
100 5
P(50 p) = =
180 9
(ii) For not a ` 5 coin:
Number of ` 5 coins = 10
Number of not ` 5 coins = 180 10 = 170
Favourable outcomes = 170
170 17
P(not 5 rupee coin) = = .
180 18
Q. 11. Gopi buys a fish from a shop for his aquarium. The shopkeeper
takes out one fish at random from a tank containing 5 male
fish and 8 female fish (see Fig.). What is the probability that
the fish taken out is a male fish?
Sol. Number of:
Male fishes = 5
Female fishes = 8
Total number of fishes = 5 + 8 = 13
Total number of outcomes = 13
For a male fish:
Number of favourable outcomes = 5
5
P(male fish) = .
13
Q. 12. A game of chance consists of spinning an arrow which comes to rest pointing at one of the numbers
1, 2, 3, 4, 5, 6, 7, 8 (see figure), and these are equally likely outcomes. What
is the probability that it will point at 8 1
(i) 8? (ii) an odd number? 7 2
(iii) a number greater than 2? (iv) a number less than 9?
Sol. Total numbers marked = 8 6 3
(i) When pointer points at 8: 5 4
Total number of outcomes = 8
Number of favourable outcomes = 1
No. of favourable outcomes
P(8) =
Total number of possible outcomes
1
=
8
(ii) When pointer points at an odd number:
Number of odd numbers from 1 to 8 = 4 [ Odd numbers are 1, 3, 5 and 7]

230 MathematicsX
Number of favourable outcomes = 4
Number of favourable outcomes 4 1
P(odd) = = =
Total number of possible outcomes 8 2
(iii) When pointer points at a number greater than 2:
Number of numbers greater than 2 = 6
[ The numbers 2, 3, 4, 5, 6, 7 and 8 are greater than 2]
Number of favourable outcomes = 6
Number of favourable outcomes
P(greater than 2) =
Total number of possible outcomes
6 3
= = .
8 4
(iv) When pointer points a number less than 9:
Number of numbers less than 9 = 8
[ The numbers 1, 2, 3, 4, 5, 6, 7 and 8 are less than 9]
Number of favourable outcome = 8
Favourable outcomes
P(greater than 9) =
Total possible outcomes
8
= = 1.
8
Q. 13. A die is thrown once. Find the probability of getting:
(i) a prime number; (ii) a number lying between 2 and 6; (iii) an odd number.
Sol. Since, numbers on a die are 1, 2, 3, 4, 5, and 6.
Number of total outcomes = 6
(i) For prime numbers:
Since 2, 3, and 5 are prime number,
Favourable outcomes = 3
Number of favourable outcomes
P(prime) =
Total number of possible outcomes
3 1
= = .
6 2
(ii) For a number lying between 2 and 6:
Since the numbers between 2 and 6 are 3, 4 and 5
Favourable outcomes = 3
Favourable outcomes
Required probability =
Total number of possible outcomes
3 1
= =
6 2
(iii) For an odd number:
Since 1, 3 and 5 are odd numbers.
Favourable outcomes = 3
3 1
Required probability = = .
6 2

Probability 231
Q. 14. One card is drawn from a well-shuffled deck of 52 cards. Find the probability of getting:
(i) a king of red colour (ii) a face card (iii) a red face card
(iv) the jack of hearts (v) a spade (vi) the queen of diamonds
Sol. Number of cards in deck = 52
Total number of possible outcomes = 52
(i) For a king of red colour:
Number of red colour kings = 2 [ Kings of diamond and heart are red]
Number of favourable outcomes = 2
2 1
E(red king) = = .
52 26
(ii) For a face card:
4 kings, 4 queens and 4 jacks are face cards
Number of face cards = 12
Number of favourable outcomes = 12
12 3
P(face) = =
52 13
(iii) For a red face card:
Since, cards of diamond and heart are red
There are [2 kings, 2 queens, 2 jacks] 6 cards are red
Favourable outcomes = 6
6 3
P(red face) = =
52 26
(iv) For a jack of hearts:
Since, there is only 1 jack of hearts.
Number of favourable outcomes = 1
Number of favourable outcomes
P(Jack of hearts) =
All possible outcomes
1
=
52
(v) For a spade:
There are 13 spades in a pack of 52 cards:
Favourable outcomes are 13.
Number of favourable outcomes
P(spade) =
Total number of possible outcomes
13 1
= =
52 4
(vi) For the queen of diamonds:
There is only one queen of diamond.
Number of favourable outcomes.
Number of favourable outcomes
P(queen of diamonds) =
Total number of possible outcomes
1
= .
52
Q. 15. Five cardsthe ten, jack, queen, king and ace of diamonds, are well-shuffled with their face
downwards. One card is then picked up at random.
(i) What is the probability that the card is the queen?
232 MathematicsX
(ii) If the queen is drawn and put aside, what is the probability that the second card picked up
is (a) an ace? and (b) a queen?
Sol. We have five cards.
All possible outcomes = 5
(i) For a queen:
Number of queens = 1
Number of favourable outcomes
P(queen) =
Total number of possible outcomes
1
=
5
(ii) The queen is drawn and put aside,
Only 5 1 = 4 cards are left,
All possible outcomes = 4
(a) For an ace:
There is only one ace
Number favourable outcomes = 1
Number of favourable outcomes
P(an ace) =
Total possible outcomes
1
=
4
(b) For a queen:
Since, the only queen has already been put aside.
Number of possible outcomes = 0
Number of favourable outcomes
P(a queen) =
Number of possible outcomes
0
= = 0.
4
Q. 16. 12 defective pens are accidentally mixed with 132 good ones. It is not possible to just look at a
pen and tell whether or not it is defective. One pen is taken out at random from this lot. Determine
the probability that the pen taken out is a good one.
Sol. We have
Number of good pens = 132
Number of defective pens = 12
Total number of pens = 132 + 12 = 144
For good pens:
There are 132 good pens
Number of favourable outcomes = 132
Number of favourable outcomes
P(good pens) =
Total possible outcomes
132 11
= = .
144 12
Q. 17. (i) A lot of 20 bulbs contain 4 defective ones. One bulb is drawn at random from the lot. What
is the probability that this bulb is defective?
(ii) Suppose the bulb drawn in (i) is not defective and is not replaced. Now one bulb is drawn at
random from the rest. What is the probability that this bulb is not defective?

Probability 233
Sol. Since, there are 20 bulbs in the lot.
Total number of possible outcomes = 20
(i) Number defective bulbs = 4
i.e., Favourable outcomes = 4
Number of favourable outcomes
P(defective bulb) =
Total number of outcomes
4 1
= =
20 5
(ii) The bulb drawn above is not included in the lot.
Remaining number of bulbs = 20 1 = 19.
Total number of possible outcomes = 19.
Number of bulbs which are not defective = 19 4 = 15
Favourable number of outcomes = 15
Number of favourable outcomes
P(not defective bulb) =
Total number of possible outcomes
15
= .
19
Q. 18. A box contains 90 discs which are numbered from 1 to 90. If one disc is drawn at random from
the box, find the probability that it bears (i) a two-digit number, (ii) a perfect square number and
(iii) a number divisible by 5.
Sol. We have:
Total number of discs = 90
Total number of possible outcomes = 90
(i) For a two-digit number:
Since the two-digit numbers are 10, 11, 12, ....., 90.
Number of two-digit numbers = 90 9 = 81
[ 1, 2, 3, 4, 5, 6, 7, 8, and 9 are 1-digit numbers]
Number of favourable outcomes = 81
Number of favourable outcomes
P(two-digit number) =
Total number of possible outcomes
81 9
= =
90 10
(ii) For a perfect square:
Perfect squares from 1 to 90 are 1, 4, 9, 16, 25, 36, 49, 64, and 81
Number of perfect numbers = 9
Number of favourable outcomes = 9
Number of favourable outcomes
P(perfect number) =
Total number of possible outcomes
9 1
= =
90 10
(iii) For a number divisible by 5:
Numbers divisible by 5 [from 1 to 90] are: 5, 10, 15, 20, 25, 30, 35, 40, 45, 50, 55, 60,
65, 70, 75, 80, 85, 90
i.e. There are 18 number (1 to 90) which are divisible by 5.
Number of favourable outcomes = 18
Number of favourable outcomes
P(Divisible by 5) =
All possible outcomes
18 1
= = .
90 5
234 MathematicsX
Q. 19. A child has a die whose six faces show the letters as given below:
A B C D E A
The die is thrown once. What is the probability of getting (i) A? and (ii) D?
Sol. Since there are six faces of the given die and these faces are marked with letters
A B C D E A
Total number of letters = 6
Number of possible outcomes = 6
(i) For the letter A
Two faces are having the letter A.
Number of favourable outcomes = 2
Number of favourable outcomes
Now, P(letter A) =
Total number of possible outcomes
2 1
= =
6 3
(ii) For the letter D:
Number of Ds = 1
Number of possible outcomes = 1
Number of favourable outcomes
P(letter D) =
Total number of possible outcomes
1
= .
6
Q. 20. Suppose you drop a die at random on the rectangular region shown in figure. What is the
probability that it will land inside the circle with diameter 1 m?
3m

2m

Sol. Here, Area of the rectangle = 3 m 2 m = 6 m2


And, the area of the circle = r2
FG 1 IJ 2
m2
=
H 2K
2
= m
4
Probability for the die to fall inside the circle
Area of the favourable region
=
Area of the whole region
Area of the circle
=
Area of the rectangle
LM OP
=
N4Q =
1
=

.
6 4 6 24
Probability 235
Q. 21. A lot consists of 144 ball pens of which 20 are defective and the others are good. Nuri will buy
a pen if it is good, but will not buy if it is defective. The shopkeeper draws one pen at random
and gives it to her. What is the probability that
(i) She will buy it?
(ii) She will not buy it? (CBSE 2012)
Sol. Total number of ball pens = 144
All possible outcomes = 144
(i) Since there are 20 defective pens
Number of good pens 144 20 = 124
Number of favourable outcomes = 124
Probability that she will buy it
124 31
= =
144 36
(ii) Probability that she will not buy it
= 1 [Probability that she will buy it]
31
= 1
36
36 31 5
= = .
36 36
Q. 22. Refer to Example 13. (i) Complete the following table:
Event:
Sum on 2 dice 2 3 4 5 6 7 8 9 10 11 12
1 5 1
Probability
36 36 36

(ii) A student argues that there are 11 possible outcomes 2, 3, 4, 5, 6, 7, 8, 9, 10, 11 and 12.
1
Therefore, each of them has a probability . Do you agree with this argument? Justify your
11
answer.
Sol. The two dice are thrown together.
Following are the possible outcomes:
(1, 1); (1, 2); (1, 3); (1, 4); (1, 5); (1, 6).
(2, 1); (2, 2); (2, 3); (2, 4); (2, 5); (2, 6).
(3, 1); (3, 2); (3, 3); (3, 4); (3, 5); (3, 6).
(4, 1); (4, 2); (4, 3); (4, 4); (4, 5); (4, 6).
(5, 1); (5, 2); (5, 3); (5, 4); (5, 5); (5, 6).
(6, 1); (6, 2); (6, 3); (6, 4); (6, 5); (6, 6).
Number of all possible outcomes is 6 6 = 36.
(i) Let the required probability be P(E).
(a) The sum on two dice is 3 for: (1, 2) and (2, 1)
Favourable outcomes = 2
2
P(E) =
36
(b) The sum on two dice is 4 for: (1, 3), (2, 2) and (3, 1).
Number of favourable outcomes = 3
3
P(2) =
36
236 MathematicsX
(c) The sum on two dice is 5 for:
(1, 4), (2, 3), (3, 2) and (4, 1)
Number of favourable outcomes = 4
4
P(E) =
36
(d) The sum on two dice is 6 for:
(1, 5), (2, 4), (3, 3), (4, 2) and (5, 1)
Number favourable outcomes = 5
5
P(E) =
36
(e) The sum on two dice is 7 for:
(1, 6), (2, 5), (3, 4), (4, 3), (5, 2) and (6, 1)
Number of favourable outcomes = 6
6
P(E) =
36
(f) The sum on two dice is 9 for:
(3, 6), (4, 5), (5, 4) and (6, 3)
Number of favourable outcome = 4
4
P(E) =
36
(g) The sum on two dice is 10 for:
(4, 6), (5, 5), (6, 4)
Number of favourable outcomes = 3
3
P(E) =
36
(h) The sum on two dice is 11 for:
(5, 6) and (6, 5)
Number of favourable outcomes = 2
2
P(E) =
36
Thus, the complete table is as under:
Event:
Sum on 2 dice 2 3 4 5 6 7 8 9 10 11 12
1 2 3 4 5 6 5 4 3 2 1
Probability
36 36 36 36 36 36 36 36 36 36 36

(ii) No. The number of all possible outcomes is 36 and not 11.
The argument is not correct.
Q. 23. A game consists of tossing a one rupee coin 3 times and noting its outcome each time. Hanif wins
if all the tosses give the same result i.e., three heads or three tails, and loses otherwise. Calculate
the probability that Hanif will lose the game.
Sol. Let T denotes the tail and H denotes the head.
All the possible outcomes are:
H H H, H H T, H T T, T T T, T T H, T H T, T T H, H T H
Number of all possible outcomes = 8
Let the event that Hanif will lose the game be denoted by E.

Probability 237
Favourable events are:
H H T, H T H, T H H, T H T , T T H, H T T
Number of favourable outcomes = 3
6 3
P (E) = = .
8 4
Q. 24. A die is thrown twice. What is the probability that
(i) 5 will not come up either time? (ii) 5 will come up at least once?
[Hint: Throwing a die twice and throwing two dice simultaneously are treated as the same
experiment.]
Sol. Since, throwing a die twice or throwing two dice simultaneously is the same.
All possible outcomes are:
(1, 1); (1, 2); (1, 3); (1, 4); (1, 5); (1, 6)
(2, 1); (2, 2); (2, 3); (2, 4); (2, 5); (2, 6)
(3, 1); (3, 2); (3, 3); (3, 4); (3, 5); (3, 6)
(4, 1); (4, 2); (4, 3); (4, 4); (4, 5); (4, 6)
(5, 1); (5, 2); (5, 3); (5, 4); (5, 5); (5, 6)
(6, 1); (6, 2); (6, 3); (6, 4); (6, 5); (6, 6)
All possible outcomes = 36
(i) Let E be the event that 5 does not come up either time, then
The favourable outcomes are [36 (5 + 6)] = 25
25
P(E) =
36
(ii) Let N be the event that 5 will come up at least once, then Number of favourable
outcomes = 5 + 6 = 11
11
P(N) = .
36
Q. 25. Which of the following arguments are correct and which are not correct? Give reasons for your
answer.
(i) If two coins are tossed simultaneously there are three possible outcomestwo heads, two tails
1
or one of each. Therefore, for each of these outcomes, the probability is .
3
(ii) If a die is thrown, there are two possible outcomesan odd number or an even number.
1
Therefore, the probability of getting an odd number is .
2
Sol. (i) Not correct.
Because, the situation one of each can result in two ways HT and TH.
1
The probability = .
4
(ii) Correct.
Because the two outcomes are possible.

238 MathematicsX
NCERT TEXTBOOK QUESTIONS SOLVED
EXERCISE 15.2
Q. 1. Two customers Shyam and Ekta are visiting a particular shop in the same week (Tuesday to
Saturday). Each is equally likely to visit the shop on any day as on another day. What is the
probability that both will visit the shop on (i) the same day? (ii) consecutive days? (iii) different
days?
Sol. Here, the number of all the possible outcomes
= 5 5 = 25
(i) For both customers visiting same day:
Number of favourable outcomes = 5
[ (Tue., Tue.), (Wed., Wed.), (Thu., Thu.), (Fri., Fri.), (Sat., Sat.)]
5 1
Required probability = =
25 5
(ii) For both the customers visiting on consecutive days:
Number of outcomes are:
(Tue., Wed.), (Wed., Thu.), (Thu., Fri.), (Fri., Sat.), (Sat., Fri.), (Wed., Tue.),
(Thu., Wed.), (Fri., Thu.)
Number of favourable outcomes = 8
8
Required probability =
25
(iii) For both the customers visiting on different days:
1
We have probability for both visiting same day =
5
Probability for both visiting on different days
= 1 [Probability for both visiting on the same day]

= 1
1
=
51 LM OP
=
4
5 5 NQ 5
4
The required probability = .
5
Q. 2. A die is numbered in such a way that its faces show the numbers 1, 2, 2, 3, 3, 6. It is thrown two
times and the total score in two throws is noted. Complete the following table which gives a few
values of the total score on the two throws:
Number in first throw
+ 1 2 2 3 3 6
Number in second throw

1 2 3 3 4 4 7

2 3 4 4 5 5 8

3 5 9

6 7 8 8 9 9 12

Probability 239
What is the probability that the total score is
(i) even? (ii) 6? (iii) at least 6?
Sol. The completed table is as under:
1 2 3 4 5 6
1 2 3 3 4 4 7
2 3 4 4 5 5 8
3 3 4 4 5 5 8
4 4 5 5 6 6 9
5 4 5 5 6 6 9
6 7 8 8 9 9 12
Number of all possible outcomes = 36
(i) For total score being even:
Favourable outcomes = 18
[ The even outcomes are: 2, 4, 4, 4, 4, 8, 4, 4, 8, 4, 6, 6, 4, 6, 6, 8, 8]
18 1
The required probability = =
36 2
(ii) For the score being 6:
In list of score, we have four 6 s.
Favourable outcomes = 4
4 1
Required probability = =
36 9
(iii) For the score being at least 6:
The favourable scores are:
7, 8, 8, 6, 6, 9, 6, 6, 9, 7, 8, 8, 9, 9 and 12
Number of favourable outcomes = 15
15 5
Required probability = = .
36 12
Q. 3. A bag contains 5 red balls and some blue balls. If the probability of drawing a blue ball is double
that of a red ball, determine the number of blue balls in the bag.
Sol. Let the number of blue balls in the bag be x.
Total number of balls = x + 5
Number of possible outcomes = (x + 5).
For a blue ball favourable outcomes = x
Probability of drawing a blue ball
x
=
x+5
Similarly, probability of drawing a red ball
5
=
x+5
Now, we have
x
= 2
5 LM OP
x+5 x+5 N Q
x 10
= x = 10
x+5 x+5
Thus the required number of blue balls = 10.

240 MathematicsX
Q. 4. A box contains 12 balls out of which x are black. If one ball is drawn at random from the box,
what is the probability that it will be a black ball?
If 6 more black balls are put in the box, the probability of drawing a black ball is now double of
what it was before. Find x.
Sol. The total number of balls in the box = 12
Number of possible outcomes = 12
Case-I: For drawing a black ball
Number of favourable outcomes = x
x
Probability of getting a black ball =
12
Case-II: When 6 more black balls are added
Now, the total number of balls
= 12 + 6
= 18
Number of possible outcomes = 18
Since, the number of black balls now
= (x + 6).
Number of favourable outcomes = (x + 6)
x+6
Required probability =
18
Applying the given condition:
x+6 FG IJ
x

18
= 2
H K
12
12 (x + 6) = 36x 12x + 72 = 36x
36x 12x = 72 24x = 72
72
x = = 3
24
Thus, the required value of x is 3.
Q. 5. A jar contains 24 marbles, some are green and others are blue. If a marble is drawn at random
2
from the jar, the probability that it is green is . Find the number of blue balls in the jar.
3
Sol. There are 24 marbles in the jar.
Number of possible outcomes = 24.
Let there are x blue marbles in the jar.
Number of green marbles = 24 x
Favourable outcomes = (24 x)
Required probability for drawing a green marble
24 x
=
24
Now, according to the condition, we have:
24 x 2
=
24 3
3 (24 x) = 2 24
72 3x = 48
3x = 72 48
3x = 24
24
x = = 8
3
Thus, the required number of blue balls is 8.

Probability 241
MORE QUESTIONS SOLVED
I. VERY SHORT ANSWER TYPE QUESTIONS
Q. 1. A letter is chosen at random from English alphabet. Find the probability that the letter chosen
precedes g.
Sol. Total number of letters in English alphabet is 26.
Total number of possible outcomes = 26
Letters preceding g are:
a, b, c, d, e and f
Favourable outcomes = 6
6 3
Required probability = = .
26 13
Q. 2. A letter of English alphabet is chosen at random. Determine the probability that the letter is a
consonant. [NCERT Exemplar]
Sol. There are 26 letters in English alphabets.
Possible outcomes = 26
Q There are 5 vowels (a, e, i, o, u) and remaining are consonants.
Number of consonants = 26 5 = 21
Favourable outcomes = 21
21
P(consonants) =
26
Q. 3. A bag contains 9 black and 12 white balls. One ball is drawn at random. What is the probability
that the ball drawn is black?
Sol. Total number of balls = 9 + 12 = 21
Number of possible outcomes = 21
Number of black balls = 9
Number of favourable outcomes = 9
9 3
Required probability = = .
21 7
Q. 4. Find the probability that a number selected from the numbers 1 to 25 which is not a prime number
when each of the given number is equally likely to be selected.
Sol. Total number of given numbers = 25
Since the numbers 2, 3, 5, 7, 11, 13, 17, 19 and 23 are prime number.
There are 9 numbers.
Number of numbers that are not prime = 25 9 = 16
Number of favourable outcomes = 16
16
Required probability = .
25

Q. 5. A die is thrown once. Find the probability of getting an odd number.


Sol. Total number of possible outcomes = 6
[ Numbers 1 to 6 are marked on the faces of a die]
odd numbers are 1, 3 and 5
Favourable outcomes = 3
3 1
Required probability = = .
6 2
242 MathematicsX
Q. 6. Cards each marked with one of the numbers 6, 7, 8, ....., 15 and placed in a box and mixed
thoroughly. One card is drawn at random from the box. What is the probability of getting a card
with number less than 10? (AI CBSE 2008 C)
Sol. There are 10 cards.
Total number of possible outcomes = 10
Cards marked with a number less than 10 are: 6, 7, 8 and 9
i.e. The number of favourable outcomes = 4
4 2
P(E) = or .
10 5
Q. 7. A card is drawn at random from a well-shuffled deck of 52 cards. What is the probability of getting
a black king? (CBSE 2009 C)
Sol. Total number of cards = 52
Number of possible outcomes = 52
Number of black king = 2
2 1
P(Black king) = = .
52 26
Q. 8. What is the probability that two different friends have different birthdays? (Ignoring leap year).
(CBSE 2009 C)
Sol. Number of days in a year = 365
Number of possible outcomes = 365
Since they have different birthdays.
Number of favourable outcomes = 365 1 = 364
364
P(E) = .
365
Q. 9. A box contains 3 blue, 2 white and 4 red marbles. If a marble is drawn at random from the box,
what is the probability that it will not be a white marble? (CBSE 2009 C)
Sol. Total number of balls = 3 + 2 + 4 = 9
Number of possible outcomes = 9
Since, number of white balls = 2
Number of balls which are not white
= 92=7
Number of favourable outcomes = 7
7
P(E) = .
9
Q. 10. From a well-shuffled pack of cards, a card is drawn at random. Find the probability of getting a
black queen. (CBSE 2012)
Sol. Total number of cards = 52
Since, the number of black queens = 2
Number of favourable outcomes = 2
2 1
P(E) = = .
52 26
Q. 11. A bag contains 4 red and 6 black balls. A ball is taken out of the bag at random. Find the
probability of getting a black ball. (AI CBSE 2008)
Sol. Total number of balls = 4 + 6 = 10
All possible outcomes = 10
Since, number of black balls = 6

Probability 243
Number of favourable outcomes = 6
6 3
P(E) = or .
10 5
Q. 12. A die is thrown once. Find the probability of getting a number less than 3. (AI F 2008)
Sol. Numbers on the faces are 1, 2, 3, 4, 5 and 6.
Number of possible outcomes = 6
Numbers less than 3 are 1 and 2.
Number of favourable outcomes = 2
2 1
P(E) = or .
6 3
Q. 13. A die is thrown once. Find the probability of getting a number greater than 5. (AI F 2008)
Sol. Total number of possible outcomes = 6
Since only one number i.e., 6 is greater than 5
Favourable number of outcomes = 1
1
P(E) = .
6
Q. 14. Find the probability of obtaining 7 on a single toss of one die.
Sol. Numbers marked on a die are:
1 , 2 , 3 , 4 , 5 , 6
There are six different possible outcomes.
But none of these outcomes would produce a 7.
Favouable outcome = 0
0
P(7) = =0
6
When an event cannot possibly succeed, we say it is an impossible event and probability of an
impossible event is zero.
i.e. P(impossible event) = 0

Q. 15. Cards bearing numbers 3 to 20 are placed in a bag and mixed thoroughly. A card is taken out
from the bag at random. What is the probability that the number on the card taken out is an even
number? (CBSE 2008 C)
Sol. Total number of cards (3 to 20) = 18
Number of possible outcomes = 18
Since cards having even numbers (4, 6, 8, 10, 12, 14, 16, 18 and 20) are 9,
Number of favourable outcomes = 9
9 1
P(E) = or .
18 2
Q. 16. Two friends were born in the year 2000. What is the probability that they have the same birthday ?
(AI CBSE 2008 C)
Sol. Since the year 2000 was a leap year,
Total number of days in the year = 366
They have the same birthday.
Number of favourable outcomes = 1
1
P(E) = .
366
244 MathematicsX
Q. 17. A box contains cards marked with numbers 5 to 20. A card is drawn from the bag at random. Find
the probability of getting a number which is a perfact square. (AI CBSE 2008 C)
Sol. Total number of cards = 16
Possible outcomes are 16.
Since the numbers 9 and 16 are perfect numbers,
Number of favourable outcomes = 2
2 1
P (E) = or .
16 8

II. SHORT ANSWER TYPE QUESTIONS


Q. 1. Two dice are thrown at the same time. Find the probability of getting different numbers on the
dice. (CBSE Sample Paper 2011)
Sol. Since the two dice are thrown simultaneously.
Total number of outcomes = 6 6 = 36
Number of outcomes for getting same numbers on both dice = 6
6 1
P (same numbers) = =
36 6
Now, P (different numbers) + P (same numbers) = 1
P (different numbers) = 1 P (same numbers)
1
= 1
6
5
= .
6
Q. 2. Two dice are thrown at the same time. Find the probability of getting same number on both dice.
Sol. Total number of outcomes = 6 6 = 36
Following are the outcomes that have same number on both dice are:
(1, 1), (2, 2), (3, 3), (4, 4), (5, 5) and (6, 6)
Favourable outcomes = 6
6 1
Required probability = = .
36 6
Q. 3. A bag contains 10 red, 5 blue and 7 green balls. A ball is drawn at random. Find the probability
of this ball being not a blue ball.
Sol. Total number of balls = 10 + 5 + 7 = 22
Number of possible outcomes = 22
Since there are 5 blue balls.
Number of balls which are not blue
= 22 5 = 17
Favourable outcomes = 17
17
Required probability = .
22
Q. 4. Two dice are thrown at the same time and the product of numbers appearing on them is noted.
Find the probability that the product is less than 9.
Sol. Total number of possible outcomes = 6 6 = 36
The outcomes such that the product of numbers appearing on the faces is less than 9
are:
(1, 1), (1, 2), (1, 3), (1, 4), (1, 5), (1, 6), (2, 1), (2, 2), (2, 3), (2, 4), (3, 1), (3, 2), (4, 1),
(4, 2), (5, 1) and (6, 1).
Probability 245
Number of favourable outcomes = 16
16 4
Required probability = = .
36 9
Q. 5. An integer is chosen between 0 and 100. What is the probability that it is divisible by 7?
Sol. Numbers between 0 and 100 are 99.
Total possible outcomes = 99
Since following numbers are divisible by 7 :
7, 14, 21, 28, 35, 42, 49, 56, 63, 70, 77, 84, 91 and 98.
Favourable outcomes = 14
14
Required probability = .
99
Q. 6. A letter of English alphabet is chosen at random. Determine the probability that the letter is
consonant.
Sol. There are 26 letters of English alphabet
Number of possible outcomes = 26
Since, there are 21 consonants of the English alphabets.
Favourable outcomes = 21
21
Required probability = .
26
Q. 7. Cards with numbers 2 to 101 are placed in a box. A card is selected at random. Find the probability
that the card has a square number. [(CBSE Sample Paper 2011, CBSE 2012)]
Sol. Number of numbers between 2 to 101 are 100
Total number of possible outcomes = 100
Since, the perfect numbers between 2 and 101 are:
4, 9, 16, 25, 36, 49, 64, 81 and 100
Number of favourable outcomes = 9
9
Required probability = .
100
Q. 8. In a game of chance there is spinning of an arrow which comes to rest pointing at one of the
numbers 1, 2, 3, 4, 5, 6, 7, 8 and there are equally likely outcomes. What is the probability that
it will point at
(i) 7? (ii) an odd number?
(iii) a number less than 9? (CBSE 2012)
Sol. Since, following numbers are marked on the disc:
1, 2, 3, 4, 5, 6, 7, 8
Possible outcomes in each case are 8.
(i) Possible outcomes = 8
Favourable otucome = 1 (\ only the number 7)
1
P(The number 7) =
8
(ii) Q Odd numbers on the disc are
1, 3, 5 and 7
Favourable outcomes = 4
4 1
P(Odd number) =
8 2

246 MathematicsX
(iii) Q The numbers less than 9 on the disc are:
1, 2, 3, 4, 5, 6, 7, 8, (i.e. 8 outcomes)
Favourable outcomes = 8
8
P(number less than 9) = =1
8
Q. 9. From a group of 2 boys and 3 girls, two children are selected at random. Find the probability such
that at least one boy is selected.
Sol. Let B1 and B2 be two boys and G1, G2 and G3 be the three girls
Since two children are selected at random,
Following are the possible groups:
B1B2, B1G1, B1G2, B1G3, B2G1, B2G2, B2G3, G1G2, G1G3, G2G3
Total number of possible outcomes = 10
Since, one boy is to be selected,
Favourable outcomes are:
B1B2, B1G1, B1G2, B1G3, B2G1, B2G2 and B2G3.
Number of favourable outcomes = 7
7
Required probability = .
10
Q. 10. A bag contains 7 red, 5 white and 3 black balls. A ball is drawn at random from the bag. Find
the probability that the drawn ball is neither white nor black.
Sol. Total number of balls
= 7+5+3
= 15
Number of white balls = 5
Number of black balls = 3
Number of balls that are neither white nor black = 15 [5 + 3]
= 15 8
= 7
7
Required probability = .
15
Q. 11. A box contains 20 cards, numbered from 1 to 20. A card is drawn from the box at random. Find
the probability that the number on the drawn card is:
(i) even (ii) multiple of 3.
Sol. Total numbers from 1 to 20 are 20
Number of possible events = 20
(i) Even numbers are:
2, 4, 6, 8, 10, 12, 14, 16, 18 and 20
Number of favourable outcomes = 10
10 1
Probability of getting an even number = =
20 2
(ii) Since, multiples of 3 are:
3, 6, 9, 12, 15 and 18
Number of favourable outcomes = 6
Probability of getting a multiple of 3
6 3
= = .
20 10

Probability 247
Q. 12. Two dice are thrown at the same time. Find the probability that the sum of the two numbers
appearing on the top of the dice is more than 9. (AI CBSE 2009 C)
Sol. Following are the possible outcomes for two dice thrown simultaneously:
(1, 1) (1, 2) (1, 3) (1, 4) (1, 5) (1, 6)
(2, 1) (2, 2) (2, 3) (2, 4) (2, 5) (2, 6)
(3, 1) (3, 2) (3, 3) (3, 4) (3, 5) (3, 6)
(4, 1) (4, 2) (4, 3) (4, 4) (4, 5) (4, 6)
(5, 1) (5, 2) (5, 3) (5, 4) (5, 5) (5, 6)
(6, 1) (6, 2) (6, 3) (6, 4) (6, 5) (6, 6)
Total number of possible outcomes = 36
Following outcomes have a sum of more than 9:
(4, 6), (6, 4), (5, 5), (5, 6), (6, 5) and (6, 6)
i.e. Favourable outcomes = 6
6 1
The required probability = or .
36 6
Q. 13. In a bag-X, there are four cards numbered 1, 3, 5 and 7 respectively. In an another bag-Y, there
are three cards numbered 2, 4 and 6 respectively. A card is drawn at random from each bag.
(a) Write all the possible outcomes.
(b) Find the probability that the sum of these two cards drawn is:
(i) 7 (ii) even (iii) more than 7
Sol. (a) There are 12 possible outcomes
Bag A 1 3 5 7
Bag B


2 2, 1 2, 3 2, 5 2, 7

4 4, 1 4, 3 4, 5 4, 7 12 Possible outcomes

6 6, 1 6, 3 6, 5 6, 7

Possible outcomes are (2, 1), (4, 1), (6, 1), (2, 3), (4, 3), (6, 3), (2, 5), (4, 5), (6, 5),
(2, 7), (4, 7) and (6, 7).
(b) (i) Q Only (6 + 1), (4 + 3) and (2 + 5) gives sum as 7
Possible outcomes = 3
3 1
P(Sum = 7) = =
12 4
(ii) Q There are no even sums
Possible outcomes = 0
0
P(Sum = an even) = =0
12
(iii) Q 6+3 = 9
2+7 = 9
4 + 7 = 11
4+5 = 9
6 + 5 = 11
6 + 7 = 13
There are six sums which are more than 7
Possible outcomes = 6
6 1
P(Sum more than 7) = =
12 2
248 MathematicsX
Q. 14. A game consists of tossing a one-rupee coin 3 times and noting its outcome each time. Hanif wins
if all the tosses give the same result, i.e., three heads or three tails, and loses otherwise, calculate
the probability that Hanif will lose the game. (AI CBSE 2009 C)
Sol. For Solution, please see the solution of Q. 23 of the Textbook Exercise 15.1.
Q. 15. Find the probability that a number selected at random from numbers 3, 4, 5, ....., 25 is prime.
(CBSE 2009 C)
Sol. Total numbers are 23.
Number of possible outcomes = 23
Since, prime numbers are 3, 5, 7, 11, 13, 17, 19 and 23.
Number of favourable outcomes = 8
8
P (E) = .
23
Q. 16. The king, queen and jack of diamonds are removed from a pack of 52 cards are then the pack is
well-shuffled. A card is drawn from the remaining cards. Find the probability of getting a card of
(i) diamonds
(ii) a Jack (AI CBSE 2008 C)
Sol. There are 52 card in the pack.
And number of cards removed = 3 [1 king + 1 queen + 1 jack = 3 cards]
Remaining cards = 52 3 = 49
13 3 10
(i) P(a diamond) = = [ Total diamonds are 13]
49 49
41 3
(ii) P(a jack) = = [ Total jacks are 4]
49 49
Q. 17. A bag contains 5 red, 4 blue and 3 green balls. A ball is taken out of the bag at random. Find
the probability that the selected ball is
(i) of red colour
(ii) not of green colour. (CBSE 2008 C)
Sol. Total number of balls = 5 + 4 + 3 = 12
Number of possible outcomes = 12
(i) Number of red balls = 5
Favourable outcomes = 5
5
P(red ball) =
12
(ii) Number of green balls = 3
Number of ball which are not green = 12 3 = 9
Favourable outcomes = 9
9 3
P(not green) = = .
12 4
Q. 18. A card is drawn at random from a well-shuffled deck of playing cards. Find the probability of
drawing a
(i) face card
(ii) card which is neither a king nor a red card. (CBSE 2008 C)
Sol. Total number of cards = 52
(i) Total number of face cards = 12 [4 Jacks + 4 Queens + 4 Kings]
Number of favourable outcomes = 12
12 3
P(face) = =
52 13

Probability 249
(ii) Number of kings = 4
Number of red cards = 13 + 13 = 26
Number of cards that are neither a red nor a king = 52 4 26 + 2 (red kings)
= 24
Favourable outcomes = 24
24 6
=
P(neither king nor red) = .
52 13
Q. 19. A bag contains tickets, numbered 11, 12, 13, ....., 30. A ticket is taken out from the bag at random.
Find the probability that the number on the drawn ticket
(i) is a multiple of 7
(ii) is greater than 15 and a multiple of 5. (AI CBSE 2008)
Sol. Total number of tickets = 20 [ Numbers from 11 to 30 are 20]
(i) Multiples of 7 are 14, 21 and 28
Number of favourable outcomes = 3
3
P(a multiple of 7) =
20
(ii) The numbers that are greater than 15 and multiples of 5 are: 20, 25 and 30
Number of favourable outcomes = 3
3
P(multiples of 5 .
and greater than 15) =
20
Q. 20. A bag contains 4 red, 5 black and 3 yellow balls. A ball is taken out of the bag at random. Find
that the ball taken out is of:
(i) yellow colour
(ii) not of red colour. (AI CBSE 2008)
Sol. Total number of balls = 4 + 5 + 3 = 12
Total number of possible outcomes = 12
(i) Number of yellow balls = 3
Number of favourable outcomes = 3
3 1
P(yellow) = =
12 4
(ii) Number of balls that are not red = 12 4 = 8 [ There are 4 red balls]
Favourable outcomes = 8
8 2
P(not red) = = .
12 3
Q. 21. A coin is tossed two times. Find the probability of getting at most one head.
(CBSE Sample Paper 2011)
Sol. Since, the coin is thrown two times.
Possible out comes = 4
Favourable outcomes are TT, TH, HT
i.e., Number of favourable outcomes = 3
3
P (atmost one head) = .
4
Q. 22. There are 40 students in class X of whom 25 are girls and 15 are boys. The class teacher has to
select one student as a class representative. She writes the name of each student on a separate card.
The cards being identical and she puts cards in a bag and stirs throughly. She then draws one
card from the bag. What is the probability that the name written on the card is the name of a:
(i) girl (ii) a boy

250 MathematicsX
Sol. Total number of students = 40
Number of possible outcomes = 40
(i) Q There are 25 girls in the class
Number of favourable outcomes = 25
25 5
P(name of a girl) =
40 8
(ii) Q Number of boys = 15
Number of favourable outcomes = 15
15 3
P(name of a boy) =
40 8
Q. 23. Cards, marked with numbers 5 to 50, are placed in a box and mixed thoroughly. A card is drawn
from the box at random. Find the probability that the number on the taken out card is:
(i) a prime number less than 10.
(ii) a number which is a perfect square. (AI CBSE 2008)
Sol. Numbers from 5 to 50 are 46.
Total number of possible outcomes = 46.
(i) Prime numbers (less than 10) are 5, 7.
Favourable outcomes = 2
2 1
P(prime number less than 10) = =
46 23
(ii) Perfect square are 9, 16, 25, 36 and 49
Number of favourable outcomes = 5
5
P(perfect square) = .
46
Q. 24. A die is thrown once. Find the probability of getting:
(i) an even prime number.
(ii) a multiple of 3. (CBSE 2008)
Sol. Total numbers on the faces of a die are 1, 2, 3, 4, 5 and 6
Number of favourable outcomes = 6
(i) Even prime number is only one i.e. 2
Favourable outcome = 1
1
P(even prime number) =
6
(ii) Multiples of 3 are 3 and 6
Favourable outcomes are 2.
2 1
P(multiple of 3) = = .
6 3
Q. 25. A die is thrown once. Find the probability of getting:
(i) a prime number
(ii) a number divisible by 2. (CBSE 2008)
Sol. The numbers on the faces of a die are 1, 2, 3, 4, 5 and 6.
Number of possible outcomes = 6
(i) Prime numbers are 2, 3 and 5
Number of prime numbers = 3

Probability 251
Number of favourable outcomes = 3
3 1
P(prime number) = =
6 2
(ii) Numbers divisible by 2 are 2, 4 and 6
Favourable outcomes are 3.
3 1
P(divisible by 2) = = .
6 2
Q. 26. Two dice are thrown simultaneously. What is the probability that
(i) 5 will not come up on either of them?
(ii) 5 will come up on at least one?
(iii) 5 will come up at both dice?
Sol. The two dice are thrown simultaneously
Possible outcomes are = 6 6 = 36
(i) When 5 will not come up on either of them:
Favourable outcomes are: 36 11 = 25
25
P(5 will not come up on either dice) =
36
(ii) When 5 will come on at least one dice:
Favourable outcomes are: 36 25 = 11
11
P(5 will come on at least one dice) =
36
(iii) When 5 will come up on either dice:
Favourable outcome is only one i.e. (5, 5)
1
P(5 on both dice) = .
36
Q. 27. Two different dice are rolled simultaneously. Find the probability that the sum of numbers
appearing on the two dice is 10. [AI. CBSE (Foreign)2014]
Sol. When two different dice are rolled then possible outcomes are :
(1, 1) (1, 2) (1, 3) (1, 4) (1, 5) (1, 6)
(2, 1) (2, 2) (2, 3) (2, 4) (2, 5) (2, 6)
(3, 1) (3, 2) (3, 3) (3, 4) (3, 5) (3, 6)
(4, 1) (4, 2) (4, 3) (4, 4) (4, 5) (4, 6)
(5, 1) (5, 2) (5, 3) (5, 4) (5, 5) (5, 6)
(6, 1) (6, 2) (6, 3) (6, 4) (6, 5) (6, 6)
Number of total outcomes = 36
Sum of (5, 5), (6, 4) and (4, 6) is 10.
No of favourable outcomes = 3

3 1
Required Probability = or
36 12

252 MathematicsX
TEST YOUR SKILLS
1. Cards bearing numbers 1, 3, 5, ..., 35 are kept in a bag. A card is drawn at random from
the bag. Find the probability of getting a card bearing:
(i) a prime number less than 15.
(ii) a number divisible by 3 and 5.
2. Red kings, queens and jacks are removed from a deck of 52 playing cards and then well-
shuffled. A card is drawn from the remaining cards. Find the probability of getting (i) King
(ii) a red card (iii) a spade.
3. One card is drawn from a well-shuffled deck of 52 cards. Find the probability of getting:
(i) A king of red suit. (ii) A queen of black suit.
(iii) A jack hearts. (iv) A red face card.
4. A bag contains 5 red balls and some blue balls. If the probability of drawing a blue ball
from the bag is thrice that of a red ball, find the number of blue balls in the bag.
Hint: Let number of blue balls = x
Total number of balls = (5 + x)
x 5
P(blue = 3 P(red ball) = 3

ball)
x+5 x + 5
5. In a throw of a coin, find the probability of getting a head.
6. Two coins are tossed together find the probability of getting:
(i) at least one tail.
(ii) one head
7. An unbiased die is thrown once, find the probability of getting:
(i) a number greater than 4.
(ii) a multiple of 3.
8. Two dice are thrown at the same time. Find the probability of getting different numbers
on both the dice. [NCERT Exemplar]
Hint: P(different numbers on both dice) = 1 P(same number on both dice)

9. Two dice are thrown at the same time. Find the probability of getting same number on
both the dice.
10. A pair of dice is thrown once. Find the probability of getting an odd number on each die.
11. A lot consists of 48 mobile phones of which 42 are good, 3 have only minor defects and
3 have major defects. Varnika will buy a phone if it is good but the trader will only buy
a mobile if it has no major defect. One phone is selected at random from the lot. What
is the probability that it is:
(i) acceptable to Varnika?
(ii) acceptable to the trader? [NCERT Exemplar]
12. Find the probability that a number selected at random from the numbers 1, 2, 3, ..., 35 is a:
(i) prime number
(ii) multiple of 7
(iii) a prime number less than 15.
13. A bag contains 5 red marbles, 8 white marbles and 4 green marbles. What is the probability
that if one marble is taken out of the bag at random it will not be a green marble?
Probability 253
14. One card is drawn from a well shuffled deck of 52 cards. Find the probability of getting:
(i) the queen of diamond
(ii) an ace of hearts
(iii) a spade.
Hint: (i) There is only one queen of diamond.
(ii) There is only one ace of hearts.
(iii) There are 13 spade cards.

15. Find the probability of getting 53 Sundays in a leap year. (CBSE 2012)
Hint: Number of days in a normal year = 365
Number of days in a leap year = 366
Number of weeks, in a normal year = 52, means 52 Sundays
[366 (52 7)] = 2 extra days in a leap year.
These two extra days may have sample-space as:
(Monday Tuesday), (Tuesday Wednesday), (Wednesday Thursday), (Thursday
Friday), (Friday Saturday), (Saturday Sunday), (Sunday Monday).
i.e. out of 7 sample spaces, only two are favourable.
2
P (53 Sundays in a leap year) =
7
16. One letter is chosen at random amongst letters of the word Mathematics. Find the
probability that the letter chosen is a:
(i) vowel (ii) consonant
17. Two coins are tossed simultaneously. Find the probability of getting:
(i) two Heads (ii) at least one Head
(iii) no Head.
Hint: In a throw of two coins simultaneously the four possible outcomes are
HH, HT, TH, TT
18. A die is thrown once. What is the probability of getting a number greater than 4?
19. What is the probability that a number selected at random from the numbers 3, 4, 5, ..., 9
is a multiple of 4?
20. From a well suffled pack of playing cards, black jacks, black kings and black aces are
removed. A card is then drawn from the pack. Find the probability of getting:
(i) a red card (ii) not a diamond card.
21. A bag contains cards which are numbered from 2 to 90. A card is drawn at random from
the bag. Find the probability that it bears.
(i) a two-digit number (ii) a number which is a perfect square.
22. Cards numbered 1 to 30 are put in a bag. A card is drawn at random from this bag. Find
the probability that the number on the drawn card is :
(i) not divisible by 3. (ii) a prime number greater than 7.
(iii) not a perfect square number. [AI CBSE (Foreign) 2014]

254 MathematicsX
Hint: Total possible outcomes = 30
(i) Numbers not divisible by 3 [1 to 30] are :
1, 2, 4, 5, 7, 8, 10, 11, 13, 14, 16, 17, 19, 20, 22, 23, 25, 26, 28, 29,
Number of favourable outcomes = 20
20 2
Required Probability = =
30 3
(ii) Prime numbers greater than 7 are : 11, 13, 17, 19, 23, 29
6 1
Required probability = =
30 5
(iii) Perfect squares are 1, 4, 9, 16, 25 Total No. = 5
30 5 25 5
Required pribability = = =
30 30 6
23. Two different dice are tossed together. Find the probability :
(i) That the numbers on either die is even.
(ii) That the sum of numbers appearing on the two dice is 5. [AI CBSE 2014]
Hint: Total possible outcomes = 36
(i) Numbers of favourable outcomes = 9
[ Q (2, 2), (2, 4), (2, 6), (4, 2), (4, 4) (4, 6) (6, 2), (6, 4) and (6, 6) are desired outcomes.
9 1
Required Probability = =
36 4
(ii) Desired (favourable) outcomes are : (1, 4), (2, 3), (3, 2), (4, 1)
4 1
Required Probability = =
36 9
24. Red queens and black jacks are removed from a pack of 52 playing cards. A card is drawn
at random from the remaining cards, after reshuffling them. Find the probability that the
card drawn is :
(i) a king (ii) of red colour
(iii) a face-card (iv) a queen [AI CBSE 2014]
Hint: A pack of playing cards consists of 52 cards. 2 red queens and 2 black jacks are removed.
Therefore, remaining number of cards = 52 - 4 = 48.
4 1
(i) Numbers of kings = 4 Required P = =
48 12
24 1
(ii) Remaining red cards = 26 2 = 24 Required P = =
48 2
8 1
(iii) Remaining face-cards = 12 4 = 8 Required P = =
48 6
2 1
(iv) Remaining queens = 4 2 = 2 Required P = =
48 24
25. Rahim tosses two different coins simultaneously. Find the probability of getting at least
one tail. [CBSE (Delhi) 2014]
Hint: Total outcomes = 4 (HH, HT, TH, TT)
Fovourable outcomes = 3 (HT, TH, TT)
Required P = 3 4
Probability 255
26. A bag contains cards numbered from 1 to 49. A card is drawn from the bag at random, after
mixing the cards thoroughly. Find the probability that the number on the drawn card is :
(i) an odd number (ii) a multiple of 5
(iii) a perfect square (iv) an even prime number [CBSE (Delhi) 2014]
Hint: Possible outcomes = 49
(i) Odd numbers are : 1, 3, 5, 7, 9, 11, 13, 15, 17, 19, 21,
Favourable outcomes = 25
23, 25, 27, 29, 31, 33, 35, 37, 39, 41, 43, 45, 57, 49
Required P = 25 49
(ii) Multiples of 5 are = 5, 10, 15, 20, 25, 30, 35, 40, 45 P = 9 49
(iii) Perfect squares are : 1, 4, 9, 16, 25, 36, 49 P = 7 49 = 1 7
1
(iv) Even Primes are : only 2. P =
49

ANSWERS
Test Your Skills
1 10 1 1 1 3
1. (i) 1 , (ii) 2. (i) 1 , (ii) , (iii) 13 3. (i) , (ii) , (iii) , (iv)
3 9 23 23 46 26 26 52 26

3
4. 15 5.
1
6. (i) , (ii) 1
2 4 2
1 1 5 1
7. (i) , (ii) 8. 9.
3 3 6 6

1 7 15 11 1 6
10. 11. (i) , (ii) 12. (i) , (ii) , (iii)
4 8 16 35 7 35
13 1 1 1 2
13. 14. (i) , (ii) , (iii) 15.
17 52 52 4 7
4 7 1 3 1
16. (i) , (ii) 17. (i) , (ii) , (iii) 18. 1
11 11 4 4 4 3
13 33 81 8
19. 2 20. (i) , (ii) 21. (i) , (ii)
7 23 46 89 89

256 MathematicsX
[Unit VI: Coordinate Geometry]

7 Coordinate Geometry

Facts that Matter


z Distance Formula
I. The distance between two points P (x1, y1) and Q (x2, y2) is given by
PQ = bx 2 x1 g + by
2
2 y1 g 2

II. The distance of P (x, y) from the origin is


OP = x2 + y 2
z Section Formula
I. If P (x, y) divides the line segment AB, joining the points A (x1, y1) and B (x2, y2) in the ratio
m : n, then
m n

A (x1, y1) P (x, y) B (x2, y2)

mx2 + nx1 my 2 + ny 1
x = , y=
m+n m+n
II. The coordinates (x, y) of the mid-point of PQ are given by:
x1 + x2 y1 + y2
x = , y=
2 2
III. If A (x1, y1), B (x2, y2) and C (x3, y3) be the vertices of ABC, then the coordinates G (x, y)
of the centroid of ABC are:
x1 + x2 + x 3 y1 + y2 + y3
x = and y =
3 3

NCERT TEXTBOOK QUESTIONS SOLVED


EXERCISE 7.1
Q. 1. Find the distance between the following pairs of points:
(i) (2, 3), (4, 1) (ii) ( 5, 7), ( 1, 3) (iii) (a, b), ( a, b)
Sol. (i) Here x 1 = 2, y1 = 3, x2 = 4 and y2 = 1
The required distance

= bx x g + by y g
2 1
2
2 1
2

= b 4 2g + b1 3g
2 2

257
= 22 + 2 b g 2

= 4+4 = 8
= 24 = 2 2
(ii) Here, x 1 = 5, y1 = 7
x 2 = 1, y2 = 3
The required distance

= bx 2 g + by y g
x1
2
2 1
2

1 b 5g + b 3 7 g
2 2
=

= b 1 + 5g + b 4 g 2 2

= 16 + 16
= 32 = 2 16 = 4 2
(iii) Here, x 1 = a, y1 = b
x 2 = a, y2 = b
The required distance

= bx x g + by y g
2 1
2
2 1
2

= b a ag + b b b g
2 2

= b 2ag + b 2bg
2 2

= 4 a 2 + 4b 2

= e
4 a2 + b 2 j = 2 ea 2
+ b2 j
Q. 2. Find the distance between the points (0, 0) and (36, 15). Can you now find the distance between
the two towns A and B discussed in Section 7.2 of the NCERT textbook?
Sol. Part-I Y
Let the points be P (0, 0) and Q (36, 15).
PQ = b36 0g + b15 0g
2 2
Q (36,15)

= a36f + a15f
2 2
X (0,0)
15 km

X
= 1296 + 225 P 36 km
= 1521
= 39 2 = 39 Y
Part-II
We have P (0, 0) and Q (36, 15) as the positions of two towns.
Here x 1 = 0, x2 = 36
y 1 = 0, y2 = 15

258 MathematicsX
PQ = bx x g + by y g
2 1
2
2 1
2

= b36 0g + b15 0g = 39 km.


2 2

Q. 3. Determine if the points (1, 5), (2, 3) and ( 2, 11) are collinear.
Sol. Let the points be A (1, 5), B (2, 3) and C ( 2, 11)
A, B and C are collinear, if
AB + BC = AC
AC + CB = AB
BA + AC = BC

AB = b2 1g + b3 5g
2 2

1 + b 2g
2 2
=

= 1+ 4 = 5

BC = b 2 2g + b 11 3g
2 2

= b 4g + b 14g
2 2

= 16 + 196 = 212

AC = b 2 1g + b 11 5g
2 2

= b 3g + b 16g
2 2

= 9 + 256 = 265
But AB + BC AC
AC + CB AB
BA + AC BC
A, B and C are not collinear.
Q. 4. Check whether (5, 2), (6, 4) and (7, 2) are the vertices of an isosceles triangle.
Sol. Let the points be A (5, 2), B (6, 4) and C (7, 2). (CBSE 2012)

AB = b6 5g + 4 b 2g
2 2

= a1f + a6f
2 2

= 1 + 36 = 37

BC = b7 6g + b 2 4 g
2 2

= a1f + b 6g
2 2

= 1 + 36 = 37

AC = b5 7g + d 2 b 2gi
2 2

= b 2g + a0f
2 2

Coordinate Geometry 259


= 4+0 = 2
We have AB = BC AC
ABC is an isosceles triangle.
Q. 5. In a classroom, 4 friends are seated at the points A, B, C and D as shown in Fig. Champa and
Chameli walk into the class and after observing for a few minutes Champa asks Chameli, Dont
you think ABCD is a square? Chameli disagrees. Using distance single formula, find which of
them is correct.
10

8
B
7

6
Rows
5
A C
4

2
D
1

1 2 3 4 5 6 7 8 9 10
Columns
Sol. Let the number of horizontal columns represent the x-coordinates whereas the vertical
rows represent the y-coordinates.
The points are:
A (3, 4), B (6, 7), C (9, 4) and D (6, 1)
AB = b6 3g + b7 4g
2 2

= a 3 f + a 3f
2 2

= 9+9 = 18 = 3 2

BC = b9 6 g + b 4 7 g
2 2

3 + b 3g
2 2
=

= 9+9 = 18 = 3 2

CD = b6 9 g + b1 4 g
2 2

= b 3g + b 3g
2 2

= 9+9 = 18 = 3 2

AD = b6 3g + b1 4g
2 2

= a 3f + b 3g
2 2

260 MathematicsX
= 9 + 9 = 18 = 3 2
Since, AB = BC = CD = AD
i.e., All the four sides are equal.
Also AC = b9 3 g + b 4 4 g
2 2

= b 6 g + a0 f = 6
2 2

and BD = b6 6g + b1 7 g
2 2

= a0 f + b 6 g = 6
2 2

i.e., BD = AC Both the diagonals are also equal.


ABCD is a square.
Thus, Champa is correct.
Q. 6. Name the type of quadrilateral formed, if any, by the following points, and give reasons for your
answer:
(i) ( 1, 2), (1, 0), ( 1, 2), ( 3, 0)
(ii) ( 3, 5), (3, 1), (0, 3), ( 1, 4)
(iii) (4, 5), (7, 6), (4, 3), (1, 2)
Sol. (i) Let the points be: A ( 1, 2), B (1, 0), C ( 1, 2) and D ( 3, 0).
AB = b1 + 1g + b0 + 2g
2 2

= a2f + a2f = 4 + 4 = 8
2 2

BC = b 1 1g + b2 0g = 4 + 4 = 8
2 2

CD = b 3 + 1g + b0 2g = 4 + 4 = 8
2 2

DA = b 1 + 3g + b 2 0 g = 4 + 4 = 8
2 2

AC = b 1 + 1g + b2 + 2g = 0 + 4 = 4
2 2 2

BD = b 3 1g + b0 0g = a4f = 4
2 2 2

AB = BC = CD = AD
i.e., All the sides are equal.
And AC = BD
Also, AC and BD (the diagonals) are equal.
ABCD is a square.
(ii) Let the points be A ( 3, 5), B (3, 1), C (0, 3) and D ( 1, 4).

AB = 3 3b g + b1 5 g 2 2

+ b 4 g
2
= 62
= 36 + 16 = 52 = 2 13

BC = b0 3g + b3 1g
2 2

= 9+4 = 13
Coordinate Geometry 261
CD = b 1 0g + b 4 3 g
2 2

= a1f + a7f = 1 + 49 =
2 2
50

3 b 1g + 5 b 4 g
2 2
DA =

= a2 f + a9f
2 2

= 4 + 81 = 85

AC = 0 3b g + b 3 5g2 2

= a 3f + b 2 g
2 2

= 9+4 = 13

BD = b 1 3g + b 4 1g
2 2
= b 4 g + b 5g
2 2

= 16 + 25 = 41
We see that:
13 + 13 = 2 13
i.e., AC + BC = AB
A, B, C and D are collinear. Thus, ABCD is not a quadrilateral.
(iii) Let the points be A (4, 5), B (7, 6), C (4, 3) and D (1, 2).

AB = b7 4 g + b6 5 g
2 2

= 3 2 + 12 = 10

BC = b 4 7 g + b 3 6g
2 2

= b 3 g + b 3g =
2 2
18

CD = b1 4g + b2 3g
2 2

= b 3g + b 1g =
2 2
10

DA = b4 1g + b5 2g
2 2

= 9+9 = 18

AC = b 4 4 g + b 3 5g
2 2

0 + b 2g = 2
2
=

BD = b1 7 g + b2 6g
2 2

= 36 + 16 = 52

262 MathematicsX
Since, AB = CD [opposite sides of the quadrilateral are equal]
BC = DA
And AC BD Diagonals are unequal
ABCD is a parallelogram.
Q. 7. Find the point on the x-axis which is equidistant from (2, 5) and ( 2, 9). (CBSE 2012)
Sol. We know that any point on x-axis has its ordinate = 0.
Let the required point be P (x, 0).
Let the given points be A (2, 5) and B ( 2, 9).

PA = b x 2g + 0 b 5g
2 2

= b x 2g + 5 = x
2 2 2
4 x + 4 + 25 = x 2 4 x + 29

x b 2g + b0 9 g
2 2
PB =

= b x + 2 g + b 9g =
2 2
x 2 + 4 x + 4 + 81 = x 2 + 4 x + 85
Since, A and B are equidistant from P,
PA = PB

x 2 4 x + 29 = x 2 + 4 x + 85
x2 4x + 29 = x2 + 4x + 85
x2 4x x2 4x = 85 29
8x = 56
56
x = = 7
8
The required point is ( 7, 0).
Q. 8. Find the values of y for which the distance between the points P (2, 3) and Q (10, y) is 10 units.
Sol. The given points are P (2, 3) and Q (10, y).

PQ = b10 2g + y b 3g
2 2

8 + b y + 3g
2
=

= 64 + y 2 + 6 y + 9

= y 2 + 6 y + 73
But PQ = 10
y 2 + 6 y + 73 = 10
Squaring both sides,
y2 + 6y + 73 = 100
y2 + 6y 27 = 0
2
y 3y + 9y 27 = 0
(y 3) (y + 9) = 0
Either y3 = 0 y=3
or y+9 = 0 y=9
The required value of y is 3 or 9.

Coordinate Geometry 263


Q. 9. If Q (0, 1) is equidistant from P (5, 3) and R (x, 6), find the values of x. Also find the distances
QR and PR.

Sol. Here, QP = b5 0g + b 3 g 1
2 2

5 + b 4 g
2 2
=

= 25 + 16 = 41

QR = bx 0g + b6 1g
2 2

x2 + 52 =
= x 2 + 25
QP = QR
2
41 = x + 25
Squaring both sides, we have:
x2 + 25 = 41
x2 + 25 41 = 0
x2 16 = 0 x = 4
4, 6)
Thus, the point R is (4, 6) or (

Now, QR = b 4g a0f + b6 1g
2 2

= 16 + 25 = 41

and PR = b 4 5 g + b6 + 3g
2 2

PR = b4 5g + b6 + 3g or b 4 5g + b6 + 3g
2 2 2 2

1 + 81 or b 9g + 9
2 2
PR =

PR = 82 or 2 92
PR = 82 or 9 2
Q. 10. Find a relation between x and y such that the point (x, y) is equidistant from the point (3, 6) and
( 3, 4).
Sol. Let the points be A (x, y), B (3, 6) and C ( 3, 4).
AB = b 3 xg + b6 y g
2 2

And AC = b 3g x + b 4 y g
2 2

Since, the point (x, y) is equidistant from (3, 6) and ( 3, 4).


AB = AC
b 3 xg + b6 y g
2 2
= b 3 xg + b 4 y g
2 2

Squaring both sides,


(3 x)2 + (6 y)2 = ( 3 x)2 + (4 y)2
(9 + x 6x) + (36 + y2 12y) = (9 + x2 + 6x) + (16 + y2 8y)
2

9 + x2 6x + 36 + y2 12y 9 x2 6x 16 y2 + 8y
6x 6x + 36 12y 16 + 8y = 0

264 MathematicsX
12x 4y + 20 = 0
3x y + 5 = 0 [Dividing by 4]
3x + y 5 = 0
which is the required relation between x and y.

NCERT TEXTBOOK QUESTIONS SOLVED


EXERCISE 7.2
Q. 1. Find the coordinates of the point which divides the join of ( 1, 7) and (4, 3) in the ratio 2 : 3.
Sol. Let the required point be P (x, y).
Here, the end points are:
( 1, 7) and (4, 3)
Ratio = 2 : 3 = m1 : m2
m1 x2 + m2 x1
x =
m1 + m2

=
b2 4g + 3 b 1g
2+3
83 5
= = = 1
5 5
m1 y 2 + m 2 y 1
And y =
m1 + m2

=
b g b
2 3 + 37 g
2+3
6 + 21 15
= = = 3
5 5
Thus, the required point is (1, 3).
Q. 2. Find the coordinates of the points of trisection of the line segment joining (4, 1) and ( 2, 3).
Sol. Let the given points be A (4, 1) and B ( 2, 3).
P Q
A B
(4, 1) (2, 3)
Let the points P and Q trisect AB.
i.e., AP = PQ = QB
i.e., P divides AB in the ratio of 1 : 2
Q divides AB in the ratio of 2 : 1
Let the coordinates of P be (x, y).
m1 x2 + m2 x1
x =
m1 + m2

=
b g af
1 2 +2 4
=
2+8
= 2
1+2 3
m 1 y 2 + m2 y 1
y = = 1
m1 + m2

Coordinate Geometry 265


=
b g
1 3 +2 1 b g =
32
=
5
1+2 3 3
5
The required co-ordinates of P are 2, 3
Let the co-ordinates of Q be (X, Y).

X =
m 1 x2 + m 2 x1
=
2 2 +1 4 b g af =
4+4
= 0
m1 + m2 2+1 3
m1 y 2 + m 2 y 1
Y =
m1 + m 2

=
b g b g
2 3 +1 1
=
6+1
=
7
2+1 3 3
7 FG IJ
The required coordinates of Q are 0,
3
.
H K
Q. 3. To conduct Sports Day activities, in your rectangular shaped school ground ABCD, lines have
been drawn with chalk powder at a distance of 1 m each. 100 flower pots have been placed at a
1
distance of 1 m from each other along AD, as shown in the figure. Niharika runs th the distance
4
1
AD on the 2nd line and posts a green flag. Preet runs th the distance AD on the eighth line
5
and posts a red flag. What is the distance between both the flags? If Rashmi has to post a blue
flag exactly halfway between the line segment joining the two flags, where should she post her
flag?
Sol. Let us consider A as origin, then AB is the x-axis. AD is the y-axis.
100 FG IJ
Now, the position of green flag-post is 2 ,
4 H
or (2, 25)
K
100 FG IJ
And the position of red flag-post is 8 ,
5 H or (8, 20)
K
D C

2
1
A B
1 2 3 4 5 6 7 8 9 10

266 MathematicsX
Distance between both the flags

= b8 2g + b20 25g
2 2

6 + b 5g = 36 + 25
2 2
= = 61
Let the mid-point of the line segment joining the two flags be M (x, y).
M

(2, 25) (x, y) (8, 20)

2+8 25 + 20
x = and y =
2 2
or x = 5 and y = (22.5).
Thus, the blue flag is on the 5th line at a distance 22.5 m above AB.
Q. 4. Find the ratio in which the line segment joining the points ( 3, 10) and (6, 8) is divided by
( 1, 6).
Sol. Let the given points are: A ( 3, 10) and B (6, 8).
Let the point P ( 1, 6) divides AB in the ratio m1 : m2.
Using the section formula, we have:
Fx m +m x , m y +m y I
( 1, 6) = GH m + m
2 1

1
2
m +m
2
JK
1 1 2

1
2

2
1

F bm 6 g + m b 3 g m b 8 g + b m 10 g IJ
( 1, 6) = GG 1
,
2 1 2
JK
H m +m 1 m +m 2 1 2

6m + b 3m g 8m + 10m
1 2 1 2
( 1, 6) = ,
m 1 + m2 m1 + m 2
6m 1 3m 2 8m1 + 10m 2
1 = and 6 =
m 1 + m2 m 1 + m2
1 (m1 + m2) = 6m1 3m2 and 6 (m1 + m2) = 8m1 + 10m2
m1 m2 6m1 + 3m2 = 0 and 6m1 + 6m2 + 8m1 10m2 = 0
7m1 + 2m2 = 0 and 14m1 4m2 = 0 or 7m1 2m2 = 0
2m2 = 7m1 and 7m1 = 2m2
m1 2 m1 2
= and =
m2 7 m2 7
m1 : m2 = 2 : 7 and m1 : m2 = 2 : 7
Thus, the required ratio is 2 : 7.
Q. 5. Find the ratio in which the line segment joining A (1, 5) and B ( 4, 5) is divided by the
x-axis. Also find the coordinates of the point of division.
Sol. The given points are: A (1, 5) and B ( 4, 5).
Let the required ratio = k : 1 and the required point be P (x, y).
Part-I: To find the ratio
Since the point P lies on x-axis,
Its y-coordinate is 0.
m1 x2 + m2 x1 m1 y 2 + m2 y 1
x = and 0 =
m1 + m2 m1 + m2

Coordinate Geometry 267


x =
b g af
k 4 +1 1
and 0 =
af b g
k 5 +1 5
k+1 k+1
4k + 1 5k 5
x = and 0 =
k+1 k+1
x (k + 1) = 4k + 1 and 5k 5 = 0 k = 1
Part II : To find coordinates of P :
x (k + 1) = 4k + 1 x (1 + 1) = 4 + 1 [ k = 1]
2x = 3
3
x =
2
The required ratio k : 1 = 1 : 1
FG 3 IJ
Coordinates of P are (x, 0) =
H 2 K
,0 .

Q. 6. If (1, 2), (4, y), (x, 6) and (3, 5) are the vertices of a parallelogram taken in order, find x and y.
Sol. We have the parallelogram vertices
A (1, 2), B (4, y), C (x, 6) and D (3, 5).
Since, the diagonals of a parallelogram bisect each other.
The coordinates of P are:
D (3,5) C (x,6)
x+1 3+4
X = =
2 2 P (X, Y)
x+1 = 7 x=6

5+y 6+2
Y = = A (1,2) B (4,y)
2 2
5+y = 8 y=3
The required values of x and y are:
x = 6, y=3
Q. 7. Find the coordinates of a point A, where AB is the diameter of a circle whose centre is (2, 3)
and B is (1, 4).
Sol. Here, centre of the circle is O (2, 3).
Let the end points of the diameter be
A (X, Y) and B (1, 4) O (2,3)
A (x,y) B (1,4)

The centre of a circle bisects the diameter.


x+1
2 = x + 1 = 4 or x = 3
2
y+4
And 3 = y + 4 = 6 or y = 10
2
Hence the coordinates of A are (3, 10).
Q. 8. If A and B are ( 2, 2) and (2, 4), respectively, find the coordinates of P such that
3
AP = AB and P lies on the line segment AB.
7

268 MathematicsX
Sol.

Here, the given points are


A ( 2, 2) and B (2, 4)
Let the coordinates of P are (x, y).
Since, the point P divides AB such that
3 AP 3
AP = AB or =
7 AB 7
Since AB = AP + BP
AP 3 AP 3
= =
AB 7 AP + AB 7
AP + BP 7
=
AP 3
BP 3+4 4
1+ = = 1+
AP 3 3
BP 4
=
AP 3
AP : PB = 3:4
i.e., P (x, y) divides AB in the ratio 3 : 4.

x =
32+4 2 b g =
68
=
2
3+4 7 7

y =
b g
3 4 +4 2 b g =
12 8
=
20
3+4 7 7
FG 2 , 20 IJ .
Thus, the coordinates of P are
H7 7K
Q. 9. Find the coordinates of the points which divide the line segment joining A ( 2, 2) and B (2, 8)
into four equal parts.
Sol. Here, the given points are:
A ( 2, 2) and B (2, 8)
Let P1, P2 and P3 divide AB in four equal parts.

AP1 = P1 P2 = P2 P3 = P3 B
Obviously, P2 is the mid point of AB
Coordinates of P2 are:
FG 2 + 2 , 2 + 8 IJ
H 2 2 K or (0, 5)

Again, P1 is the mid point of AP2.


Coordinates of P1 are:
FG 2 + 0 , 2 + 5 IJ FG 1, 7 IJ
H 2 2 K or
H 2K
Coordinate Geometry 269
Also P3 is the mid point of P2 B.
Coordinates of P3 are:
FG 0 + 2 , 5 + 8 IJFG 1, 13 IJ
H 2 2 K H 2K
or

Thus, the coordinates of P , P and P are:


1 2 3
F 7 I F 13 I
(0, 5), G 1, J and G 1, J respectively.
H 2K H 2 K
Q. 10. Find the area of a rhombus if its vertices are (3, 0), (4, 5), ( 1, 4) and ( 2, 1) taken in order.
1
[Hint: Area of a rhombus = (product of its diagonals)]
2
Sol. Let the vertices of the given rhombus are:
A (3, 0), B (4, 5), C ( 1, 4) and D ( 2, 1)
B (4,5)

O
A (3,0) (1,4)

D (2,1)
AC and BD are the diagonals of rhombus ABCD.
Diagonal AC = b 1 3g + b 4 0 g
2 2
= b 4g + a 4 f
2 2

= 16 + 16 = 4 2

Diagonal BD = b 2 4 g + b 1 5g
2 2

= b 6g + b 6g = 36 + 36
2 2
= 2 2
For a rhombus,
1
Area = (Product of diagonals)
2
1
= AC BD
2
1
= 4 2 6 2
2
1
= 2 4 6 Square units.
2
= 4 6 = 24 Square units.
z Area of Triangle
I. If A (x1, y1); B (x2, y2) and C (x3, y3) are the vertices of ABC, then
1
the area of ABC = [x1 (y2 y3) + x2 (y3 y1) + x3 (y1 y2)].
2
II. The three points A, B and C are collinear if and only if area of ABC = 0.

270 MathematicsX
NCERT TEXTBOOK QUESTIONS SOLVED
EXERCISE 7.3
Q. 1. Find the area of the triangle whose vertices are:
(i) (2, 3), ( 1, 0), (2, 4) (ii) ( 5, 1), (3, 5), (5, 2)
Sol. (i) Let the vertices of the triangle be
A (2, 3), B ( 1, 0) and C (2, 4)
Here, x 1 = 2, y1 = 3
x 2 = 1, y2 = 0
x 3 = 2, y3 = 4
1
Area of a = [x1 (y2 y3) + x2 (y3 y1) + x3 (y1 y2)]
2
1
Area of ABC = [2 {0 ( 4)} + ( 1) { 4 (3)} + 2 {3 0}]
2
1
= [2 (0 + 4) + ( 1) ( 4 3) + 2 (3)]
2
1
= [8 + 7 + 6]
2
1 21
= 21 = sq. units.
2 2
(ii) Let the vertices of the triangle be
A ( 5, 1), B (3, 5) and C (5, 2)
i.e., x 1 = 5, y1 = 1
x 2 = 3, y2 = 5
x 3 = 5, y3 = 2
1
Area of a = [x1 (y2 y3) + x2 (y3 y1) + x3 (y1 y2)]
2
Area of ABC
1
= [ 5 { 5 2} + 3 {2 ( 1)} + 5 { 1 ( 5)}]
2
1
= [ 5 { 7} + 3 {2 + 1} + 5 { 1 + 5}]
2
1
= [( 5) ( 7) + 3 (3) + 5 (4)]
2
1
= [35 + 9 + 20]
2
1
= 64 = 32 sq. units.
2
Q. 2. In each of the following find the value of k, for which the points are collinear.
(i) (7, 2), (5, 1), (3, k) (ii) (8, 1), (k, 4), (2, 5)
Sol. The given three points will be collinear if the formed by them has zero area.
(i) Let A (7, 2), B (5, 1) and C (3, k) be the vertices of a triangle.
The given points will be collinear, if ar ( ABC) = 0
or 7 (1 k) + 5 (k + 2) + 3 ( 2 1) = 0

Coordinate Geometry 271


7 7k + 5k+ 10 + ( 6) 3 = 0
17 9 + 5k 7k = 0
8 2k = 0
2k = 8
8
k = = 4
2
The required value of k = 4.
(ii) Let (8, 1), (k, 4) and (2, 5) be the verticles of a triangle.
For the above points being collinear, ar ( ABC) = 0
i.e., 8 ( 4 + 5) + k ( 5 1) + 2 [1 ( 4)] = 0
8 (+ 1) + k ( 6) + 2 (5) = 0
8 + ( 6k) + 10 = 0
6k + 18 = 0
k = ( 18) ( 6) = 3
Thus, k = 3.
Q. 3. Find the area of the triangle formed by joining the mid-points of the sides of the triangle whose
vertices are (0, 1), (2, 1) and (0, 3). Find the ratio of this area to the area of the given triangle.
Sol. Let the vertices of the triangle be A (0, 1), B (2, 1) and C (0, 3).
Let D, E and F be the mid-points of the sides BC, CA and AB respectively. Then:
FG 2 + 0 , 1 + 3 IJ i.e., FG 2 , 4 IJ
Coordinates of D are
H 2 2 K H 2 2K or (1, 2)

0 + 0 3 + a 1f
Coordinates of E are: , i.e., (0, 1)
2 2
2 + 0 1 + b 1g
Coordinates of F are: , i.e., (1, 0)
2 2 A (0,1)

(1,0) F E (0,1)

B (2,1) D (1,2) C (0,3)

1
Now, ar ( ABC) = [0 (1 3) + 2 {3 ( 1)} + 0 ( 1 1)]
2
1
= [0 ( 2) + 8 + 0 ( 2)]
2
1 1
= [0 + 8 + 0] = 8 = 4 sq. units
2 2
1
And ar (DEF) = [1 (1 0) + 0 (0 2) + 1 (2 1)]
2
1
= [1 (1) + 0 + 1 (1)]
2
1 1
= [1 + 0 + 1] = 2 = 1 sq. unit
2 2
272 MathematicsX

b
ar DEF g 1
b
ar ABC g =
4
ar ( DEF) : ar ( ABC) = 1 : 4.
Q. 4. Find the area of the quadrilateral whose vertices, taken in order, are ( 4, 2), ( 3, 5),
(3, 2) and (2, 3).
Sol. Let A ( 4, 2), B ( 3, 5), C (3, 2) and D (2, 3) be the vertices of the quadrilateral.
Let us join diagonal BD.
C (3, 2)
D (2,3)

A (4,2) B (3,5)

1
ABD)
Now, ar ( = [( 4) { 5 3} + ( 3) {3 ( 2)} + 2 {( 2) ( 5)}]
2
1
= [( 4) ( 8) + ( 3) (5) + 2 ( 2 + 5)]
2
1
= [32 + ( 15) + 6]
2
1 23
= [23] = sq. units
2 2
1
CBD)
ar ( = [3 ( 5 3) + ( 3) {3 ( 2)} + 2 {( 2) ( 5)}]
2
1
= [3 ( 8) + 3 (5) + 2 (3)]
2
1
= [ 24 15 + 6]
2

=
1
2
[ 33] =
33
2
b
sq. units, numerically g
Since, ar (quad ABCD) = ar ( ABD) + ar ( CBD)
FG 23 + 33 IJ sq. units
ar (quad ABCD) = H2 2K
56
= sq. units = 28 sq. units.
2
Q. 5. You have studied in class IX (Chapter 9, Example-3) that, a median of a triangle divides it into
two triangles of equal areas. Verify this result for ABC whose vertices are A (4, 6),
B (3, 2) and C (5, 2).
Sol. Here, the vertices of the triangle are A (4, 6), B = (3, 2) and C (5, 2).
Let D be the mid-point of BC.

Coordinate Geometry 273


The coordinates of the mid point D are:
RS 3 + 5 , 2 + 2 UV or (4, 0).
T 2 2 W
Since AD divides the triangle ABC into two parts i.e., ABD and ACD,
1
ABD)
Now, ar ( = [4 {( 2) 0} + 3 (0 + 6) + 4 ( 6 + 2)]
2
1
= [( 8) + 18 + ( 16)]
2

=
1
2
b g
6 = 3 sq. units.
= 3 sq. units (numerically) ...(1)

A (4,6)

B (3,2) D C (5,2)
(4,0)
1
ACD)
ar ( = [4 (0 2) + 4 (2 + 6) + 5 ( 6 0)]
2
1
= [ 8 + 32 30]
2
1
= 6 = 3 sq. units
2
= 3 sq. units (numerically) ...(2)
From (1) and (2)
ar ( ABD) = ar ( ACD)
i.e. A median divides the triangle into two triangles of equal areas.

NCERT TEXTBOOK QUESTIONS SOLVED


EXERCISE 7.4
Q. 1. Determine the ratio in which the line 2x + y 4 = 0 divides the line segment joining the points
A (2, 2) and B (3, 7).
Sol. Let the required ratio be k : 1 and the point C divides them in the above ratio.
Coordinates of C are:
F 3k + 2 , 7k 2 I
GH k + 1 k+1 JK
274 MathematicsX
Since the point C lies on the given line 2x + y 4 = 0,
We have:
3k + 2 7k 2
2 + 4 = 0
k + 1 k + 1
2 (3k + 2) + (7k 2) = 4 (k + 1)
6k + 4 + 7k 2 4k 4 = 0
(6 + 7 4) k + (4 2 4) = 0
9k + ( 2) = 0
9k 2 = 0
2
k =
9
The required ratio
= k:1
2
= : 1
9
= 2:9
Q. 2. Find a relation between x and y if the points (x, y), (1, 2) and (7, 0) are collinear.
Sol. The given points are:
A (x, y), B (1, 2) and C (7, 0)
The points A, B and C will be collinear if
x (2 0) + 1 (0 y) + 7 (y 2) = 0
or if 2x y + 7y 14 = 0
or if 2x + 6y 14 = 0
or if x + 3y 7 = 0
which is the required relation between x and y.
Q. 3. Find the centre of a circle passing through the points (6, 6), (3, 7) and (3, 3).
Sol. Let P (x, y) be the centre of the circle passing through
A (6, 6), B (3, 7) and C (3, 3)
AP = BP = CP
Taking AP = BP, we have AP2 = BP2
(x 6)2 + (y + 6)2 = (x 3)2 + (y + 7)2
x 12x + 36 + y2 + 12y + 36 = x2 6x + 9 + y2 + 14y + 49
2

12x + 6x + 12y 14y + 72 58 = 0


6x 2y + 14 = 0
3x + y 7 = 0 ...(1) [Dividing by ( 2)]
Taking BP = CP, we have BP = CP 2 2

(x 3)2 + (y + 7)2 = (x 3)2 + (y 3)2


x 6x + 9 + y2 + 14y + 49 = x2 6x + 9 + y2 6y + 9
2

6x + 6x + 14y + 6y + 58 18 = 0
20y + 40 = 0
40
y = = 2 ...(2)
20
From (1) and (2),
3x 2 7 = 0
3x = 9 x = 3

Coordinate Geometry 275


i.e., x = 3 and y = 2
The required centre is (3, 2).
Q. 4. The two opposite vertices of a square are ( 1, 2) and (3, 2). Find the coordinates of the other two
vertices.
D C (3,2)
Sol. Let us have a square ABCD such that A ( 1, 2)
and C (3, 2) are the opposite vertices.
Let B (x, y) be an unknown vertex.
Since all sides of a square are equal,
AB = BC
AB2 = BC2
B
(x + 1)2 + (y 2)2 = (x 3)2 + (y 2)2 A (1,2)
2x + 1 = 6x + 9
8x = 8 x = 1 ...(1)
Since each angle of a square = 90,
ABC is a right angled triangle.
Using Pythagoras theorem, we have:
AB2 + BC2 = AC2
[(x + 1) + (y 2)2] + [(x 3)2 + (y 2)2] = [(3 + 1)2 + (2 2)2]
2

2x2 + 2y2 + 2x 4y 6x 4y + 1 + 4 + 9 + 4 = 16
2x2 + 2y2 4x 8y + 2 = 0
x2 + y2 2x 4y + 1 = 0 ...(2)
Substituting the value of x from (1) into (2) we have:
1 + y2 2 4y + 1 = 0
y2 4y + 2 2 = 0
y2 4y = 0
y (y 4) = 0
y = 0 or y = 4
Hence, the required other two vertices are: (1, 0) and (1, 4).
Q. 5. The Class X students of a secondary school in Krishinagar have been allotted a rectangular plot
of land for their gardening activity. Saplings of Gulmohar are planted on the boundary at a
distance of 1 m from each other. There is a triangular grassy lawn in the plot as shown in the
Fig. The students are to sow seeds of flowering plants on the remaining area of the plot.
(i) Taking A as origin, find the coordinates of the vertices of the triangle.
(ii) What will be the coordinates of the vertices of PQR if C is the origin?
Also calculate the areas of the triangles in these cases. What do you observe?
B C

P
R

Q
A 1 2 3 4 5 6 7 8 9 10 D

Sol. (i) By taking A as the origin and AD and AB as the coordinate axes, we have
P (4, 6), Q (3, 2) and R (6, 5) as the vertices of PQR.

276 MathematicsX
(ii) By taking C as the origin and CB and CD as the coordinate axes, then the vertices
of PQR are
P (12, 2), Q (13, 6) and R (10, 3)
Now, ar ( PQR) [when P (4, 6), Q (3, 2) and R (6, 5) are the vertices]
1
= [4 (2 5) + 3 (5 6) + 6 (6 2)]
2
1
= [ 12 3 + 24]
2
9
= sq. units. [taking numerical value]
2
ar ( PQR) [when P (12, 2), Q (13, 6) and R (10, 3) are the vertices.]
1
= [12 (6 3) + 13 (3 2) + 10 (2 6)]
2
1
= [36 + 13 40]
2
9
= sq. units.
2
Thus, in both cases, the area of PQR is the same.
Q. 6. The vertices of a ABC are A (4, 6), B (1, 5) and C (7, 2). A line is drawn to intersect sides AB
AD AE 1
and AC at D and E respectively, such that = = . Calculate the area of the
AB AC 4
ADE and compare it with the area of ABC. (Recall Theorem 6.2 and Theorem 6.6).
AD 1
Sol. We have =
AB 4
AB 4
=
AD 1
AD + DE 4
=
AD 1
AD DE 4 3
+ = = 1+
AD AD 1 1
DE 3 DE 3
1+ = 1+ =
AD 1 AD 1
AD : DE = 1 : 3
Thus, the point D divides AB in the ratio 1 : 3
The coordinates of D are:
LM b1 1g + b3 4g , b1 5g + b3 6g OP
MN 1 + 3 1+3 PQ
or LM
1 + 12 5 + 18 O
N 4 , 4 PQ
or FG ,
13 23 I
H 4 4 JK
Similarly, AE : EC = 1 : 3
i.e., E divides AC in the ratio 1 : 3

Coordinate Geometry 277


Coordinates of E are:
LM b1 7g + b3 4g , 1 2 + 3 6 OP
MN 1 + 3 1+3 PQ
or M
L 7 + 12 , 2 + 18 OP
N 4 4 Q

or LM , 5OP
19
N4 Q
Now, ar ( ADE)
1 L F 23 I 13 b5 6g + 19 FG 6 23 IJ OP
4G 5J +
2 MN H 4 K 4 4 H 4 KQ
=

1L
=
2N
Mb23 20g + 134 a1f + 194 FGH 24 4 23 IJK OPQ A (4,6)

1F 13 19 I
= G3 + J
2H 4 16 K
1 L 48 + 52 + 19 O 15 D E
= M P =
2N 16 Q 32 sq. units.
Area of ABC
1
= [4 (5 2) + 1 (2 6) + 7 (6 5)] C
2 B (1,5) (7,2)
1
= [(4 3) + 1 ( 4) + 7 1]
2
1
= [12 + ( 4) + 7]
2
=
1
2
a f
15 =
15
2
sq. units.

15
Now,
b
ar ADE g
= 32 =
15

2
=
1
b
ar ABC g 15 32 15 16
2
ar ( ADE) : ar ( ABC) = 1 : 16.
Q. 7. Let A (4, 2), B (6, 5) and C (1, 4) be the vertices of ABC.
(i) The median from A meets BC at D. Find the coordinates of the point D.
(ii) Find the coordinates of the point P on AD such that AP : PD = 2 : 1.
(iii) Find the coordinates of points Q and R on medians BE and CF respectively such that
BQ : QE = 2 : 1 and CR : RF = 2 : 1.
(iv) What do you observe?
[Note: The point which is common to all the three medians is called the centroid and this
point divides each median in the ratio 2 : 1.]
(v) If A (x1, y1), B (x2, y2) and C (x3, y3) are the vertices of ABC, find the coordinates of
the centroid of the triangle.
Sol. We have the vertices of ABC as A (4, 2), B (6, 5) and C (1, 4).
(i) Since AD is a median

278 MathematicsX
FG 6 + 1 , 5+4 IJ FG 7 , 9 IJ
Coordinates of D are:
H 2 2 K or
H 2 2K
A (4,2)

F
E
Q

B (6,5)
D
C (1,4)

(ii) Since AP : PD = 2 : 1 i.e., P divides AD in the ratio 2 : 1.


Coordinates of P are:
LM 2 F 7 I + b1 4g 2 F 9 I + 1 2 OP
MM H 2 K2 + 1 , H 22K + 1 PP or FGH 113 , 113IJK
MN PQ
(iii) BQ : QE = 2 : 1 [The point Q divides BE in the radio 2 : 1]
Coordinates of Q are:
LM 2 F 5 I + 1 6 O
H 2K b 2 3g + b1 5g P
MM 2 + 1 , 2 + 1 PP
MN PQ
or M
L 5 + 6 , 6 + 5 OP
N 3 3 Q
or M ,
L 11 11 OP
N3 3Q
Coordinates of Q are:
FG 4 + 6 , 2 + 5 IJ or FG 5, 7 IJ
H 2 2 K H 2K
Coordinates of R are:
LM 7 O
2 5 + 1 1 2 2 + 1 4P
MM 2 1 , 2 + 1 PP
N Q
or M
L , OP
10 + 1 7 + 4
N 3 3 Q
or M ,
L 11 11 OP
N3 3Q
(iv) We observe that P, Q and R represent the same point.

Coordinate Geometry 279


(v) Here, we have A (x , y)
A (x1, y1), B (x2, y2), C (x3, y3) as the
vertices of ABC. Also AD, BE and
CF are its medians.
D, E and F are the mid points of F
BC, CA and AB respectively. E
We know, the centroid is a point on
a median, dividing it in the ratio
2 : 1.
B (x2 , y2)
Concidering the median AD, D
C (x3 , y3)
Coordinates of AD are:
LM
x2 + x 3 y 2 + y 3
,
OP
q
N 2 2 Q
Let G be the centroid.
Coordinates of the centroid are:
LM b1 x g + 2 FG x + x IJ b1 y g + 2 FG y
2 3 2 y3 IJ OP
MM 1
H 2 K, H 1
2 KP
MN 1+2 1+2 PP
Q
= M
L x + x + x , y + y + y OP
1 2 3 1 2 3
N 3 3 Q
Similarly, considering the other medians we find that in each the coordinates of G
x + x 3 + x3 y 1 + y 2 + y 3
are 1 , .
3 3
i.e., The coordinates of the centroid are
x1 + x2 + x 3 y 1 + y 2 + y 3
,
LM. OP
3 3 N Q
Q. 8. ABCD is a rectangle formed by the points A ( 1, 1), B ( 1, 4), C (5, 4) and D (5, 1).
P, Q, R and S are the mid points of AB, BC, CD and DA respectively. Is the quadrilateral PQRS
a square? a rectangle? or a rhombus? Justify your answer.
Sol. We have a rectangle whose vertices are A ( 1, 1), B ( 1, 4), C (5, 4) and D (5, 1).
A (1,1) P (1,3/2)
B (1,4)

S (2,1) Q (2,4)

D (5,1) R (5, 3 ) C (5,4)


2
P is mid-point of AB
Coordinates of P are:
LM 1 1 ,
1+4 3 OP FG IJ
N 2 2
or 1,
2 Q H K
Similarly, the coordinates of Q are:
280 MathematicsX
FG 1 + 5 , 4 + 4 IJ or (2, 4)
H 2 2 K
Coordinates of R are:
FG 5 + 5 , 1 + 4 IJ or FG 5, 3 IJ
H 2 2 K H 2K
Coordinates of S are:
FG 1 + 5 , 1 1 IJ or (2, 1)
H 2 2 K

2 F 3I
PQ = b2 + 1g + G 4 J = 9 +
2
25 61
Now,
H 2K 4
=
2

2 F3 I
SR = b5 2g + G 4J = 9 +
2
25 61
H2 K 4
=
2

2 R F 3I U
RS = b2 5g + S 1 + G J V = 9 +
25
2
61
T H K
2 W 4
=
2

2 F 3I
SP = b2 + 1g + G 1 J = 9 +
25
2
61
H 2K 4
=
2

F 3 3I
SR = b5 + 1g + G J = 6 + 0 = 6
2
2
2
H 2 2K
QS = b2 2g + b 4 + 1g = 0 + 5 = 5
2 2

We see that:
PQ = QR = RS = SP
i.e., all sides of PQRS are equal.
It can be a square or a rhombus.
But its diagonals are not equal.
i.e., PR QS
PQRS is a rhombus.

MORE QUESTIONS SOLVED


I. VERY SHORT ANSWER TYPE QUESTIONS
Q. 1. Find a point on the y-axis equidistant from ( 5, 2) and (9, 2). (CBSE 2012)
Sol. Let the required point on the y-axis be P (0, y)
PA = PB

b0 + 5g + b y 2g
2 2
= b0 9 g + b y + 2g
2 2

52 + y 2 + 4 4y = b 9g + y + 4 + 4 y
2 2

25 + y2 + 4 4y = 81 + y2 + 4 + 4y
y2 y2 4y 4y = 81 + 4 4 25
8y = 85 29
8y = 56
Coordinate Geometry 281
56
= 7y =
8
The required point is (0, 7).
Q. 2. Find a point on x-axis at a distance of 4 units from the point A (2, 1).
Sol. Let the required point on x-axis be P (x, 0).
PA = 4
bx 2g + b0 1g
2 2
= 4
x2
4x + 4 + 1 = 42 = 16
x2 4x + 1 + 4 16 = 0
x2 4x 11 = 0
x = 2 15
Thus, the coordinates of P are: 2 15 , 0 . e j
Q. 3. Find the distance of the point (3, 4) from the origin.
Sol. The coordinates of origin (0, 0).
Distance of (3, 4) from the origin
= b 3 0g + b 4 0g
2 2

= a3f + b 4g = 9 + 16
2 2
= 25 = 5 .
Q. 4. For what value of x is the distance between the points A ( 3, 2) and B (x, 10) 10 units?
Sol. The distance between A ( 3, 2) and B (x, 10)
= bx + 3g + b10 2g
2 2

bx + 3g + b10 2g
2 2
= 10
(x + 3)2 + (8)2
= 10 2
(x + 3)2 = 102 82
(x + 3)2 = (10 2) (10 + 8) = 36
x + 3 = 36 = 6
For +ve sign, x = 63=3
For ve sign, x = 63=9
Q. 5. Find a point on the x-axis which is equidistant from the points A (5, 2) and B (1, 2).
Sol. The given points are:
A (5, 2) and B (1, 2)
Let the required point on the x-axis be C (x, 0).
Since, C is equidistant from A and B.
AC = BC
b x 5g + b0 2g
2 2
= bx 1g + b0 + 2g
2 2

(x 5)2 + ( 2)2 = (x 1)2 + (2)2


x2 + 25 10x + 4 = x2 + 1 2x + 4
10x + 2x = 5 29
8x = 24
24
x = = 3
8
The required point is (0, 3).
282 MathematicsX
Q. 6. Establish the relation between x and y when P (x, y) is equidistant from the points A ( 1, 2) and
B (2, 1).
Sol. P is equidistant from A and B
PA = PB

bx + 1g + by 2g
2 2
= bx 2g + by + 1g
2 2

(x + 1)2 + (y 2)2 = (x 2)2 + (y + 1)2


x2 + 1 + 2x + y2 4y + 4 = x2 + 4 4x + y2 + 1 + 2y
2x 4x + 5 = 4x + 2y + 5
2x + 4x + 5 = 2y + 4y + 5
6x = 6y
x = y
which is the required relation.
Q. 7. Show that the points (7, 2), (2, 3) and ( 1, 6) are collinear.
Sol. Here, the vertices of a triangle are (7, 2), (2, 3) and ( 1, 6)
Area of the triangle
1
= [x1 (y2 y3) + x2 (y3 y1) + x3 (y1 y2)]
2
1
= [7 (3 6) + 2 (6 + 2) + ( 1) ( 2 3)]
2
1
= [7 ( 3) + 2 8 + ( 1) ( 5)]
2
1
= [ 21 + 16 + 5]
2
1
= [ 0] = 0
2
Since area of triangle = 0
The vertices of the triangle are collinear.
Thus, the given points are collinear.
Q. 8. Find the distance between the points
FG 8 , 2IJ FG 2 , 2IJ
H5 K H5 K
and (CBSE 2009)

Distance between G
F 8 , 2IJ FG 2 , 2IJ
Sol.
H5 K and
H5 K is given by

FG 2 + 8 IJ + b2 2g
2
2
2 2 0 2 = 2 units.
H 5 5K =

Q. 9. If the mid point of the line joining the points P (6, b 2) and Q ( 2, 4) is (2, 3), find the
value of b. (CBSE 2009 F)
Sol. Here, P (6, b 2) and Q ( 2, 4) are the given points.
Mid point of PQ is given by:

Coordinate Geometry 283


LM 6 + b 2g , 4 + b 2 OP
MN 2 2 PQ
or M
L 6 2 , 4 2 + b OP
N 2 2 Q
L 2 + b OP
or M2 ,
N 2 Q
2+b
= 3 2+b=6
2
b = 62
b = 8
Q. 10. In the given figure, ABC is a triangle. D and E are the mid points of the sides BC and AC
1
respectively. Find the length of DE. Prove that DE = AB (CBSE 2011)
2
Sol. Co-ordinates of the mid point of BC are: C
-6 + 2 -1 + ( -2) (4, 2)
= ,
2 2

-3 -3
= -2,

E -2, D
2 2
Co-ordinates of the mid point of AC are:
-6 + 4 -1 + ( -2) (2, 2)
= , C
2 2 (6, 1) E B

-3 -3
= -1, D -1,
2 2

3
2
Now, .
DE = ( -2 + 1)2 + -3 / 2 +
2
1
= ( -1)2 + 0 = 1 DE = AB Hence proved.
2
AB = (4 - 2)2 + ( -2 + 2)2

= 22 + 0 = 2

II. SHORT ANSWER TYPE QUESTIONS


Q. 1. Points P (5, 3) is one of the two points of trisection of the line segment joining the points A (7, 2)
and B (1, 5) near to A. Find the coordinates of the other point of trisection.
(AI CBSE 2010)
Sol.

Since P is near to A
284 MathematicsX
other point Q is the mid point of PB
5+1
x = =3
2
3 5 8
y = = =4
2 2
Thus, the point Q is (3, 4)
Q. 2. Find the area of the quadrilateral ABCD whose vertices are A (1, 0), B (5, 3), C (2, 7) and
D ( 2, 4). [AI CBSE 2009]
1
Sol. Area of ABC = [1 (3 7) + 5 (7 0) + 2 (0 3)]
2
1 1
= [ 4 + 35 6] = 25
2 2
25
= sq. units.
2
1
Area of ACD = [1 (7 4) + 2 (4 0) + ( 2) (0 7)]
2
1 1 D C
= [3 + 8 + 14] = 25
2 2
25
= sq. units.
2
A
Area of the quad. ABCD
25 25
= sq. units + sq. units. B
2 2
= 25 sq. units.
Q. 3. Points P, Q, R and S, in this order, divide a line segment joining A (2, 6), B (7, 4) in five equal
parts. Find the coordinates of P and R. [AI CBSE 2009 Comptt.]
Sol.

A (2, 6) P Q R S B (7, 4)
P, Q, R and S divide AB in five equal parts.
AP = PQ = QR = RS = SB
Now, P divides AB in the ratio 1 : 4
Coordinates of P are:
LM 1 7 + 4 2 , 1 b 4g + 4 6 OP
MN 1 + 4 1+4 PQ
or M
L 7 + 8 , 4 + 24 OP or (3, 4)
N 5 5 Q
Again, R divides AB in the ratio 3 : 2
Coordinates of R are:
LM 2 2 + 3 7 , 2 6 + 3 b 4g OP L 4 + 21 0 O
MN 2 + 3 2+3 PQ or MN 5 , 5 PQ or (5, 0)
Coordinate Geometry 285
Q. 4. A ( 4, 2), B ( 3, 5), C (3, 2) and D (2, k) are the vertices of a quad. ABCD. Find the
value of k, if the area of the quad is 28 sq. units.
Sol. Area of quad ABCD = 28 sq. units
[ar ( ABD)] + [ar ( BCD)] = 28 sq. units
1
[ 4 ( 5 k) 3 (k + 2) + 2 ( 2 + 5)]
2
1
+ [ 3 ( 2 k) + 3 (k + 5) + 2 ( 5 + 2)] = 28
2
1 1 C (3, 2)
[20 + 4k 3k 6 + 6] + [6 + 3k + 3k + 15 6] = 28
2 2
1 D (2,k)
1
[k + 20] + [6k + 15] = 28
2 2
k + 20 + 6k + 15 = 56
7k + 35 = 56
B (3, 5)
7k = 56 35 = 21
21
k = = 3 A (4,2)
7
Q. 5. Find the point on y-axis which is equidistant from the points (5, 2) and ( 3, 2).
Sol. Let the required point be P (0, y)
The given points are A (5, 2) and B ( 3, 2)
PA = PB
PA2 = PB2
(5 0)2 + ( 2 y)2 = ( 3 0)2 + (2 y)2
52 + ( 2 y)2 = ( 3)2 + (2 y)2
25 + 4 + y2 + 4y = 9 + 4 + y2 4y
25 + 4y = 9 4y
8y = 16 y = 2
Thus, the required point is (0, 2)
Q. 6. Find the point on y-axis which is equidistant from ( 5, 2) and (9, 2). (CBSE 2009 C)
Sol. Let the required point on Y-axis be P (0, y).
The given points are A ( 5, 2) and B (9, 2)
AP = BP
b0 + 5g + b y 2g
2 2
= b0 9 g + b y + 2g
2 2

52 + (y 2)2 = 92 + (y + 2)2
25 + y2 4y + 4 = 81 + y2 + 4 + 4y
4y 4y = 81 + 4 4 25
8y = 56
56
y = = 7
8
The required point = (0, 7)
Q. 7. Find the value of x for which the distance between the points P (4, 5) and Q (12, x) is 10 units.
(CBSE 2009 C)
Sol. The given points are P (4 5) and Q (12, x) such that PQ = 10

286 MathematicsX
b12 4g + bx + 5g
2 2
= 10
(12 4)2 + (x + 5)2 = 10 2
82 + (x + 5)2 = 100
2
64 + x + 25 + 10x = 100
x2 + 10x 11 = 0
(x 1) (x + 11) = 0
x = 1 or x = 11
Q. 8. Find the relation between x and y if the points (2, 1), (x, y) and (7, 5) are collinear.
(AI CBSE 2009)
Sol. Here, x 1 = 2, y1 = 1
x 2 = x, y2 = y
x 3 = 7, y3 = 5
1
Area of triangle = [x1 (y2 y3) + x2 (y3 y1) + x3 (y1 y2)]
2
1
= [2 (y 5) + x (5 1) + 7 (1 y)]
2
1
= [2y 10 + 5x x + 7 7y]
2
1
= [ 5y + 4x 3]
2
ar () = 0
1
[ 5y + 4x 3] = 0
2
4x 5y 3 = 0
which is the required relation.
Q. 9. If A ( 2, 4), B (0, 0) and C (4, 2) are the vertices of ABC, then find the length of the median
through the vertex A. (CBSE 2009 C)
Sol. AD is the median on BC
D is the mid-point of BC. A (2 ,4)
Coordinates of D are:
FG 0 + 4 ,
0+2 IJ
H 2 2 K
i.e., (2, 1)

Now, the length of the median


AD = b2 + 2g + b1 4 g
2 2

= a 4 f + b 3g
2 2 B
(0,0) D
C
(4,2)
= 16 + 9 = 25 = 5 units.
Q. 10. If the points A (4, 3) and B (x, 5) are on the circle with the centre O (2, 3), find the value of x.
(AI CBSE 2009)
Sol. Let O (2, 3) be the centre of the circle.
OA = OB OA2 = OB2
(4 2) + (3 3)2 = (x 2)2 + (5 3)2
2

Coordinate Geometry 287


2 2 = (x 2)2 + 22
(x 2)2 = 0
x2 = 0
x = 2
Thus, the required value of x is 2.
Q. 11. If the vertices of a are (2, 4), (5, k) and (3, 10) and its area is 15 sq. units, then find the value
of k. (AI CBSE 2008)
Sol. The area of the given
1
= [2 (k 10) + 5 (10 4) + 3 (4 k)]
2
1
= [2k 20 + 50 20 + 12 3k]
2
1
= [ k + 22]
2
But ar () = 15 [given]
1
[ k + 22] = 15
2
k + 22 = 30
k = 30 22 = 8
k = 8
Q. 12. The vertices of a triangle are:
(1, k), (4, 3), ( 9, 7) and its area is 15 sq. units. Find the value of k. (AI CBSE 2008)
Sol. Area of the given triangle
1
= [1 ( 3 7) + 4 (7 k) 9 (k + 3)] = 15
2
1
[ 10 + 28 4k 9k 27] = 15
2
13k 9 = 30
13k = 39
39
k = k=3
13
Q. 13. Find the area of a ABC whose vertices are A ( 5, 7), B ( 4, 5) and C (4, 5).
(AI CBSE 2008)
Sol. Here, x 1 = 5, y1 = 7
x 2 = 4, y2 = 5
x 3 = 4, y3 = 5
1
Now, ar () = [x1 (y2 y3) + x2 (y3 y1) + x3 (y1 y2)]
2
1
= [( 5) ( 5 5) + ( 4) (5 7) + 4 (7 + 5)]
2
1
= [50 + 8 + 48]
2
1
= 106 = 53
2
The required ar ( ABC) = 53 sq. units.

288 MathematicsX
Q. 14. Find the value of k such that the points (1, 1), (3, k) and ( 1, 4) are collinear.
(AI CBSE 2008)
Sol. For the three points, to be collinear, the area of triangle formed by them must be zero.
Area of triangle = 0
1
[1 (k 4) + (3) (4 1) + ( 1) (1 k)] = 0
2
1
[k 4 + 9 1 + k] = 0
2
1
[2k + 4] = 0
2
k+2 = 0
k = 2
Q. 15. For what value of p, the points ( 5, 1), (1, p) and (4, 2) are collinear? (CBSE 2008)
Sol. Since the points are collinear,
The area of the formed by these points must be zero.
1
i.e., [ 5 (p + 2) + 1 ( 2 1) + 4 (1 p)] = 0
2
5p 10 3 + 4 4p = 0
9p 9 = 0
9p = 9
9
p = = 1
9
Q. 16. For what value of p, are the points (2, 1), (p, 1) and ( 1, 3) collinear? (CBSE 2008)
Sol. The given points are collinear.
The area of a triangle formed by these points must be zero.
i.e., Area of triangle = 0
1
[2 ( 1 3) + p (3 1) + ( 1) (1 + 1)] = 0
2
1
[8 + 2p 2] = 0
2
1
[ 10 + 2p] = 0
2
5+p = 0
p = 5
Q. 17. Find the ratio in which the line 3x + 4y 9 = 0 divides the line segment joining the points (1, 3)
and (2, 7). (CBSE 2008)
Sol. Let the ratio be k : 1.

Coordinates of R are:
F 2k + 1 , 7k + 3 I
GH k + 1 k+1 JK
Coordinate Geometry 289
Since, R lies on the line 3x + 4y 9 = 0
F 2k + 1 I + 4 F 7 k + 3 I 9
3 GH k + 1 JK GH k + 1 JK = 0

6k + 3 + 28k + 12 9k + 9 = 0
(6k + 28k 9k) + (3 + 12 9) = 0
25k + 6 = 0
6
k =
25
The required ratio is
6 : 25 or 6 : 25
Q. 18. If the point P (x, y) is equidistant from the points A (3, 6) and B ( 3, 4), prove that
3x + y 5 = 0. (AI CBSE 2008)
Sol.

P is equidistant from A and B.


AP = BP
AP2 = BP2
(x 3) + (y 6)2 = (x + 3)2 + (y 4)2
2

x 6x + 9 + y2 12y + 36 = x2 + 6x + 9 + y2 8y + 16
2

x2 + y2 6x + 45 = x2 + y2 + 6x 8y + 25
( 6x 6x) + ( 12y + 8y) + 45 25 = 0
12x + 20 4y = 0
3x y + 5 = 0
or 3x + y 5 = 0
Q. 19. The coordinates of A and B are (1, 2) and (2, 3). If P lies on AB, then find the coordinates of P
such that:
AP 4
= (AI CBSE 2008)
PB 3
AP 4
Sol. =
PB 3
AP : PB = 4 : 3

Here, P divides AB internally in the ratio 4 : 3.


P has coordinates as:
LM 4 2 + 3 1 , 4 3 + 3 2 OP
N 4+3 4+3 Q
or M
L 8 + 3 , 12 + 6 OP
N 7 7 Q
or LM ,
11 18 O
N 7 7 PQ
290 MathematicsX
Q. 20. If A (4, 8), B (3, 6) and C (5, 4) are the vertices of a ABC, D(4, 1) is the mid-point of BC
AP
and P is a point on AD joined such that = 2 , find the coordinates of P.
PD
(AI CBSE 2008)
Sol. D is the mid-point of B

We have D LM 3 + 5 ,64 OP or D [4, 1] A (4,8)

N 2 2 Q
AP 2 2
Since, =
PD 1
AP : PD = 2:1 P

Coordinates of P are:
LM 2 4 + 1 4 , 2 1 + 1 b 8g OP 1

MN 2 + 1 2+1 PQ B (3,6) D (4,1) C (5,4)

or M
L 8 + 4 , 2 8 OP
N 3 3 Q
L 12 6 OP or [4, 2]
or M ,
N3 3 Q
Q. 21. Show that the triangle PQR formed by the points P( 2 , 2 ), Q( - 2 , - 2 ) and R( - 6 , - 6 )
is an equilateral triangle.
OR
Name the type of triangle PQR formed by the points P( 2 , 2 ), Q( - 2 , - 2 ) and

R( - 6 , - 6 ). [NCERT Exemplar]

Sol. We have, P( 2 , 2 )

Q( - 2 , - 2 )

and R( - 6 , - 6 )
PQ = ( 2 + 2 )2 + ( 2 + 2 )2 = (2 2 )2 + (2 2 )2
= 4 2 + 4 2 = 8 + 8 = 16 = 4
PR = ( 2 + 6 )2 + ( 2 - 6 )2

= 2 + 6 + 2 12 + 2 + 6 - 2 12 = 2 + 6 + 2 + 6 = 16 = 4
2
RQ = ( - 2 ) + 6 + ( - 2 - 6 ) 2

=2 + 6 - 2 12 + 2 + 6 + 2 12 = 2 + 6 + 2 + 6 = 16 = 4
Since, PQ = PR = RQ = each (= 4)
PQR is an equilateral triangle.
Q. 22. The line joining the points (2, 1) and (5, 6) is bisected at P. If P lies on the line
2x + 4y + k = 0, find the value of k. (AI CBSE 2008)
Sol. We have A (2, 1) and B (5, 6).
Coordinate Geometry 291
P is the mid point of AB,

Coordinates of P are:
LM 2 + 5 ,
16 7 7
or ,
OP
N 2 2 2 2 Q
Since P lies on the line 2x + 4y + k = 0
We have:
FG 7 IJ + 4 FG 7 IJ + k = 0
2x + 4y + k = 0 2
H 2K H 2 K
7 14 + k = 0
7+k = 0 k=7
Q. 23. Find the point on y-axis which is equidistant from the points (5, 2) and ( 3, 2).
(CBSE 2009)
Sol. Let P is on the y-axis
Coordinates of P are: (0, y)
Since, PA = PB
PA2 = PB2
(5 0)2 + ( 2 y)2 = ( 3 0)2 + (2 y)2
25 + 4 + 4y + y2 = 9 + 4 4y + y2
25 + 4y = 9 4y
8y = 16
16
y = = 2
8
The required point is (0, 2).
Q. 24. The line joining the points (2, 1) and (5, 8) is trisected at the points P and Q. If point P lies
on the line 2x y + k = 0, find the value of k. (CBSE 2009)
Sol.

AB is trisected at P and Q
Coordinates of P are:
LM 1 5 + 2 2 , 1 b 8g + 2 1 OP
MN 1 + 2 1+2 PQ
F 9 6 IJ or (3, 2)
or G ,
H3 3 K
Since, P (3, 2) lies on 2x y + k = 0
We have:
2 (3) ( 2) + k = 0
6+2+k = 0
8+k = 0 k=8

292 MathematicsX
Q. 25. If P (x, y) is any point on the line joining the points A (a, 0) and B (0, b), then show that:
x y
+ = 1 (CBSE 2009)
a b
Sol. P lies on the line joining A and B.
A, B and P are collinear.
The area of a formed by A (a, 0), B (0, b) and P (x, y) is zero.
We have:
x1 [y2 y3] + x2 [y3 y2] + x3 [y1 y2] = 0
x [0 b] + a [b y] + 0 [y 0] = 0
bx + ab ay = 0
(bx + ay) = ab
bx + ay = ab
bx ay ab
+ = [Dividing by ab]
ab ab ab
x y
+ = 1
a b
Q. 26. Find the point on x-axis which is equidistant from the points (2, 5) and ( 2, 9).
(CBSE 2009)
Sol. The required point P is on x-axis.
Coordinates of P are (x, 0).
We have
AP = PB
AP2 = PB2
(2 x)2 + ( 5 + 0)2 = ( 2 x)2 + (9 0)2
4 4x + x2 + 25 = 4 + 4x + x2 + 81
4x + 25 = 4x + 81
8x = 56
56
x = = 7
8
The required point is ( 7, 0).
Q. 27. The line segment joining the points P (3, 3) and Q (6, 6) is trisected at the points A and B such
that A is nearer to P. It also lies on the line given by 2x + y + k = 0. Find the value
of k. (CBSE 2009)
Sol. PQ is trisected by A such that

PA : AQ = 1 : 2
The coordinates of A are:
LM 1 6 + 2 3 , 1 b 6g + 2 3 OP
MN 1 + 2 1+2 PQ
or LM 6 + 6 , 6 + 6 OP
N 3 3 Q

or LM 12 , 0 OP or (4, 0).
N 3 3Q
Coordinate Geometry 293
Since, A (4, 0) lies on the line 2x + y + k = 0
2 (4) + (0) + k = 0
8+k = 0 k=8
Q. 28. Find the ratio in which the points (2, 4) divides the line segment joining the points A ( 2, 2)
and B (3, 7). Also find the value of y. (AI CBSE 2009)
Sol. Let P (2, y) divides the join of A ( 2, 2) and B (3, 7) in the ratio k:1
Coordinates of P are:
3k 2 7 k + 2
,
k+1 k+1
3k 2 7k + 2
= 2 and =y
k+1 k+1
3k 2
Now, = 2 3k 2 = 2k + 2 k = 4
k+1

And
7k + 2
= 7
af
7 4 +2
=y
k+1 4+1
30
= y 6=y
5
Thus, y = 6 and k = 4
Q. 29. Find the area of the quadrilateral ABCD whose vertices are:
A ( 4, 2), B ( 3, 5), C (3, 2) and D (2, 3) (AI CBSE 2009)
1
Sol. Area of ( ABC) = [( 4) ( 5 + 2) + ( 3) ( 2 + 2) + 3 ( 2 + 5)]
2
1
= [ 4 ( 3) + ( 3) (0) + 3 (3)]
2
1
= [ 12 + 0 + 9]
2
1 21
= 21 = sq. units.
2 2
C (3, 2)

D (2 , 3)

B (3,5)
A (4,2)
1
Also, ar ( ACD) = [(4) (2 3) + 3 (3 + 2) + 2 (2 + 2)]
2
1
= [20 + 15 + 0]
2
35
= sq. units.
2
ar (quad. ABCD) = ar ( ABC) + ar ( ACD)

294 MathematicsX
21 35
= + sq. units.
2 2
56
= = 28 sq. units.
2
Q. 30. Find the ratio in which the point (x, 2) divides the line segment joining the points ( 3, 4) and
(3, 5). Also find the value of x. (AI CBSE 2009)
Sol. Let the required ratio = k : 1

Coordinates of the point P are:


F 3k 3 ,5k 4 I
GH k + 1 k+1 JK
But the coordinates of P are (x, 2)
5k 4
= 2 5k 4 = 2k + 2
k+1
3k = 6
6
k = = 2
3
The required ratio is 2 : 1
3k 3
Now, x =
k+1

=
af
3 2 3
2+1
63 3
= = = 1
3 3
Q. 31. Find the area of the triangle formed by joining the mid-points of the sides of triangle whose vertices
are (0, 1), (2, 1), and (0, 3). (AI CBSE 2009)
Sol. We have the vertices of the given triangle as A (0, 1), B (2, 1) and C (0, 3). Let D, E and
F be the mid-points of AB, BC and AC.

Coordinates of D are
LM 0 + 2 , 1+1 OP or (1, 0)
N 2 2 Q
E are
LM 2 + 0 , 1 + 3 OP or (1, 2)
N 2 2 Q
L 0 + 0 , 3 + b 1g OP or (0, 1)
F are M
MN 2 2 PQ
Coordinates of the vertices of DEF are (1, 0), (1, 2) and (0, 1).
1
Now, area of DEF = [1 (2 1) + 1 (1 0) + 0 (0 2)]
2
1
= 2 = 1 sq. units.
2
Q. 32. Find the area of the ABC with A(1, 4), and the mid-point of sides through A being (2, 1) and
(0, 1). [NCERT Exemplar]

Coordinate Geometry 295


Sol. Let the co-ordinates of B and C are (a, b) and (x, y) respectively. C (x, y)
Sides through A are AB and AC
1 + a -4 + b
(2, 1) = ,
2 2
1+ a -4 + b
= = 2 and = -1
2 2
= 1 + a = 4 and 4 + b = 2
= a = 3 and b = 2 (a, b)
A
1 + x -4 + y B
Also, (0, 1) =
(1, 4)
,
2 2
-4 + y
= 1 + x = 0 and = -1
2 2
= 1 + x = 0 and 4 + y = 2
= x = 1 and y = 2
Thus, the co-ordinates of the vertices of ABC are: A(1, 4), B(3, 2) and C(1, 2)
Area of ABC
1
=
2
[1(2 - 2) + 3(2 + 4) - 1(-4 - 2)]
1
= [0 + 18 + 6]
2
1
= [24]
2
= 12 sq. units
Q. 33. Find the ratio in which the point (x, 1) divides the line segment joining the points ( 3, 5) and
(2, 5). Also find the value of x. (AI CBSE 2009)
Sol. Let the required ratio is k : 1

The coordinates of P are:


LM 2k 3 , 5k + 5 OP
N k+1 k+1 Q
But the coordinates of P are (x, 1)
5k + 5
= 1 5k + 5 = k 1
k+1
3
2k = 3 or k=
2
F 3I 3
Also, x =
2k 3
=
2
H 2K =
33
= 0
k+1 3 5
+1
2 2
x = 0
3
And k =
2

296 MathematicsX
Q. 34. If the mid-point of the line segment joining the point A(3, 4) and B(k, 6) is P(x, y) and
x + y 10 = 0, then find the value of k. [NCERT Exemplar]
Sol. Q Mid point of the line segment joining A(3, 4) and B(k, 6)
3 + k 4 + 6 3 + k
=
2
, = , 5
2 2
3+ k 3+ k
, 5 = ( x , y ) = x and 5 = y
2 2
Since, x + y 10 = 0
3+ k 3+ k

2
+ 5 - 10 = 0 Q x = 2 and y = 5
3 + k + 10 20 = 0
3+k = 10
k = 10 3 = 7
Thus, the required value of k=7
Q. 35. Point P, Q, R and S divide the line segment joining the points A (1, 2) and B (6, 7) in 5 equal
parts. Find the co-ordinates of the points P, Q and R. [AI. CBSE (Foreign) 2014]
Sol. A (1, 2)
P Q R S
B (6, 7)

P, Q, R and S, divide AB into five equal parts.


AP = PQ = QR = RS = SB
Now, P divides AB in the ratio 1 : 4
Let, the co-ordinates of P be x and y.
Using the section formula i.e.,
mx2 + nx1 my 2 + ny 1
x = ,y= , we have
m+n m+n
1(6) + 4(1) 6 + 4
x = = =2
1+ 4 5
1(7) + 4(2) 7 + 8
y = = =3
1+ 4 5
(x, y ) = ( 2, 3)
Next, Q divides AB in the ratio 2 : 3
Co-ordinates of Q are :
2(6) + 3(1) 2(7) + 3(2) 15 20
2+3 ;
5 or 5 , 5 or (3, 4)

Now, R divides AB in the ratio 3:2
Co-ordinates of R are :
3(6) + 2(1) 3(7) + 2(2) 20 25
3+2 , or , or ( 4, 5)
3 + 2 5 5
The co-ordinates of P, Q and R are respectively :
(2, 3), (3, 4) and (4, 5).

Coordinate Geometry 297


TEST YOUR SKILLS
1. The line-segment joining the points (3, 4) and (1, 2) is trisected at the points P and Q.
5 FG IJ
If the coordinates of P and Q are (p, 2) and
3 H K
, q respectively, find the values of
p and q. [CBSE 2005]
2. In the figure, find the coordinates of A. [AI CBSE 2005]
A (x , y)

G
(2, 1)

B (3, 5) D C (7, 4)

3. The line joining the points (2, 1) and (5, 8) is trisected at the points P and Q. If P lies
on the line 2x y + k = 0, find the value of k. [AI CBSE 2006]
4. If the coordinates of the mid-points of the sides of a are (10, 5), (8, 4) and (6, 6), then
find the coordinates of its vertices. [AI CBSE 2006]
5. Find the coordinates of the points which divide the line segment joining the points
( 4, 0), and (0, 6) in three equal pasts. [CBSE 2005C]
6. Find the coordinates of the point equidistant from the points A (1, 2), B (3, 4) and
C (5, 6). [CBSE 2005C]
7. Prove that the points A ( 4, 1), B ( 2, 4), C (4, 0) and D (2, 3) are the vertices of a
rectangle. [CBSE 2005C]
8. Find the coordinates of the points which divide the line-segment joining the points
( 4, 0) and (0, 6) in four equal parts. [CBSE 2005C]
9. Find the coordinates of the points which divide the line-segment joining the points
(2, 2) and (7, 4) in three equal parts. [CBSE 2011]
10. Find the coordinates of the point equidistant from the points A (5, 1), B ( 3, 7) and
C (7, 1). [AI CBSE 2005 Comptt]
FG 2 IJ .
11. The vertices of a ABC and given by A (2, 3) and B ( 2, 1) and its centroid is G 1,
H 3K
Find the coordinates of the third vertex C of the ABC. [AI CBSE 2005C]
12. If the points (x, y) is equidistant from the points (a + b, b a) and (a b, a + b). Prove that
bx = ay. [AI CBSE 2005 Comptt]
13. Two vertices of a ABC are given by A (6, 3) and B (1, 7) and its centroid is G (1, 5).
Find the coordinates of the third vertex of the ABC. [AI CBSE 2005C]
14. Two of the vertices of a ABC are given by A (6, 4) and B ( 2, 2) and its centroid is
G (3, 4). Find the coordinates of the third vertex C of the ABC. [AI CBSE 2005C]
15. The coordinates one end point of a diameter of a circle are (4, 1). If the coordinates of
the centre be (1, 3) find the coordinates of the other end of the diameter. [CBSE 2006]
16. Show that the points A (1, 2), B (5, 4) C (3, 8) and D ( 1, 6) are the vertices of a square.
[CBSE 2006]

298 MathematicsX
17. Find the value of P for which the points ( 1, 3), (2, p) and (5, 1) are collinear.
[CBSE 2006]
18. Find the distance of the point ( 6, 8) from the origin. [AI CBSE 2006]
19. Find the coordinates of the point equidistant from three given points A (5, 3), B (5, 5)
and C (1, 5). [AI CBSE 2006]
20. Find the value of p for which the points ( 5, 1), (1, p) and (4, 2) are collinear.
[AI CBSE 2006]
21. Find the coordinates of the point on the line joining P (1, 2) and Q (4, 7) that is twice
as far from P as from Q. [AI CBSE 2006]
22. Find the perimeter of a triangle with vertices (0, 4), (0, 0) and (3, 0).
[CBSE 2011, NCERT Exemplar Problem]
23. In the following figure, find the length of the median AD. [CBSE 2006C]
A (5,7)

B (4,0) D C (0,6)

24. Prove that the points (3, 0), (6, 4) and ( 1, 3) are vertices of right angled triangle. Also
prove that these are the vertices of an isosceles triangle. [CBSE 2006C]
25. In what ratio is the line segment joining the points ( 2, 3) and (3, 7) divided by the
y-axis? Also, find the coordinates of the point of division. [CBSE 2006C]
26. If A (5, 1), B ( 3, 2) and C ( 1, 8) are the vertices of ABC, find the length of median
through A and the coordinates of the centroid. [CBSE 2006C]
27. Find the value of k if the points A (2, 3), B (4, k) and C (6, 3) are collinear.
[CBSE 2006C]
28. In the figure, A and B are the end points of a diameter
of a circle having its centre at (1, 2). If the coordinates of
A are ( 3, 5) find the the coordinates of point B.
[AI CBSE 2006C] A (1 , 2)
B (x , y)
(3, 5) C
29. If ( 2, 1), (a, 0), (4, b) and (1, 2) are the vertices of a
parallelogram, find the value of a and b.
[AI CBSE 2006C]
30. The vertices of a triangle are ( 1, 3), (1, 1) and (5, 1). Find the lengths of medians through
vertices ( 1, 3) and (5, 1). [AI CBSE 2006C]
31. By distance formula, show that the points (1, 1); (5, 2) and (9, 5) are collinear.
[AI CBSE 2006C]
32. Show that the points (7, 10), ( 2, 5) and (3, 4) are the vertices of an isosceles right
triangle. [CBSE 2007]
33. In what ratio the line x y 2 = 0 divides the line segment joining (3, 1) and (8, 9)?
[CBSE 2007]
34. Find the ratio in which the line joining the points (6, 4) and (1, 7) is divided by x-axis.
[CBSE 2007]

Coordinate Geometry 299


35. Find the ratio in which the point ( 3, k) divides the line segment joining the points
( 5, 4) and ( 2, 3). Hence find the value of k. [AI CBSE 2007]
36. Three consecutive vertices of a parallelogram are ( 2, 1), (1, 0) and (4, 3). Find the
coordinates of the fourth vertex. [AI CBSE 2007]
37. For what value P, are the points (2, 1), (p, 1) and ( 1, 3) collinear?
38. Show that the point P ( 4, 2) lies on the line segment joining the points A ( 4, 6) and
B ( 4, 6).
39. Find the value (s) of k for which the points [(3k 1), (k 2)], [k, (k 7)] and [(k 1),
( k 2)] are collinear. [AI CBSE (Foreign) 2014]
Hint: Here, x1 = (3k 1) x2 = k x3 = (k 1)
y1= (k 2) y2 = (k 7) y3 = ( k 2)
For the given points to be collinear, we have
(3k 1) [(k 7) ( k 2)] + k [( k 2)(k 2)] + (k 1) [(k 2) (k 7)] = 0
6k2 17k + 5 2k2 + 5k 5 = 0 or 4k2 12 = 0 k = 0, 3
40. If the point A (0, 2) is equidistant from the point B(3, p) and C (p, 5), find p.
[CBSE (Delhi) 2014]
Hint: Here, AB = AC

32 + ( p 2)2 = p 2 + 32 9 + ( p 2)2 = p 2 + 9
(p 2)2 = p2 p2 + 4 4p = p2 p = 1
41. Find the ratio in which the point P(x, 2) divides the line segment joining the points
A(12, 5) and B(4, 3). Also find the value of x. [CBSE (Delhi) 2014]
Hint: Here, x 1 = 12, x2 = 4, y1 = 5 and y2 = 3
Let P(x, 2) divide the given line segment in the ratio k : 1
co-ordinates of P are :
k(4) + 1(12) k( 3) + 1(5)
x = , 2=
k+1 k+1
[x(k + 1) = 4k + 12], [2(k + 1) = 3k + 5]
3
2(k + 1) = 3k + 5 2k + 2 = 3k + 5 5k = 3 or k =
5

3 3
x(k +1) = k(4) + 12 x + 1 = (4) + 12
5 5

8 72 72 5
x= or x= =9
5 5 5 8
i.e., x = 9 and ratio = 3 : 5

42. If the point P(k 1, 2) is equidistant from the point A (3, k) and B(k, 5) then find the values
of k. [AI. CBSE 2014]
A(3, k) B(k, 5)
Hint:
P(k 1, 2)

300 MathematicsX
AP = BP ( k 1) 3]2 + (2 k )2 = ( k 1 k )2 + (2 5)2

( k 4) 2 + (2 k )2 = ( 1) 2 + ( 3) 2
Solving it we get k = 5

43. Find the ratio in which the line segment joining the points A(3, 3) and B(2, 7) is divided
by x axis. Also find the co-ordinates of the point of division. [AI CBSE 2014]
Hint: Let the xaxis meets AB at P(x, O) B(2, 7)
Let (k : 1) is the required ratio.
k
Co-ordinates of P are given as
(x, 0)
3k 2 x -axis
x= P
1
k+1
3k + 7 7
y= =0k= A(3, 3)
k+1 3
7
3 2
3 3
=
Now, x= 7 2
+1
3
3
We get x = and Required ratio as 7 : 3
2
3
co-ordinates of the point of division , 0
2
44. The mid-point P of the line segment joining the points A(10, 4) and B(2, 0) lies on the
line segment joining the points C(9, 4) and D(4, y). Find the ratio in which P divides
CD. Also find the value of y. [AI. CBSE (Foreign) 2014]
Hint: Mid point of AB [where A (10, 4) and B (2, 0)] is P(6, 2)
Let P(6, 2) divide CD, [where C(9, 4) and D(4, y)] in k : 1
k( 4) + 1( 9) k ( y ) + 1( 4)
= 6 k = 3 and 2 = y=6
k+1 2 k+1
Required ratio = 3 : 2 and y = 6.

ANSWERS
Test Your Skills
7
1. p = ; q=0 2. A(10, 2) 3. k = 8 or k = 13
3

-8 -4
4. (4, 5), (8, 3) and (12, 3) 5. , 2 , , 3 6. (1, 2)
3 3

8. -3, 3 9
7. ___
, (2, 3) and -1, 9. (1, 0) and (4, 2)
2 2

Coordinate Geometry 301


10. (2, 4) 11. (3, 2) 12. ___
13. (2, 5) 14. (5, 6) 15. (2, 5)
16. ___ 17. p=1 18. 10
19. (3, 1) 20. p = 1 21. (2, 1)
22. 12 units 23. 5 cm 24. ___

25. 2 : 3, (0, 1) 26. 1 5 27. k = 0


65; ,
3 3

5
28. (5, 1) 29. a = 1, b = 3 30. , 1
3
31. ___ 32. ___ 33. 2 : 3
2
34. 4 : 7 35. 2 : 1, k = 36. (1, 2), (1, 4)
3
37. p = 5 38. ___

302 MathematicsX
[Unit VII: Mensuration]

12 Areas Related to Circles

Facts that Matter


z Perimeter and Area of a Circle
The total distance (perimeter) around a circle is called its circumference.
The plane surface enclosed in a circle is called its area.
If r be the radius of a circle, then
(i) Circumference of the circle = 2r
(ii) Area of the circle = r2
NOTE:
I. The interior of a circle along with its boundary is called the circular region of the circle.
II. By the area of a circle, we mean the area of the circular region.

NCERT TEXTBOOK QUESTIONS SOLVED


EXERCISE 12.1
22
Unless stated otherwise, use =
7
Q. 1. The radii of two circles are 19 cm and 9 cm respectively. Find the radius of the circle which has
circumference equal to the sum of the circumferences of the two circles.
Sol. We have, r1 = 19 cm
r2 = 9 cm
Circumference of circle-I = 2 r1 = 2 (19) cm
Circumference of circle-II = 2 r2 = 2 (9) cm
Sum of the circumferences of circle-I and circle-II
= 2 (19) + 2 (9) = 2 (19 + 9) cm = 2 (28) cm
Let R be the radius of the circle-III.
Circumference of circle-III = 2 R
According to the condition,
2 R = 2 (28)

R =
a f
2 28
= 28 cm
2
Thus, the radius of the new circle = 28 cm.
Q. 2. The radii of two circles are 8 cm and 6 cm respectively. Find the radius of the circle having area
equal to the sum of the areas of the two circles.
Sol. We have,
Radius of circle-I, r1 = 8 cm

303
Radius of circle-II, r2 = 6 cm
Area of circle-I = r12 = (8)2 cm2
Area of circle-II = r22 = (6)2 cm2
Let the area of the circle-III be R
Area of circle-III = R2
Now, according to the condition,
r12 + r22 = R2
i.e. (8)2 + (6)2 = R2
(82 + 62) = R2
82 + 62 = R 2
64 + 36 = R 2
100 = R 2
10 2 = R2 R = 10
Thus, the radius of the new circle = 10 cm.
Q. 3. Figure depicts an archery target marked with its five scoring
regions from the centre outwards as Gold, Red, Blue, Black and
White. The diameter of the region representing gold score is 21
cm and each of the other bands is 10.5 cm wide. Find the area
of each of the five scoring regions.
Sol. Diameter of the innermost region = 21 cm
21
Radius of the innermost (Gold Scoring) region = = 10.5 cm
2
Area of Gold region = (10.5)2 cm2
22 FG IJ
105
2
cm 2 =
22 105 105
cm 2
=
7

H K
10 7

10

10
22 15 105
= cm 2 = 346.50 cm2
100
Area of the Red region = (10.5 + 10.5)2 (10.5)2
= (21)2 (10.5)2
22
= [(21)2 (10.5)2] = [(21 + 10.5) (21 10.5)] cm2
7
22 315 15
= 31.5 10.5 cm2 = 22 cm 2 = 1039.5 cm2
7 10 10
Area of Blue region = [(21 + 10.5)2 (21)2] cm2
22 22
= [(31.5)2 (21)2] cm2 = [(31.5 + 21) (31.5 21)] cm2
7 7
22 75 105
= 52.5 10.5 cm2 = 22 cm 2 = 1732.5 cm2
7 10 10
Area of Black region = [(31.5 + 10.5)2 (31.5)2] cm2
22 22
= [(42)2 (3.15)2] cm2 = [(42 31.5) (42 + 31.5)] cm2
7 7
22 15 735
= 10.5 73.5 cm2 = 22 cm 2 = 2425.5 cm2
7 10 10
Area of White region = [(42 + 10.5)2 = (42)2] cm2

304 MathematicsX
= [(52.5)2 (42)2] cm2 = [(52.5 + 42) (52.5 42)]
22 945 15
= 94.5 10.5 = 22 = 3118.5 cm2.
7 10 10
Q. 4. The wheels of a car are of diameter 80 cm each. How many complete revolutions does each wheel
make in 10 minutes when the car is travelling at a speed of 66 km per hour?
Sol. Diameter of a wheel = 80 cm
80
Radius of the wheel = = 40 cm
2
Circumference of the wheel
22
= 2 40 = 2 40 cm
7
2 22 40
Distance covered by a wheel in one revolution = cm
7
Distance travelled by the car in 1hr
= 66 km = 66 1000 100 cm
Distance travelled in 10 minutes
66 1000 100
= 10 cm = 11 100000 cm
60
Now,
Number of revolutions
Distance travelled in 10 minutes
=
Distance travelled in one revolution

1100000 1100000 7
= = = 4375
LM 2 22 40
PQO
2 22 40
N 7
Thus, the required number of revolutions = 4375.
Q. 5. Tick the correct answer in the following and justify your choice: If the perimeter and the area of
a circle are numerically equal, then the radius of the circle is
(A) 2 units (B) units (C) 4 units (D) 7 units
Sol. We have:
[Numerical area of the circle] = [Numerical circumference of the circle]
r2 = 2 r
r2 2 r = 0
r2 2r = 0
r (r 2) = 0
r = 0 or r = 2
But r cannot be zero
r = 2 units.
Thus, the option (A) 2 units is correct.
z Area of Sector and Segment of a Circle
The portion (or part) of the circular region enclosed by two radii and the corresponding
arc is called a sector of the circle.

Areas Related to Circles 305


Q
Major
Sector

Minor
Sector B

A P

The portion (or part) of the circular region enclosed between a chord and the corresponding
arc is called a segment of the circle.
Q

Major
Segment

or
Min ent
g m
A Se P

NOTE:
I. AOB is called the angle of sector.
II. OAPB is the minor sector and OAQB is the major sector.
III. APB is the minor segment and AQB is the major segment.
IV. When we write sector and segment we will mean the minor-sector and the minor-
segment respectively.
Let us remember that
2
(i) Area of the sector of angle = r
360

(ii) Length of the arc of a sector of angle = 2 r
360
(iii) Area of a segment = [Area of the corresponding sector] [Area of the corresponding triangle]

NCERT TEXTBOOK QUESTIONS SOLVED


(EXERCISE 12.2)
22
Use = (unless stated otherwise)
7
Q. 1. Find the area of a sector of a circle with radius 6 cm if angle of the sector is 60.

306 MathematicsX
Sol. Here, r = 6 cm
= 60

Using, the Area of a sector = r2
360
We have,
Area of the sector with r = 6 cm and = 60
60 22 22 132
= 6 6 cm 2 = 6 cm 2 = cm 2 .
360 7 7 7
Q. 2. Find the area of a quadrant of a circle whose circumference is 22 cm.
Sol. Let radius of the circle = r
2 r = 22
22
2 r = 22
7
22 1 7
r = 2 = cm
7 2 2
Here = 90
FG IJ
1
Area of the quadrant
4H K
th of the circle,


r2 =
90 22 7 FG IJ 2
1 11 7
cm 2 = 77
=
360

360 7

2 H K 42
cm 2 =
8
cm 2 .

Q. 3. The length of the minute hand of a clock is 14 cm. Find the area swept by the minute hand in
5 minutes.
Sol. [Length of minute hand] = [radius of the circle]
r = 14 cm
Angle swept by the minute hand in 60 minutes = 360
360
Angle swept by the minute hand in 5 minutes = 5 = 30
60
Now, area of the sector with r = 14 cm and = 30
30 22 11 14 154
r2 = 14 14 cm 2 = cm 2 = cm 2
360 360 7 3 3
154
Thus, the required area swept by the minute hand by 5 minutes = cm 2 .
3
Q. 4. A chord of a circle of radius 10 cm subtends a right angle at the centre. Find the area of the
corresponding (i) minor segment (ii) major sector. (Use = 3.14)
Sol. Length of the radius (r) = 10 cm
Sector angle = 90
Area of the sector with = 90 and r = 10 cm
90 314 1 157
= 10 10 cm 2 = 314 cm 2 = cm 2 = 78.5 cm2
360 100 4 2
Now,
(i) Area of the minor segment
= [Area of minor sector] [Area of rt. AOB]

= [78.5 cm2]
LM 1 10 10 cm OP = 78.5 cm
2 2 50 cm2 = 28.5 cm2.
N2 Q
Areas Related to Circles 307
(ii) Area of major segment
= [Area of the circle] [Area of the minor segment]
= r2 78.5 cm2

=
LM 314 10 10 78.5OP cm 2
= (314 78.5) cm2 = 235.5 cm2.
N 100 Q
Q. 5. In a circle of radius 21 cm, an arc subtends an angle of 60 at the centre. Find:
(i) the length of the arc [CBSE 2013]
(ii) area of the sector formed by the arc
(iii) area of the segment formed by the corresponding chord
Sol. Here, radius = 21 cm and = 60
(i) Circumference of the circle = 2 r
22
= 2 21 cm = 2 22 3 cm = 132 cm
7 O
60
Length of APB = 132 cm 60 21
360 21 cm
1 cm
= 132 cm = 22 cm B
6
(ii) Area of the sector with sector angle 60 A P
60 60 22
= r2 = 21 21 cm 2 = 11 21 cm2 = 231 cm2
360 360 7
(iii) Area of the segment APQ
= [Area of the sector AOB] [Area of AOB] ...(1)
In AOB, OA = OB = 21 cm
A = B = 60 [ O = 60]
AOB is an equilateral ,
AB = 21 cm
Draw OM AB such that
OM 3 3
= sin 60 = OM = 21 cm
OA 2 2
1 1 3
Now area of OAB = AB OM = 21 21 cm 2
2 2 2
441 3
= cm 2 ...(2)
4
From (1) and (2), we have:
LM 441 3 OP = F 231 441 3 I cm
PQ GH 4 JK
2
Area of segment = [231 cm2] cm 2 .
MN 4
Q. 6. A chord of a circle of radius 15 cm subtends an angle of 60 at the centre. Find the areas of the
corresponding minor and major segments of the circle. (Use = 3.14 and 3 = 1.73)
Sol. Here, radius (r) = 15 cm
Sector angle = 60
Area of the sector with = 60
60 314 11775
= r2 = 15 15 cm 2 = cm 2 = 117.75 cm2
360 360 100 100

308 MathematicsX
Since O = 60 and OA = OB = 15 cm
AOB is an equilateral triangle.
AB = 15 cm and A = 60
Draw OM AB
O
OM 3
= sin 60 = 60
OA 2 15 cm 15 cm
3 15 3
OM = OA = cm
2 2 A M B
1
Now, ar ( AOB) = AB OM
2
1 3 225 3
= 15 15 cm 2 = cm 2
2 2 4
225 1.73
= cm 2 = 97.3125 cm2
4
Now area of the minor segment
= (Area of minor sector) (ar AOB)
= (117.75 97.3125) cm2 = 20.4375 cm2
Area of the major segment
= [Area of the circle] [Area of the minor segment]

= r2 20.4375 cm2 =
LM
314 OP
15 2 20.4375 cm2
N
100 Q
= 706.5 20.4375 cm2 = 686.0625 cm2.
Q. 7. A chord of a circle of radius 12 cm subtends an angle of 120 at the centre. Find the area of the
corresponding segment of the circle. (Use = 3.14 and 3 = 1.73)
Sol. Here, = 120 and r = 12 cm

Area of the sector = r2
360
120 314
= 12 12 cm 2
360 100
314 4 12 15072 O
= cm 2 = cm 2 = 150.72 cm2 ...(1) 12
100 100 cm
cm

12
1 B
Now, area of AOB = AB OM [ OM AB] ...(2) M
2 A

In OAB, O = 120
A + B = 180 120 = 60
OB = OA = 12 cm A = B = 30
OM 1 1
So, = sin 30 = OM = OA
OA 2 2
1
OM = 12 = 6 cm
2
In right AMO, 122 62 = AM2
144 36 = AM2
108 = AM2

Areas Related to Circles 309


AM = 108 = 6 3
2 AM = 12 3
AB = 12 3 cm
Now, from (2),
1 1
Area of AOB = AB OM = 12 3 6 cm 2 = 36 3 cm 2
2 2
= 36 1.73 cm2 = 62.28 cm2 ...(3)
From (1) and (3)
Area of the minor segment
= [Area of minor segment] [Area of AOB]
= [150.72 cm2] [62.28 cm2] = 88.44 cm2.
Q. 8. A horse is tied to a peg at one corner of a square shaped grass
field of side 15 m by means of a 5 m long rope (see figure). Find:
(i) the area of that part of the field in which the horse can
graze.
(ii) the increase in the grazing area if the rope were 10 m long
instead of 5 m. (Use = 3.14)
Sol. Here, Length of the rope = 5 m
Radius of the circular region grazed by the horse = 5 m
(i) Area of the circular portion grazed
90
= r2 [ = 90 for a square field.]
360
90 314 1 314 2 157 2
= 5 5 m2 = m = m = 19.625 m2
360 100 4 16 8
(ii) When length of the rope is increased to 10 m,
r = 10 m
Area of the circular region where = 90.
90 314 1
r2 = ( 10 ) m 2 =
2
= 314m2 = 78.5m2
360 360 100 4
Increase in the grazing area
= 78.5 19.625 m2 = 58.875 m2.
Q. 9. A brooch is made with silver wire in the form of a circle with
diameter 35 mm. The wire is also used in making 5 diameters which
divide the circle into 10 equal sectors as shown in Fig. Find:
(i) the total length of the silver wire required.
(ii) the area of each sector of the brooch.
Sol. Diameter of the circle = 35 mm
35
Radius (r) = mm
2
22 35
(i) Circumference = 2 r = 2 mm = 22 5 = 110 mm
7 2
Length of 1 piece of wire used to make diameter to divide the circle into 10 equal
sectors = 35 mm
Length of 5 pieces = 5 35 = 175 mm
Total length of the silver wire = 110 + 175 mm = 285 mm

310 MathematicsX
(ii) Since the circle is divided into 10 equal sectors,
360
Sector angle = = 36
10
36 22 35 35
Area of each sector = r2 = mm 2
360 360 7 2 2
11 35 385
= mm 2 = mm 2 .
4 4
Q. 10. An umbrella has 8 ribs which are equally spaced (see figure).
Assuming umbrella to be a flat circle of radius 45 cm, find the
area between the two consecutive ribs of the umbrella.
Sol. Here, radius (r) = 45 cm
Since circle is divided in 8 equal parts,
Sector angle corresponding to each part
360
= = 45
8
Area of a sector (part)
45 22
= r2 = 45 45 cm 2
360 360 7
11 45 45 22275
= cm 2 = cm 2
47 28
22275
The required area between the two ribs = cm 2 .
28
Q. 11. A car has two wipers which do not overlap. Each wiper has a blade of length 25 cm sweeping
through an angle of 115. Find the total area cleaned at each sweep of the blades.
Sol. Here, radius (r) = 25 cm
Sector angle () = 115
Area cleaned by each sweep of the blades

=
LM OP
r2 2 [ Each sweep will have to and fro movement]
N
360 Q
LM 25 25OP 2 cm
115 22 2
=
N 360 7 Q
23 11 25 25 158125
= cm 2 = cm 2 .
18 7 126
Q. 12. To warn ships for underwater rocks, a lighthouse spreads a red coloured light over a sector of
angle 80 to a distance of 16.5 km. Find the area of the sea over which the ships are warned.
(Use = 3.14)
Sol. Here, Radius (r) = 16.5 km
Sector angle () = 80
Area of the sea surface over which the ships are warned

= r2
360
80 314 165 165
= km 2
360 100 10 10
157 11 11 18997
= km 2 = km 2 = 189.97 km2.
100 100
Areas Related to Circles 311
Q. 13. A round table cover has six equal designs as shown in Fig. If the
radius of the cover is 28 cm, find the cost of making the designs at
the rate of ` 0.35 per cm2. (Use 3 = 1.7)
Sol. Here, r = 28 cm
Since, the circle is divided into six equal sectors.
360
Sector angle = = 60 .
6
Area of the sector with = 60 and r = 28 cm
60 22
= 28 28 cm 2
360 7
44 28
= cm 2 = 410.67 cm2 ...(1)
3
Now, area of 1 design
= Area of segment APB
= Area of sector Area of AOB ...(2)
In AOB, AOB = 60, OA = OB = 28 cm
OAB = 60 and OBA = 60
AOB is an equilateral triangle.
AB = AO = BO
AB = 28 cm
Draw OM AB
In right AOM, we have
OM 3 3
= sin 60 = OM = OA cm
OA 2 2
3
OM = 28 cm
2
OM = 14 3 cm
1 1
Area of AOB = AB OM = 28 14 3 cm 2
2 2
= 14 14 3 cm 2
= 14 14 1.7 cm2 = 333.3 cm2 ...(3)
Now, from (1), (2) and (3), we have:
Area of segment APQ = 410.67 cm2 333.2 cm2 = 77.47 cm2
Area of 1 design = 77.47 cm2
Area of the 6 equal designs = 6 (77.47) cm2
= 464.82 cm2
Cost of making the design at the rate of ` 0.35 per cm2,
= ` 0.35 464.82
= ` 162.68.
Q. 14. Tick the correct answer in the following:
Area of a sector of angle p (in degrees) of a circle with radius R is
p p p p
(A) 2 R (B) R 2 (C) 2 R (D) 2 R 2
180 180 360 720
Sol. Here, radius (r) = R
Angle of sector () = p
312 MathematicsX
2 p
R2 = p 2 R
2
p
Area of the sector = r2 = R 2 =
360 360 2 360 720
p
Thus, the option (D) 2 R2 is correct.
720

NCERT TEXTBOOK QUESTIONS SOLVED


EXERCISE 12.3
22
Unless stated otherwise, use = Q
7
Q. 1. Find the area of the shaded region in Fig. PQ = 24 cm, PR = 7 cm
and O is the centre of the circle. O

Sol. Since O is the centre of the circle,


QOR is a diameter.
RPQ = 90 [Angle in a semi-circle] R

Now, in right RPQ, P

RQ2 = PQ2 + PR2


RQ2 = 242 + 72 = 576 + 49 = 625
RQ = 625 = 25
1 1
Area of RPQ = RQ RP = 24 7 cm2 = 12 7 cm2 = 84 cm2
2 2
1 1 22 25 25 11 625
Now, area of semi-circle = r2 = = cm 2
2 2 7 2 2 74
6875
= cm 2 = 245.54 cm2
28
Area of the shaded portion = 245.54 cm2 84 cm2 = 161.54 cm2.

Q. 2. Find the area of the shaded region in figures, if radii of the two
concentric circles with centre O are 7 cm and 14 cm respectively and
AOC = 40. [AI. CBSE 2014]
Sol. Radius of the outer circle = 14 cm
O
Here, = 40 B
40
A
40 22
Area of the sector AOC = 14 14 cm 2 D
360 7
1 616 C
= 22 2 14 cm2 = cm 2
9 9
Radius of the inner circle = 7 cm
Here, also = 40
Area of the sector BOD
40 22 1 154
= 7 7 cm 2 = 22 7 cm2 = cm 2
360 7 9 9
Now, area of the shaded region

Areas Related to Circles 313


= [Area of sector AOC] [Area of sector BOD]
616 154 1 1
= cm 2 = [616 154] cm2 = 462 cm2
9 9 9 9
1
= 154 cm2.
3
Q. 3. Find the area of the shaded region in figure, if ABCD is a square of side 14 cm and APD and
BPC are semi-circles. [CBSE 2012]
Sol. Side of the square = 14 cm
Area of the square ABCD=14 14 cm2 = 196 cm2
Now, diameter of the circle = (Side of the square) = 14 cm
14
Radius of each of the circles = = 7 cm
2
1 1 22
Area of the semi-circle APD = r 2 = 7 7 = 77 cm2
2 2 7
1 22
Area of the semi-circle BPC = 7 7 = 77 cm2
2 7
Area the shaded region
= [Area of the square] [Area of semi-circle APD + Area of semi-circle BPC]
= 196 [77 + 77] cm2 = 196 154 cm2 = 42 cm2.
Q. 4. Find the area of the shaded region in figure, where a circular arc
of radius 6 cm has been drawn with vertex O of an equilateral
triangle OAB of side 12 cm as centre. (CBSE 2012)
Sol. Area of the circle with radius = 6 cm.
22 792
= r2 = 6 6 cm2 = cm 2
7 7
Area of equilateral triangle, having side a = 12 cm, is given
by
3 2 3
a = 12 12 cm 2 = 36 3 cm 2
4 4
Each angle of an equilateral triangle = 60
AOB = 60
60 22
Area of sector COD = r2 = 6 6 cm 2
360 360 7
22 6 132
= cm 2 = cm 2
7 7
Now, area of the shaded region,
= [Area of the circle] + [Area of the equilateral triangle] [Area of the sector COD]
792
+ 36 3
132
cm 2 =
LM
660 OP
+ 36 3 cm 2 . A B
=
7 7 N
7 Q
Q. 5. From each corner of a square of side 4 cm a quadrant of a circle
of radius 1 cm is cut and also a circle of diameter 2 cm is cut as
shown in figure. Find the area of the remaining portion of the
square. [CBSE 2012]
Sol. Side of the square = 4 cm
Area of the square ABCD = 4 4 cm2 = 16 cm2 D C

314 MathematicsX
Each corner has a quadrant circle of radius 1 cm.
Area of all the 4 quadrant squares
1 22 22
= 4 r 2 = r2 = 1 1 cm 2 = cm 2
4 7 7
Diameter of the middle circle = 2 cm
Radius of the middle circle = 1 cm
Area of the middle circle = r2
22 22
= 7 1 cm 2 = cm 2
7 7
Now, area of the shaded region
= [Area of the square ABCD] [(Area of the 4 quadrant circles) + (Area of the middle circle)]

= 16 cm 2
LM 22 + 22 cm OP = 16 cm
2 2
2
22
cm 2
N7 7 Q 7
44 112 44 68 A
= 16 cm 2 cm 2 = cm 2 = cm 2 .
7 7 7
Q. 6. In a circular table cover of radius 32 cm, a design is formed leaving
an equilateral triangle ABC in the middle as shown in figure. Find
the area of the design.
Sol. Area of the circle having radius r = 32 cm.
= r2
22 22528 B C
= 32 32 cm2 = cm 2
7 7
O is the centre of the circle,
AO = OB = OC = 32 cm A
AOB = BOC = AOC = 120
Now, in AOB, 1 = 30 1
1 + 2 = 60
Also OA = OB 1 = 2 32 cm
If OM AB, then M
32 cm
OM 1 1 O
= sin 30 = OM = OA 2 32 cm C
OA 2 2
1 B
OM = 32 = 16 cm
2
AM 3
Also, = cos 30 =
AO 2
3 3
AM = AO = 32
2 2
F 3 I
2 AM = AB = 2 GH 2 JK
32 = 32 3 cm

1 1
Now, area of AOB, = OM AB = 16 32 3 = 256 3 cm 2
2 2
Since area ABC = 3 [area of AOB] = 3 256 3 cm 2 = 768 3 cm2
Now, area of the design = [Area of the circle] [Area of the equilateral triangle]
FG 22528 768 3 IJ cm 2
=
H 7 K .

Areas Related to Circles 315


Q. 7. In figure, ABCD is a square of side 14 cm. With centres A, B, C and D, four circles are drawn
such that each circle touch externally two of the remaining three circles. Find the area of the shaded
region. (CBSE 2012)
Sol. Side of the square ABCD = 14 cm
Area of the square ABCD = 14 14 cm2 = 196 cm2.
A B
Circles touch each other
14
Radius of a circle = = 7 cm
2
Now, area of a sector of radius 7 cm and sector angle as D C
90
90 22 11 7
= 7 7 cm 2 = cm 2
360 7 2

Area of 4 sectors = 4
LM
11 7 OP
= 2 11 7 cm2 = 154 cm2
N2 Q
Area of the shaded region = [Area of the square ABCD] [Area of the 4 sectors]
= 196 cm2 154 cm2 = 42 cm2.
Q. 8. The figure depicts a racing track whose left and right ends are semicircular.

The distance between the two inner parallel line segments is 60 m and they are each 106 m long.
If the track is 10 m wide, find:
(i) the distance around the track along its inner edge
(ii) the area of the track. (CBSE 2012)

Sol. (i) Distance around the track along its inner edge
= BC + EH + BPE + CQH
1 1
= 106 m + 106 m + (2 r) + (2 r)
2 2
1 FG 22 IJ
1 22 FG IJ LM r = 1
BE =
1
60 = 30 m
OP
= 212 m +
2 H
2
7
30 +
2K 2
7 H
30
K N 2 2 Q
1320 2804
= 212 m + m = m
7 7

316 MathematicsX
(ii) Now, area of the track
= Area of the shaded region
= (Area of rectangle ABCD) + (Area of rectangle EFGH)
LMF Area of 2 semi-circles I F Area of 2 sem- circles I OP
+2
MNGH each of radius 40 m JK GH each of radius 30 cm JK PQ [ The track is 10 m wide]

Area of the track

= (106 10 m2) + (106 10 m2) + 2


LM 1 22 a40f 2

1 22
a
30 f OPQ m
2 2
N2 7 2 7

= 1060 m2 + 1060 m2 L 1 22 e40 30 jOP m


2M 2 2 2
N2 7 Q
1 22
= 2120 m2 + 2 [(40 + 30) (40 30)] m2
2 7
22
= 2120 m2 + 70 10 m2 = 2120 m2 + 2200 m2 = 4320 m2.
7
B
Q. 9. In the figure, AB and CD are two diameters of a circle (with centre
O) perpendicular to each other and OD is the diameter of the
smaller circle. If OA = 7 cm, find the area of the shaded region.
(CBSE 2012, 2013)
Sol. O is the centre of the circle. D
O
C
OA = 7 cm
AB = 2 OA = 2 7 = 14 cm
OC = OA = 7 cm
AB and CD are perpendicular to each other A
OC AB
1 1
Area ABC = AB OC = 14 cm 7 cm = 49 cm2
2 2
Again OD = OA = 7 cm

Radius of the small circle =


1
2
a f 1
OD = 7 =
2
7
2
cm

22 7 7 11 7 77
Area of the small circle = cm 2 = = cm 2
7 2 2 2 2
14
Radius of the big circle = cm = 7 cm
2
1 22 FG
7 7 cm 2 =
IJ 11 7 7
cm 2
Area of semi-circle OABC =
2 7 H K 7
= 11 7 cm2 = 77 cm2
Now, Area of the shaded region
= [Area of the small circle] + [Area of the big semi-circle OABC] [Area of ABC]
77 77 + 154 98
= cm 2 + 77 cm2 49 cm2 = cm 2
2 2
231 98 133
= cm 2 = cm 2 = 66.5 cm2.
2 2

Areas Related to Circles 317


Q. 10. The area of an equilateral triangle ABC is 17320.5 cm2. With each
vertex of the triangle as centre, a circle is drawn with radius equal to A
half the length of the side of the triangle (see Fig.). Find the area of the
shaded region. (Use = 3.14 and 3 = 1.73205). (CBSE 2012)
Sol. Area of ABC = 17320.5 cm2
ABC is an equilateral triangle and area of an B C

3
equilateral = (side)2
4


4
3
sidea f 2
= 17320.5


1.73205
4
sidea f 2
= 17320.5 [ 3 = 1.73205 (given)]
173205

173205
400000
sidea f 2
=
10
173205 400000
(side)2 =
10 173205
(side)2 = 40000
(side)2 = (200)2 side = 200 cm
200
Radius of each circle = = 100 cm
2
Since each angle of an equilateral triangle is 60,
A = B = C = 60
Area of a sector having angle of sector as 60 and radius 100 cm.
60 314 1 314 15700
= 100 100 cm 2 = 100 100 cm 2 = cm 2
360 100 3 100 3
15700
Area of 3 equal sectors = 3 cm 2 = 15700 cm2
3
Now, area of the shaded region
= [Area of the equilateral triangle ABC] [Area of 3 equal sectors]
= 17320.5 cm2 15700 cm2 = 1620.5 cm2.
Q. 11. On a square handkerchief, nine circular designs each of radius
7 cm are made (see figure). Find the area of the remaining
portion of the handkerchief.
Sol. The circles touch each other.
The side of the square ABCD
= 3 diameter of a circle
= 3 (2 radius of a circle) = 3 (2 7 cm)
= 42 cm
Area of the square ABCD = 42 42 cm2 = 1764 cm2.
22
Now, area of one circle = r2 = 7 7 cm2 = 154 cm2
7
There are 9 squares
Total area of 9 circles = 154 9 = 1386 cm2
Area of the remaining portion of the handkerchief = 1764 1386 cm2 = 378 cm2.

318 MathematicsX
Q. 12. In the figure, OACB is a quadrant of a circle with centre O and
radius 3.5 cm. If OD = 2 cm, find the area of the
(i) quadrant OACB, (ii) shaded region. (CBSE 2012)
Sol. Here, centre of the circle is O and radius = 3.5 cm.
1
Area of the quadrant OACB = r2
4
1 22 35 35 11 35 11 7 77
= cm 2 = = cm 2 = cm 2
4 7 10 10 2 20 8 8
1
Now, ar ( BOD) = OB OD
2
1
= 3.5 2 cm2 [ OB = 3.5 cm = radius and OD = 2 cm (given)]
2
1 35 7
= 2 cm 2 = cm 2
2 10 2
Area of the shaded region
= (Area of the quadrant OACB) (Area of BOD)
FG 77 7 IJ cm 2 =
77 28
cm 2 =
49
=
H 8

K
2 8 8
cm 2 .
Q. 13. In the figure, a square OABC is inscribed in a quadrant OPBQ. If
OA = 20 cm, find the area of the shaded region. (Use = 3.14)
(CBSE Sample Paper 2011) [CBSE (Delhi) 2014]
Sol. OABC is a square such that its side OA = 20 cm.
OB2 = OA2 + OB2
= [202 + 202] = [400 + 400] = [800]
OB = 800 = 20 2 cm .
Radius of the circle = 20 2 cm .
1
Now, area of the quadrant OPBQ = r2
4
1 314
= 800 cm 2 = 314 2 = 628 cm2
4 100
Area of the square OABC = 20 20 cm2 = 400 cm2
Area of the shaded region = 628 cm2 400 cm2 = 228 cm2.
Q. 14. AB and CD are respectively areas of two concentric circles of radii
21 cm and 7 cm and centre O (see figure). If AOB = 30, find the
area of the shaded region. (CBSE 2012)
Sol. Radius of bigger circle
R = 21 cm
and sector angle = 30
30 22
Area of the sector OAB = 21 21 cm 2
360 7
11 21 231
= cm 2 = cm 2
2 2
Again, radius of the smaller circle
r = 7 cm
Areas Related to Circles 319
Here also, the sector angle is 30
30 22 77
Area of the sector OCD = 7 7 cm 2 = cm 2
360 7 6
Area of the shaded region
231 77 693 77 616 308
= cm 2 = cm 2 = cm 2 = cm 2 .
2 6 6 6 3
Q. 15. In the figure, ABPC is a quadrant of a circle of radius 14 cm and
a semi-circle is drawn with BC as diameter. Find the area of the
shaded region.
Sol. Radius of the quadrant = 14 cm
Therefore, area of the quadrant ABPC
LM 90 22 14 4OP cm LMusing r OP
2 2
=
N 360 7 Q N 360 Q
= 22 7 cm2 = 154 cm2
1
Area of right ABC = 14 14 cm2 = 98 cm2
2
Area of segment BPC = 154 cm2 98 cm2 = 56 cm2
Now, in right ABC,
AC2 + AB2 = BC 2
142 + 142 = BC 2
196 + 196 = BC 2
BC 2 = 392 BC = 14 2 cm .

14 2
Radius of the semi-circle BQC = cm = 7 2 cm
2
1 2 1 22
Area of the semi-circle BQC =
2
r =
2 7
e j
7 2
2

1 22
= 7 2 7 2 = 11 2 7 2 cm 2
2 7
= 11 7 2 cm2 = 154 cm2
Now, area of the shaded region
= [Area of segment BQC] [Area of segment BPC]
= 154 cm2 56 cm2 = 98 cm2.
Q. 16. Calculate the area of the designed region in the figure, common
between the two quadrants of circles of radius 8 cm each.
Sol. Side of the square = 8 cm
Area of the square (ABCD) = 8 8 cm2
= 64 cm2
Now, radius of the quadrant ADQB = 8 cm
90 22
Area of the quadrant ADQB = 8 2 cm 2
360 7
1 22
= 64 cm 2
4 7

320 MathematicsX
8 cm
22 16 D C
= cm 2
7 Q

22 16
Similarly, area of the quadrant BPDC = cm 2 8 cm 8 cm
7

Sum of the two quadrants = 2


LM 22 16 OP cm 2
=
704
cm 2
P

N 7 Q 7 A
8 cm
B

Now, area of design


= [Sum of the areas of the two quadrants] [Area of the square ABCD]
704 704 448 256
= cm 2 64 cm 2 = cm 2 = cm 2 .
7 7 7

MORE QUESTIONS SOLVED


I. VERY SHORT ANSWER TYPE QUESTIONS
Q. 1. PQRS is a diameter of a circle of radius 6 cm. The equal
lengths PQ, QR and RS are drawn on PQ and QS as diameters,
as shown in figure. Find the perimeter of the shaded region.
Q O
Sol. Diameter PS = 12 cm [ Radius OS = 6 cm] P
R
S

Since PQ, QR and RS are three equal parts of diameter,


12
PQ = QR = RS = cm = 4 cm
3
QS = 8 cm [ QS = 2 QR]
Now, the total required perimeter
= PS + QS + PQ
1 1 1
= (2 6) + (2 4) + (2 2) = 6 + 4 + 2
2 2 2
= 12 cm.
Q. 2. A sheet of paper is in the form of a rectangle D C
ABCD in which AB = 40 cm, and BC = 28 cm.
A semi-circlular portion with BC as diameter is
cut off. Find the area of the remaining paper. 28 cm
Sol. Length of the paper = 40 cm, width of the 14cm
paper = 28 cm.
Area of the rectangle = length breadth A 40 cm B
=40 28 cm2 = 1120 cm2
Again, diameter of semi-circle = 28 cm.
28
Radius of the semi-circle = = 14 cm.
2
Area of the semi-circle
1 2 1 22
= r = 14 14 cm 2 = 11 2 14 cm2 = 308 cm2
2 2 7
Area of the remaining paper = 1120 cm2 308 cm2 = 812 cm2.

Areas Related to Circles 321


Q. 3. Find the area of the shaded region in the figure, if ABCD is a square D C
of side 14 cm and APD and BPC are semi-circles.
Sol. Side of the square = 14 cm
Area of the square = 14 14 cm2 = 196 cm2 P
Also, diameter of each semi-circle = side of the square = 14 cm.
14 A B
Radius = = 7 cm.
2
1 2 1 22
Area of 1 semi-circle = r = 7 7 cm 2 = 11 7 cm2 = 77 cm2
2 2 7
Area of both semi-circles = 2 77 cm2 = 154 cm2.
Now, the area of the shaded region
= [Area of the square ABCD] [Area of both the semi-circles]
= (196 154) cm2 = 42 cm2.
Q. 4. A park is in the form of a rectangle 120 m long and 100 m wide. At the centre, there is a circular

lawn of radius 1050 m . Find the area of the park excluding the lawn.
LMTake = 22 OP
N 7Q
Sol. Length of the park l = 120 m
Breadth of the park b = 100 m
Area of the park = l b = 120 100 m2
100
= 12000 m2 m

Now, radius of the lawn = 1050 m


Area of the lawn = r2 120 m

22 22
=
7
e
1050 m 2 =
2
j 7
1050 m 2

= 22 150 m2 = 3300 m2 P

Area of the park excluding the central park


= 12000 3300 m2 = 8700 m2. O
Q. 5. In the given figure, OAPB is a sector of a circle of radius 3.5 cm Q
with the centre at O and AOB = 120. Find the length of 120 3.5 cm
OAPBO. A
B
Sol. Here, the major sector angle is given by
= 360 120 = 240
35 7
Radius = 3.5 cm = cm = cm
10 2
Circumference of the sector APB
240 22 7 2 44
= 2 r = 2 cm = 22 cm = cm
360 360 7 2 3 3
Perimeter of OAPBO
= [Circumference of sector AOB] + OA + OB
44 7 7 44 44 + 21 65 2
= + + cm = +7 = = or 21 cm .
3 2 2 3 3 3 3

322 MathematicsX
Q. 6. Find the area of the shaded region of the following figure, if the
diameter of the circle with centre O is 28 cm and
1
AQ = AB .
4 O
Sol. We have AB = 28 cm A B
Q
1 1
AQ = AB = 28 cm = 7 cm
4 4
BQ = 28 7 = 21 cm
Area of the semi-circle having diameter as 21 cm
1 22 2.1 21 11 3 21
= cm 2 = cm 2
2 7 2 2 4
Also area of the semi-circle having diameter as 7 cm
1 22 7 7 11 7
= cm 2 = cm 2
2 7 2 2 4
Thus the area of the shaded region

=
LM 11 3 21 + 11 7 OP cm 2
=
11
[3 21 + 7] cm2
N 4 4 Q 4
11 11 770
= [63 + 7] cm2 = 70 = cm 2 = 192.5 cm2.
4 4 4
Q. 7. PQRS is a square land of side 28 m. Two semi-circular grass covered
postions are to be made on two of its opposite sides as shown in the

figure. How much area will be left uncovered?


LMTake =
22 OP
Sol. Side of the square = 28 m
N 7 Q
Area of the square PQRS = 28 28 m2
Diameter of a semi-circle = 28 m
28
Radius of a semi-circle = = 14 m
2
1 2 1 22
Area of 1 semi-circle = r = 14 14 = 22 14 m2 = 308 cm2
2 2 7
Area of both the semi-circles
= 2 308 m2 = 616 m2
Area of the square left uncovered
= (28 28) 616 m2 = 784 616 m2 = 168 m2.
Q. 8. Find the area of a square inscribed in a circle of radius 10 cm.
Sol. Let ABCD be the square such that
AB = BC = 10 cm D C
AC2 = AB2 + BC2
cm

AB2 + BC2 = (10 2)


10

x2 + x2 = (20)2 [Let AB = BC = x]
2x 2 = 400
cm
10

400
x2 = = 200 cm2
2 A B
Area of the square = 200 cm2.
Areas Related to Circles 323
Q. 9. In the given figure, O is the centre of a circular arc and AOB is C
a straight line. Find the perimeter of the shaded region.
Sol. O is the centre of the circle. 16 cm

cm
AB is its diameter.

12
In right ABC,
AC2 + BC2 = AB2 A O B
2 2
12 + 16 = AB 2

144 + 256 = AB2 AB2 = 400


AB = 400 = 20cm
Circumference of semi-circle ACB
1 1 22 20 11 20 220
= (2r) = 2 cm = cm = cm
2 2 7 2 7 7
Perimeter of the shaded region
220
= cm + 12 cm + 16 cm = 31.43 cm + 12cm + 16cm = 59.43 cm
7

II. SHORT ANSWER TYPE QUESTIONS


Q. 1. In a circular table cover of radius 70 cm, a design is formed leaving an equilateral triangle ABC
in the middle as shown in the figure. Find the total area of the design.
22
[Use 3 = 1.73 and = 7 ] (AI CBSE 2009 C, 2013 compt.)
A
Sol. Radius of the circle = 70 cm and O is the centre of the circle.
OA = OB = OC
Since ABC is an equilateral triangle,
ABO = BOC = CDA = 120
Draw OD BC
Now in right BDO,
BD 3
= sin 60 = B C
BO 2
3
BD = BO = 70 3 = 35 3 cm C
2
Also BC = 2 BD = 2 35 2 = 70 3 cm
D
3
Area of equilateral ABC = (side) 2
4 O

3 3 A B
= (70 3 ) 2 = 70 70 3 cm2
4 4
= 3 3 35 35 cm2 = 3675 3 cm2
Also, area of the circle = r2
22
= 70 70 cm2 = 22 10 70 cm2 = 15400 cm2
7
Area of the shaded region
= 15400 cm2 (3675 3 ) cm2 = 15400 cm2 (3675 1.73) cm2
= 15400 cm2 6357.75 cm2 = 9042.25 cm2.

324 MathematicsX
Q. 2. Calculate the area other than the area common between two quadrants of the circles of radius 16
cm each, which is shown as the shaded region in the figure. (AI CBSE 2009 C)
Sol. Area of sector ADB
90 22
= 16 16 cm2 [using area = r 2 ]
360 7 360
1 22
= 16 16 cm2
4 7
22 4 16
= cm 2
7
1408
= cm 2
7

Area of the shaded region-I =


LM Area of the OP LMArea of the OP
Nsquare ABCDQ Nsector ADBQ
1408 1792 1708 384
= 256 cm2 cm 2 = cm 2 = cm 2
7 7 7
384
Similarly, in area of the shaded region-II = cm 2
7
Total area of the shaded region
= [Area of shaded region-I] + [Area of shaded region-II]
384 384 768
= cm 2 + cm 2 = cm 2 .
7 7 7
Q. 3. In the figure, PQ = 24 cm, PR = 7 cm and O is the centre of the circle. Find the area of shaded
region. (Take = 3.14) (CBSE 2009)
Sol. In right RPQ,
PR2 + PQ2 = RQ2 Q

72 + 242 = RQ2
49 + 576 = RQ2
O
625 = RQ2 RQ = 625 = 25 cm
25
Radius of semi-circle = cm
2 R
1 2 1 314 25 25 P
Area of semi-circle RQP = r = cm 2
2 2 100 2 2
157 25 3925
= cm 2 = cm 2 = 245.31 cm2
4 4 16
Areas Related to Circles 325
1 1
Area of right RPQ = RP PQ = 7 24 cm2 = 7 12 = 84 cm2
2 2
Area of the shaded region = Ar of semi-circle RQP ar (right RPQ)
= 245.31 cm2 = 84 cm2 = 161.31 cm2.
Q. 4. The area of an equilateral triangle is 49 3 cm 2 . Taking each angular point as centre, circles are
drawn with radius equal to half length of the side of the triangle. Find the area of triangle not
included in the circles. [Take 3 = 1.73] (AI CBSE 2009)
Sol. Let the given equilateral triangle be ABC, such that its
side = 14 cm.
A
Area of ABC
3 3
= (side)2 = (14)2 cm2
4 4
1.73
= 14 14 cm2 = 49 1.73 cm2 60
4 B C
= 84.77 cm 2

Since each angle of an equilateral triangle = 60,


14
Area of a sector having as 60 and radius i.e. 7 cm.
2

=
60

22
7 7 cm 2 =
11 7
cm 2 using area =
LM
r 2
OP
360 7 3 360 N Q
Area of 3 sectors = 3
LM
11 7 OP
cm 2 = 77 cm2
N3 Q
Area of the shaded region
= [Area of equilateral ABC] [Area of 3 sectors]
= 84.77 cm2 77 cm2 = 7.77 cm2.
Q. 5. A square OABC is inscribed in a quadrant OABQ of a circle as shown in the figure. If OA = 14 cm,

find the area of the shaded region.


LMuse =
22 OP (AI CBSE 2008 C, CBSE Delhi 2014)
N 7 Q
Sol. OABC is a square with side = 14 cm.
Area of the square OABC = 14 cm 14 cm = 196 cm2
Now, the diagonal of the square OABC
OB = OA2 + AB 2 = 14 2 + 14 2

= 14 2 cm
Radius of the quadrant OPBQ = 14 2 cm
Area of the quadrant OABQ
1 2 1 22
=
4
r =
4

7
e2
14 2 cm 2 j
11 14 11
= 14 2 cm 2 = 2 14 2 cm2
7 2 2
= 11 14 2 cm2 = 308 cm2
Area of the shaded region
= 308 cm2 196 cm2 = 112 cm2.

326 MathematicsX
Q. 6. In the figure, ABDC is a quadrant of a circle of radius 14 cm and a semi-circle is drawn with
diameter BC. Find the area of the shaded region.[AI. CBSE (Foreign 2014)] (CBSE 2008 C)
Sol. We have, in the right ABC,
BC 2 = AB2 + AC2 = 142 + 142 = 2 (14)2
BC = 14 2 cm
BC 14
Radius of the semi-circle = = 2 = 7 2 cm
2 2
Area of semi-circle BEC
1 22 1 22
=
2 7
e j2
7 2 cm 2 =
2 7
7 7 2 cm2 = 154 cm2
Area of the quadrant with radius 14 cm,
1 22
= 14 14 cm2 = 154 cm2
4 7
1
Area of right ABC = 14 14 cm2 = 98 cm2
2
Area of the shaded region
Area of semicircle Area of Area of
= with diameter BC + ABC quadrant ABDC

= 154 cm2 + 98 cm2 154 cm2 = 98 cm2.
Q. 7. In the figure, find the perimeter of the shaded region where, ADC, AEB and BFC are semi-circles
on diameters AC, AB and BC respectively. (CBSE 2008, CBSE Delhi 2014)
Sol. Diameter of semi-circle ADC
= 2.8 cm + 1.4 cm = 4.2 cm
4.2
Radius of semi-circle ADC = cm = 2.1 cm
2
Circumference of semi-circle ADC
2 22 2 22 21 66
= 2.1 cm = = cm = 6.6 cm
2 7 2 7 10 10
Diameter of semi-circle AEB = 2.8 cm
2.8
Radius of semi-circle AEB = = 1.4 cm
2
Circumference of semi-circle AEB
2 2 22 2
= r= 1.4 cm = 22 cm = 4.4 cm
2 2 7 10
Diameter of semi-circle BFC = 1.4 cm
Radius of semi-circle BFC = 0.7 cm
2 22
Circumference of semi-circle BFC = 0.7 = 2.2 cm.
2 7
Total perimeter of the shaded region = 6.6 cm + 4.4 cm + 2.2 cm = 13.2 cm.
Q. 8. In the figure, arcs are drawn by taking vertices A, B and C of an equilateral triangle of side 10
cm to intersect the sides BC, CA and AB at their respective mid-points D, E and F. Find the area
of the shaded region. [use = 3.14] [NCERT Exemplar, CBSE 2012]
Sol. Q ABC is an equilateral triangle.

Areas Related to Circles 327


A = B = C = 60 A

Area of sector AFEA


q
= pr 2 cm 2
360 F E
60
= p(5)2 cm 2
360
B C
D
1 1
Q AF = 2 AB = 2 (10) cm = 5 cm
A

1
= 3.14 5 5 cm 2
6
F E
78.5
= cm 2
6
78.5
Area of all the three sectors = 3 cm 2 = 39.25 cm 2 . B C
6 D
Thus, area of the shaded region = 39.25 cm . 2

Q. 9. In figure OABC is a quadrant of a circle of radius 7 cm. If OD = 4 cm, find the area of the shaded
22
region. [Use = ] [CBSE (Foreign) 2014]
7
Sol. We have, the centre of the circle as O and radius (r) = 7 cm
1 2
Area of the quadrant OABC = r
4
1 22 11 77 A
= 7 7 cm = 7cm =
2 2
cm 2 B
4 7 2 2
Now, the ar(rt COD)
D
1
= OC OD
2 4 cm
1 Q OC = radius = r = 7cm
= 7 4 cm
2

2 OD = 4cm (given)
O C
= 7 2 = 14 cm 2

Area of the shaded region


= (Area of the quadrant OABC) (Area of COD)
77 77 28 49
= 14 cm = cm 2 =
2
cm 2 = 24.5 cm2
2 2 2

TEST YOUR SKILLS


1. Four equal circles are described about the four corners of a square
so that each touches two of the others, as shown in the figure.
Find the area of the shaded region, each side of the square
measuring 14 cm.

FG Take = 22 IJ
H 7K
(CBSE 2012)
328 MathematicsX
2. In the figure, MN and PQ are the arcs of two concentric circles of radii 7 cm and 3.5 cm
LM
respectively and MON = 30. Find the area of the shaded region. Use = 22 OP
N 7 Q
M

P
5 cm
3.
O 30

7 cm
Q
N
(CBSE 2012)
3. A playground is in the form of a rectangle having semi-circles on the shorter sides as
shown in the figure. Find its area when the length of the rectangular portion is 38 m and
the breadth is 10 m. [use = 3.14]

[NCERT Exemplar, CBSE 2006]

4. A chord of a circle of radius 20 cm subtends an angle of 90 at the centre. Find the area
of the corresponding major segment of the circle.
[se = 3.14] [NCERT Exemplar]
5. In the figure, AB is a diameter of the circle with centre O and OA = 7 cm. Find the area
LMuse = 22 OP
of the shaded region.
N 7Q
(CBSE 2012)

B O A

Areas Related to Circles 329


6. With the vertices A, B and C of a triangle ABC as centres A
arcs are drawn with radii 5 cm each as shown in the figure.
If AB = 14 cm, BC = 48 cm and CA = 50 cm, then find the
area of the shaded region. [Use = 3.14] 50

14 cm
cm
[NCERT Exemplar]
Hint:
a = 48 cm, b = 50 cm and c = 14 cm
48 + 50 + 14 B C
s= cm = 56 cm 48 cm
2
Area of ABC
= s(s - a) (s - b) (s - c ) =56 8 6 42 cm2
= 336 cm2
A B C
Area of three sectors = pr 2 + pr 2 + pr 2
360 360 360
2 2
pr pr
= [A + B + C ] = 180
360 360
180 314 25
= 3.14 5 2 = = 39.25 cm2
360 100 2
Area of shaded region = 336 cm2 39.25 cm2 = 295.75 cm2
7. Area of a sector of a circle of radius 36 cm is 54 cm2. Find the length of the corresponding
arc of the sector. [NCERT Exemplar]
Hint:
q
Length of an arc = 2 pr
360
q
Area of a sector = pr 2
360
q = 54
p (36)2
360
54 360
= = 15
36 36
15
Now, length of the arc = 2 p 36 cm
360
= 3 cm
8. A bicycle wheel makes revolutions per minute to maintain a speed of 8.91 km per hour.
Find the diameter of the wheel. (CBSE 2012)
9. In the figure, AB is a diameter of the circle, AC = 6 cm and BC = 8 cm. Find the area of
the shaded region.[Use = 3.14] [NCERT Exemplar]
C

A B

330 MathematicsX
Hint:
AB is diameter
ABC is in a semicircle
C = 90
AB2 = AC2 + BC2

10. Find the area of the shaded region in the given figure, if PQ = 24 cm, PR = 7 cm and O
is the centre of the circle. (CBSE 2012)
Q

R
Hint:
Q O is the centre
RQ is a diameter
PQR is a rt being in a semi circle

RQ = RP 2 + PQ 2
Area of the shaded region = [Area of semi circle ar PQR]

11. A calf is tied with a rope of length 6 m at the corner of a square grassy lawn of side
20 m. If the length of the rope is increased by 5.5 m, find the increase in area of the grassy
lawn in which the calf can graze. [NCERT Exemplar]
Hint:
Increased radius = 6 m + 5.5 m = 11.5 m
Area of sector of Area of sector of
Increase in area = central angle 90 -
central angle 90

and radius 11.5 cm and radius 6m

90 90
= p(11.5)2 - p 6 2 m2
360 360
p
= (11.5 )2 - (6)2 m2
4
22 1
= (11.5 + 6) (11.5 - 6) m2
7 4
12. In the figure, from a rectangular region ABCD with AB = 20 cm, a right triangle AED with
AE = 9 cm and DE = 12 cm, is cut off. On the other end, taking BC as diameter, a semi-
circle is added on outside the region. Find the area of the shaded region.
[Use = = 3.14 ] [AI. CBSE Foreign 2014]

Areas Related to Circles 331


Hint:
AD = DE2 + AE2 = 12 2 + 9 2 = 15cm
ar (ABCD) = AB AD = 20 15 cm2 = 300 cm2
1 1
ar (rt AED) = AE DE = 9 12 = 88.31cm2
2 2
1 1
(r2) = 3.14 (7.5) = 88.31cm2
2
ar (semi circle with diameter BC) =
2 2
Area of shaded region
= ar.(ABCD) + ar. (Semi circle) ar.(rt )
= 300cm2 + 88.31cm2 54cm2 = 334.31cm2

13. In the figure, two concentric circles with centre O, have radii
21 cm and 42 cm. If AOB = 60 , find the area of the shaded
22
region. [Use = ]. [AI. CBSE 2014]
7

Hint:

Area of shaded region = [Area of larger circle area of smaller circle


sector AOB + sector COD]

22 1 22 1 22
42 2 212 ( 42) + ( 21) = 5544 1386 924 + 231 = 3465cm 2
2 2

7 6 7 6 7

14. In the figure, a circle is inscribed in an equilateral triangle ABC of side 12 cm. Find the
radius of the inscribed circle and the area of the shaded region. [Use = 3.14 and 3 = 1.73]
A

B C

332 MathematicsX
Hint: A
3 3
Area of equilatral ABC = (side)2 = 12 12 cm2
4 4
= 1.73 3 12cm2 = 62.28cm2
N M
1 1
Area of BOC = ar ( ABC ) = 62.28 = 20.76cm2 O
3 3
1 r
BC r = 20.76 r = 3.46cm
2
Area of the circle with radius 3.46 cm B L C
2
= r = 3.14 3.46 3.46 cm 2

= 37.6 cm2
Now, area of shaded region = (ar Equilateral ABC)
(Circle with radius 3.46 cm)
= 62. 28 cm2 37.6 cm2 = 24.68cm2

ANSWERS
Test Your Skills
1. 12 cm2 2. 9.625 cm2 3. (380 + 25 ) m2
4. 285.5 cm2 5. 66.5 cm2 6. 295.75 cm2
7. 3 cm 8. 0.63 m 9. 54.5 cm2
10. 161.54 cm2 11. 75.625 m2

Areas Related to Circles 333


13 Surface Areas and Volumes

Facts that Matter


z An object having definite shape and size is called a solid.
Solids like a book, a tile, a match box, an almirah, a room, etc. are called cuboids.
Solids like dice, ice-cubes, sugar-cubes, etc. are called cubes.
Solids like jars, circular pillars, circular pipes, circular pencils, gas jars, road rollers, etc. are
called cylinders.
Solids like conical tents, ice-cream cones, funnels, etc. are called cones.
Solids like cricket balls, footballs etc., are called spheres.
When a cone is cut by a plane parallel to the base of the cone then the portion between the
plane and the base is called the frustum of the cone.

NOTE:
I. Solids like rubber tubes, iron pipes etc., are called hollow cylinders.
II. A plane through the centre of a sphere cuts it into two equal parts. Each part is called a
hemisphere.
We also know that:
I. For a cuboid of length = l, breadth = b and height = h, we have:
(i) Volume = (l b h) cu. units.
(ii) Lateral surface area = [2 (l + b) h] sq. units.
(iii) Total surface area = [2 (lb + bh + hl] sq. units.
II. For a cylinder of base radius = r and height (or length) = h, we have:
(i) Volume = r2h cu. units.
(ii) Curved surface area = 2rh sq. units.
(iii) Total surface area = 2r (h + r) sq. units.
III. For a hollow cylinder having external radius = R, internal radius = r and
height = h, we have:
(i) Volume
= [External volume] [Internal volume]
= (R2h r2h) cu. units = (R2 r2) cu. units.
(ii) Curved surface area
= [External S.A.] + [Internal S.A.]
= [2Rh + 2rh] sq. units = 2h (R + r) sq. units

334
(iii) Total surface area
= (curved S.A.) + (area of the base-ring)
= [(2Rh + 2rh) + 2 (R2 r2)] sq. units
= [(2h (R + r) + 2 (R2 r2)] sq. units

IV. For a cone of base radius = r, height = h and slant height l = h 2 + r 2 , we have:

1
(i) Volume = r2h cu. units.
3
(ii) Curved surface area = rl sq. units.
(iii) Total surface area = [Curved surface area] + [Area of base]
= [rl + r2] sq. units = r (l + r) sq. units.
V. For a sphere of radius r, we have:
4 3
(i) Volume = r cu. units.
3
(ii) Surface area = 4r2 sq. units.
VI. For a hemisphere of radius r, we have:
2 3
(i) Volume = r cu. units.
3
(ii) Surface area = 2r2 sq. units.
(iii) Total surface area = 3r2 sq. units.
VII. For a frustum of a cone of base radius = R, top radius = r, height = h and
slant height = l, we have:
h
(i) Volume = [R2 + r2 + Rr] cu. units.
3
(ii) Lateral surface area
= l (R + r) sq. units, where l2 = h2 + (R r)2
(iii) Total surface area
= [(area of base) + (area of top) + (lateral surface area)]
= [R2 + r2 + l (R + r)] sq. units.
= [R2 + r2 + l (R + r)] sq. units.

NCERT TEXTBOOK QUESTIONS SOLVED


EXERCISE 13.1
22
[Unless stated otherwise, take = ]
7
Q. 1. Two cubes each of volume 64 cm3 are joined end to end. Find the surface area of the resulting
cuboid.

Surface Areas and Volumes 335


Sol. Volume of each cube = 64 cm3
2x
Total volume of the two cubes = 2 64 cm3 x
= 128 cm3 x

Let the edge of each cube = x


x 3 = 64 = 43
x = 4 cm
Now, Length of the resulting cuboid l = 2x cm
Breadth of the resulting cuboid b = x cm
Height of the resulting cuboid h = x cm
Surface area of the cuboid = 2 (lb + bh + hl) = 2 [(2x x) + (x x) + (x 2x)]
= 2 [(2 4 4) + (4 4) + (4 2 4)] cm2
= 2 [32 + 16 + 32] cm2 = 2 [80] cm2 = 160 cm2.
Q. 2. A vessel is in the form of a hollow hemisphere mounted by a hollow cylinder. The diameter of the
hemisphere is 14 cm and the total height of the vessel is 13 cm. Find the inner surface area of
the vessel.
Sol. For cylindrical part:
Radius (r) = 7 cm
Height (h) = 6 cm
Curved surface area
6 cm
= 2rh
22
= 2 7 6 cm2 = 264 cm2 13
7 cm
7 cm 7 cm
For hemispherical part:
Radius (r) = 7 cm
Surface area = 2r2
7 cm
22
= 2 7 7 cm2 = 308 cm2
7
Total surface area
14 cm
= (264 + 308) cm2 = 572 cm2.

Q. 3. A toy is in the form of a cone of radius 3.5 cm mounted


on a hemisphere of same radius. The total height of the toy
is 15.5 cm. Find the total surface area of the toy.
[CBSE 2012]
12.0
Sol. Here, r = 3.5 cm cm
15.5
h = (15.5 3.5) cm = 12.0 cm cm
Surface area of the conical part
= rl
Surface area of the hemispherical part
= 2r2
3.5
cm

336 MathematicsX
Total surface area of the toy
= rl + 2r2 = r (l + 2r) cm2
l2 = (12)2 + (3.5)2
22 35
= (12.5 + 2 3.5) cm2
7 10
l2 = 144 + 12.25 = 156.25
= 11 (12.5 + 7) cm2 l = 12.5 cm
= 11 19.5 cm2 = 214.5 cm2.
Q. 4. A cubical block of side 7 cm is surmounted by a hemisphere. What is the greatest diameter the
hemisphere can have? Find the surface area of the solid.
Sol. Side of the block = 7 cm
The greatest diameter of the hemisphere = 7 cm
Surface area of the solid
= [Total S.A. of the cubical block] 7 cm

+ [S.A. of the hemisphere]


[Base area of the hemisphere]
= (6 l2) + 2r2 r2 7 cm

7
[where l = 7 cm and r = cm ]
2

e2
= 67 + 2j FGH 22 7 7
7

2 2

IJ FG
22 7 7
7K H

2 2
cm 2
IJ
K
F 7I
= (6 49) + (11 7) G 11 J cm
FG 294 + 77 77 IJ cm 2
H 2K
2
H 2K = = 332.5 cm2.

Q. 5. A hemispherical depression is cut out from one face of a cubical wooden block such that the diameter
l of the hemisphere is equal to the edge of the cube. Determine the surface area of the remaining
solid. [AI. CBSE (Foreign) 2014]
Sol. Let l be the side of the cube.
The greatest diameter of the curved hemisphere = l
l
Radius of the curved hemisphere =
2 l

Surface area of hemisphere = 2r2 l

l l l 2
= 2 =
2 2 2

FG l IJ 2
l 2
Base area of the hemisphere =
H 2K =
l

4
l
Surface area of the cube = 6 l2 = 6l2
Surface area of the remaining solid

Surface Areas and Volumes 337


l 2 l 2 24l 2 + 2 l 2 l 2 24l 2 + l 2
= 6l 2 + = =
2 4 4 4
l2
= (24 + ) sq. units.
4
Q. 6. A medicine capsule is in the shape of a cylinder with two
hemispheres stuck to each of its ends (see Fig.). The
5 mm
length of the entire capsule is 14 mm and the diameter
of the capsule is 5 mm. Find its surface area.
[CBSE 2012] 14 mm
Sol. Radius of the hemispherical part
5
= mm = 2.5 mm
2
14 mm

5mm 2.5mm 2.5


mm

9 mm

Surface area of one hemispherical part = 2r2


Surface area of both hemispherical parts
22 FG IJ
25
2
mm 2
= 2 (2r2) = 4r2 = 4
7

H K
10
22 25 25
= 4 mm 2
7 10 10
22 22 25
Area of cylindrical part = 2rh = 2 2.5 9 mm2 = 2 9 mm 2
7 7 10

LM2 22 25 9OP + LM4 22 25 25 OP mm 2


Total surface area =
N 7 10 Q N 7 10 10 Q
FG 2 22 25 IJ LM9 + 50 OP mm = 44 25 14 mm
2 2
=
H 7 10 K N 10 Q 70
44 25 2
= mm 2 = 44 5 cm2 = 220 mm2.
10
Q. 7. A tent is in the shape of a cylinder surmounted by a conical top. If the height and diameter of
the cylindrical part are 2.1 m and 4 m respectively, and the slant height of the top is 2.8 m, find
the area of the canvas used for making the tent. Also, find the cost of the canvas of the tent at the
rate of ` 500 per m2. (Note that the base of the tent will not be covered with canvas.)

338 MathematicsX
Sol. For cylindrical part:
4
Radius (r) = m =2m
2
Height (h) = 2.1 m
22 21 2
Curved surface area = 2rh = 2 2 m
7 10
For conical part:
Slant height (l) = 2.8 m
Base radius (r) = 2 m
22 28 2
Curved surface area = rl = 2 m
7 10
Total surface area
= [Surface area of the cylindrical part] + [Surface area of conical part]
LM
= 2
22
2
OP LM
21
+
22
2
28
m2
OP
N 7 Q N
10 7 10 Q
22 L 42 28 O 22 70 2
7 MN 10 10 PQ
2
= 2 + m = 2 m = 44 m 2
7 10
Cost of the canvas used:
Cost of 1 m2 of canvas = ` 500
Cost of 44 m2 of canvas = ` 500 44 = ` 22000.

Q. 8. From a solid cylinder whose height is 2.4 cm and diameter 1.4 cm, a conical cavity of the same
height and same diameter is hollowed out. Find the total surface area of the remaining solid to
the nearest cm2. [CBSE 2012, CBSE (Delhi) 2014]
Sol. For cylindrical part:
0.7 cm
Height = 2.4 cm
Diameter = 1.4 cm
Radius (r) = 0.7 cm
Total surface area of the cylindrical part
= 2rh + 2r2 = 2r [h + r] 2.4 cm
22 7
= 2 [2.4 + 0.7] cm2
7 10
44 44 31 1364
= 3.1 cm 2 = = cm 2
10 100 100
For conical part:
Base area (r) = 0.7 cm 1.4 cm
Height (h) = 2.4 cm

Slant height (l) = r 2 + h2 = a0.7f + a2.4f


2 2
= 0.49 + 5.76 = 6.25 = 2.5 cm
Curved surface area of the conical part
22 22 7 25 22 25 550
= rl = 0.7 2.5 cm2 = cm 2 = cm 2 = cm 2
7 7 10 10 100 100
Surface Areas and Volumes 339
Base area of the conical part
22 FG IJ
7
2
cm 2 =
22 7
cm 2 =
154
cm 2
= r2 =
7

H K
10 100 100
Total surface area of the remaining solid
Total SA of cylindrical Curved surface area Base area of the
= +
part of conical part conical part

=
LM
1364
cm 2 +
550 OP
cm 2
154
cm 2 =
1914
cm 2
154
cm 2 =
1760
cm 2 = 17.6 cm 2 .
N
100 100 Q 100 100 100 100
Q. 9. A wooden article was made by scooping out a hemisphere from each end of a
solid cylinder, as shown in Fig. If the height of the cylinder is 10 cm, and its
base is of radius 3.5 cm, find the total surface area of the article.
Sol. Radius of the cylinder (r) = 3.5 cm
Height of the cylinder (h) = 10 cm
Total surface area = 2rh + 2r2 = 2r (h + r)
22 35 FG
35
cm 2
IJ
= 2
7

10
10 +
10H K
FG 135 IJ cm 2
= 22
H 10 K = 297 cm2

Curved surface area of a hemisphere = 2r2


Curved surface area of both hemispheres
22 35 35
= 2 2r2 = 4r2 = 4 cm 2 = 154 cm2
7 10 10
Base area of a hemisphere = r2
Base area of both hemispheres = 2r2
2 22 35 35
= 2
22
7
3.5 a f 2
=
7 100
cm 2 = 77 cm2

Total surface area of the remaining solid


= 297 cm2 + 154 cm2 77 cm2 = (451 77) cm2 = 374 cm2.

z Volume of a Combination of Solids


Now, we shall calculate the volumes of combinations of two basic solids. The volume of the
solid formed by joining two basic solids will actually be the sum of the volumes of the
constituents.

NCERT TEXTBOOK QUESTIONS SOLVED


EXERCISE 13.2
22
[Unless stated otherwise, take = ]
7
Q. 1. A solid is in the shape of a cone standing on a hemisphere with both their radii being equal to
1 cm and the height of the cone is equal to its radius. Find the volume of the solid in terms of .

340 MathematicsX
Sol. Here, r = 1 cm and h = 1 cm.
1 2
Volume of the conical part = r h
3
2 3
Volume of the hemispherical part = r
3
1 2 2 1 2
Volume of the solid shape = r h + r 3 = r [h + 2r]
3 3 3
1 1
= (1)2 [1 + 2 (1)] cm3 = 1 [3] cm3
3 3
3
= cm 3 = cm3.
3
Q. 2. Rachel, an engineering student, was asked to make a model shaped like a cylinder with two cones
attached at its two ends by using a thin aluminium sheet. The diameter of the model is 3 cm and
its length is 12 cm. If each cone has a height of 2 cm, find the volume of air contained in the model
that Rachel made. (Assume the outer and inner dimensions of the model to be nearly the same.)
Sol. Here, diameter = 3 cm
3
Radius (r) = cm
2
Total height = 12 cm
Height of a cone (h1) = 2 cm
Height of both cones = 2 2 = 4 cm
Height of the cylinder (h2) = (12 4) cm = 8 cm.
Now, volume of the cylindrical part = r2h2

Volume of both conical parts = 2


LM 1 r h OP
2
N3 Q 1

Volume of the whole model

= r2h2 +
2 2 2
r2h1 = r h2 + h1
LM OP
3 3 N Q
22 FG IJ LM8 + 2 a2fOP cm
3
2
3 22 9

FG
24 + 4 IJ
cm 3
=
7

2H K N 3 Q =
7 4 3H K
22 3
= 28 cm 3 = 22 3 cm3 = 66 cm3.
7 4
Q. 3. A gulab jamun, contains sugar syrup up to about 30% of its volume. Find
approximately how much syrup would be found in 45 gulab jamuns, each
shaped like a cylinder with two hemispherical ends with length 5 cm and
diameter 2.8 cm (see figure). [CBSE 2008]
Sol. Since, a gulab jamun is like a cylinder with hemispherical ends.
Total height of the gulab jamun = 5 cm.
Diameter = 2.8 cm
Radius = 1.4 cm
Length (height) of the cylindrical part = 5 cm (1.4 + 1.4) cm
Surface Areas and Volumes 341
= 5 cm 2.8 cm = 2.2 cm
Now, volume of the cylindrical part
= r2h
2 3
Volume of a hemispherical end = r 1.4 cm
3
Volume of both the hemispherical ends
FG 2 r IJ
3 4 3
= 2
H3 K =
3
r
2.2
Volume of a gulab jamun cm 5 cm
2 4 3
= r h + r
3

= r
2 LM OP
4
h+ r
N 3Q
22 L 4 O
a1.4 f M2.2 + a1.4 fP cm
2 3 1.4 cm
=
7 N 3 Q
22 14 14 L 22 56 O
10 10 MN 10 30 PQ
3
= + cm 2.8 cm
7
22 2 14 L 66 + 56 O 44 14 122
= M
10 10 N 30 Q P cm = 3
100

30
cm 3

Volume of 45 gulab jamuns

= 45
LM 44 14 122 OP cm 3
=
15 44 14 122
cm 3
N 100 30 Q 1000
Since, the quantity of syrup in gulab jamuns

= 30% of [volume] = 30% of


LM 15 44 14 122 OP cm
3
N 1000 Q
30 15 44 14 122
= cm 3 = 338.184 cm3 = 338 cm3 (approx.)
100 1000
Q. 4. A pen stand made of wood is in the shape of a cuboid
with four conical depressions to hold pens. The
dimensions of the cuboid are 15 cm by 10 cm by 3.5
cm. The radius of each of the depressions is 0.5 cm
and the depth is 1.4 cm. Find the volume of wood in
the entire stand (see Fig.). [NCERT Exemplar]
Sol. Dimensions of the cuboid are 15 cm, 10 cm and
3.5 cm.
35
Volume of the cuboid = 15 10 cm 3
10
= 15 35 cm3
= 525 cm3
Since each depression is conical with base radius (r) = 0.5 cm and depth (h) = 1.4 cm,

342 MathematicsX
Volume of each depression (cone)

1 2 1 22 5 FG IJ 2
14
cm 3
=
3
r h =
3

7

10 H K
10
Since there are 4 depressions,
Total volume of 4 depressions
1 22 5 5 14 4 11 44
= 4 cm 3 = cm 3 = cm 3
3 7 10 10 10 3 10 30
Now, volume of the wood in entire stand
= [Volume of the wooden cuboid] [Volume of 4 depressions]
44 15750 44 15706
= 525 cm 3 cm 3 = cm 3 = cm 3 = 523.53 cm3.
30 30 30
Q. 5. A vessel is in the form of an inverted cone. Its height is 8 cm and the radius of its top, which is
open, is 5 cm. It is filled with water up to the brim. When lead shots, each of which is a sphere
of radius 0.5 cm are dropped into the vessel, one-fourth of the water flows out. Find the number
of lead shots dropped in the vessel.
Sol. Height of the conical vessel (h) = 8 cm
5 cm
Base radius (r) = 5 cm
1 2
Volume of the cone = r h
3
=
1 22
3 7
af 2
5 8 cm 3
8 cm

4400
= cm 3
21
Since, Volume of the cone = [Volume of water in the cone]
4400
[Volume of water in the cone] = cm 3
21
1
Now, Total volume of lead shots = of [Volume of water in the cone]
4
1 4400 1100
= cm 3 = cm 3
4 21 21
Since, radius of a lead shot (sphere) (r) = 0.5 cm
4 3 4 22 5 5 5
Volume of 1 lead shot = r = cm 3
3 3 7 10 10 10
1100 LM OP
Number of lead shots =
Total volume of lead shots
=
21 N Q
Volume of 1 lead shot LM
4 22 5 5 5 OP
N
3 7 1000 Q
1100 3 7 1000
= = 100
21 4 22 5 5 5
Thus, the required number of lead shots = 100.

Surface Areas and Volumes 343


Q. 6. A solid iron pole consists of a cylinder of height 220 cm and base 8 cm
diameter 24 cm, which is surmounted by another cylinder of height
60 cm and radius 8 cm. Find the mass of the pole, given that 1 cm3 of
60 cm
iron has approximately 8 g mass. (Use = 3.14) [CBSE 2012]
Sol. Height of the big cylinder (h) = 220 cm
24
Base radius (r) = cm = 12 cm 220 cm
2
24 cm
Volume of the big cylinder = r2h = (12)2 220 cm3
Also, height of smaller cylinder (h1) = 60 cm 12 cm
Base radius (r1) = 8 cm
Volume of the smaller cylinder r12h1 = (8)2 60 cm3
Volume of iron
= [Volume of big cylinder] + [Volume of the smaller cylinder]
= 220 122 + 60 82 cm3
314
= 3.14 [220 12 12 + 60 8 8] cm3 = [20 144 + 60 64] cm3
100
314 314
= [31680 + 3840] cm3 = 35520 cm 3
100 100
8 314 35520 89226240 8922624
Mass of iron = g = g = g
100 100 10000
= 892.2624 kg = 892.26 kg.

Q. 7. A solid consisting of a right circular cone of height 120 cm and radius 60 cm standing on a
hemisphere of radius 60 cm is placed upright in a right circular cylinder full of water such that
it touches the bottom. Find the volume of water left in the cylinder, if the radius of the cylinder
is 60 cm and its height is 180 cm.
Sol. Height of the conical part = 120 cm.
Base radius of the conical part = 60 cm.
1 22
Volume of the conical part = 60 2 120 cm 3
3 7
Radius of the hemispherical part = 60 cm.
2 22
Volume of the hemispherical part = 60 3 cm 3
3 7
Volume of the solid
= [Volume of conical part] + [Volume of hemispherical part]

LM 1 22 60 2 OP LM 2 22 60 OP cm
120 + 3 3 2 22
602 [60 + 60] cm3
=
N3 7 Q N3 7 Q =
3 7
2 22 2 22 60 60 40 6336000
= 60 60 120 cm3 = cm 3 = cm 3
3 7 7 7
Volume of the cylinder = r2h
344 MathematicsX
22 22 60 60 180
= 602 180 cm3 = cm 3 60 cm
7 7
14256000
= cm 3 120
7 cm
180 cm
14256000
Volume of water in the cylinder = cm 3
7
Volume of the water left in the cylinder
LM 14256000 6336000 OP cm 3
=
7920000
cm 3
60 cm
=
N 7 7 Q 7
= 1131428.57142 cm3
1131428.57142 3
= m [ 1000000 cm3 = 1 m3]
1000000
= 1.13142857142 m3 = 1.131 m3 (approx).

Q. 8. A spherical glass vessel has a cylindrical neck 8 cm long, 2 cm in diameter; the diameter of the
spherical part is 8.5 cm. By measuring the amount of water it holds, a child finds its volume to
be 345 cm3. Check whether she is correct, taking the above as the inside measurements, and
= 3.14.
Sol. Volume of the cylindrical part
2
= r2h = 3.14 12 8 cm3 [ Radius = = 1 cm, height (h) = 8 cm]
2
314
= 8 cm 3
100
Volume of the spherical part
4 3 8.5
= r1 Here r1 = cm
3 2
4 314 85 85 85
= cm 3 2cm
3 100 20 20 20
Total volume of the glass-vessel

=
LM 314 8OP + LM 314 4 85 85 85 OP 8 cm

N 100 Q N 100 3 8000 Q


314 L 4 85 85 85 O
=
100 NM 8+
24000 PQ cm 3

314 L 614125 O
=
100 NM8+
6000 PQ
3
cm

314 L 48000 + 614125 O 314 L 662125 O


=
100 NM 6000 P
Q
3
cm =
100 MN 6000 PQ
cm 3

314 5297 157 5297 831629


= cm 3 = = cm 3
100 48 100 24 2400
= 346.51 cm3 (approx.)
Volume of water in the vessel = 346.51 cm3 8.5 cm

Surface Areas and Volumes 345


Since, the child finds the volume as 345 cm3
The childs answer is not correct
The correct answer is 346.51 cm3.

z Conversion of Solids from One Shape to Another


We generally come across situations where objects are converted from one shape to another, or
a liquid, originally filled in a container of a particular shape is poured into another container
of a different shape or size. In such situations the volume of the material remains the same.

NCERT TEXTBOOK QUESTIONS SOLVED


EXERCISE 13.3
22
[Take = , (Unless stated otherwise)]
7
Q. 1. A metallie sphere of radius 4.2 cm is melted and recast into the shape of a cylinder of radius
6 cm. Find the height of the cylinder.
Sol. Radius of the sphere (r1) = 4.2 cm
FG 4 r IJ
3 4 22 42 42 42
cm 3
Volume of the sphere
H3 K 1 =
3 7

10 10 10
Radius of the cylinder (r2) = 6 cm
Let h be the height of the cylinder
22
Volume of the cylinder = r2h = 6 6 h cm3
7
Since, Volume of the metallic sphere = Volume of the cylinder.
4 22 42 42 42 22
= 66h
3 7 10 10 10 7
4 22 42 42 42 7 1 1
h = cm
3 7 10 10 10 22 6 6
4774 2744
= cm = cm = 2.744 cm.
10 10 10 1000

Q. 2. Metallic spheres of radii 6 cm, 8 cm and 10 cm, respectively, are melted to form a single solid
sphere. Find the radius of the resulting sphere.
Sol. Radii of the given spheres are:
r1 = 6 cm
r2 = 8 cm
r3 = 10 cm
Volume of the given spheres are:
4 3 4 3 4 3
V1 = r1 , V2 = r2 and V3 = r3
3 3 3
Total volume of the given spheres

346 MathematicsX
= V1 + V2 + V3
4 3 4 3 4 3 4 4 22
= r1 + r2 + r3 = r13 + r23 + r33 = [63 + 83 + 103] cm3
3 3 3 3 3 7
4 22 4 22
= [216 + 512 + 1000] cm3 = 1728 cm 3
3 7 3 7
Let the radius of the new big sphere be R.
Volume of the new sphere
4 4 22 2 1728
= R3 = R3 2 864
3 3 7
Since, the two volume must be equal. 2 432
2 216
4 22 4 22
R3 = 1728 cm 3 2 108
3 7 3 7
R 3 = 1728 2 54
3 27
R 3 = 23 23 33
3 9
R 3 = (2 2 3)3
3
R = 223
R = 12 cm
Thus, the required radius of the resulting sphere = 12 cm.

Q. 3. A 20 m deep well with diameter 7 m is dug and the earth from digging is evenly spread out to
form a platform 22 m by 14 m. Find the height of the platform.
Sol. Diameter of the cylindrical well = 7 m
7
Radius of the cylinder (r) = m
2
Depth of the well (h) = 20 m
22 7 7
Volume = r2h = 20 m 3 = 22 7 5 m3
7 2 2
Volume of the earth taken out = 22 7 5 m3
Now this earth is spread out to form a cuboial platform having
length = 22 m
breadth = 14 m
Let h be the height of the platform.
Volume of the platform = 22 14 h m3
22 14 h = 22 7 5
22 7 5 5
h = = m = 2.5 m
22 14 2
Thus, the required height of the platform is 2.5 m.

Q. 4. A well of diameter 3 m is dug 14 m deep. The earth taken out of it has been spread evenly all
around it in shape of a circular ring of width 4 m to form an embankment. Find the height of the
embankment. [(CBSE Sample Paper 2011) CBSE 2012]

Surface Areas and Volumes 347


Sol. Diameter of cylindrical well (d) = 3 m
3
Radius of the cylindrical well (r) = m = 1.5 m
2
Depth of the well (h) = 14 m
22 FG IJ
15
2
14 m 3
Volume = r2h =
7

H K
10
22 15 15 14 3
= m = 11 3 3 m3 = 99 m3
7 10 10
Let the height of the embankment = H metre.
Internal radius of the embankment (r) = 1.5 m.
External radius of the embankment R = (4 + 1.5) m = 5.5 m.
Volume of the embankment
= R2H r2H = H [R2 r2] = H (R + r) (R r)
22 22
= H (5.5 + 1.5) (5.5 1.5) = H 7 4 m3
7 7
Since, Volume of the embankment = Volume of the cylindrical well
LM
22 OP
H 7 4 = 99

7N Q
7 1 1 9
H = 99 m = m = 1.125 m
22 7 4 8
Thus, the required height of the embankment = 1.125 m.

Q. 5. A container shaped like a right circular cylinder having diameter 12 cm and height 15 cm is full
of ice cream. The ice cream is to be filled into cones of height 12 cm and diameter 6 cm, having
a hemispherical shape on the top. Find the number of such cones which can be filled with ice cream.
[CBSE 2009]
Sol. For the circular cylinder:
12
Diameter = 12 cm Radius = = 6 cm
2
Height (h) = 15 cm
Volume = r2h
22
Volume of total ice cream = 6 6 15 cm3
7
For conical + hemispherical ice-cream cone:
Diameter = 6 cm radius (R) = 3 cm
Height of conc. (H) = 12 cm
Volume = (Volume of the conical part) + (Volume of the hemispherical part)
1 2 2 3 1
= R H + R = R 2 H + 2 R
3 3 3
1 22 22 3
= 3 3 [12 + 2 3] cm3 = 18 cm 3
3 7 7
Let number of ice-cream cones required to fill the total ice cream = n.

348 MathematicsX
n
LM 22 3 18OP =
22
6 6 15
N 7 Q 7
22 7 1 1
n = 6 6 15
7 22 3 18
n = 2 5 = 10
Thus, the required number of cones is 10.
Q. 6. How many silver coins, 1.75 cm in diameter and of thickness 2 mm, must be melted to form a
cuboid of dimensions 5.5 cm 10 cm 3.5 cm ?
Sol. For a circular coin:
Diameter = 1.75 cm
175
Radius (r) = cm
200
2
Thickness (h) = 2 mm = cm
10
Volume = r2h
22 FG IJ
175
2
2
cm 3
=
7

H K
200

10
| A coin is like a cylinder

For a cuboid:
Length (l) = 10 cm, Breadth (b) = 5.5 cm
and Height (h) = 3.5 cm
55 35
Volume = 10 cm 3
10 10
Number of coins
Let the number of coins need to melt be n
LM10 55 35 OP LM 22 175 175 2 OP
n =
N 10 10 Q N 7 200 200 10 Q
55 35 7 200 200 10
= 10
= 16 25 = 400
10 10 22 175 175 2
Thus, the required number of coins = 400.
Q. 7. A cylindrical bucket, 32 cm high and with radius of base 18 cm, is filled with sand. This bucket
is emptied on the ground and a conical heap of sand is formed. If the height of the conical heap
is 24 cm, find the radius and slant height of the heap. [CBSE 2014]
Sol. For the cylindrical bucket:
Radius (r) = 18 cm
Height (h) = 32 cm
Volume = r2h
22
= (18)2 32 cm3
7
22 FG
18 18 32 cm 3
IJ
Volume of the sand =
7 H K
Surface Areas and Volumes 349
For the conical heap:
Height (H) = 24 cm
Let radius of the base be (R).
1
R 2 H =
1 22

LM R 2 24 cm 3
OP
Volume of conical heap =
3 3 7 N Q
Radius of the conical heap of sand:
Volume of the conical heap of sand = Volume of the sand
1 22 27
R 2 24 = 18 18 32
3 7 7
22 7 1
R2 = 18 18 32 3 = 18 18 4 = 182 22
7 22 24
R = 18 2 2 2 = 18 2 cm = 36 cm
Slant Height
Let l be the slant height of the conical heap of the sand.
l 2 = R2 + H2
l 2 = 242 + 362 l H
l 2 = (12 2)2 + (12 3)2
l 2 = 122 [22 + 32]
(R)
l 2 = 122 13

l = 12 2 13 = 12 13
Thus, the required height = 36 cm and slant height = 12 13 cm.
Q. 8. Water in a canal, 6 m wide and 1.5 m deep, is flowing with a speed of 10 km/h. How much area will
it irrigate in 30 minutes, if 8 cm of standing water is needed ? [CBSE 2012, AI. CBSE 2014]
Sol. Width of the canal = 6 m
Depth of the canal = 1.5 m
Length of the water column in 1 hr = 10 km
FG i. e. , 1 hrIJ
Length of the water column in 30 minutes
H 2 K
10 10000
=km = m = 5000 m
2 2
1
Volume of water flown in hr
2
15
= 6 1.5 5000 m3 = 6 5000 m 3 = 45000 m3
10
Since the above amount (volume) of water is spread in the form of a cuboid of height as
FG = 8 IJ
8 cm
H 100 K
m .

Let the area of the cuboid = a


8
Volume of the cuboid=Area Height = a m3
100
350 MathematicsX
8
Thus, a = 45000
100
45000 100 4500000 2
a = = m
8 8
562500
a = 562500 m2 = hectares = 56.25 hectares
10000
Thus, the required area = 56.25 hectares.

Q. 9. A farmer connects a pipe of internal diameter 20 cm from a canal into a cylindrical tank in her
field, which is 10 m in diameter and 2 m deep. If water flows through the pipe at the rate of
3 km/h, in how much time will the tank be filled ?
Sol. Diameter of the pipe = 20 cm
20
Radius of the pipe (r) = cm = 10 cm
2
Since, the water flows through the pipe at 3 km/hr.
Length of water column per hour
(h) = 3 km = 3 1000 m = 3000 100 cm = 300000 cm.
Volume of water = r2h = 102 300000 cm3 = 30000000 cm3
Now, for the cylindrical tank,
Diameter = 10 m
10
Radius (R) = m = 5 100 cm = 500 cm
2
Height (H) = 2 m = 2 100 cm = 200 cm
Volume of the cylindrical tank
= R2H = (500)2 200 cm3
Now, time required to fill the tank
Volume of the tank 500 500 200
= = hrs
Volume of water flown in 1 hour 30000000
552 5 5
= hrs = hrs = 60 minutes = 100 minutes.
30 3 3
z Frustum
Let us consider a right circular cone, which is sliced through by a plane parallel to its base,
when the smaller conical portion is removed, the resulting solid is called a Frustum of right
circular cone. The formulae involving the frustum of a cone having:
h = the height
l = the slant height r2
r1 and r2 = the radii of the ends (r1 > r2), are given below: h
1
(i) Volume of the frustum of the cone = h (r12 + r22 + r1 r2) r1
3
(ii) Curved surface area of the frustum of the cone
= (r1 + r2) l
LMwhere l = b
h 2 + r1 r2 g OPQ
2

N
(iii) Total surface area of the frustum of the cone = l (r1 + r2) + r12 + r22

Surface Areas and Volumes 351


NCERT TEXTBOOK QUESTIONS SOLVED
EXERCISE 13.4
22
[Use = , unless stated otherwise]
7
Q. 1. A drinking glass is in the shape of a frustum of a cone of height 14
cm. The diameters of its two circular ends are 4 cm and 2 cm. Find 1cm
the capacity of the glass.
Sol. We have: r1 = 2 cm
r2 = 1 cm
h = 14 cm 14 cm

Volume of the glass


= h (r12 + r22 + r1 r2)

1 22
= 14 [22 + 12 + 2 1] cm3 2 cm
3 7
22 22 308 2
= 2 [4 + 1 + 2] cm3 = 2 [7] cm3 = cm 3 = 102 cm 3 .
3 3 3 3
Q. 2. The slant height of a frustum of a cone is 4 cm and the perimeters (circumference) of its circular
ends are 18 cm and 6 cm. Find the curved surface area of the frustum.
Sol. We have:
Slant height (l) = 4 cm
2r1 = 18 cm
and 2r2 = 6 cm
4 cm
18
r1 = = 9 cm
2
6
r2 = = 3 cm
2
Curved surface area of the frustum of the cone
= (r1 + r2) l = (r1 + r2) l = (9 + 3) 4 cm2 = 12 4 cm2 = 48 cm2.

Q. 3. A fez, the cap used by the Turks, is shaped like the frustum of a
cone (see Fig.). If its radius on the open side is 10 cm, radius at
the upper base is 4 cm and its slant height is 15 cm, find the area
of material used for making it. [CBSE 2012]
Sol. Here, the radius of the open side (r1) = 10 cm
The radius of the upper base (r2) = 4 cm
Slant height (l) = 15 cm
Area of the material required
= [Curved surface area of the frustum] + [Area of the top end]
= (r1 + r2) l + r22

=
22
7
b g
10 + 4 15 +
22
7
44

352 MathematicsX
LM 22 14 15 + 22 16OP cm 2 LMb22 2 15g + FG 22 16 IJ OP cm 2
=
N7 7 Q =
N H 7 KQ
352 4620 + 352 4972 2
= 660 + cm 2 = cm 2 = cm 2 = 710 cm 2 .
7 7 7 7
Q. 4. A container, opened from the top and made up of a metal sheet, is in the form of a frustum of a
cone of height 16 cm with radii of its lower and upper ends as 8 cm and 20 cm, respectively. Find
the cost of the milk which can completely fill the container, at the rate of ` 20 per litre. Also find
the cost of metal sheet used to make the container, if it costs ` 8 per 100 cm2. (Take = 3.14)
[AI. CBSE 2014]
Sol. We have: r1 = 20 cm, r2 = 8 cm 20 cm
and h = 16 cm
Volume of the frustum
1
= h [r12 + r22 + r1 r2]
3
16 cm
1 314
= 16 [202 + 82 + 20 8] cm2
3 100
1 314
= 16 [400 + 64 + 160] cm2
3 100 8
1 314
16 624 cm 3 =
LM 314 16 208OP cm 3
cm
=
3 100 N 100 Q
=
LM 314 16 208OP 1000 litres = 314 16 208
litres
N 100 Q 100000
314 16 208
Cost of milk = ` 20
100000
628 16 208 2089984
= ` = ` = ` 208.998 ` 209.
10000 10000
Now, slant height of the given frustum

l = b
h 2 + r1 r2 g 2
= b
16 2 + 20 8 g 2
= 16 2 + 12 2

= 256 + 144 = 400 = 20 cm


Curved surface area
314
= (r1 + r2) l = (20 + 8) 20 cm2
100
314 314 8792
= 28 20 cm2 = 28 cm 2 = cm 2 = 1758.4 cm2
100 5 5
Area of the bottom
314 20096
= r2 = 8 8 cm 2 = cm 2 = 200.96 cm2
100 100
Total area of metal required = 1758.4 cm2 + 200.96 cm2 = 1959.36 cm2
8
Cost of metal required = ` 1959.36 = ` 156.75.
100
Surface Areas and Volumes 353
Q. 5. A metallic right circular cone 20 cm high and whose vertical angle is 60 is cut into two parts
at the middle of its height by a plane parallel to its base. If the frustum so obtained be drawn into
1
a wire of diameter cm , find the length of the wire. [CBSE 2012]
16
Sol. Let us consider the frustum DECB of the metallic cone ABC

Here, r1 = BO and r2 = DF
r1 1
In AOB,
bh 1 + h2 g = tan 30 =
3
1 1
r1 = (h1 + h2) = 20
3 3
r2
In ADF, = tan 30
h1
1 1
r2 = h1 = 10
3 3
Now, the volume of the frustum DBCE

1 1 LMF 20 I + F 10 I 2 2
20 10 OP
=
3
h2 [r12 + r22 + r1 r2] =
3
10
MNGH 3 JK GH 3 JK +
3

3 PQ

L 400 + 100 + 200 OP = 10 L 700 O


10 M
=
3 N 3 3 3 Q 3 MN 3 PQ
Let l be the length and D be diameter of the wire drawn from the frustum. Since the wire
is in the form of a cylinder,
Volume of the wire = r2l
FG D IJ
D 2 l
=
2
l
QD =
1
=
H 2K
4
l =
4 16 16 16
[Volume of the frustum] = [Volume of the wire]
LM 10 700 OP l

N3 3 Q
=
4 16 16
l 10 700
=
4 16 16 33

354 MathematicsX
10 700 7168000
l = 4 16 16 = = 7964.44 m
33 9 100
Thus, the required length of the wire = 7964.44 m.

NCERT TEXTBOOK QUESTIONS SOLVED


EXERCISE 13.5
Q. 1. A copper wire, 3 mm in diameter, is wound about a cylinder whose length is 12 cm, and diameter
10 cm, so as to cover the curved surface of the cylinder. Find the length and mass of the wire,
assuming the density of copper to be 8.88 g per cm3.
Sol. Since, diameter of the cylinder = 10 cm
10
Radius of the cylinder (r) = cm = 5 cm
2
Length of wire in completely one round
2r = 2 3.14 5 cm = 31.4 cm
3
Diameter of wire = 3 mm = cm
10
3
The thickness of cylinder covered in one round = cm
10
Number of rounds (turns) of the wire to cover 12 cm
12 10
= = 12 = 40
3 / 10 3
Length of wire required to cover the whole surface = Length of wire required to complete
40 rounds
= 40 31.4 cm = 1256 cm
3 3
Now, radius of the wire = mm = cm
2 20
Volume of wire = r2l
22 3 3
= 1256 cm3
7 20 20
Density of wire = 8.88 gm/cm3
Weight of the wire = [Volume of the wire] Density

LM 22 3 3 1256OP 8.88 gm = 22 3 3 1256 888


=
N 7 20 20 Q 7 20 20 100
gm

= 788 g (approx.)
Q. 2. A right triangle, whose sides are 3 cm and 4 cm (other than hypotenuse) is made to revolve about
its hypotenuse. Find the volume and surface area of the double cone so formed. Choose value of
as found appropriate. [CBSE 2012]

Surface Areas and Volumes 355


A

3 cm 4 cm

D
B C

A
Sol. Let us consider the rt BAC, rt. angled at A such that
AB = 3 cm, AC = 4 cm

Hypotenuse BC = 3 2 + 4 2 = 5 cm
Obviously, we have obtained two cones on the same base AA such that
the radius = DA or DA
AD AB
Now, = | ADB ~ CAB
CA CB
AD 3 3 12
= AD = 4 = cm
4 5 5 5
DB AB
Also, =
AB CB
DB 3 33 9
= DB = = cm
3 5 5 5
9 16
Since, CD = BC DB CD = 5 = cm
5 5
Now, volume of the double cone
LM 1 FG 12 IJ 2
9 1 12FG IJ 2
16 OP 1 FG IJ LM 9 + 16 OP cm
12
2

MN 3 H 5 K H K
+ 3
= 5 3 5 5 PQ cm
3 =
3

H K N5 5 Q
5
1 144 1 314 144
= 5 cm 3 = 5 cm 3 = 30.14 cm3
3 25 3 100 25
Surface area of the double cone
FG 12 3IJ + FG 12 4IJ cm 2 12
=
H 5 K H 5 K =
5
[3 + 4] cm2

314 12
= 7 cm 2 = 52.75 cm2
100 5
Q. 3. A cistern, internally measuring 150 cm 120 cm 110 cm, has 129600 cm3 of water in it. Porous
bricks are placed in the water until the cistern is full to the brim. Each brick absorbs one-seventeenth
of its own volume of water. How many bricks can be put in without overflowing the water, each
being 22.5 cm 7.5 cm 6.5 cm?
356 MathematicsX
Sol. Dimensions of the cistern are 150 cm, 120 cm and 110 cm.
Volume of the cistern = 150 120 110 cm3 = 1980000 cm3
Volume of water contained in the cistern = 129600 cm3
Free space (volume) which is not filled with water
= 1980000 129600 cm3 = 1850400 cm3
Now, Volume of one brick = 22.5 7.5 6.5 cm3
225 75 65
= cm 2 = 1096.875 cm3
10 10 10
l
Volume of water absorbed by one brick = (1096.875) cm 3
17
Let n bricks can be put in the cistern.
n
Volume of water absorbed by n bricks= (1096.875)
17
[Volume occupied by n bricks] = [(free space in the cistern)
+ (volume of water absorbed by n-bricks)]
n
[n (1096.875)] = [1850400 + (1096.875)]
17
n
1096.875 n (1096.875) = 1850400
17
FG n IJ
n
H 17 K
1096.875 = 1850400

16 1850400
n =
17 1096.875
1850400 1000 17
n = = 1792.4102 1792
1096875 16
Thus, 1792 bricks can be put in the cistern.

Q. 4. In one fortnight of a given month, there was a rainfall of 10 cm in a river valley. If the area of
the valley is 97280 km2, show that the total rainfall was approximately equivalent to the addition
to the normal water of three rivers each 1072 km long, 75 m wide and 3 m deep.
Sol. Volume of three rivers = 3 {(Surface area of a river) Depth}
RSFG 1072 km 75 kmIJ 3 kmUV = 3 RS 241200 km UV 3
= 3
TH 1000 K 1000 W T 1000000 W
723600
= km 3 = 0.7236 km3
1000000
Volume of rainfall = (surface area) (height of rainfall)
10
km 3
LMQ10 cm = 10 kmOP
= 97280
100 1000 N 100 1000 Q
9728
= km 3 = 9.728 km3
1000
Since, 0.7236 km3 9.728 km3
The additional water in the three rivers is not equivalent to the rainfall.
Surface Areas and Volumes 357
Q. 5. An oil funnel made of tin sheet consists of a 10 cm long cylindrical 18 cm
portion attached to a frustum of a cone. If the total height is 22 cm,
diameter of the cylindrical portion is 8 cm and the diameter of the
top of the funnel is 18 cm, find the area of the tin sheet required to
make the funnel (see Fig.). 22 cm
Sol. We have, for the cylindrical part
10 cm
Diameter = 8 cm
Radius (r) = 4 cm
Height = 10 cm 8 cm
22 22
Curved Surface area = 2rh = 2 4 10 cm2 = 80 cm2
7 7
18
For the frustum: r1 = cm = 9 cm
2
8 cm
and r2 = = 4 cm
2
Height (H) = 22 10 = 12 cm
Slant height (l) = H 2 + ( r1 r2 ) 2 = 12 2 + (9 4 ) 2

= 144 + 5 2 = 144 + 25 = 169 = 13


22
Surface area, (r1 + r2) l = (4 + 9) 13 cm2
7
22 22
= 13 13 cm2 = 16.9 cm2
7 7
Area of tin required = [Area of the frustum] + [Area of cylindrical portion]
22 22 22
= 169 cm2 + 80 cm2 = (169 + 80) cm2
7 7 7
22 5478 4
= 249 cm2 = cm 2 = 782 cm2
7 7 7
Q. 6. Derive the formula for the curved surface area and total surface area of the frustum of a cone, given
to you in Section 13.5, using the symbols as explained.
C1 r1 Q
P

l h

C2 r2
R S
l1 h1

l2 h2

358 MathematicsX
Sol. We have,
Curved surface area of the frustum PQRS
LMcurved surface area OP LMcurved surface area OP
=
MN of the rt circular
cone OPQ PQ MN of theconert circular
ORS PQ
= r1 l1 r2 l2 ...(1)
Now, OC1 Q ~ OC2S
OQ QC1 OC1
= =
OS SC2 OC2
l1 r h1
= 1 =
l2 r2 h2

l + l2 r1
= | l1 = l + l2
l2 r2
l r l r
+1 = 1 = 1 1
l2 r2 l2 r2


FG r r IJ l
1 2
l =
H r K 2
2 ...(2)

Now, from (1),


curved surface area of the frustum
FG r l IJ r l = l L r r O = l F r r I
2 2 2
= r1
Hr K
1

2
2 MN r PQ GH r JK
2 2 2
1

2
2 2
1

2
2

= l M
L (r + r ) (r r ) OP = FG r r IJ l (r + r )
1 2 1 2 1 2

N
2
r Q H r K
2 2
2 1 2

= l (r + r ) | G
F r r IJ l 1 2
1 2
H r K 2
2 = l From (2)

Now, the total surface area of the frustum


= (curved surface area) + (base surface area) + (top surfaces area)
= l (r1 + r2) + r22 + r12 = (r1 + r2) l + (r12 + r22)
= [(r1 + r2) l + r12 + r22]

Q. 7. Derive the formula for the volume of the frustum of a cone, given to you in Section 13.5, using
the symbols as explained.
Sol. We have,
[Volume of the frustum RPQS]
= [Volume of right circular cone OPQ] [Volume of right circular cone ORS]
1 1
= r12 h1 r22 h2
3 3
1
= [r12 h r22 h2] ...(1)
3
Surface Areas and Volumes 359
Since OC1Q ~ OC2S C1 r1 Q
P
OQ QC1 OC1
= =
OS SC2 OC2


l1 r h
= 1 = 1 h1 =
FG r 1
h2
IJ l h

l2 r2 h2 Hr 2 K C2 r2
...(2)
R S
r1 h + h2 l1 h1
=
r2 h2
l2 h2
r1 h
= +1
r2 h2


h
=
r1
1 h=
LM r 1OP
1
h2
h2 r2 Nr Q
2
O

h2
h = (r1 r2) ...(3)
r2
From (1) and (2), we have

{volume of the frustum RPQS} =


1 LM
r
r12 1 h2 r22 h2
OP
3 N
r2 Q
=
1 r3 LM
2
1 r2 h2
OP
3 r2 N Q
1 h
= [ r13 r23 ] 2
3 r2

=
1 2 2
( r1 + r2 + r1 r2 ) ( r1 r2 )
h2LM OP
3 r2 N Q
1
= (r12 + r22 +r1r2) h | From (3)
3

MORE QUESTIONS SOLVED


I. VERY SHORT ANSWER TYPE QUESTIONS
Q. 1. A cylinder, a cone and a hemisphere have the same values for r and h. Find the ratio of their
volumes.
Sol. Let radius be r and height h.
r = h
[Volume of a cylinder] : [Volume of a cone] : [Volume of a hemisphere]

1 2 2 3
r2h : r h : r
3 3

1 2
h : h : r | Dividing by r2
3 3
360 MathematicsX
1 2
h : h : h | r = h
3 3
1 2
1 : : | Dividing by h
3 3
3 : 1 : 2 | multiplying by 3
The required ratio = 3 : 1 : 2

Q. 2. If two identical solid cubes of side x are joined end to end, then the total surface area of the
resulting cuboid is 12x2. Is it true?
Sol. The total surface area of a cube of side x is 6x2
When they are joined end to end, the lenght bcomes 2x
Total surface area = 2[lh + bh + hl] = 2 [(2x x) + (x x) + (2x x)]
= 2 [2x2 + x2 + 2x2] = 2 [5x2] = 10x2 12x2
False
Q. 3. A spherical ball is melted to make eight new identical balls. Then the radius of each new ball is
1
th of the radius of the original ball. Is it true?
8
4
Sol. Radius R of original ball volume = R 3
3
4 3
And Radius r of the new ball volume = r
3


4
R 3 = 8
4LM
r 3
OP
3 3 N Q
1
R 3 = (2r)3 R = 2r or r = R
2
Radius of the new ball = Half the radius of original ball.
False
Q. 4. If a solid cone of base radius r and height h is placed over a solid cylinder having same base
radius r and height - h as that of the cone, then the curved surface area of the shape is
r ( h 2 + r 2 ) + 2 rh. Is it true?
Sol. Curved surface area of a cone = rl = r h2 + r2 | l = r 2 + h2
And curved surface area of the cylinder = 2rh
The curved surface area of the combination = r ( h 2 + r 2 ) + 2rh
True.
Q. 5. A cylinder and a cone are of the same base radius and same height. Find the ratio of the volumes
of the cylinder of that of the cone. (CBSE 2009)
Sol. Let the base radius = r and height = h
Volume of cylinder r 2 h 1 3 3
= = =1 =
Volume of cone 1 2 1/3 1 1
r h
3
The required ratio = 3 : 1
Surface Areas and Volumes 361
II. SHORT ANSWER TYPE QUESTIONS

Q. 1. A metallic sphere of radius 4.2 cm is melted and recast into the shape of a cylinder of radius
6 cm. Find the height of the cylinder.
Sol. Here, radius (r) = 4.2 cm
4 3 4
r = r 3 FG IJ
42
3
cm 3
Volume of the sphere =
3 3 H K
10
Let the height of the cylinder = h
Radius of the cylinder (R) = 6 cm
Volume of the cylinder = R2 h = (6)2 h
Since, [Volume of the cylinder] = [Volume of the sphere]
4 42 42 42
62 h =
3 10 10 10
4 42 42 42 1 2744
h = = cm = 2.744 cm
3 1000 36 1000
Q. 2. The figure shows a decorative block which is made of two
solidsa cube and a hemisphere. The base of the block is
a cube with edge 5 cm and the hemisphere, fixed on the top,
has a diameter of 4.2 cm. Find the total surface area of the
22
block. [Take =
] (AI CBSE 2009)
7
Sol. Total surface area of a cube = 6 (side)2
Total surface area of the cubical block
= 6 (5)2 [Base area of the hemisphere]
= 150 r2
Now, total surface area of the solid
= 150 r2 + 2r2 = 150 + r2
22 21 21
= 150 d = 4.2
7 10 10
4.2 21
r= =
2 10
1386
= 150 + = 150 + 13.86 = 163.86 cm2
100
Q. 3. A toy is in the form of a cone mounted on a hemisphere with same radius. The diameter of the
base of the conical portion is 7 cm and the total height of the toy is 14.5 cm. Find the volume
22
of the toy. [Use = ] (AI CBSE 2007)
7

Sol.
LMRadius of the base of OP =
LMRadius of the OP
N the conical portion Q N hemisphere Q
r = 3.5 cm,

362 MathematicsX
Height of the conical portion (h)
= 14.5 3.5 = 11 cm
Now, Volume of the toy
11 cm
Volume of the Volume of the
= + hemispherical part
conical part 3.5 cm
14.5 cm

2 3 1 2 1
= r + r h = r 3 2 r + h
3 3 3

=
1 22 7 7

7 LM
2 + 11 =
11 7 OP
18 cm3
3 7 2 2 2 N 32 Q
= 11 7 3 cm3 = 231 cm3.
Q. 4. In the figure, the shape of a solid copper piece (made of two pieces) with dimensions as shown.
The face ABCDEFA has uniform cross section. Assume that the angles at A, B, C, D, E and F are
right angles. Calculate the volume of the piece. (AI CBSE 2007)
22 cm
A 2 cm
B
5 cm
F
C

8 cm

D
E 3 cm

Sol. Since, volume of a cuboid = lbh


Volume of horizontal piece = 22 (8 + 2) 3 cm3 = 22 10 3 cm3 = 660 cm3
Volume of the vertical piece = 22 2 5 cm3 = 220 cm3
Total volume of the piece = 660 cm3 + 220 cm3 = 880 cm3
Q. 5. A toy is in the form of a cone mounted on a hemisphere of common base radius 7 cm. The total
22
height of the toy is 31 cm. Find the total surface area of the toy. [Use = ] (CBSE 2007)
7
Sol. Height of the cone, h = 31 7 = 24 cm
Radius of the cone = Radius of the hemisphere
r = 7 cm.
Now, Slant height, l = r 2 + h2 = 7 2 + 24 2 = 49 + 576 cm = 625 cm = 25 cm
Total surface area of the toy
= 2r2 + rl = r [2r + l]
22
= 7 [2 7 + 25] cm2 = 22 [14 + 25] cm2
7
= 22 39 cm2 = 858 cm2

Surface Areas and Volumes 363


III. LONG ANSWER TYPE QUESTIONS
Q. 1. A bucket is in the form of a frustum of a cone. Its depth is 15 cm and the diameters of the top
and the bottom are 56 cm and 42 cm respectively. Find how much water can the bucket hold?
22
[use = ]
7
56
Sol. Here r1 = = 28 cm
2
42
r2 = = 21 cm
2
Height h = 15 cm
1 2 2
Since the volume of a frustum of a cone = h ( r1 + r2 + r1 + r2 )
3
Volume of the bucket
1 22
= 15 [282 + 212 + 28 21] cm3
3 7
110 110
= [784 + 441 + 588] cm 3 = 1813 cm 3
7 7
= 110 259 cm3 = 28490 cm3
28490
= litres = 28.49 litres A
1000
Q. 2. The height of a cone is 30 cm. A small cone is cut off h 1
at the top by a plane parallel to the base. If its volume B 2 C
1
be th of the volume of the given cone, at what H = 30 cm
27
height above the base is the section made?
Sol. In the figure, we have
ABC ~ ADE [By AA similarity]
3
BC AB D R
= E
DE AD
[ corresponding sides of similar S are proportional.]
r h
= ...(1)
R 30
1 2
Here, volume of the small cone = r h
3
1 2
Volume of the given cone = R H
3
1
Since, [Volume of the small cone] = [Volume of the given cone]
27
1 2 1 1 FG
R2 H
IJ

3
r h =
27 3 H K
1 1
r2 H
27 3 1 30 10
2 = = =
R 1 27 h 9h
h
3
364 MathematicsX
r h
But = , from (1)
R 30
FG h IJ 2
10

H 30 K =
9h
h2 10
=
900 9h
9 h3 = 10 900 = 9000
h 3 = 1000 = 103
h = 10 AB = 10 cm
The required height BD = AD AB = (30 10) cm = 20 cm
Q. 3. A hollow cone is cut by a plane parallel to the base and the upper portion is removed. If the curved
8
surface of the remainder is of the curved surface of the whole cone, find the ratio of the line
9
segments into which the altitude of the cone is divided by the plane.
Sol. In the figure, we have
ABC ~ ADE [AA similarity]
BC AB AC
= = [ sides of similar s are proportional.]
DE AD AE
r h l
= = ...(1)
R H L
Now, curved surface area of the small cone = rl
Curved surface area of the whole cone = RL
Since, A
[Surface area of the remainder (frustum)]
8 h l
= [curved surface area of the given cone]
9 C
[curved surface area of the frustum] B r
L
8 H
= [curved surface area of the given cone]
9
[curved surface area of the small cone]
1
= [curved surface area of the whole cone] R
E
9 D
1
rl = RL
9
rl 1
=
RL 9
FG IJ FG IJ
r l 1

RH K H K

L
=
9
FG h IJ FG h IJ 1

H HK H HK =
9
h2 FG 1 IJ 2

H2
= H 3K
Surface Areas and Volumes 365
h 1
=
H 3
H = 3h
Now, BD = H h = 3h h = 2h
AB h 1
= 2h =
BD 2
AB : BD = 1 : 2

Q. 4. A circus tent is cylindrical to a height of 3 m and conical above it. If its base radius is 52.5 m
and slant height of the conical portion is 53 m, find the area of the canvas needed to make
22
the tent. [Use = ].
7
Sol. For cylindrical part:
We have, radius (r) = 52.5 m
Height (h) = 3 m
Curved surface area = 2rh
For the conical part
Slant height (l) = 53 m
Radius (r) = 52.5 m
Curved surface area = rl
Area of the canvas = 2rh + rl
= r (2h + l)
22 22 525
= 52.5 (2 3 + 53) m2 = 59 m2
7 7 10
22 75 59 2
= m = 11 15 59 m2 = 9735 m2
10
Q. 5. An iron pillar has some part in the form of a right circular cylinder and remaining in the form
of a right circular cone. The radius of the base of each of cone and cylinder is 8 cm. The cylindrical
part is 240 cm high and the conical part in 36 cm high. Find the weight of the pillar if one
cu. cm of iron weighs 7.8 grams.
Sol. Here, height of the cylindrical part
h = 240 cm
Height of the conical part, H = 36 cm
Radius r = 8 cm
Now, the total volume of the pillar
LMvolume of theOP LM OP
volume of the
= cylindrical part + conical part
N Q N Q
L1 O L 1 O
= [r h] + M r H P = r M rh + H P
2
2
N3 Q N 3 Q
2

22 L 1 O
8 M8 240 + 36P cm =
2 22 64
3
240 + 12 cm 3
=
7 N 3 Q 7
22 64
= 252 cm 3 = 22 64 36 cm3 = 50688 cm3
7

366 MathematicsX
78 50688
Weight of the pillar = 7.8 50688 g = kg
10 1000
3953664
= kg = 395.3664 kg 395.36 kg.
10000
Q. 6. An open container made up of a metal sheet is in the form of a frustum of a cone of height 8 cm
with radii of its lower and upper ends as 4 cm and 10 cm respectively. Find the cost of oil which
can completely fill the container at the rate of ` 50 per litre. Also, find the cost of metal used, if
it costs ` 5 per 100 cm2. [Use = 3.14]
Sol. Here, for the frustum,
r1 = 10 cm, r2 = 4 cm and h = 8 cm
l = h 2 + ( r1 r2 ) 2
10 cm
= 8 2 + (10 4 ) 2

= 64 + 6 2 cm 8 cm

= 64 + 36 cm = 100 cm
= 10 cm 4 cm

Volume of the container


1
= h (r12 + r22 + r1r2)
3
1 314
= 8 [102 + 42 + 10 4] cm3
3 100
314 8 314 8
= [100 + 16 + 40] cm3 = 156 cm 3
3 100 3 100
130624 130624
= cm 3 = 1306.24 cm3 =
100 100 1000 litres
= 1.30624 litres 1.31 approx.
Cost of oil = ` (1.31 50) = ` 65.50
Now, the total surface area of the container (excluding upper base)
314 314
= l (r1 + r2) + r22 = 10 (10 + 4) + 42 cm2
100 100
314 314 48984
= [10 14 + 16] cm2 = 156 cm2 = cm2 = 489.84 cm2
100 100 100
5
Cost of metal = ` ( 489.84 ) = ` 24.492 = ` 24.49 (appx.)
100
Q. 7. A building is in the form of a cylinder surmounted by a hemispherical dome as shown in the figure.
2
The base diameter of the dome is equal to of the total height of the building. Find the height
3
1
of the building, if it contains 67 m3 of air. [NCERT Exemplar]
21

Surface Areas and Volumes 367


Sol. Here, radius of the hemispherical part = r (say)
r
Let the total height of the building = h
And the height of the cylindrical part = H r
h


LM Diameter of OP =
2 LMTotal height of OP
Nbase of the domeQ 3 N the building Q H

2
2r = h
3
1 2 h
r = h =
2 3 3
FG h h IJ metres = 2 h metres
H =
H 3K 3

Volume of air LM Volume of OP LVolume of air inside theO



Now, inside the = MMair inside thePP MN cylindrical part PQ
building N dome Q
2 3
= r + r2H
3

2 h

LM OP 3

LM h OP LM 2 hOP
2
=
3 3 NQ N3Q N3 Q
8
= h 3 m3
81
But, volume of the air in the building
1 3
= 67 m
21
8 1
h3 = 67
81 21
22 8 1408
h3 =
7 81 21
1408 7 81
h3 =
21 22 8
h 3 = 8 27
= 216
h3 = (6)3
h = 6m
Thus, the required height of the building is 6 metres.

368 MathematicsX
Q. 8. A solid toy is in the form of a hemisphere surmounted by a right circular cone. The height of the
cone is 4 cm and diameter of the base is 8 cm. Determine the volume of the toy. If the cube
circumscribes the toy, then find the difference of the volumes of the cube and the toy. Also, find
the total surface area of the toy. [NCERT Exemplar]
Sol. Let the radius of the hemisphere = r
And height of the cone = h

Now,
LMVolume of OP = LMVolume of OP + LM Volume of OP
N the toy Q N the cone Q Nthe hemisphereQ
1 2 2
= r h r = M 3 L 1 22 4 4OP LM 2 22 4 OP cm
2 3 3
3 3 N3 7 Q N3 7 Q
1 22 1 22
= 43 1 2 cm 3 = 4 4 4 3 cm3
3 7 3 7
64 22 1408
= cm3 = cm3
7 7
Since, the cube circumscribing the given solid must have its edge as (4 + 4) cm i.e., 8 cm,
Volume of the cube = (edge)3 = (8)3 cm3 = 512 cm3
Now, difference in volumes of the cube and the toy

LM512 1408 OP cm 3
=
N 2 Q

3584 1408
= cm3
7
2176
= cm3 = 310.86 cm3
7
Also, total surface area of the toy

=
LMcurved surface areaOP + LM curved surface area OP
N of conical part Q Nof the hemispherical partQ
= rl + 2r2 [where, l = h 2 r 2 ]

22
= r [l + 2r] = 4 ( 4 2 4 2 ) 2 4 cm 2
7

22
= 4 [4 2 8] cm 2
7
22 352
= 4 4 [ 2 2] cm 2 = ( 2 + 2) = 171.68 cm2
7 7

Surface Areas and Volumes 369


TEST YOUR SKILLS
1. The diameter of a sphere is 28 cm. Find the cost of painting it all around at ` 0.10 per
square cm.
2. The perimeter of one face of a wooden cube is 20 cm. Find its weight if 1 cm3 of wood
weighs 8.25 g.

3. The radii of two cylinders are in the ratio of 1 : 3 . If the volumes of two cylinders be
same, find the ratio of their respective heights.
4. If the radius of the base of a cone is doubled keeping the height same. What is the ratio
of the volume of the larger cone to the smaller cone?
5. If the length, breadth and height of a solid cube are in the ratio 4 : 3 : 2 and total surface
area is 832 cm2. Find its volume.
6. Three cubes of a metal whose edges are in the ratio 3 : 4 : 5 are melted and converted into
a single cube whose diagonal is 12 3 cm. Find the edges of the three cubes.
[NCERT Exemplar]
Hint:
Let the edges of the given cubes be 3x, 4x and 5x.
\ Sum of volumes of the given cubes = (3x)3 + (4x)3 + (5x)3 = 216x3
Let edge of the new cube be a.

\ Diagonal of the new cube = a2 + a2 + a2 = 3a 2 = a 3

a 3 = 12 3 or a = 12
Also, volume of the new cube = a3 = 216 x3
123 = 216 x3 x = 2
\ 3x = 6, 4x = 8 and 5x = 10

7. A toy is in the form of a cone mounted on a hemisphere of common base of diameter 7 cm.
22
If the height of the toy is 15.5 cm, find the total surface area of the toy. [Take p = ]
7
[CBSE 2012]
8. A circular tent is cylindrical up to a height of 3 m and conical above it. If the diameter
of the base of the cone and cylinder is 105 m and the slant height of the conical part is
53 m, find the total canvas used in making the tent. [CBSE 2012]
9. A solid composed of a cylinder with hemispherical ends. If the whole length of the solid
is 108 cm and the diameter of the hemispherical end is 36 cm, find the cost of polishing
its surface at the rate of 70 paise per square cm. [CBSE 2012]
10. A 5 m wide cloth is used to make a conical tent of base diameter 14m and height 24m.
22
Find the cost of cloth used at the rate of ` 25 per metre [Use ]
7
[AI. CBSE (Foreign) 2014]

370 MathematicsX
Hint:
Slant height = h2 + r2 = 24 2 + 7 2 = 25 m

24 m
22
Curved S.A = rl = 25 7 m2 = 550 m2
7
Area of the cloth required = length breadth
550
= l 5 = 550 l = = 110 m
5
Cost of the cloth = ` 25 110 = ` 2750 per metre. 14 m

11. A girl empties a cylindrical bucket, full of sand, of base radius 18 cm and height 32 cm,
on the floor to form a conical heap of sand. If the height of this conical heap is 24 cm,
then find its slant height correct up to one place of decimal. [AI. CBSE (Foreign) 2014]
Hint:
Let radius and height of cylindrical bucket = R and H respectively
Let r and h be radius and height of conical heap.

1 2 3 H R 2
R2H = r h r =
3 h
3 18 1832
= = 36
24

l = r 2 + h 2 = 36 2 + 24 2
= 1296+ 574
= 1872 = 43.3cm

12. The largest possible sphere is carved out of a wooden solid cube of side 7 cm. Find the
22
volume of the wood left. [Use ] [AI. CBSE 2014]
7
Hint:
Volume of the wooden cube = 7 7 7cm3 = 343cm3
1 1
Radius of the sphere = side of the cube = (7 cm) = 3.5 cm
2 2
3
4 3 4 22 7 3
Volume of the sphere = r = cm = 179.66 cm3
3 3 7 2

volume of volume
of sphere = (343 179.66)cm
3
Volume of the wood left = cube

= 163.34 cm3

Surface Areas and Volumes 371


13. 150 spherical marbles, each of radius 1.4 cm, are dropped in a cylindrical vessel of radius
7 cm containing some water, which are completely immersed in water. Find the rise in
the level of water in the vassel. [AI. CBSE 2014]

Hint:
Volume of 150 spherical marbles = 150 [volume of a spherical marble]
4 22
1.4 = 548.8 cm 3
3
= 150
3 7
Let the level of water rises h cm.
22
Volume of h high water in the vessel = r h cm =
2 3
7 7 h = 154 h cm 3
7
5488 1
154 h = 548.8 h = = 3.56 cm
10 154

ANSWERS
Test Your Skills

1. ` 246.40 2. 1031.25 g 3. 3 : 1 4. 4 : 1 5. 1536 cm3


6. 6 cm, 8 cm, 10 cm 7. 214.5 cm2 8. 9735 m2 9. ` 8553.60

372 MathematicsX
Value Based Questions (Solved)

Q1. Rahul plans to grow organic vegetables in a 100 sq. m


rectangular plot. He has only 30 m of barbed wire
which can fence its three sides. Fourth side of his plot
touches Rehaman's compound wall. He requests Rehaman
to allow his compound wall be used as fencing to his plot.
(a) Find the dimensions of the plot.
(b) Which mathematical concept is used in above problem?
(c) By allowing the compound wall to act as fencing,
which value is depicted by Rehaman?
Sol. Let x metres be the width and y metres be the
length of the vegetable plot.
Area = xy sq. m
xy = 100 ... (i)
length of the barbed wire = 30 m
x y + y = 30 m
x 2y = 30
30 x
y = ... (ii)
2
From (i) and (ii), we have
30 x
x
2 = 100
x2 30x + 200 = 0
(x 10) (x 20) = 0
x = 10 or x = 20
100 100
For x = 10, we get y = = = 10
x 10
100 100
For x = 20, we get y = = =5
x 20
Thus,
(a) The dimensions of the plot are: 10m, 10m or 20m, 5m.
(b) Quadratic Equations
(c) Co-operation.
Q2. A shopkeeper buys a certain number of books from a publisher for ` 80. The publisher gives him 4
more books for same amount with a condition that the shopkeeper would donate ` 1 per book to an
orphanage.
(a) How many books did he buy?
(b) Which mathematical concept is used in this problem?
(c) By allowing ` 1 per book towards an orphanage, which value is depicted by the publisher?
373
Sol. Let the number of books bought = x
Cost of x books = ` 80
80
Cost of 1 book = `
x
Again,
Cost of (x + 4) books = ` 80
80
Cost of 1 book = ` ( x + 4)

Since, the shopkeeper donates ` 1 per book to an orphanage.


80 80 1 1
= 1 80 = 1
x ( x + 4) x x 4
x 4 x 320
80 2 = 1 =1
x 4x 2
x 4x
x2 + 4x = 320 x2 + 4x 320 = 0
2
x + 20x 16x 320 = 0 x (x + 20) 16 (x + 20) = 0
(x + 20) (x 16) = 0 x = 20 or x = 16
x cannot of negative
x = 16
Thus,
(a) Number of books bought = 16 (b) Quadratic Equations (c) Charity
Q3. Radha wants to buy a piece of cloth for ` 200. She bought another 5m piece of cloth for donation
to a blind-school. For this, the shopkeeper reduces the cost by ` 2 per metre such that the total cost
remains the same (` 200).
(a) What is the original rate per metre?
(b) Which mathematical concept is used in the above problem?
(c) By donating a piece of cloth to the blind-school, which value is depicted?
Sol. Let the original length of cloth = x metres
200
Original rate = `
x
New length of the cloth = (x + 5) metres
200
New length of cloth = ` ( x 5)

The new rate is ` 2 less than the original rate

200 200 1 1
= 2 200 x ( x 5) = 2
x ( x 5)
x 5 x
200 2 = 2 1000 = 2 (x2 + 5x)
x 5x
1000 = 2x2 + 10x
x2 + 5x 500 = 0 x2 + 25x 20x 500 = 0
(x + 25) (x 20) = 0 x = 25, which is not desirable or x = 20
374 MathematicsX
Now,
200
(a) Original rate = ` per metre = ` 10 per metre
20
(b) Quadratic Equations
(c) Charity
Q4. Ranjeet wants to go by car from place A to B. He has two options.
B
(i) He can go straight from A to B.
(ii) He goes to C due east and then from C to B due north.
The distance between A to B exceeds the distance between A to C

North
by 2 km. The distance between A to B exceeds twice the
distance between C and B by 1 km.
He decided to choose the option (i) for going from A to B.
(a) Find the distance difference in the above two options.
A East C
(b) Which mathematical concept is used in the above
problem?
(c) By choosing the option (i), which value is depicted by Ranjeet?
Sol. Let the distance between C and B = x km
Distance between A and B = (2x + 1) km
And distance between A and C = (2x + 1) 2 km
B
= (2x 1) km
The direction East and North are perpendicular to each other.
AC BC
) km
ABC is a right angle , right angled at C +1

x km
(2x
Using Pythagoras theorem, we have:
x2 + (2x 1)2 = (2x + 1)2
x + 4x2 4x + 1 = 4x2 + 4x + 1
2

x2 8x = 0 A (2x 1) km C
x (x 8) = 0
Either x = 0, [Which is not possible as
distance cannot be zero]
Or x = 8
Now, AB = (2x + 1) km = (16 + 1) km = 17 km
AC = (2x 1) km = (16 1) km = 15 km
BC = x km = 8 km
Now, (a) Difference in distance = (AC + BC) AB = (15 + 8) km 17 km
= 23 km 17 km = 6 km
(b) Quadratic Equations
(c) Saving of National resource (fuel consumption)
Q5. Savita has two options to buy a house:
(a) She can pay a lumpsum amount of ` 22,00,000
Or
(b) She can pay ` 4,00,000 cash and balance in 18 annual instalments of ` 1,00,000 plus 10%
interest on the unpaid amount.
Value Based Questions 375
She prefers the option (i) and donates 50% of the difference of the costs in the above two options
to Prime Minister Relief Fund.
(i) What amount was donated to Prime Minister Relief Fund?
(ii) Which mathematical concept is used in the above problem?
(iii) By choosing to pay a lumpsum amount and donating 50% of the difference to Prime
Minister Relief Fund, which value is depicted by Savita?
Sol. (a) Total cost of the house = ` 22,00,000
(b) Cash payment = ` 4,00,000
Balance = ` 22,00,000 ` 4,00,000 = ` 18,00,000
1st instalment = ` [1,00,000 + 10% of balance]
10
= ` [1,00,000 + 18,00,000]
100
= ` [1,00,000 + 1,80,000] = ` 2,80,000
Balance after 1st instalment = ` [18,00,000 1,00,000] = ` 17,00,000
2nd instalment = ` [1,00,000 + 10% of 17,00,000]
= ` [1,00,000 + 1,70,000] = ` [2,70,000]
Balance after 2nd instalment = ` 17,00,000 ` 1,00,000 = ` 16,00,000
3rd instalment = ` [1,00,000 + 10% of 16,00,000]
= ` [1,00,000 + 1,60,000] = ` 2,60,000
... and so on.
Total amount in instalments = ` 2,80,000 + ` 2,70,000 + ` 2,60,000 + ..... to 18 terms
n
=
2
2 a (n 1)d , where a = 2,80,000, d = 10,000, n = 18
18
= ` [2(2,80,000) + (18 1) (10,000)
2
= ` 9 5, 60, 000 17 (10, 000)

= ` 9 5, 60, 000 1, 70, 000

= ` 9 3, 90, 000 = ` 35,10,000


Total cost of house = ` 35,10,000 + 4,00,000 = ` 39,10,000
Difference in costs of the house in two options
= ` 39,10,000 ` 22,00,000 = ` 17,10,000
(i) Amount donated towards Prime Minister Relief Fund = 50% of ` 17,10,000
50
= ` 17,10,000 = ` 8,55,000
100
(ii) Arithmetic Progressions
(iii) National Loyality
Q6. Shankar wants a lawn to be developed in a corner of his square plot. He gave following three options
to the lawn-developer:

376 MathematicsX
7m 7m 7m

7m 7m 7m

I II III
The lawn developing rate is ` 150 per sq. m and the lawn under option-III was developed, however,
Shankar paid for the option having largest area. The lawn-developer realized the mistake and
refunded the balance back to Shankar.
(a) Find the area in each of the above three options.
(b) What amount was refunded back to Shankar by the lawn-developer?
(c) Which mathematical concept is used in above problem?
(d) Refunding the difference, which value is depicted by the lawn-developer?
1
Sol. (a) Area of lawn in option-I = (r2)
4
1 22 154
= 7 7 m2 = 4 m2 = 38.5 m2
4 7
Area of lawn in option-II = 7 7 m2 = 49 m2
1 49 2
Area of lawn in option-III = 7 7 m2 = m = 24.5 m2
2 2
(b) Largest area of the lawn = 49 m2
Cost of lawn-developing in option-II
= ` 150 49 = ` 7350
Area of lawn in option-III = 24.5 m2
Cost of lawn-developing in option-III
245
= ` 150 24.5 = ` 150
10
= ` 15 245 = ` 3675
Amount refunded back by the lawn-developer
= ` 7350 ` 3675 = ` 3675
(c) Areas related to plane surfaces.
(d) Honesty
Q7. Shivram has a piece of land in the form of sector OBPQ
adjoining to a temple. He donates a part of it to the
temple such that a square plot OABC is left with him.
If OA = 20 m. Then,
(a) Find the area of the shaded region (donated to the
temple). [use = 3.14]
(b) Which mathematical concept is used in above
problem?
Value Based Questions 377
(c) By donating land to a temple by Shivram, which value is depicted?
Sol. (a) OABC is a square and OA = 20 m
Diagonal OB of the square OABC
= 202 + 20 2 = 400 + 400 = 800 m
Radius of the quadrant OPBQ = 800 m
1 2 1 314 2
Area of the quadrant OPBQ = r = 800 m 2
4 4 100
1 314 1
= 800 m 2 = 314 8 m2
4 100 4
= 314 2 m2 = 628 m2
Also, area of square OABC = 20 20 m2 = 400 m2
Area of the shaded region = [Area of the quadrant OPBQ] [Area of the square OABC]
= 628 m2 400 m2 = 228 m2
Thus, area of the land donated to the temple = 228 m2
(b) Areas related to circles
(c) Charity.
Q8. Rajat has a piece of land in the shape of a rectangle with two semicircles on its smaller sides,
as diameters, added to its outside. The sides of the rectangle are 36 m and 21 m. He lets out
the two semicircular parts at the rate of ` 100 per sq. metre and donates the proceeds to
an orphanage.

22
(a) Find the total area of his land. [use =]
7
(b) What amount does he donate to orphanage?
(c) Which mathematical concept is used in the above problem?
(d) By donating the rent proceeds to an orphanage which value is depicted by Rajat?
Sol. (a) Area of the rectangular part = 36 24.5 m2
245
= 36 m2 = 18 49 m2 = 882 m2
10
1 2
Area of both the semicircular parts = 2 r
2
1 22 2
= 2 21 21 m = 22 3 21 m2 = 1386 m2
2 7
Area of Rajats total land = 882 m2 + 1386 m2 = 2268 m2
378 MathematicsX
(b) Rent rate = ` 100 per m2
Total rent = ` 1386 100 = ` 1,38,600
Amount donated to orphanage by Rajat = ` 1,38,600
(c) Surface areas related to circles
(d) Charity
Q9. Rajesh has a circular plot of radius 105m. He donates a 7m wide
track along its boundary for community-track.

22
(a) Find the area of the track. [use = ]
7
(b) Which mathematical concept is used in the above problem?
(c) By donating a community-track, which value is depicted by
Rajesh?
Sol. (a) Radius of the total circular plot = 105 m
Area of the total circular plot = (105)2 sq. m
Width of the track = 7 m
Radius of the circular plot excluding the track
= 105 m 7 m = 98 m
Area of the inner circular plot = (98)2 sq. m.
Now, area of the track = [Area of total circular plot] [Area of inner circular plot]
= (105)2 (98)2 sq. m
= [1052 982] sq. m
= [(105 98) (105 + 98)] sq. m
[using a2 b2 = (ab) (a+b)]
= [7 203] sq. m
22
= 7 203 sq. m
7
= 22 203 sq. m
= 4466 sq. m
(b) Areas related to circles
(c) Community service
Q10. Rohan is in the business of supplying water. He has three types of tankers of inner diameter 1.0 m
to supply water to the customers. The length of the tankers is 6 m. [use = 3.14]
He decided to serve his customers with type A tankers.

Value Based Questions 379


(a) Find the volume of tanker of type A.
(b) Which tanker has the minimum capacity?
(c) Which mathematical concept is used in the above problem?
(d) By choosing a tanker of type-A, which value is depicted by Rohan?
Sol. Diameter = 1.0 m
1
Radius = m
2
Length of the tank = 6 m h = 6 m
(a) Volume of the tanker of type-A = r2h
1 1
= 3.14 6 cu. m
2 2
314 1 1 471
= 6 cu.m = cu. m = 4.71 cu. m
100 2 2 100
(b) Volume of the tanker of type-B=[Volume of the tanker-A] [Volume of hemisphere]
2 3
= 4.71 cu. m r
3
2 314 1 1 1
= 4.71 cu. m cu.m
3 100 2 2 2
157
= 4.71 cu. m cu. m
600
471 157
= cu. m cu. m
100 600
2669
= cu. m = 4.45 cu. m
600
Volume of the tanker of type-C = [Volume of the tanker-A] [Volume of the cone]

380 MathematicsX
1 2
= 4.71 cu. m r h
3
1 1 1
= 4.71 cu. m 3.14 0.6 cu.m
3 2 2
= [4.71 0.157] cu. m = [0.157] cu. m = 4.553 cu. m
Thus, the tanker of type-B has the minimum capacity of 4.45 cu. m.
(c) Mensuration [volume of solid figures]
(d) Honesty
Q11. Shankar, a Kulfi-vendor has three types (spherical, conical and cuboidal) of containers for making Kulfi :

7.0 cm

10 cm
10 cm

cm 5
3.
7.0 cm 3.5
cm

Type I Type II Type III

He decided to serve the customers with type-I container.


(a) Find the volume of the container type-I.
(b) Which container has the minimum capacity?
(c) Which mathematical concept is used in this problem?
(d) By choosing to prepare to sell Kulfi using container of type-I, which value is depicted by Shankar?
Sol. (a) Finding the volume of Type-I container:
Diameter = 7 cm
7
Radius = cm
2
4 3
Volume of the container of type-I = r
3
4 22 7 7 7
= cu.cm
3 7 2 2 2
117 7
= cu. cm = 179.66 cu. cm
3
1 2
(b) Volume of conical (type-II) container = r h
3
1 22 7 7
= 10 cu.cm
3 7 2 2

Value Based Questions 381


11 35
= cu.cm = 128.33 cu. cm
3
Volume of cuboidal (type-III) container = l b h
35 35
= 3.5 3.5 10 cu. cm = cu.cm = 122.5 cu. cm
10
Obviously, the capacity of the cuboidal (type-III) container is minimum, which is
122.5 cu. cm.
(c) Volume of solid bodies (mensuration)
(d) Honesty
Q12. Prashant has undertaken a contract to build a wall of 9m long, 2.5m thick and 6m high. His labour
is to be calculated according to the number of bricks used to complete the wall. In the market three
types of bricks are available.
Type-I : Each measuring 25cm 11.25cm 6cm
Type-II : Each measuring 20cm 8cm 10cm
Type-III : Each measuring 25cm 10cm 9cm
Prashant used bricks of type-III.
(a) Find the number of bricks of type-III required to build the wall.
(b) In which case, maximum number of bricks will be used?
(c) Which mathematical concept is used in the above problem?
(d) By using the bricks of type-III, which value is depicted by Prashant?
Sol. Volume of a brick of type-I = l b h

11.25
cm

6.0
cm

25.0 cm
= 25 cm 11.25 cm 6 cm
= 1687.50 cm3
Volume of a brick of type-II = l b h
= 20 cm 8 cm 10 cm = 1600 cm3
Volume of a brick of type-III = l b h
= 25 cm 10 cm 9 cm = 2250 cm3
Volume of the wall = l b h
= 9 2.5 6 m3 = 135 106 cm3
(a) Number of bricks of type-III required for building the wall
Volume of the wall
= Volume of a brick of type-III

382 MathematicsX
135 106
= = 60,000
2250
(b) Number of bricks required for building the wall using type-I bricks:
Volume of the wall
= Volume of a brick of type-I
135 10, 00, 000
= = 80,000
1687.50
Number of bricks of type-II required for building the wall
Volume of the wall
= Volume of a brick of type-II
135 10, 00, 000
= = 84,375
1600
In case of type-II, the maximum number of bricks will be required.
(c) Volume of solid bodies (mensuration)
(d) Honesty
Q13. Sampat has set up his juice shop. He has three types of cylindrical glasses as given below :

A cylindrical glass with inner diameter 7 cm and height as 10 cm.

A cylindrical glass with inner diameter 4 cm and height as 14 cm.

A cylindrical glass with inner diameter 14 cm and height as 4 cm.

Value Based Questions 383


He decided to serve the customers in type-I of glasses.
(a) Find the volume of the glass of type-I.
(b) Which glass has the minimum capacity?
(c) Which mathematical concept is used in above problem?
(d) By choosing a glass of type-I, which value is depicted by juice seller Sampat?
Sol. (a) Diameter of glass of type-I = 7 cm
7
Radius = cm
2
Height = 10 cm
Volume = r2h
22 7 7
= 10 cm 3
7 2 2
= 385 cm3
(b) Diameter of glass of type-II=4 cm
Radius = 2cm
Height = 14 cm
Volume = r2h
22
= 2 2 14 cm 3
7
= 176 cm3
Diameter of glass of type-III = 14 cm
14
Radius = cm = 7 cm
2
Height = 4 cm
Volume = r2h
22
= 7 7 4 cm 3
7
= 2228 cm3
= 308 cm3
The glass of type-II has the minimum capacity.
(c) Mensuration [volume of solid bodies]
(d) Honesty.
Q14. A community-well with 10 m inside diameter is dug 14 m deep. For spreading the earth taken out
of it, there are two options :
(i) It is spread all around the well to a width of 6 m to form an embankment.
(ii) It is spread evenly on a rectangular surface of 25m 11m.
The contractor charges according to the height of the spreadover mud. He charges by choosing the
option-(i).
(a) Find the height of the spreadover mud in both options.

384 MathematicsX
(b) Which mathematical concept is involved in the above problem?
(c) In which case cost of digging the well is less?
(d) Which values are depicted by the contractor by charging according to option-(i)?
Sol. Inside diameter of the well = 10 m 10.0 m
Inside radius of the well = 5 m
Depth of the well = 14 m
Volume of the earth dugout = r2h
22
= 5 514 m 3 14.0
7 m
= 22 5 5 2 m3
= 1100 m3
(a) Height of the spreadover mud:
option-(i)
Area of the shaded-region (base of the spreadover mud)

5m
11 m

= R2 r2
= [R2 r2] [Here: R = 11 m and r = 5 m]
= [(R r) (R + r)]
22
= [(11 5) (11 + 5)] m2
7
22
= 616 m 2
7
22 96
= m2
7
Let height of the spreadover mud (embankment) be h1,
22 96
h1 = 1100
7
11007
h1 =
22 96
Value Based Questions 385
= 3.64 m 25 m

option-(ii)
Let the height of the spreadover mud = h2,
25 11 h2 = 1100
1100 11.0 m
h2 =
2511
= 4m
(b) Mensuration [volume of solid bodies]
(c) The height of spreadover mud is less in option-(i).
The cost of digging the well is less in option-(i).
(d) Community-service and honesty.
Q15. A contractor is entrusted to erect a tent for flood-victims. He is allowed a fixed amount for this task.
He has two options :
(i) to erect a tent which is cylindrical upto a height of 3 m and conical above it. The diameter of
the base is 105 m and slant height of the conical part is 53 m.
m
.0
53

52.5 m

3.0 m

105 m

(ii) to erect the tent as described in option-(i), only replacing the conical part as a hemispherical part.

52.5 m

3m

105 m

The contractor chooses the option-(ii) and decides to donate the extra (difference) canvas to be used
in this case.
(a) How much canvas is donated by the contractor?
(b) Which mathematical concept is used in the above problem?
(c) By choosing the option-(ii) to erect the tent, which value is depicted by the contractor?

386 MathematicsX
Sol. (a) Total canvas used in option-(i) = [Curved surface area + [Curved surface area
of the cylindrical part] of the conical part]
= [2rh + rl]
22 22
= [(2 52.5 3) + ( 52.5 53)] m2
7 7

22
= 52.5 [(23) + 53] m2
7

22
= 52.5 59 m2
7
= 9733 m2
Total canvas used in option-(ii)
= [Curved surface area + [Curved surface area
of cylindrical part] of hemispherical part]
= [2rh + 2r2]

22 22
= [(2 52.5 3) + (2 52.5 52.5)] m2
7 7
22
= 2 52.5 [3 + 52.5] m2
7
2 22 52.5
= [55.5] m2
7
2 22 52.5 55.5 2
= m
7
= 18315 m2
Difference in areas of canvas required in the above two options.
= 18315 9733 m2
= 8582 m2
The contractor donated 8582 m2 of canvas.
(b) Mensuration [volume of solid bodies]
(c) Patriotism
Q16. A fire at a building B is reported on telephone to two fire stations F1 and F2, 10 km apart from each
other on a straight road.

45 60
F2 F1
10.0 km
Value Based Questions 387
F1 observes that the fire is at an angle of 60 to the road and F2 observes that it is at angle of 45
from it. The station F1 sends its team.
(a) Why the team of station F1 was sent?
(b) How much distance the station F1 team will have to travel?
(c) Which mathematical concept is involved in the above problem?
(d) By sending its team, which value is depicted by the fire-station F1?
Sol. Let BL be the perpendicular from B on F1F2.

45 L 60
F2 F1
10.0 km

(a) BL F1F2
and LF1B = 60;
LF2B = 45
From right-angled F1LB and F2LB, we have:
BL BL
sin 60 = F B and sin 45 = F B
1 2

BL BL
F1B = sin 60 and F2B = sin 45

sin increases as increases from 0 to 90.


sin 45 < sin 60
BL BL
sin 45 > sin 60 or F2B >F1B

Distance of B from F2 is more than that of from F1. That is why the station F1 sent
its team to B.
F1 L
(b) In right F1LB, cos 60 = F B
1
Let F1B be x km
x cos 60 = F1L ..........(1)
BL
Also, BF = sin 60
1

BL = BF1sin 60
F2L = x sin 60 ..........(2)
BL
In right F2LB, tan 45 = F L
2

388 MathematicsX
BL
1 = FL
2
BL = F2L ..........(3)
From (2) and (3), we get
F1L + F2L = 10
1
x cos 60 + x sin 60 = 10 cos 60 =
2
3
x [cos 60 + sin 60] = 10 sin 60 =
2
1 3
x 2 + 2 = 10

1+ 3
x 2 = 10

1 + 1.732
x = 10
2
x 2.732 = 20
20
x =
2.732
x = 7.32 km
Station F1 team travelled 7.32 km
(c) Trigonometry [Heights and Distances]
(d) Promptness.
Q17. Roshan is a wholesaler of electric-bulbs. He sells a box of 600 electric bulbs which contains 12
defective bulbs to an electrician. He gives 12 extra bulbs to the electrician. In the process of replacing
the defective bulbs by non-defective bulbs, the electrician takes out one bulb at random from the box.
(a) What is the probability that it is non-defective bulb?
(b) Which mathematical concept is used in the above problem?
(c) By giving 12 extra non-defective bulbs to the electrician, which value is depicted by Roshan?
Sol. (a) Total number of bulbs in the box = 600
Number of defective bulbs = 12
Number of non-defective bulbs = 600 12
= 588
Number of possible events = 600
Number of favourable events = 588
[Number of favourable events]
Probability of non-defective bulbs = [Number of possible events]
588 49
= =
600 50
(b) Probability
(c) Honesty
Value Based Questions 389
Q18. There are 40 students in class X, of whom 25 are girls and 15 are boys. They plan to go to help
the earthquake victims in a camp. Their class-teacher has to select one student at random for the class
representative.
(a) What is the probability that the student selected for class-representative is
(i) a girl (ii) a boy
(b) Which mathematical concept is used in the above problem?
(c) By planning to go for the help of earthquake victims, which value is depicted by the students
of class X?
Sol. (a) Total number of students in the class = 40
Number of boys = 15
Number of girls = 25
(i) Since, the class representative selected at random is to be a girl
Number of possible outcomes = 40
Number of favourable outcomes = 25 [ There are 25 girls in the class]
Probability of a girl to be the representative
Favourable outcomes
=
Possible outcomes
25 5
or =
40 8
(ii) Since, the class-representative selected at random is to be a boy
Number of favourable outcomes = 15
15 3
Probability of a boy to be the representative = or
40 8
(b) Probability
(c) Helping the persons, in need.
Q19. In a school, students decided to plant trees in and around the school to reduce air pollution. It was decided
that the number of trees, that each section of each class will plant, will be double of the class in which
they are studying. If there are 1 to 12 classes in the school and each class has two sections, find how
many trees were planted by the students. Which value is shown in this question? [AI. CBSE 2014]
Sol. There are 12 classes in all.
Each class has 2 sections.
Number of plants planted by class I = 1 2 = 2
Number of plants planted by class II = 2 2 = 4
Number of plants planted by class III = 3 2 = 6
Number of plants planted by class IV = 4 2 = 8
......................................................................................................
......................................................................................................
Number of plants planted by class XII = 12 2 = 24
The numbers 2, 4, 6, 8, ........................ 24 forms an A.P.
Here, a = 2, d=42=2
Number of classes = 12
Number of terms (n) = 12

390 MathematicsX
n
Now, the sum of n terms of the above A.P., is given by Sn =
2
2 a (n 1)d
12
S12 = 2 2 12 1 2
2
= 6 [4 + (11 2)]
= 6 26 = 156
Thus, the total number of trees planted = 156
Value shown: To enrich polution free environment.
Q20. Sushant has a vessel, of the form of an inverted cone, open at the top, of height 11 cm and radius of top
as 2.5 cm and is full of water. Metallic spherical balls each of diameter 0.5 cm are put in the vessel due
2
to which th of the water in the vessel flows out. Find how many balls were put in the vessel. Sushant
5
made the arrangement so that the water that flows out irrigates the flower beds. What value has been
shown by sushant? [CBSE Delhi 2014]
Sol. Base radius of the cone (r) = 2.5cm
Height of the concial part (h) = 11cm
1 2
Using V = r h , the volume of the conical vessel.
3
1 22 25 25 11 5 25 11
3
= 11 cm = 3710 cm 3
3 7 10 10
2 11 5 2511 11 5 11
of = 37 cm 3
5 37 10
11511 3
cm of water is flown out due to n spherical balls each of radius = 0.5 cm
37
11 511 3
Volume of n balls =
37 cm

11 511
n volume of one ball = cm 3
37
4 22 5 5 5 11 5 11
n =
3 7 10 10 10 37
11 5 11 3 7 10 10 10
n=
37 4 22 5 5 5
n = 11 5 = 55
Thus, the required number of leadshots = 55
Value : To keep the plants green for pollution free surroundings.
Q21. The angle of elevation of the top of a chimney from the foot of a tower is 60 and the angle of depression
of the foot of the chimney from the top of the tower is 30. If the height of the tower is 40m, find the
height of the chimney. According to pollution control norms, the minimum height of a smoke emitting
chimney should be 100m. State if the height of the above mentioned chimney meets the pollution norms.
What value is discussed in this question? [AI. CBSE (Foreign) 2014]

Value Based Questions 391


Sol. In the figure, the height of the tower (AB) = 40 m
Let the height of the chimney (CD) = h metre.
D
Now, in rt. ABC, we have :
Let BC = x
40 1
= tan 30 =
x 3
x = 40. 3 ...(1)
In rt. BCD, A h
h 30
= tan 60 = 3
x
h

40m
x = ...(2)
3
From (1) and (2) we have, 60 30
h B x C
= 40 3 h 40 3 3
3
= 40 3 m = 120 m
Thus, the height of the chimney is 120 m.
As, the minimum height of a chimney (according to the pollution control norms) should be
100 m
120 m > 100 m
Thus, the above mentioned chimney meets the pollution norms.
Value : To keep pollution free environment.

392 MathematicsX

Potrebbero piacerti anche